This page intentionally left blank
Endocrinology and Diabetes
CLINICAL CASES UNCOVERED
This book is dedicated to my daughter Nour and wife Manar, for their care,
patience and support, and to my parents for their constant encouragement
Endocrinology
and Diabetes
CLINICAL CASES UNCOVERED
Ramzi Ajjan
MRCP, MMed Sci, PhD
Senior Lecturer and Honorary Consultant
in Diabetes and Endocrinology
Department of Health Clinician Scientist
The LIGHT Laboratories
University of Leeds
Leeds, UK
A John Wiley & Sons, Ltd., Publication
This edition first published 2009, © 2009 by R. Ajjan
Blackwell Publishing was acquired by John Wiley & Sons in February 2007. Blackwell’s publishing
program has been merged with Wiley’s global Scientific, Technical and Medical business to form
Wiley-Blackwell.
Registered office: John Wiley & Sons Ltd, The Atrium, Southern Gate, Chichester, West Sussex, PO19
8SQ, UK
Editorial offices: 9600 Garsington Road, Oxford, OX4 2DQ, UK
The Atrium, Southern Gate, Chichester, West Sussex, PO19 8SQ, UK
111 River Street, Hoboken, NJ 07030-5774, USA
For details of our global editorial offices, for customer services and for information about how
to apply for permission to reuse the copyright material in this book please see our website at
www.wiley.com/wiley-blackwell
The right of the author to be identified as the author of this work has been asserted in accordance
with the Copyright, Designs and Patents Act 1988.
All rights reserved. No part of this publication may be reproduced, stored in a retrieval system, or
transmitted, in any form or by any means, electronic, mechanical, photocopying, recording or
otherwise, except as permitted by the UK Copyright, Designs and Patents Act 1988, without the prior
permission of the publisher.
Wiley also publishes its books in a variety of electronic formats. Some content that appears in print
may not be available in electronic books.
Designations used by companies to distinguish their products are often claimed as trademarks. All
brand names and product names used in this book are trade names, service marks, trademarks or
registered trademarks of their respective owners. The publisher is not associated with any product or
vendor mentioned in this book. This publication is designed to provide accurate and authoritative
information in regard to the subject matter covered. It is sold on the understanding that the
publisher is not engaged in rendering professional services. If professional advice or other expert
assistance is required, the services of a competent professional should be sought.
Library of Congress Cataloging-in-Publication Data
Ajjan, Ramzi.
Endocrinology and diabetes : clinical cases uncovered / Ramzi Ajjan.
p. ; cm.
Includes index.
ISBN 978-1-4051-5726-1
1. Endocrinology - Case studies. 2. Diabetes - Case studies. I. Title.
[DNLM:
1. Endocrine System Diseases - diagnosis - Case Reports.
2. Diabetes Mellitus -
diagnosis - Case Reports.
3. Diabetes Mellitus - therapy - Case Reports.
4. Endocrine System
Diseases - therapy - Case Reports. WK 140 A312e 2009]
RC649.5.A35 2009
616.4 - dc22
2008033368
ISBN: 978-1-4051-5726-1
A catalogue record for this book is available from the British Library.
Set in 9/12pt Minion by SNP Best-set Typesetter Ltd., Hong Kong
Printed and bound in Singapore by Ho Printing Singapore Pte Ltd
1
2009
Contents
Preface, vii
Acknowledgements, viii
How to use this book, ix
List of abbreviations, x
Part 1 Basics, 1
The pituitary gland, 1
The thyroid, 13
Bone and calcium metabolism, 23
The adrenal glands, 30
The reproductive system, 36
The pancreas, 46
Lipid abnormalities and obesity, 60
The neuroendocrine system, 63
Part 2 Cases, 66
Case 1 A 19-year-old with abdominal pain and vomiting, 66
Case 2 A 35-year-old woman with palpitation and irritability, 73
Case 3 A 61-year-old man with polyuria, polydipsia, cough and weight loss, 79
Case 4 A 44-year-old woman with visual problems, 82
Case 5 A 20-year-old man with recent diagnosis of diabetes, 86
Case 6 Tiredness and weight gain in a 30-year-old woman with diabetes, 89
Case 7 Acute confusion in an 82-year-old with known type 2 diabetes, 92
Case 8 A 42-year-old man with headaches, increased sweating and sexual dysfunction, 98
Case 9 Amenorrhoea in an 18-year-old, 102
Case 10 A 28-year-old with tiredness and abnormal thyroid function postpartum, 106
Case 11 A 33-year-old man with polyuria and polydipsia, 109
v
vi
Contents
Case 12 A 62-year-old man with tiredness and hyponatraemia, 113
Case 13 Excess hair in a 29-year-old woman, 117
Case 14 A 52-year-old woman with paroxysmal atrial fibrillation and abnormal thyroid
function, 120
Case 15 A 22-year-old man with hypertension, 123
Case 16 A 20-year-old woman with polyuria and polydipsia, 126
Case 17 A 78-year-old man with pain in the leg and knee, 132
Case 18 A 32-year-old woman with a lump in the neck, 135
Case 19 A 26-year-old with headaches and hypertension, 139
Case 20 Sweating, nausea and hand tremor in a 24-year-old woman, 142
Case 21 A 19-year-old man with sexual dysfunction, 146
Case 22 A 38-year-old woman with muscular aches and weakness, 151
Case 23 A wrist fracture in a 56-year-old woman, 154
Case 24 A 37-year-old woman with recurrent flushing, 158
Case 25 A 46-year-old man with an abnormal lipid profile, 161
Part 3 Self-assessment, 164
MCQs, 164
EMQs, 169
SAQs, 174
Answers, 176
Index of cases by diagnosis, 187
Index, 189
Colour plate section can be found facing p. 84.
Preface
Almost two decades have passed since my medical student
diabetes and endocrine conditions is provided, including
days and I still remember how difficult, and often tedious,
basic science, symptoms and signs, investigations and
it was to read and understand some of the clinical topics
treatment.
presented in textbooks.
In Part 2, diabetes and endocrinology are covered
Having been fortunate enough for my career to develop
using ‘real life’ cases, which I encountered during my
in academic medicine, part of my work involves regular
clinical practice. Each case is divided into a number of
teaching and lecturing at different levels, ranging from
sections/questions, which you should read carefully and
medical students to experienced physicians and health
make an attempt to give a differential diagnosis or for-
care professionals.
mulate a management plan. You will notice I have varied
Despite a variety of audience, there has always been a
the amount of background information, depending on
general enthusiasm for further learning when clinical
the importance and the prevalence of the medical condi-
tutorials/lectures were not only presented as ‘facts’ but
tion under discussion. In common clinical scenarios,
also as case-based studies. Moreover, I realised during my
comprehensive management plans are given, whereas in
clinical practice that various medical conditions are best
less common and more specialised cases, diagnostic and
remembered by discussing and fully evaluating real life
treatment strategies are only briefly touched upon. Take
cases. Putting things together, I felt a case-based book
your time with each case and remember that these are
would offer a unique opportunity to facilitate under-
real life cases, which you may be attending to as a junior
standing of clinical diabetes and endocrinology, and
medical doctor.
make the learning process an enjoyable experience.
In Part 1 of the book, a simple reminder of clinical
Ramzi Ajjan
vii
Acknowledgements
My thanks and appreciation extend to a large number of
up with my repeated requests. I acknowledge the help of
individuals who contributed to this book by providing
my Registrar, Dr Thet Koko, for sourcing appropriate
appropriate cases and different illustrations, including Dr
illustrations. Special thanks go to my Secretary, Krystyna
Steve Orme, Dr Paul Belchetz, Dr Carol Amery, Dr
Pierzchalski for her patience and invaluable support.
Michael Waller, Dr Robert Bury, Mr Bernard Chang, Pro-
Finally, I would like to thank Professor Anthony
fessor David Gawkrodger and Professor Steve Atkin. I am
Weetman and Professor Peter Grant for their guidance
indebted to the Radiology and Radionuclide Depart-
over the years, which has been vital for my academic
ments at Leeds General Infirmary and I also wish to
progress, and Dr Steve Orme for his unwavering support
thank the Medical Photography Department for putting
through my clinical career.
viii
How to use this book
Clinical Cases Uncovered (CCU) books are carefully
test your learning with several question styles (MCQs,
designed to help supplement your clinical experience and
EMQs and SAQs), each with a strong clinical focus.
assist with refreshing your memory when revising. Each
Whether reading individually or working as part of a
book is divided into three sections: Part 1, Basics; Part 2,
group, we hope you will enjoy using your CCU book.
Cases; and Part 3, Self-Assessment.
If you have any recommendations on how we could
Part 1 gives a quick reminder of the basic science,
improve the series, please do let us know by contacting
history and examination, and key diagnoses in the area.
us at: medstudentuk@oxon.blackwellpublishing.com.
Part 2 contains many of the clinical presentations you
would expect to see on the wards or crop up in exams,
Disclaimer
with questions and answers leading you through each
CCU patients are designed to reflect real life, with their
case. New information, such as test results, is revealed as
own reports of symptoms and concerns. Please note that
events unfold and each case concludes with a handy case
all names used are entirely fictitious and any similarity to
summary explaining the key points. Part 3 allows you to
patients, alive or dead, is coincidental.
ix
List of abbreviations
ABG
arterial blood gas (analysis)
5HIAA
5-hydroxyindolacetic acid
ACEI
angiotensin converting enzyme inhibitors
HNF
hepatic nuclear factor
ACR
albumin/creatinine ratio
HMG
CoA 3-hydroxy, 3-methylglutaryl coenzyme A
ACTH
adrenocorticotrophic hormone
HONK
hyperosmolar non-ketotic hyperglycaemia
AD
autosomal dominant
HRT
hormone replacement therapy
ADH
antidiuretic hormone
IHD
ischaemic heart disease
AH
autoimmune hypothyroidism
IHH
idiopathic hypogonadotrophic hypogonadism
AP
alkaline phosphatase
IST
insulin stress test
AR
autosomal recessive
IUI
intrauterine insemination
ARB
angiotensin receptor blocker
i.v.
intravenous
BMD
bone mineral density
IVF
in vitro fertilization
BMI
body mass index
LADA
latent autoimmune diabetes of adults
CAH
congenital adrenal hyperplasia
LDLc
low-density lipoprotein cholesterol
CCF
congestive cardiac failure
LDST
low dose synacthen test
CRH
corticotrophin releasing hormone
LFT
liver function test
CRP
C-reactive protein
LH
luteinizing hormone
CT
computed tomography
MEN
multiple endocrine neoplasia
CVA
cerebrovascular accident
MIBG
meta-iodobenzylguanidine
DEXA
dual energy X-ray absorptiometry
MODY
maturity onset diabetes of the young
DHEA
dehydroepiandrosterone
MRA
magnetic resonance angiography
DI
diabetes insipidus
MRI
magnetic resonance imaging
DKA
diabetic ketoacidosis
MTC
medullary thyroid cancer
DOC
deoxycorticosterone
NF
neurofibromatosis
DPP
dipeptidyl peptidase
OCP
oral contraceptive pill
ECG
electrocardiogram
OGT
oral glucose tolerance (test)
ESR
erythrocyte sedimentation rate
PCOS
polycystic ovary syndrome
FBC
full blood count
PE
pulmonary embolus
FHH
familial hypocalciuric hypercalcaemia
PRA
plasma renin activity
FNA
fine needle aspiration
PRL
prolactin
FSH
follicle stimulating hormone
PSA
prostate specific antigen
GAD
glutamic acid decarboxylase
PTH
parathyroid hormone
GGT
gamma glutamyl transpeptidase
RAI
radioactive iodine
GH
growth hormone
SHBG
sex hormone binding globulin
GHRH
growth hormone releasing hormone
SIADH
syndrome of inappropriate ADH secretion
GLP
glucagon-like peptide
TC
total cholesterol
GnRH
gonadotrophin releasing hormone
T1DM
type 1 diabetes mellitus
GO
Graves’ ophthalmopathy
T2DM
type 2 diabetes mellitus
GST
glucagon stimulation test
TFT
thyroid function test
hCG
human chorionic gonadotrophin
TG
thyroglobulin
x
List of abbreviations xi
TIA
transient ischaemic attack
TSH thyroid stimulating hormone (thyrotropin)
TMNG toxic multinodular goitre
U&Es urea and electrolytes
TN
toxic solitary nodule
UTI
urinary tract infection
TPO
thyroid peroxidase
VIP
vasoactive intestinal peptide
TRH
thyrotropin releasing hormone
This page intentionally left blank
The pituitary gland
Understanding the pituitary gland is probably the hardest
Antidiuretic hormone
(ADH): stimulates water
part of endocrinology as it controls most of the endocrine
reabsorption by the kidneys
glands in the body and disease may arise due to both over-
Oxytocin: helps uterine contractions during labour
secretion and undersecretion of a particular hormone. A
The anterior pituitary gland is under the control of the
full understanding of the hormonal tests in this section
hypothalamus as shown in Fig. 3.
will make interpretation of the endocrine tests in the rest
Corticotrophin releasing hormone (CRH): stimulates
of the book an easy and pleasant experience.
ACTH secretion
Growth hormone releasing hormone (GHRH): stimu-
Anatomy
lates GH secretion
The pituitary gland is situated in the pituitary fossa and
Thyrotrophin releasing hormone (TRH): stimulates
is surrounded by (see Fig. 1):
TSH secretion
Below: sphenoid air sinus
Gonadotrophin releasing hormone (GnRH): stimu-
Either side: cavernous sinus and carotid artery
lates FSH and LH secretion
Above: the pituitary stalk extending into the
Prolactin releasing hormone does not exist and prolac-
hypothalamus
tin is under the inhibitory effect of the hypothalamus
Cortisol, GH, thyroid hormones and sex hormones all
Physiology
have a negative feedback effect on corresponding
The pituitary gland can be functionally divided into two
pituitary (ACTH, GH, TSH and FSH/LH respectively)
lobes (Fig. 2)
and hypothalamic
(CRH, GHRH, TRH and GnRH
The anterior pituitary, which produces the following
respectively) hormone release.
hormones
Growth hormone
(GH): resulting in skeletal
Clinical disease
growth
Clinical disease results from oversecretion or undersecre-
Adrenocorticotrophic hormone (ACTH): stimulates
tion of pituitary hormones, in addition to the local com-
the adrenals to produce steroids
pressive effects of a pituitary tumour. A pituitary tumour
Gonadotrophins (FSH and LH): stimulate the testi-
may secrete excessive hormones but it may also be non-
cles or ovaries to produce sex hormones
functional, in which case the clinical presentation
Thyroid stimulating hormone or thyrotrophin
consists of pituitary failure associated with compressive
(TSH): stimulates the thyroid to produce thyroid
effects.
hormones
Prolactin (PRL): stimulates breast milk production
Pituitary oversecretion
The posterior pituitary, which stores the hormones
Usually due to pituitary tumours overproducing one
produced in the hypothalamus
(does not produce
hormone (sometimes more than one) resulting in typical
them)
clinical entities, which are described below
Very rarely, overproduction of pituitary hormones
Endocrinology and Diabetes: Clinical Cases Uncovered. By R. Ajjan.
may be due to increased production of pituitary hormone
Published 2009 by Blackwell Publishing, ISBN: 978-1-4051-5726-1
releasing hormones (CRH, GHRH)
1
2
Part 1: Basics
Pituitary
Optic nerve
Carotid artery
Sphenoid crest
Sphenoid sinus
Sphenoid sinus ostium
Maxillary sinus
Vomer
Figure 1 Position of the pituitary gland.
Paraventricular nuclei
Supraoptic nuclei
Hypothalamic /
Optic chiasm
hypophyseotropic area
Superior hypophyseal
Primary capillary plexus
artery
Median eminence
Pituitary stalk
Hypophyseal portal
Supraoptic -
vessels
hypothalamic tract
Anterior pituitary
Posterior pituitary
Secretory cells
Capillaries
Efferent veins
Figure 2 Two lobes of the pituitary
gland. From Holt, RIG & Hanley, NA
Efferent veins
(2007) Essential Endocrinology and
Inferior hypophyseal
Diabetes, 5th edition. Blackwell
artery
Publishing, Oxford.
Hypothalamus
GHRH
CRH
TRH
GnRH
Pituitary failure
Acquired pituitary hormonal deficiency is commonly
Pituitary
GH
ACTH
TSH
FSH/LH
due to a pituitary tumour compressing and compromis-
ing the activity of normal cells
It may also be secondary to:
Developmental abnormalities
Liver
Adrenals
Thyroid
Ovary/testicle
(IGF-1)
(steroids)
(T3 and T4)
(sex hormones)
Autoimmune conditions
Head injury
Figure 3 Control of hormone secretion by the hypothalamus
Vascular disorders and severe blood loss (resulting
and pituitary (see text). GHRH, CRH, TRH and GnRH, secreted
in infarction of the pituitary)
by the hypothalamus, stimulate GH, ACTH, TSH and FSH/LH
Infiltrative disease and infection
(sarcoidosis,
production by the pituitary respectively, which in turn
tuberculosis)
stimulate the liver, adrenal glands, thyroid and ovaries/testicles
Radiotherapy
to produce their hormones. GH, adrenal steroids, thyroid
It should be noted that pituitary hormonal deficiency
hormones and sex steroids in turn have a negative feedback
commonly involves multiple hormones and, therefore,
effect (reduce hormone production) on the corresponding
deficiency of one hormone warrants full pituitary
hypothalamic/pituitary hormone release. The pituitary hormone
prolactin (which is not shown here) is unique as there is no
investigations.
hypothalamic hormone to stimulate its release but it is rather
Local effects of all pituitary tumours include:
under inhibitory control.
Headaches
The pituitary gland
3
Visual field defects (usually bitemporal hemianopia)
High sex steroids with elevated gonadotrophin suggest
Deficiency of other hormones (due to pressure effect
gonadotrophin-secreting pituitary tumour
(these are
on normal pituitary tissue)
rare and often clinically silent)
Cranial nerve palsies:
3rd,
4th and 6th in large
Low sex hormones with raised gonadotrophins, indi-
pituitary tumours
cate primary gonadal failure and this is seen in physio-
logical menopause (women above the age of 50 usually
Investigations of the pituitary gland
have raised gonadotrophin levels with low oestradiol)
This involves investigations of hormonal abnormalities
Prolactin
and imaging of the pituitary gland.
Raised serum prolactin may be due to a pituitary pro-
lactinoma (this is fully discussed later in this chapter)
Hormonal investigation of suspected pituitary
hormone abnormality
Stimulation tests in suspected hypopituitarism
In general, there are three ways to investigate hormonal
The two main stimulation tests used are:
abnormalities in endocrine disease:
Static hormone measurements: this is a “one off ” mea-
Insulin stress test
surement of a particular hormone. Examples include
This is the gold standard test to assess pituitary func-
measurement of thyroid function (TSH and T4), gonadal
tion but it has a number of contraindications (see below)
function (sex steroids and gonadotrophins) and mea-
and therefore it is not always used first line
surement of prolactin
Insulin injection results in hypoglycaemia creating a
Stimulation tests: if deficiency of a particular hormone
stressful environment with consequent release of ACTH
is suspected, stimulation tests are carried out. Failure of
and GH
a particular hormone level to rise after stimulation tests
0.1-0.3 U/kg of insulin is injected
(high doses are
confirms hormonal deficiency. Examples include growth
required in those with insulin resistance) to render the
hormone and cortisol deficiency
patient hypoglycaemic and GH/cortisol are measured
Suppression test: if oversecretion of a hormone is sus-
GH >20 mIU/L and cortisol >580 nmol/L indicate
pected, suppression tests can be carried out. Failure of
adequate hormonal reserve
suppression of a particular hormone indicates overpro-
Contraindications
duction. Examples include growth hormone oversecre-
History of epilepsy
tion (acromegaly) and ACTH oversecretion (Cushing’s
Abnormal ECG or ischemic heart disease
disease)
Untreated hypothyroidism
Basal cortisol < 100 nmol/L
Static pituitary function tests
Thyroid function tests (TFTs)
Glucagon stimulation test
Low free T4 (FT4) with low or low normal TSH:
Injection of glucagon results in:
This should alert to the possibility of pituitary
Release of growth hormone and ACTH
(GH
failure
>20 mIU/L or cortisol >580 nmol/L indicate normal
Differential diagnosis includes abnormal TFTs due
GH and ACTH reserve)
to non-thyroidal illness
(described in the thyroid
The test is not always reliable (up to 20% of normal
section)
individuals fail to fully respond) and in case of any doubts
Raised TSH and raised FT4: possible TSH-secreting
insulin stress test should be performed
pituitary tumour
Contraindications
Raised TSH with low or normal FT4: primary
The test is less reliable in subjects with diabetes
hypothyroidism
Suppressed TSH with high or normal FT4: primary
Other stimulation tests
hyperthyroidism
These are quite specialized and beyond the scope of
Sex hormones (testosterone or oestradiol)
this book and include:
Low sex hormones with low or low normal gonadotro-
TRH stimulation test
phins (FSH and LH) should raise the possibility of pitu-
GnRH stimulation test
itary failure
Arginine stimulation test
4
Part 1: Basics
Table 1 Main tests for pituitary functions.
Static tests
Stimulation tests
Suppression tests
Thyroid function tests
Insulin stress test
Glucose tolerance test
Low FT4 and low or low-normal TSH
Failure of GH and cortisol to rise after
Failure of GH suppression after oral GTT
suggests hypopituitarism
insulin injection suggests
suggests GH oversecretion (acromegaly)
hypopituitarism
Sex hormones
Glucagon stimulation tests
Low- and high-dose dexamethasone
Low sex hormones with low or low-normal
Failure of GH and cortisol to rise after
suppression test
gonadotrophins suggests hypopituitarism
glucagon injection suggests
(see text)
hypopituitarism
Prolactin
Raised prolactin suggests pituitary
prolactinoma
Suppression tests in suspected hormonal
High levels of pituitary hormones in the petrosal sinus
overproduction
compared with a peripheral vein, confirm the diagnosis
Oral glucose tolerance test
of pituitary secreting hormones
This is used in suspected GH oversecretion
The test is often used to differentiate pituitary-
Failure to suppress GH to <2 mIU/L after 75 g oral
dependent Cushing’s disease from ectopic ACTH
glucose tolerance test strongly suggests the diagnosis of
secretion. Higher ACTH levels in the petrosal sinus
acromegaly
compared with venous ACTH, after CRH stimulation
confirms pituitary-dependent Cushing’s disease
Dexamethasone suppression test
This is used to diagnose Cushing’s syndrome but may
Treatment
also be able to differentiate between pituitary and non-
Non-functioning pituitary tumours or those associ-
pituitary causes of Cushing’s syndrome
ated with increased hormone production (except for
Low dose dexamethasone suppression test: failure to
prolactinomas, see below) are usually treated surgically:
suppress cortisol to <50 nmol/L after giving 0.5 mg of
Transphenoidal surgery (in most cases)
dexamethasone 6 hourly for 2 days, suggests the diag-
Transcranial surgery (rarely, in very large tumours)
nosis of Cushing’s syndrome
Pituitary hormone deficiency should be treated by
Suppression of cortisol to >50% of basal levels after
hormone replacement (pituitary failure is usually associ-
giving 2 mg of dexamethasone 6 hourly for 2 days
ated with multiple hormonal deficiencies)
suggest pituitary cause (i.e. Cushing’s disease)
The main tests for pituitary function are summarized
Clinical disease of the anterior
in Table 1.
pituitary gland
This section discusses the effects of over- and underpro-
Imaging of the pituitary gland
duction of a particular hormone.
Magnetic resonance imaging (MRI)
This is the gold standard for imaging of the pituitary
Abnormalities of growth hormone secretion
gland (Fig. 4 shows a pituitary adenoma that enhances
Growth hormone excess
after godalinium injection)
In childhood or adolescence growth hormone excess
results in:
Combination of imaging with stimulation tests
Excessive growth spurt
In some complicated cases it may be necessary to
Increased size of feet and hands
perform inferior petrosal sinus sampling under radio-
General skeletal enlargement
logical guidance followed by stimulation tests
Increased skin thickness
The pituitary gland
5
Thickening of lips and tongue
Dental malocclusion and widely spaced teeth
Wide and large hands/feet (enlargement of soft tissue,
skin and cartilage), typically presenting with
Increasing glove size
Tight-fitting rings
Increasing shoe size
Deep voice
Nerve entrapment: carpal tunnel syndrome is not
uncommon (soft tissue enlargement)
Increased sweating (common complaint)
Organomegaly
Goitre
Cardiomegaly
Hepatomegaly
Splenomegaly
(a)
Complications of acromegaly include (may be the
presenting feature of the disease):
Hypertension
Diabetes
Obstructive sleep apnoea
Increased risk of heart disease
Increased risk of colonic polyps and colonic
carcinoma
Investigations
Random GH
A random GH of <1 mIU/L makes the diagnosis of
acromegaly unlikely
A random GH >1 mIU/L does not help in making a
diagnosis
Glucose tolerance test
Failure of GH suppression after GTT suggests the
diagnosis of acromegaly
Insulin-like growth factor-1 (IGF-1) levels
(b)
These are elevated in acromegaly but this is mainly
Figure 4 MRI of the pituitary showing a pituitary adenoma,
used to monitor response to therapy
before (a) and after (b) gadolinium injection.
Imaging
If left untreated, growth hormone excess in this period
Pituitary MRI: this usually shows a pituitary tumour
of life leads to gigantism, the most serious consequence
of the disease
Treatment
In adults, growth hormone excess affects the skin, soft
Transphenoidal surgery: the treatment of choice
tissue and skeleton resulting in acromegaly, which has the
Radiotherapy: in patients with failed surgery or if
following features:
surgery is contraindicated
Acromegalic face (coarse facial features, see Fig. 5,
Medical treatment
colour plate section)
Somatostatin analogues: used in patients with
Prominent supraorbital ridges
residual tumour post surgery or in whom surgery is
Large nose
contraindicated. It is effective at reducing GH levels in
Lower jaw pushed forward (prognathism)
around 60% of patients
6
Part 1: Basics
Dopamine agonists (cabergoline, bromocriptine):
Table 2 Main symptoms, signs and complications of growth
effective in a minority of patients
hormone excess and deficiency.
Pegvisomant: relatively new and effective treatment
Growth hormone excess
Growth hormone deficiency
that blocks the growth hormone receptor but has no
effect on growth hormone levels. The effect of this
Symptoms
Symptoms
treatment on tumour size remains controversial
Fast growth (in children)
Failure of growth (in children)
Monitoring response to treatment
GH day curve: mean GH <5 mIU/L defines cure
Headaches (independent of
Tiredness
local tumour effect)
from the disease
Depression
IGF-1 levels: the aim is to normalize IGF-1 levels
Increased sweating
Decreased body mass
Due to increased risk of colonic cancer, acromegaly
Musculoskeletal pains
patients should undergo regular colonoscopy for early
detection of the disease
Change in glove/ring and
shoe size
Growth hormone deficiency
Signs
Signs
In childhood, growth hormone deficiency (GHD) results
Facial appearance (see text)
Failure of growth and thin
in:
skin in children
Soft tissue and skeletal
Failure of growth
changes
No specific signs in adults
Thin skin
Organomegaly
Hypoglycaemia (particularly in the presence of ACTH
deficiency)
Visual field defect
Delayed puberty (particularly in the presence of sex
Deficiency of other pituitary
hormone deficiency)
hormones
In adults, GHD results in non-specific symptoms:
Complications
Complications
Tiredness
Hypertension
Short stature in untreated
Depression
children
Reduction in muscle and increase in fat mass
Diabetes
The main clinical features of growth hormone excess/
Hypoglycaemia (mainly in
Colonic polyps and colonic
deficiency are summarized in Table 2.
children)
carcinoma
Osteoporosis in adults
Obstructive sleep apnoea
Investigations
Glucagon stimulation test or insulin stress tests
Failure of GH to rise after these stimulation tests
suggests GHD
Abnormalities of ACTH secretion
IGF-1 levels
ACTH excess
Low IGF-1 aids in the diagnosis. However, normal
Pituitary tumours producing ACTH result in excessive
IGF-1 levels do not rule out the possibility of GHD
cortisol production by the adrenals, consequently leading
Imaging
to pituitary-dependent Cushing’s syndrome (or Cush-
Pituitary MRI should be performed in subjects
ing’s disease), which must be differentiated from other
with GHD to rule out the possibility of pituitary
causes of Cushing’s syndrome, including:
tumour causing GHD by compressing GH-producing
Ectopic ACTH syndrome: due to the presence of
cells
malignant cells producing ACTH
(lung cancer for
example)
Treatment
Adrenal tumours: excess cortisol production is associ-
Childhood GHD
ated with suppression of ACTH production and, there-
GH replacement is necessary
fore, these tumours are usually referred to as non-ACTH
Adult GHD
dependent Cushing’s syndrome
Only symptomatic patients are usually offered GH
Pseudo-Cushing’s: excessive alcohol consumption or
replacement therapy
severe depression can result in symptoms and signs
The pituitary gland
7
similar to Cushing’s syndrome, and differentiating this
from “real” Cushing’s can sometimes be difficult even for
Red cheeks
an experienced endocrinologist
Moon face
Fat pads
Box 1 Clinical features of Cushing’s syndrome
Bruisabillity
with ecchymoses
Growth arrest in children
Typical facial appearance (see Fig. 6)
Striae
Round (moon-like) face
Acne
Pendulous
Thin skin
Hirsutism
abdomen
Thinning of scalp hair
Fat redistribution
Truncal obesity
Proximal
Thin extremities
myopathy
Supraclavicular fat pads
Skin abnormalities
Thin skin
Poor wound
Striae on abdomen, breast and axillae (due to central
healing
obesity and thinning of the skin)
Mood problems
Depression
Psychosis
Figure 6 Typical facial appearance of Cushing’s disease.
Insomnia
Sexual dysfunction
Low libido and impotence
Menstrual problems
Complications
doses) and check cortisol levels thereafter, which
Hypertension
should be undetectable in the absence of Cushing’s
Diabetes mellitus
syndrome
Osteoporosis
Differentiate between different causes of Cushing’s
High risk of infections
syndrome
Poor wound healing
ACTH levels: these are suppressed in adrenal Cush-
ing’s but detectable in pituitary Cushing’s disease or
Investigations
cases due to ectopic ACTH production
Confirm the presence of excess cortisol
High dose dexamethasone suppression test: give
24-hour urinary cortisol: high levels are suggestive
2 mg dexamethasone every 6 h for 2 days. If cortisol is
of Cushing’s syndrome
suppressed to more than 50% of basal value, it suggests
Midnight cortisol: in normal individuals, cortisol
a diagnosis of pituitary Cushing’s disease
levels at midnight during sleep are undetectable. This
Imaging
test may be difficult to arrange as the patient needs to
MRI of the pituitary: may show a pituitary tumour
be admitted and a blood sample should be taken
but it can sometimes be normal (tumour too small to
immediately after the patient is woken up
visualize)
Overnight dexamethasone suppression test: give
Petrosal sinus sampling: this may need to be under-
0.5-1.0 mg of dexamethasone at 23:00 and measure
taken in difficult cases to differentiate ectopic ACTH
cortisol at 09:00. Cortisol levels less than 50 nmol/L
secretion from pituitary-dependent Cushing’s disease
effectively rule out the diagnosis of Cushing’s
syndrome
Treatment of Cushing’s disease
Low dose dexamethasone suppression test: give
Transphenoidal surgery to remove the pituitary
0.5 mg dexamethasone ever 6 hours for 2 days (eight
tumour
8
Part 1: Basics
Radiotherapy: in relapsed disease or in those whom
Table 3 Main symptoms, signs and complications of ACTH
surgery is contraindicated
excess and deficiency.
Adrenalectomy: in difficult cases
(to stop cortisol
ACTH excess
ACTH deficiency
secretion), but this is rarely performed
Symptoms
Symptoms
ACTH deficiency
Failure of growth (in children)
Failure of growth (in children)
This results in the failure of cortisol production by the
Weight gain
General malaise and weakness
adrenal glands. This results in:
Failure of growth in children
Thin skin and easy bruising
Dizziness
Malaise and tiredness
Muscle weakness
Generalized aches and pains
Weight loss
Mood disturbances
Abdominal pain, diarrhoea
Hypoglycaemia
and vomiting
Hypotension
Reduced libido and menstrual
Confusion
irregularities
Reduced libido and menstrual
The main clinical features of ACTH excess/deficiency are
irregularities
summarized in Table 3.
Signs
Signs
Facial appearance (see text)
Postural hypotension
Investigations
Truncal obesity, buffalo hump
Decreased axillary and pubic
Pituitary stimulation tests (insulin stress test or gluca-
hair
Thin and fragile skin
gons stimulation test) fail to show adequate rise in serum
cortisol levels
Abdominal and axillary striae
The possibility of primary hypoadrenalism should be
Increased pigmentation due
ruled out, in which case there is:
to high ACTH (skin and
Low cortisol
mucous membranes)
High ACTH
Proximal muscle weakness
ACTH deficiency is usually part of panhypopituitarism
and, therefore, deficiency of other hormones should be
Visual field defect
investigated
Deficiency of other pituitary
In subjects with pure ACTH deficiency a CRH test may
hormones
be necessary to confirm the diagnosis (failure of ACTH
Complications
Complications
and cortisol to rise confirm ACTH deficiency)
Hypertension
Hypoglycaemia
Imaging
Pituitary MRI to investigate the possibility of
Diabetes
Death
pituitary tumour
Osteoporosis
Infections
Treatment
Cortisol replacement is necessary and usually oral
hydrocortisone is used in two to three divided doses
Causes of raised plasma prolactin concentration
seem to be a popular question in postgraduate medical
Abnormalities of prolactin secretion
examinations
Prolactin excess
Prolactinomas are the commonest functioning
Clinical presentation
pituitary tumours
Prolactinomas result in:
Microprolactinomas are detected in up to 10% of the
Galactorrhoea (90% of women and 15% of men)
population in post-mortem studies
Sexual dysfunction
Serum prolactin concentration may be elevated due to
Decreased libido
a large number of factors (summarized in Table 4), which
Menstrual irregularities
should be differentiated from a prolactinoma.
Local tumour effects
The pituitary gland
9
Table 4 Causes of high plasma prolactin levels.
Treatment
Pituitary prolactinomas are usually treated medically
Physiological
Pregnancy
with dopamine agonists (cabergoline or bromocriptine),
Nipple stimulation
which result in both reduced hormone secretion and
shrinkage of the tumour
Sexual intercourse
Surgery is reserved for severe cases that are not
Stress (simple venepuncture may cause PRL
responding to medical treatment (and these are fortu-
elevation)
nately rare)
Pituitary
Prolactinoma
It should be noted that prolactinomas are the only
tumours
pituitary tumours where medical therapy, rather than
Non-functioning tumours (elevation of
surgery, is first-line treatment and, therefore, it is impor-
prolactin is usually modest due to stalk
tant to make the correct diagnosis in these cases
compression and lack of inhibition of
prolactin secretion)
Drugs
Large list including:
Prolactin deficiency
Deficiency of prolactin results in failure of lactation in
Anti-emetics
women with no other systemic effects
Antidepressants and antipsychotics
This is usually part of other pituitary hormonal
deficiency
Opiates
Can result from severe blood loss during childbirth,
Anti-HIV treatment
resulting in pituitary infarction, which is called Sheehan’s
Hypothalamic
Tumours compressing the hypothalamus
syndrome
disease
There is no prolactin replacement therapy and defi-
Infiltrative disease (sarcoidosis)
ciency of this hormone is not treated
Large pituitary tumours causing stalk
compression
Abnormalities in TSH secretion
Metabolic
Hypothyroidism
TSH excess
Chronic renal disease
TSH excess is rare and is usually due to a pituitary tumour
(TSH-oma). It results in:
Features of hyperthyroidism (clinical presentation is
discussed in the chapter on the thyroid)
Mass effect of the pituitary tumour (particularly as
Investigations
these tumours tend to be large)
Raised serum prolactin is suggestive of the diagnosis,
provided other causes for raised prolactin are ruled out
Investigations
(see Table 4)
High FT4 with high or normal TSH in subjects not on
Imaging
thyroxine replacement is suggestive of TSH-producing
MRI of the pituitary usually shows a pituitary
pituitary tumour
tumour, particularly in those with very high prolactin
Imaging
levels
MRI of the pituitary: this usually shows a large
In some patients no tumour can be identified but
tumour
this does not rule out the diagnosis of prolactinoma
(tumour can be too small)
In patients with a large pituitary tumour and only
Treatment
mild elevation of prolactin, a non-functioning
Transphenoidal surgery
pituitary adenoma rather than a prolactinoma should
Somatostatin analogues for recurrent or incompletely
be suspected (raised prolactin in this case is due to stalk
removed tumours
compression and ‘escape’ from the inhibitory effects of
Radiotherapy in case of recurrent tumour or unsuc-
hypothalamus)
cessful surgery
10
Part 1: Basics
TSH deficiency
Treatment
TSH deficiency causes hypothyroidism (usually associ-
Treat the underlying cause
ated with other pituitary hormone deficiency).
Sex hormone replacement
The clinical features of hypothyroidism are discussed
Testosterone
in the chapter on the thyroid.
Oestrogen and progesterone
Investigations
Non-functioning pituitary adenoma
Low FT4 with low or normal TSH is suggestive of TSH
These are the commonest of pituitary macroadenomas.
deficiency and the pituitary gland should be fully evalu-
They present clinically with:
ated for deficiencies of other pituitary hormones.
Mass effect
Visual field defect
Treatment
Headaches
Thyroid hormone replacement in the form of synthetic
Cranial nerve palsies
T4 (levothyroxine)
Hypopituitarism: resulting in GH, ACTH, TSH and
It should be noted that TSH measurements cannot be
gonadotrophin deficiencies (variable degrees), with the
relied upon for monitoring the thyroxine dose, which is
clinical manifestations described above
simply done by measuring FT4 levels and assessing the
patient clinically
Investigations
In patients with combined ACTH and TSH deficiency,
Static pituitary function tests
cortisol therapy should be started first and thryoxine
TFTs
replacement introduced a few days later to avoid precipi-
Sex hormones and gonadotrophin levels
tating an adrenal crisis
Prolactin (may be mildly elevated in non-functioning
tumours; see section on prolactinoma)
Abnormalities of gonadotrophin secretion
Gonadotrophin excess
Stimulation tests
Tumours producing FSH or LH are extremely rare and
Insulin stress test or glucagon stimulation test to
usually behave similarly to a non-functioning pituitary
assess:
tumour. In men, FSH-secreting tumours may result in
Cortisol (ACTH) reserve
testicular enlargement.
GH reserve
Suppression tests: only if there is a clinical suspicion
Gonadotrophin deficiency
of:
This results in sex hormone deficiency.
Clinical presentation
Box 2 Pituitary tumours
Decreased libido, impotence and menstrual
irregularities
Pituitary tumours may be:
Loss of secondary sexual hair
Functional, secreting one or more hormones resulting in:
Loss of muscle mass in men
Galactorrhoea (prolactin)
In children
Acromegaly (GH)
Delayed puberty and sexual infantilism
Cushing’s syndrome (ACTH)
Hyperthyroidism (TSH)
Primary amenorrhoea
Non-functional, causing:
Mass effect
Investigations
Anterior pituitary failure: this can be partial (one or
Low testosterone in men and oestradiol in women with
two hormones) or total (involving all pituitary
low or normal gonadotrophin levels, suggest secondary
hormones)
gonadal failure
Suspected pituitary tumours should be investigated with
Imaging
hormonal tests (rule out hyper- and hyposecretion of
Pituitary MRI should be performed in subjects with
hormones) as well as imaging tests
secondary gonadal failure
The pituitary gland
11
Cushing’s syndrome
Table 5 Causes of syndrome of inappropriate ADH (SIADH)
Acromegaly
secretion.
Imaging
Tumours
Cancers: Lung malignancy, haematological
Pituitary MRI: shows a pituitary macroadenoma
malignancies, etc.
Treatment
Central nervous
Infection (meningitis, encephalitis)
Surgery: usually transphenoidal but transcranial
system
Head injury
surgery may be needed for larger tumours
abnormalities
Vascular disorders
Radiotherapy: for recurrence
Hormone replacement therapy: these patients usually
Respiratory
Infections
end up with a mixture of pituitary hormonal deficiencies,
abnormalities
Positive pressure ventilation
which should be replaced
Drugs
Chemotherapy
Anti-epileptic (carbamazepine)
Box 3 Other causes of pituitary failure
Oral hypoglycaemic (chlorpropamide)
Pituitary infarction, characterized by:
Antipsychotics
Sudden onset headache
Endocrine
Hypothyroidism
Cranial nerve palsies
Symptoms and signs of cortisol deficiency
Metabolic
Acute intermittent porphyria
Pituitary infiltration
Idiopathic
All above causes need ruling out before
Sarcoidosis
making this diagnosis
Haemochromatosis
Trauma
Pituitary infection
Abscess
Tuberculosis
Abnormalities of ADH secretion
Head irradiation
Arginine-vasopressin or antidiuretic hormone
Unknown causes
This hormone is secreted secondary to osmotic
Treatment of pituitary failure includes one or a cocktail
changes
of hormone replacement therapies:
Mediates free water reabsorption in the kidneys
Steroids (hydrocortisone): anyone with suspected
pituitary failure should be given hydrocortisone and
Excessive ADH secretion - syndrome of
investigated later (failure to give hydrocortisone in
inappropriate ADH secretion (SIADH)
suspected deficiency may result in death)
Thyroxine: should only be given after adequate cortisol
This is not uncommonly seen on the medical wards and
replacement
results in:
Testosterone (males), oestrogen and progesterone
Dilutional hyponatraemia
(females)
Low plasma osmolarity and inappropriately high urine
Growth hormone: this is routinely given to children with
osmolarity
(secondary to water reabsorption in the
GH deficiency but in adults, only those with symptoms
kidneys)
receive this expensive form of treatment
Causes of inappropriate ADH secretion (known as
syndrome of inappropriate ADH or SIADH) are sum-
marized in Table 5.
The posterior pituitary
In contrast to the anterior pituitary, the posterior pitu-
Investigations
itary does not synthesize hormones but stores hormones
Hyponatraemia is commonly seen in hospitalized
produced in the hypothalamic region. These hormones
patients. A common ‘knee jerk reaction’ is to label these
include:
patients as having SIADH and start fluid restriction,
Antidiuretic hormone (ADH)
which can be detrimental if the patient is not assessed
Oxytocin
properly
12
Part 1: Basics
It should be remembered that patients with SIADH are
Functional ADH deficiency may occur if the kidneys
euvolemic and therefore:
fail to respond to the hormone (ADH production is
It is important to rule out dehydration before start-
normal). This is called nephrogenic DI, which may be:
ing investigations for SIADH (are they on diuretics? is
Congenital or familial
there a history of recent fluid loss?)
Acquired
It is also important to rule out fluid overload before
Drugs (lithium or demeclocycline)
starting investigations for SIADH (is there advanced
Electrolyte abnormalities: hypercalcaemia, hypo-
heart, liver or renal failure?)
kalaemia
In euvolemic patients, SIADH should be suspected in
Chronic renal disease
the presence of:
Hyponatraemia with low plasma osmolarity
Box 4 Abnormalities of oxytocin secretion
Inappropriately high urine osmolarity
High urinary sodium excretion
In women, oxytocin:
In patients with suspected SIADH, we need to exclude:
Helps contraction of the pregnant uterus
Hypothyroidism (TFTs)
Helps contraction of breast duct smooth muscle aiding
breast feeding
Hypoadrenalism (short synacthen test)
Deficiency of this hormone has no effect on parturition
Once the diagnosis of SIADH is made, it is necessary
or breast feeding
to establish the cause (see Table 5)
In men, the role of this hormone is unclear
Careful history and examination of the patient
Double check drug history
Computed tomography
(CT) head, chest and
Special cases in pituitary disease
abdomen are frequently requested to rule out a
What is pituitary apoplexy?
malignant cause
This is caused by infarction of the pituitary gland, con-
sequently resulting in failure of hormone production
Treatment
Can occur in patients with large pituitary tumours
In confirmed SIADH:
Any individual with known or suspected pituitary
Restrict oral fluid to 750-1500 mL of oral fluid/day
tumour complaining of sudden onset severe headache
Treat the cause
with or without cranial nerve palsies (III, IV and VI)
Demeclocycline, which induces nephrogenic diabetes
should be suspected as having pituitary apoplexy
insipidus, can help in difficult cases
Urgent MRI of the pituitary should be requested
ADH deficiency
These patients should be given parenteral steroids
This results in the passage of large volume of dilute urine,
What is a craniopharyngioma?
resulting in:
A tumour arising from the epithelial remnants of
Polyuria
Rathke’s pouch
Nocturia
Can be present at any age and half the subjects are
Thirst
children
Enuresis in children
Clinical presentation is similar to a non-functioning
Causes of ADH deficiency, also known as cranial
pituitary adenoma
diabetes insipidus (DI) are:
The presence of calcification in a pituitary tumour
Congenital or familial
should raise the suspicion of a craniopharyngioma
Acquired
Treated surgically but recurrence rates are high
Head injury
What is lymphocytic hypophysitis?
Tumours infiltrating the posterior pituitary
A rare inflammatory condition of the pituitary, likely
Infiltrative conditions, such as sarcoidosis or
to be autoimmune in origin
histiocytosis
Results in pituitary hormonal failure and can cause a
Infections such as meningitis, encephalitis or
mass effect
tuberculosis
Spontaneous recovery may occur
Vascular
Usually treated with replacement of deficient
Idiopathic
hormone(s)
The thyroid
Anatomy
Pathophysiology of the thyroid
The thyroid is composed of a midline isthmus just
Disorders of the thyroid gland include:
below the cricoid cartilage (Fig. 7), a right and a left lobe,
Hormonal hypersecretion (hyperthyroidism): with or
extending from the isthmus laterally
without thyroid gland enlargement (thyroid goitre)
Thyroid cells are arranged in follicles and produce
Hormonal hyposecretion (hypothyroidism): with or
thyroid hormones, which are stored in the lumen of the
without thyroid goitre
follicle
Thyroid nodules/goitre with normal thyroid hormone
The thyroid also contains C cells, which produce
levels
calcitonin
Thyroid cancers
Physiology
An approach to a patient with suspected
Iodine is an essential component of thyroid
thyroid disease
hormones
A proper history is important particularly in relation
The thyroid gland traps iodine from the plasma, a
to:
process mediated by the sodium iodide symporter
Symptoms of hyper- or hypothyroidism (see below)
Iodine is then organified and iodothyronines (thyroid
In the case of thyroid nodules or goitre:
hormones) are formed, a process mediated by the enzyme
Recent change in size
thyroid peroxidase (TPO)
Recent hoarse voice
Thyroid hormones are stored in thyroid follicles bound
Compressive symptoms (difficulty in breathing or
to thyroglobulin (TG)
swallowing)
In response to demand, TG is internalized by thyroid
follicular cells, and thyroid hormones are liberated into
the blood stream
Thyroid hormone secretion is constituted of 20% T3
Box 5 Examination of the thyroid
and 80% T4
T4 is converted in peripheral tissue to the active
Assessment of thyroid status
hormone T3, through the action of deiodinase enzymes
General
Thyroid hormones are bound to plasma proteins (thy-
Is the patient fidgety or agitated?
roxine binding globulin, albumin) and their levels can be
Facial appearances
influenced by plasma protein concentrations. Therefore,
Is the patient’s clothing appropriate? (Wearing a
free thyroid hormone levels should be measured in cases
t-shirt in December suggests hyperthyroidism!)
Hand tremor: best checked by placing a piece of paper
of suspected thyroid hormone abnormalities
on outstretched arms
Thyroid hormone production is regulated by the
Sweaty or dry skin (check palms)
hypothalamus and pituitary gland as shown in Fig. 8.
Feel pulse (tachycardia, atrial fibrillation or bradycardia)
Assess for lid lag
Check for signs of proximal myopathy
Endocrinology and Diabetes: Clinical Cases Uncovered. By R. Ajjan.
Tendon reflexes (ask the patient to kneel on a chair and
Published 2009 by Blackwell Publishing, ISBN: 978-1-4051-5726-1
check ankle reflexes)
13
14
Part 1: Basics
Hypothlamus
Superior
-
thyroid artery
Larynx
Thyroid gland
TRH
+
Common
Isthmus
carotid
Pituitary
artery
-
Inferior thyroid
artery
Trachea
TSH
Figure 7 Anatomy of the thyroid gland. The isthmus of the
+
gland thyroid is located just below the cricoid cartilage. The
right and left lobes extend laterally and some individuals have
Thyroid
a small conical lobe extending from the isthmus upwards
called the pyramidal lobe.
Assessment of the thyroid gland
Inspection
Observe for any neck swelling; ask the patient to swallow
Thyroid Hormones
and observe for a neck mass that moves with swallowing
(T3 and T4)
A midline structure moving by tongue protrusion
suggests a thyroglossal cyst
Figure 8 Regulation of thyroid hormone production. The
Palpation
hypothalamus produces thyrotropin releasing hormone (TRH),
Palpate the thyroid starting in the isthmus and moving
which stimulates the pituitary to secrete thyroid stimulating
out laterally and upwards. Use the pulp not the tip of
your fingers
hormone (TSH), which in turn stimulates the thyroid gland to
Feel the neck for lymphadenopathy
synthesize and release thyroid hormones. T3 and T4 have a
Percussion: gives limited information about the possibility
negative feedback effect on TRH and TSH production.
of retrosternal extension of a goitre
Auscultation: bruit over the thyroid gland suggest a
diagnosis of Graves’ disease (due to increased gland
Hyperthyroidism
vascularity)
Causes of hyperthyroidism are summarized in Table 6.
Check for Pemberton’s sign: raising both arms above the
head results in venous obstruction, which can be seen in
Graves’ disease (GD)
large goitres with retrosternal extension
The commonest cause of hyperthyroidism (80% of
Assessment for signs of extrathyroidal disease (in
cases)
suspected Graves’ disease)
An autoimmune condition characterized by the pres-
Graves’ ophthalmopathy
Proptosis
ence of thyroid stimulating antibodies (TSAb), mimick-
Periorbital oedema
ing the action of TSH and resulting in hyperthyroidism
Conjunctival injection
Can be associated with extrathyroidal manifestations
Chemosis
(summarized in Table 7, Fig. 9, colour plate section)
Corneal ulceration
Inability to fully close the eyes
Clinical presentation
Pretibial myxoedema
Patient usually presents with classical symptoms
Thyroid acropachy
hyperthyroidism (summarized in Table 8)
The thyroid
15
Table 6 Causes, aetiology and diagnosis of hyperthyroidism.
Cause of hyperthyroidism
Frequency and aetiology
Diagnosis
Graves’ disease
80%, thyroid stimulating antibodies
Clinical examination
Thyroid autoantibodies
Thyroid uptake scan in uncertain cases
Toxic nodule or toxic
15%, activating mutations in TSH receptor
Clinical examination
multinodular goitre
and Gsα protein
Thyroid uptake scan
Thyroiditis
2-4%, autoimmune, viral or drug-related
Clinical examination
(amiodarone)
Thyroid uptake scan
ESR
TSH-secreting tumour
<1%
Raised TSH and thyroid hormones; pituitary imaging
Exogenous thyroid hormone
Variable, excess ingestion of thyroid
Clinical assessment
administration
hormones
Hyperemesis gravidarum
Rare, raised hCG (mimicking TSH action)
Clinical assessment
Choriocarcinoma
Absence of thyroid autoimmunity
Known pregnancy
Imaging of the pelvis
Struma ovarii
Rare, ectopic ovarian thyroid tissue
Clinical assessment
Thyroid/pelvic uptake scan
Imaging of the pelvis
Thyroid hormone resistance
Rare, pituitary resistance to thyroid
Clinical assessment
hormones
Family history
ESR, erythrocyte sedimentation rate; hCG, human chorionic gonadotrophins.
Neck palpation reveals a smooth, uniform goitre in the
Thyroid uptake scan: in Graves’ disease this shows
majority of cases
uniform uptake of technetium or iodine (Fig. 10)
Around half the patients will have extrathyroidal man-
ifestations of the disease (summarized in Table 7 and
Treatment
shown in Fig. 9 and Fig. 40)
GD can be treated with antithyroid drugs, radioactive
iodine and surgery. Symptomatic treatment is also
Investigations
possible in patients with severe symptoms; β-adrenergic
Confirm the presence of hyperthyroidism:
blocking agents (propranolol) can be used but these are
Suppressed TSH
only required in a minority.
Raised thyroid hormones (T4 and/or T3)
Detection of thyroid stimulating antibodies: not
essential for making the diagnosis and usually reserved
Antithyroid drugs (thionamides)
for atypical cases. These are positive in 95-99% of GD
Include propylthiouracil, carbimazole and its active
cases depending on the type of assay used
metabolite methimazole
In uncertain cases (no or asymmetrical goitre, negative
These agents interfere with the action of thyroid peroxi-
antibodies):
dase, thereby inhibiting thyroid hormone production
16
Part 1: Basics
Table 7 Extrathyroidal manifestations of Graves’ disease.
Table 8 Symptoms and signs of Graves’ disease.
Extrathyroidal disease, usually Graves’ ophthalmopathy (GO)
Hyperthyroidism due to other causes presents with similar
can be seen even in individuals with normal thyroid function.
symptoms and signs except for the absence of GO, PTM and
acropachy.
Extrathyroidal
Clinical findings
manifestations of
Symptoms
Frequency
Graves’ disease
Nervousness, heat intolerance and perspiration,
85-95%
Graves’
Clinically evident in 50% of Graves’
palpitations, fatigue
ophthalmopathy
disease patients but can be seen in 90%
Increased frequency of stools, weight loss with
75-85%
using imaging techniques
increased appetite
Characterized by swelling of the
Symptoms of Graves’ ophthalmopathy
50%
extraorbital muscles and proliferation of
adipose and connective tissue in the
Insomnia, polyuria and menstrual irregularities
20-30%
orbit
Signs
Frequency
The above results in proptosis of the
eyes and in severe cases exposure
Hyperkinetic behaviour, tachycardia or atrial
90-95%
keratitis. Also, it may result in
fibrillation
ophthalmoplegia and optic neuropathy
Goitre, warm and moist skin, hand tremor
90%
Graves’ dermopathy
Rare, usually affects the shins (hence
pretibial myxoedema)
Signs of Graves’ ophthalmopathy
50%
Skin looks discoloured, indurated and
Pretibial myxoedema
5%
can be itchy
Onycholysis, acropachy
<5%
Graves’ dermopathy is almost always
Rare presentations
associated with GO
Thyroid storm: an extreme form of thyrotoxicosis presenting
Graves’ acropachy
Very rare, usually associated with GO
with fever, cardiovascular collapse, confusion, psychosis, severe
and Graves’ dermopathy
weakness and even coma. This is a life-threatening emergency
that requires urgent medical treatment (see text).
Characterized by clubbing and
subperiostal new bone formation
Apathetic hyperthyroidism: the adrenergic hyperactivity
manifestations are absent and this presentation can be confused
with depression (usually occurs in the elderly).
Antithyroid drugs can be given as
Titration regime (usually for 18 months): enough
antithyroid drug is given to keep the thyroid hormones
in the normal range
Box 6 Side effects of antithyroid drugs
Block and replace regime (usually for 6 months): a
large dose of antithyroid drug is given to fully block
Agranulocytosis: the most serious complication and all
patients are advised to immediately report to their
thyroid hormone production and thyroxine replace-
physician in case they develop a temperature, sore
ment therapy is added to ensure adequate plasma
throat or mouth ulcers. Agranulocytosis with either
thyroid hormone levels
propylthiouracil or carbimazole represents a
After 6-18 months, treatment is stopped and disease
contraindication to the use of these agents
remission is achieved in less than 50%
Minor side effects such as rash, musculoskeletal pain,
deranged liver function. If these occur, it is possible to
Radioactive iodine (RAI)
switch between antithyroid drugs
Safe and effective treatment (up to 90% respond after
one dose)
Used as second line in Europe but frequently as first
RAI treatment destroys the thyroid gland and can take
line in America
up to 6 months to have full effect
The thyroid
17
Surgery
This is reserved for those with:
Large goitre
Personal preference
Severe hyperthyroidism and intolerance to antithyroid
drugs
Toxic solitary nodule (TN) and toxic
multinodular goitre (TMNG)
Around 15% of hyperthyroid patients have a TN or
TMNG.
Clinical presentation
Symptoms and signs of hyperthyroidism
Neck palpation reveals an irregular goitre or a thyroid
nodule
(a)
There are no extrathyroidal signs
Investigations
Confirm the presence of biochemical hyper-
thyroidism
Thyroid uptake scan shows one or more thyroid
nodules with increased uptake, which are often referred
to as “hot” nodules
Hyperthyroidism due to toxic nodule(s) must be dif-
ferentiated from GD and a cold nodule (an uptake scan
can be used), due to the relatively high prevalence of
malignancy in GD-associated cold nodules
A thyroid uptake scan for a toxic nodule is shown in
(Fig. 11)
Treatment
Toxic solitary nodule or toxic multinodular goitre can
be treated with antithyroid drugs but the disease relapses
(b)
once medical treatment is stopped
Figure 10 Technetium scan in an individual with Graves’
The best treatment option is radioactive iodine, which
disease, demonstrating uniform uptake and thyroiditis
often restores euthyroidism
showing lack of uptake. Courtesy of Dr R. Bury, the
Surgery is also an option but is reserved for a minority
Radionuclide Department, University of Leeds.
of patients, usually those with large disfiguring goitres
Fine needle aspiration
(FNA) is only required in
selected cases (malignancy in toxic nodules is rare) and
Induces long-term hypothyroidism (patients need to
this is discussed below
be warned that they will potentially need lifelong treat-
ment with thyroxine)
Contraindications include:
Thyroiditis
Absolute: pregnancy
A relatively rare cause of hyperthyroidism
Relative: active eye disease (eye disease may worsen
May be autoimmune in nature, follow a viral disease
after RAI)
or can be drug-related
18
Part 1: Basics
Table 9 Symptoms and signs of hypothyroidism.
Symptoms
Frequency
Weakness/lethargy, dry skin
95%
Sensation of cold and decreased sweating,
80%
oedema of the face
Impaired memory, constipation, hair loss and
65%
weight gain
Deafness and non-specific chest pain
30%
Signs
Frequency
Thick tongue, facial oedema, dry and cold skin
80%
Slow relaxing reflexes
75%
Bradycardia, skin pallor, coarse hair, depression
60%
Figure 11 Radioactive iodine uptake in a subject with
hyperthyroidism shows a toxic nodule with suppression of
uptake activity in the rest of the gland. Courtesy of Dr. R. Bury
the Radionuclide Department, University of Leeds.
Thyrotoxic phase is usually followed by a hypothyroid
phase, which may require a short course of thyroid
Clinical presentation
hormone replacement until the thyroid follicular cells are
Symptoms and signs of thyrotoxicosis as discussed
fully recovered
above
Commonly secondary to a viral infection; therefore,
Hyperthyroidism secondary to TSH-secreting
thyrotoxic symptoms following a flu-like illness should
tumours (TSH-oma)
raise the suspicion of thyroiditis
This is a rare cause of hyperthyroidism
Individuals may experience pain and tenderness in
It should be suspected in individuals with raised
the region of the thyroid gland, a condition called De
thyroid hormones and detectable TSH levels
Quervain thyroiditis
TSH-oma is discussed in the pituitary section
Diagnosis is made by demonstrating biochemical thyro-
toxicosis, associated with lack of uptake on thyroid scan
Hypothyroidism
(Fig. 10)
A common disease, mainly affecting the female popula-
Postpartum thyroiditis
tion. Causes include:
Occurs in
5-10% of women within 1 year of
Autoimmune hypothyroidism: by far the commonest
delivery
cause
Characterized by a hyperthyroid phase within 4-6
Atrophic (no goitre palpable)
months of delivery followed by a hypothyroid phase
Goitrous (Hashimoto’s thyroiditis)
with subsequent restoration of normal thyroid
Postpartum thyroiditis
function
Post-radiation
Permanent hypothyroidism eventually develops in
Iodine deficiency
around one-third of patients
Drugs (amiodarone, lithium)
Treatment
Congenital developmental and biosynthetic defects
The disease is self-limiting and treatment is not usually
Secondary (due to pituitary or hypothalamic defects)
required
For neck pain and tenderness, non-steroidal anti-
Clinical presentation
inflammatory agents can be used, whereas steroids are
This can be very variable and the commonest symptoms
reserved for severe cases
and signs are summarized in Table 9.
The thyroid
19
Investigations
It may be due to:
Biochemical testing shows low plasma thyroid hormone
Graves’ disease
levels with raised TSH
Toxic multinodular goitre
Some individuals may have high TSH with normal
Usually occurs in older individuals who may display
thyroid hormone levels, a condition known as subclinical
mild symptoms of hyperthyroidism but may be
hypothyroidism which is discussed below
asymptomatic
Thyroid antibodies
(TPO antibodies) are usually
Subjects with SHyper are at increased risk of:
detected
in
individuals
with
autoimmune
Atrial fibrillation
hypothyroidism
Osteoporosis
Any subject with low plasma thyroid hormone levels
Radioactive iodine is usually the best treatment option
with low or normal TSH should be suspected of having
for these individuals
secondary hypothyroidism (i.e. pituitary failure) and
urgent investigations/endocrine referral should be
Amiodarone-induced thyroid dysfunction
made
This can be a difficult condition to manage even for an
experienced endocrinologist. Amiodarone can result in
both hypo- and hyperthyroidism through:
Treatment
High iodine content of the drug (40% of its weight)
This is relatively simple and consists of replacing
Direct toxic effect of amiodarone on thyroid follicular
thyroid hormone
cells
L-thyroxine (T4) is usually given, which is converted
in the periphery to the active hormone T3
Amiodarone-induced hypothyroidism
Combination therapy with T3 and T4 is very rarely
Occurs in up to 15% of patients on the drug
used and only in selected patients who remain symptom-
This can be simply managed by giving thyroid hormone
atic on T4 replacement alone
replacement similarly to individual with primary
The appropriate dose of thyroxine should titrated to
hypothyroidism
suppress TSH below
2 mIU/L but full suppression
Discontinuation of amiodarone (which is not always
should be avoided (usual replacement dose is around
possible) can result in restoration of normal thyroid
1.4 mcg/kg)
function
Special cases of abnormal thyroid function
Amiodarone-induced hyperthyroidism (AIT)
Subclinical hypothyroidism (SHypo)
This occurs in less than 5% of patients on amiodarone
Raised TSH levels in the presence of normal thyroid
treatment, and can be divided into:
hormones is defined as SHypo
AIT type I
SHypo is usually due to early autoimmune
Similar to autoimmune hyperthyroidism
hypothyroidism
Can be managed with antithyroid drugs
The term SHypo suggests the absence of symptoms but
RAI is usually ineffective (need to stop amiodarone
this is somewhat misleading as a large proportion of these
for a year before considering RAI)
patients are symptomatic
Thyroidectomy should be considered for difficult
Thyroid function should be repeated within 3 months
cases
and if TSH remains elevated (or it is increasing), then
AIT type II
treatment is advised, particularly in patients with positive
This is due to thyroiditis and thyroid destruction
TPO antibodies
Usually managed with high doses of steroids
Some studies suggest an association between subclini-
Mixed type I and type II AIT can occur and is best
cal hypothyroidism and atherosclerotic disease
managed by a combination of antithyroid drugs and
The aim of treatment is to normalize TSH
steroids
Amiodarone withdrawal is advisable in subjects with
Subclinical hyperthyroidism (SHyper)
AIT but this is not always possible
Suppressed TSH with normal thyroid hormone levels
Any patient planned for amiodarone treatment should
(both T4 and T3) is defined as SHyper
have:
20
Part 1: Basics
Table 10 Important characteristics of type I and type II AIT.
The dose of thyroxine replacement is usually
increased in pregnancy by up to 50%
Type I AIT
Type II AIT
Thyroid hormone levels should be kept at the upper
end of normal (without TSH suppression) in women
Goitre
Frequent
Rare
with hypothyroidism receiving thyroxine replacement
Thyroid antibodies
Positive
Negative
during pregnancy
Graves’ disease in pregnancy
Plasma CRP
Normal
High
Block and replace is contraindicated (antithyroid
Plasma IL-6
Normal
High
drugs cross the placenta whereas thyroxine does not,
Vascularity (Doppler studies)
Increased
Reduced
potentially resulting in fetal hypothyroidism)
Propylthiouracil is probably safer to use than carbima-
CRP, C-reactive protein.
zole due to reported congenital abnormalities with the
latter
Thyroid function and thyroid antibody screen done
The lowest dose of antithyroid drugs should be used
prior to starting treatment
to keep thyroid hormones at the upper end of normal
Thyroid function tested every 6 months whilst on this
range
therapy and for 12 months after discontinuing the drug
Table 10 summarizes the important characteristics of
Thyroid nodular disease in euthyroid subjects
type I and type II AIT.
(thyroid nodules and multinodular goitre)
Very common, clinically evident in around 10% of the
Thyroid storm
UK population
A rare, severe and life-threatening case of hyperthyroid-
A thyroid goitre can be physiological:
ism characterized by:
Puberty
Reduced conscious level
Pregnancy
Hyperthermia
Non-physiological causes of thyroid goitre include:
Multisystem decompensation (cardiac failure, renal
Thyroid autoimmunity
failure, etc.)
Iodine deficiency (endemic goitre)
Treatment consists of:
Drugs (lithium)
High-dose antithyroid drugs
Other (unknown)
Potassium iodine
A thyroid nodule can be:
β-blockers
Solid: composed of thyroid tissue
Steroids
Cystic: usually filled with brown fluid
Mixed: solid/cystic
Myoedema coma
Thyroid nodules/goitres are more common in
A rare, severe and life-threatening case of hypo-
women
thyroidism.
Only a minority of thyroid nodules are malignant
Often precipitated by infection
(<5%)
Characterized by:
Thyroid nodules are more likely to be malignant
Reduced conscious levels
in:
Hypothermia
Young (<20 years) or older (>60 years) subjects
Respiratory depression and associated CO2
Rapidly growing nodule
retention
Compressive symptoms: hoarse voice, dysphagia,
Treatment consists of:
breathing difficulties
T3 and T4
Family history of endocrine malignancy
Antibiotic cover after appropriate cultures
Cold nodule in an individual with Graves’ disease
Steroid cover
(associated adrenal dysfunction is
History of familial polyposis coli (papillary carci-
common)
noma), Hirshprung’s disease
(medullary thyroid
Hypo- and hyperthyroidism during pregnancy
cancer) or Hashimoto’s thyroiditis
(thyroid
Hypothyroidism in pregnancy
lymphoma)
The thyroid
21
Table 11 Classification of thyroid cancers.
Characteristics
Papillary
Follicular
Anaplastic
Medullary* Lymphoma
Cell type
Thyroid cells
Thyroid cells
Thyroid cells
C cells
Lymphocytes
Age at presentation
30-50
40-50
>60
Any age
>40
Frequency
75%
15%
<5%
<5%
<5%
Prognosis
Good
Good
Very poor
Variable
Variable
Treatment
Surgery and RAI ablation Surgery and RAI ablation Surgery
Surgery
Chemotherapy
radiation
Chemotherapy
External radiation
*May be part of MEN II or familial medullary carcinoma and is associated with raised serum calcitonin levels.
Clinical presentation
iodine ablation (high doses of radioactive iodine) if his-
Patient presents with a history of lump in the neck, which
tology confirms malignancy (up to 10% of FNA gives
is:
false-positive results)
Observed by the patient
Nodules with benign cytology can be followed up
Observed by a family member/friend
medically with regular examination and repeat FNA as
Detected during investigations for other pathologies
necessary
(ultrasound or CT neck)
Alarming features include:
Thyroid cancers
Predisposition to thyroid malignancy as above
Thyroid cancers are rare and mortality is low as most
Rapidly growing goitre or nodule
carry a good prognosis
Compressive symptoms or hoarse voice
Occur more commonly in women but a thyroid nodule
Very hard nodule
in man is more likely to be malignant
Fixation of skin above the nodule
Risk factors and indicators of malignancy in thyroid
Presence of neck lymphadenopathy
nodules are discussed above
Classification of thyroid cancers is summarized in
Investigations of thyroid nodules/multinodular
Table 11
goitre
Fine needle aspiration (FNA) of the solitary nodule or
Clinical presentation
dominant nodule in a multinodular. A simple test, usually
A thyroid nodule or goitre: a rapidly growing thyroid
done in a clinic
nodule should raise the suspicion of malignancy
Benign cytology: follow-up with repeat FNA in 6
Post-mortem examination has shown that thyroid
months is required
cancers are not uncommon and individuals die with,
Inconclusive: repeat FNA (if repeat is undetermined
rather than from, the disease
then refer to surgery)
Risk factors for thyroid cancers should be elicited in
Features of malignancy: surgery
the history
CT scan in large goitres and in the presence of com-
Hard nodules and cervical lymphadenopathy should
pressive symptoms
raise the suspicion of malignancy
Pulmonary function tests to establish the presence of
respiratory compromise
Investigations
Treatment
FNA as above
Clinically and/or cytologically suspicious nodules
Ensure that patient is not thyrotoxic before performing
should be treated with surgery, followed by radioactive
FNA
22
Part 1: Basics
Treatment
Palliative radiotherapy can be arranged, whereas che-
Patients with cytologically proven papillary or follicular
motherapy is generally ineffective
thyroid malignancy should undergo:
Lymphoma
Total thyroidectomy
Usually treated with radio- and chemotherapy
Radioactive iodine ablation
Patients with strong clinical suspicion of malignancy
This should be followed by treatment with TSH-
but negative FNA should still be considered for surgery
suppressive doses of thyroxine (i.e. supraphysiological
as FNA can give false-negative results in a minority of
doses of throxine are given to keep TSH suppressed)
cases (5-10%).
Patients with medullary carcinoma should undergo:
Patients with previous thyroid cancer should be moni-
Total thyroidectomy and lymph node dissection
tored for life
Suppressive therapy with thyroxine is not needed
Regular examination
(C cells are not controlled by TSH)
Thyroglobulin plasma levels: detectable thyroglobulin
Appropriate testing should be arranged to rule out
plasma levels after surgery and radioactive ablation
MEN II (see neuroendocrine section)
therapy indicate the presence of residual thyroid tissue
Anaplastic carcinoma
and, hence, recurrence of the disease
Prognosis is very poor and surgery is rarely successful
Bone and calcium metabolism
Parathyroid hormone (PTH), secreted by the parathy-
from bone, increased intestinal absorption and reduced
roid glands, is the main hormone responsible for calcium
urinary excretion, and, hence, increases plasma calcium
haemostasis. There are five organs involved in calcium
levels (low blood calcium levels result in increased secre-
metabolism:
tion of PTH, whereas high levels lead to suppression of
Parathyroid gland, through the secretion of PTH,
PTH release)
which increases plasma calcium levels
Gastrointestinal tract: Vitamin D plays an important
Gastrointestinal tract (absorption of calcium)
role in controlling absorption of calcium in the gut
Renal tract (excretion of calcium)
Vitamin D undergoes
25-hydroxylation and
1-
Bone (storage of calcium)
hydroxylation in the liver and kidneys, respectively, to
Thyroid gland, through the secretion of calcitonin by
form active vitamin D [1,25-(OH)2D]
C cells
Vitamin D enhances intestinal calcium absorption
Calcitonin has a weak calcium-lowering effect
Kidneys: calcium is reabsorbed by the kidneys, a
Plasma calcitonin levels are only used for the diag-
process regulated by PTH
nosis of medullary thyroid cancer and have no role in
PTH results in increased calcium and decreased phos-
investigations of disorders of calcium metabolism
phate reabsorption (i.e. phosphate loss) by the kidneys.
Therefore, in primary hyperparathyroidism, hypercal-
Anatomy
caemia is often associated with hypophosphataemia
Usually, there are four parathyroid glands located at the
back of the thyroid gland (see Fig. 12). Rarely, ectopic
Pathophysiology
parathyroid tissue can be identified in the thoracic cavity,
Hypocalcaemia and hypercalcaemia are relatively
which is due to abnormal parathyroid gland migration
common clinical conditions and are discussed below
during embryogenesis.
Osteoporosis is characterized by an increase in osteo-
clast over osteoblast activity, resulting in reduction in
Physiology
bone mass (quantitative change), consequently predis-
Bones are in constant turnover, through the action of:
posing to fractures
Osteoclasts: these cells are responsible for bone
Osteomalacia is characterized by insufficient calcium
resorption
in bone tissue with normal bone mass
(qualitative
Osteoblasts: these are responsible for bone formation
change)
Calcium is important for:
In Paget’s disease, the activity of osteoblasts and osteo-
Bone health
clasts is disorganized resulting in both bone resorption
Neuromuscular conduction
and new bone formation in an uncoordinated manner
Plasma calcium levels, which should always be cor-
This chapter will discuss a number of different clinical
rected for plasma albumin, are kept in check by a number
entities including:
of mechanisms/organs:
Hypocalcaemia
Parathyroid gland: PTH results in calcium liberation
Hypercalcaemia
Osteomalacia and rickets
Osteoporosis
Endocrinology and Diabetes: Clinical Cases Uncovered. By R. Ajjan.
Paget’s disease
Published 2009 by Blackwell Publishing, ISBN: 978-1-4051-5726-1
Inherited bone abnormalities
23
24
Part 1: Basics
Clinical presentation
This can be variable from one person to another and is
related to the degree of hypocalcaemia. Symptoms
include:
Thyroid gland
Tingling and numbness (often described by the patient
as pins and needles sensation) in the fingers, toes and
lips
Parathyroid gland
Cramps
In severe cases, stridor (due to spasm of laryngeal
muscles) and/or seizures
Low calcium results in neuromuscular irritability and,
Figure 12 The four parathyroid glands are located on the
therefore, the following signs can be found:
posterior aspect of the thyroid gland.
Chvostek’s sign: tapping on the facial nerve in front
of the ear results in twitching of the corner of the
mouth
Hypocalcaemia
Trousseau’s sign: inflation of the sphygmomanometer
This is not an uncommon condition and causes
above the arterial pressure results in carpopedal
include:
spasm
Vitamin D deficiency: probably the commonest cause,
secondary to:
Investigations
Poor sunlight exposure (commonly seen in Asian
Plasma calcium: diagnosis is confirmed by demon-
women who cover their bodies with clothes)
strating low plasma calcium (make sure corrected calcium
Malabsorption
(coeliac disease is a common
levels are assessed)
cause)
Establish the cause:
Poor diet (frequently seen in the elderly)
Check PTH levels: low PTH levels in the presence of
Kidney disease (failure of 1-hydroxylation of vitamin
hypocalcaemia indicate parathyroid failure
D)
Vitamin D levels: patients with low vitamin D levels
Hypoparathyroidism
should be investigated for the possibility of coeliac
Congenital
disease
Destruction of the glands by radiation, surgery or
Renal function
autoimmunity
Magnesium levels
Hypomagnesaemia (inhibits PTH secretion), which
may be due to:
Treatment
Gastrointestinal loss
Acute symptomatic hypocalcaemia (tetany, seizures) is
Renal loss
a medical emergency and should be treated with i.v.
Drugs (diuretics, cyclosporine, alcohol)
calcium
Drugs
20 mL of 10% calcium diluted in 100-200 saline
Calcitonin
should be infused over 10-20 min
Increased calcium uptake by bone
Further calcium infusion may be needed (24 h slow
Hungry bone syndrome (following thyroid or para-
calcium infusion is frequently used)
thyroid surgery)
Regular monitoring of calcium levels should be
Osteoblastic bony metastasis (prostate cancer)
organized (every 4-8 h)
Complexing of calcium from the circulation
Care should be taken against extravasation of
Acute pancreatitis (calcium soap formation due to
calcium into interstitial tissue, which may cause
fat autodigestion)
necrosis (a large vein should be used for i.v. calcium
Multiple blood transfusions (complex of calcium
administration)
with citrate)
Intravenous treatment should be followed by oral
Functional: inability of PTH to exert its effect (PTH
calcium administration and correction of the precipi-
resistance), also known as pseudohypoparathyroidism
tating cause
Bone and calcium metabolism
25
Acute hypocalcaemia with mild symptoms
Abdominal pain
Oral therapy with calcium and vitamin D is usually
Constipation
given
Central nervous system
Correction of the underlying cause
Confusion
Patient should be carefully monitored
Depression
Chronic hypocalcaemia
Treatment should be directed at correcting the
Investigations
underlying cause
Plasma calcium levels: these are elevated (make sure
corrected calcium levels are used)
Hypercalcaemia
PTH
Hypercalcaemia is commonly seen on the general medical
Raised: hyperparathyroidism
wards. Causes include:
Suppressed: non-parathyroid cause
Increased secretion of PTH
Normal: early hyperparathyroidism (usually calcium
Primary hyperparathyroidism
levels are only mildly elevated) or familial hypocalciu-
Tertiary hyperparathyroidism
ric hypercalcaemia (FHH)
Malignancy
Establish the cause
Secretion of PTH-related peptide
History and full examination: this is important as it
Bony invasion in metastatic disease
may give clues to the presence of a malignant
Familial, e.g. familial hypocalciuric hypercalcaemia
disorder
(secondary to low urinary calcium excretion)
In those with elevated PTH, the most likely diagnosis
Autosomal dominant disease due to mutation in the
is a parathyroid adenoma and localization of this can
calcium-sensing receptor
be done with: CT scan of the neck and chest, ultra-
PTH levels are usually in the normal range
sound of the neck and 99mTc-cestamibi scan, which
It must be differentiated from primary hyperpara-
relies on concentration of the radioactive material in
thyroidism, otherwise the patient may undergo unnec-
the parathyroid tissue
essary surgery
Renal function: chronic renal failure may result in
Patient usually asymptomatic and diagnosis is made
tertiary hyperparathyroidism
by demonstrating reduced urinary calcium excretion
Chest X-ray: particularly in those with respiratory
with high plasma calcium
symptoms (exclude a malignant lung condition)
Medications
Myeloma screen: hypercalcaemia can be one of the
Thiazide diuretics
early manifestations of multiple myeloma
Vitamin D intoxication
Vitamin D levels: to rule out vitamin D
Vitamin A intoxication
intoxication
Granulomatous disease
24-h urinary calcium: low urinary calcium excretion
Sarcoidosis
in FHH (important to make this diagnosis as no treat-
Endocrine causes (rare)
ment is usually required)
Hyperthyroidism
In case of suspicion, rule out endocrine causes of
Addison’s disease
hypercalcaemia: hyperthyroidism
(TFTs), adrenal
Acromegaly
failure
(synacthen test) and acromegaly
(glucose
tolerance test, if history is suggestive)
Determine end organ damage
Clinical presentation
Ultrasound of the renal tract
Symptoms are usually insidious and include:
Skeletal radiographs
Osmotic symptoms
Check bone mineral density
Polyuria
Polydipsia
Dehydration
Treatment
Gastrointestinal
For severe symptomatic hypercalcaemia:
Anorexia and vomiting
Rehydrate patient with i.v. fluid
26
Part 1: Basics
Intravenous bisphosphonates
(pamidronate is fre-
Table 12 Symptoms and signs of osteomalacia and rickets.
quently used): these agents should only be given after
Osteomalacia
Rickets
adequate hydration
Treat the cause of hypercalcaemia
Bone pain
Growth retardation
In resistant cases calcitonin can be used
For moderate hypercalcaemia:
Fractures
Bone pain and fractures
Ensure adequate patient hydration
Proximal myopathy
Skeletal deformities:
Treat the underlying cause:
Bowing of tibia
Surgery for hyperparathyroidism: in mild cases, this
is not always necessary and patient can be simply fol-
Rickety rosary
lowed up with regular calcium checks and monitoring
Widening of wrists
for end organ damage
Hypercalcaemia of malignancy may partly respond to
systemic steroids, which can be given until specific cancer
treatment is introduced
Osteomalacia and rickets
Osteomalacia and rickets are due to inadequate mineral-
ization of bone. The former occurs in mature bone,
whereas the latter occurs in growing bone. Causes of
osteomalacia and rickets include:
Associated with low phosphate
Vitamin D deficiency (the commonest cause): low
phosphate is due to increased PTH secretion
Vitamin D-dependent rickets: due to deficient
vitamin D receptor or inadequate conversion of
vitamin D to the active form (rare)
Excessive loss of urinary phosphate (rare): onco-
genic osteomalacia (seen in malignant disease), X-
Figure 13 X-ray changes in osteomalacia. A partial fracture in
linked hypophosphataemia, renal tubular acidosis and
the femur, known as a Looser zone or pseudofracture, can be
drugs (diuretics)
seen in subjects with osteomalacia.
Decreased phosphate availability: starvation, malnu-
trition
(alcoholism in the UK is one cause) and
malabsorption
PTH: usually elevated (except for some rare causes of
Associated with high phosphate
osteomalacia)
Renal failure
Vitamin D levels
The vast majority of patients will have osteomalacia/
Bone X-ray
rickets due to vitamin D deficiency with or without renal
Pseudofractures or Looser zones: these are patho-
disease, this is what you need to remember.
gnomic of osteomalacia (see Fig. 13)
Widening of growth plates and bone deformity: seen
Clinical presentation
in rickets
Symptoms and signs of osteomalacia/rickets are sum-
Table
13 summarizes the biochemical findings in
marized in Table 12.
common causes of osteomalacia/rickets.
Investigations
Treatment
Calcium: low or low-normal
Vitamin D replacement
Phosphate: usually low, except for osteomalacia due to
Calciferol can be given orally often with calcium
renal failure
supplementation
Bone and calcium metabolism
27
Table 13 Biochemical findings in osteomalacia/rickets caused by vitamin D deficiency and renal failure. It is important to
remember that longstanding renal failure can be associated with hypercalcaemia (rather than hypocalcaemia) due to tertiary
hyperparathyroidism.
Calcium
Phosphate
Alkaline phosphatase
25-OH-Vit D
PTH
Vit D deficiency (dietary, malabsorption)
Renal failure
N
In renal failure
1-OH vitamin D (1-α-calciferol)
Hyperthyroidism
should be given as the kidneys are unable to convert
Growth hormone deficiency
25-OH vitamin D to the active 1, 25-OH vitamin D
Inflammatory conditions
Treat the underlying cause
Rheumatoid arthritis
Neoplastic disease
Osteoporosis
Multiple myeloma
This is a very common condition, which is due to a
Clinical presentation
reduction in bone mass
The disease is usually silent until the occurrence of frac-
tures (it does not cause skeletal pain).
Box 7 Loss of bone mass and osteoporosis
Low trauma fractures are a common presentation
Vertebral fractures are also common resulting in:
From the age of 40, there is a gradual loss in bone
Back pain
(usually sudden onset and well
mass (around 0.5% annually)
Due to the protective effects of sex hormones,
localized)
osteoporosis is common in women after the menopause
Loss of height (this explains why older individuals
and is called postmenopausal osteoporosis
shrink in size!)
In women, bone mass loss after the menopause is
Kyphosis
accelerated, which explains the higher rate of
Spinal cord compression in severe cases
osteoporotic fractures in older women compared with
men
Investigations
Measure bone mineral density (BMD) using dual energy
Causes include:
X-ray absorptiometry (known as DEXA scan):
Gonadal failure
Establish the cause
Women: premature menopause
(physiological
Box 8 Bone mineral density (BMD)
menopause in the older age group) and any cause of
amenorrhoea
BMD > −1: normal
Men: Kleinfelter’s syndrome and acquired
BMD < −1 but > −2.5: osteopenia
hypogonadism
BMD < −2.5: osteoporosis
Drugs (long-term use)
Steroids
Full blood count (FBC), ESR (rule out inflammatory
Heparin
condition)
Anticonvulsants
Calcium levels
Gastrointestinal and nutritional
Renal function
Malabsorption due to any cause (for example coeliac
Thyroid function
disease)
Testosterone levels in men
Malnutrition (excessive alcohol, anorexia nervosa)
Oestradiol and FSH/LH levels in women with early
Endocrine disease
menopause
Hyperparathyroidism
X-rays to rule out fractures. Back X-rays (see Fig. 14)
Cushing’s syndrome
are requested to rule out vertebral crush fractures in
28
Part 1: Basics
Figure 15 Typical “frontal bossing” and bowing of the bones.
Paget’s disease
The main abnormality in Paget’s disease is overactivity
of the osteoblasts resulting in bone resorption. This in
turn stimulates osteoblast function resulting in the for-
mation of new bone. Bone resorption and formation is
Figure 14 The x-ray shows a crush fracture of the vertebral
disorganized, consequently leading to bony deformities.
body (can be seen as shorter vertebral body with increased
bone density). In the presence of a crush fracture, DEXA scan
Clinical presentation
may be inaccurate in measuring vertebral bone density.
Paget’s disease mainly affects the skull, vertebrae, pelvis,
those with: significant loss of height and/or sudden-onset
femur and tibia. Therefore, the patient presents with:
back pain.
Bone deformities (skull abnormalities, bowing of tibia)
More tests may be required to exclude specific
(see Fig. 15)
conditions
Bone pain
Fractures
Treatment
Complications
Hormone replacement therapy (HRT)
Nerve compression (may result in deafness)
Postmenopausal women
Spinal cord compression (may result in paralysis)
Women with gonadal failure
Sarcomatous transformation (osteogenic sarcoma):
Testosterone replacement
fortunately, a rare event
Men with gonadal failure
It should be noted that the vast majority of patients
Calcium and vitamin D
(up to 90%) are asymptomatic and the disease is diag-
Bisphosphonates: remain the mainstay of osteoporosis
nosed during routine laboratory investigations (raised
treatment usually in combination with calcium and
alkaline phosphatase).
vitamin D
Strontium can be effective but makes BMD measure-
Investigations
ments unreliable (the drug is incorporated into the bone
Alkaline phosphatase: usually elevated and can reach
structure)
very high levels
Calcitonin can be effective in those with vertebral
Bone X-ray:
crush fractures as it partially relieves the pain
Lytic (bone resorption) lesions
Treatment is monitored
Sclerotic (new bone formation) lesions (Fig. 16)
Clinically: no further fractures/loss of height
Bone isotope scan: helpful to fully assess the extent of
Repeat BMD 1-2 years after starting the treatment
bony involvement (Fig. 17)
Bone and calcium metabolism
29
Figure 16 Typical lytic and sclerotic lesions in Paget’s disease
of the pelvis.
Treatment
High dose oral bisphosphonate is the mainstay of
(a)
(b)
treatment
Figure 17 An isotope bone scan showing an increased uptake
Indications for treatment include:
in the left femur and tibia, and right elbow and (b) vertebral
Bone pain
body. Courtesy of Dr R. Bury, Radionuclide Department,
Neurological complications
University of Leeds.
Fractures
Hypercalcaemia (which is a rare complication of
Paget’s disease)
Main abnormality is in bone architecture, resulting in:
Monitoring therapy
Severe osteoporosis and easy fractures: these indi-
Clinical improvement
viduals usually develop multiple fractures during their
Alkaline phosphatase levels: these usually fall with
lifetime
successful treatment
Associated abnormalities include: blue sclera, abnor-
malities in teeth and hearing loss
Osteogenesis imperfecta
A familial disease that can be inherited as autosomal
Treatment
dominant (AD) or autosomal recessive (AR)
Bisphosphonates can partly help by:
Several mutations are recognized leading to different
Increasing bone mass
clinical presentations
Reducing the incidence of fractures
The adrenal glands
Anatomy
Adrenal disorders with no hormonal abnormality
The adrenal glands are situated above the kidneys
Adrenal tumours
(Fig. 18)
The aorta and renal arteries provide the arterial blood
Investigation for adrenal dysfunction
supply to the adrenal glands, whereas venous drainage
Hormonal tests
usually occurs into the inferior vena cava on the right and
Static tests
left renal vein on the left
Plasma renin activity, aldosterone, androgens
The adrenal glands are composed of:
(DHEA, testosterone), urinary catecholamines
Adrenal cortex, which represents 90% of the gland
Suppressive tests
and produces corticosteroids (cortisol), mineralocor-
Dexamethasone suppression test
ticoid
(aldosterone) and androgens
[dehydroepi-
Stimulating tests
androsterone (DHEA) and androstenedione]
Synacthen test
Adrenal medulla, which represents
10% of the
adrenal gland and produces catecholamines (adrena-
Imaging tests
line, noradrenaline and dopamine)
CT or MRI: can detect lesions >5 mm in diameter
The adrenal gland is under the control of:
Ultrasound: can detect lesions >20 mm in diameter
Pituitary adrenocorticotrophic hormone
(ACTH)
but can be technically difficult
(steroid and androgen production)
Venous sampling: can be used to investigate hormonal
Renal renin (aldosterone production)
production by the glands, particularly in the presence
of pathology in both adrenals (helps to differentiate
Pathophysiology of the adrenal glands
unilateral from bilateral disease)
Disorders of the adrenal glands include:
Full details of the above tests will be given under each
Excess production of adrenal hormones, which result
disease entity (described below).
in different clinical entities:
Glucocorticoids: Cushing’s syndrome
Overproduction of adrenal hormones
Mineralocorticoids: hyperaldosteronism
Glucocorticoid excess
Androgens: adrenal tumours, congenital adrenal
Excessive secretion of glucocorticoid by an adrenal
hyperplasia
adenoma results in Cushing’s syndrome with similar
Catecholamines: pheochromocytoma
symptoms to ACTH-dependent Cushing’s syndrome,
Hormonal undersecretion (adrenal failure)
except for the absence of pigmentation (because ACTH
Glucocorticoid and mineralocorticoid: Addison’s
levels are suppressed). Causes of Cushing’s syndrome are
disease
summarized in Table 14.
Lack of androgen and catecholamine production
is
not believed to have significant clinical
Clinical presentation
manifestations
Onset is often insidious and symptoms may fluctuate.
Patient presents with:
Weight gain
Endocrinology and Diabetes: Clinical Cases Uncovered. By R. Ajjan.
Central obesity with thin extremities
Published 2009 by Blackwell Publishing, ISBN: 978-1-4051-5726-1
Acne and hirsutism
30
The adrenal glands
31
Cortex
Medulla
Red cheeks
Moon face
Fat pads
Cut-section of adrenal gland
Left adrenal
Right adrenal
gland
Bruisabillity
gland
with ecchymoses
Striae
Pendulous
Thin skin
abdomen
Proximal
myopathy
Right kidney
Left kidney
Figure 18 Anatomy of the adrenal glands.
Poor wound
healing
Table 14 Causes of ACTH-dependent and ACTH-independent
Cushing’s syndrome.
Figure 19 Clinical features of Cushing’s syndrome (see text for
ACTH-dependent
ACTH-independent Cushing’s
full explanation).
Cushing’s syndrome
syndrome
Investigations
Pituitary tumour secreting
Adrenal tumour secreting
Confirm the presence of excess cortisol
ACTH (Cushing’s disease)
glucocortocoid (ACTH levels are
Midnight cortisol: in normal individuals, cortisol
suppressed)
levels at midnight during sleep are undetectable. This
Ectopic ACTH production
Long-term steroid treatment
test may be difficult to arrange as the patient needs to
(malignant tumours)
(respiratory disease, connective
be admitted and a blood sample should be taken
tissue disease)
immediately after the patient is woken up at midnight
(the patient should not be warned about having this
test)
24-h urinary cortisol: high levels are suggestive of the
Easy bruising
diagnosis
Low libido and menstrual irregularities
Overnight dexamethasone suppression test: give
Growth arrest in children
0.5-1.0 mg of dexamethasone at 23:00 and measure
Signs include (see Fig. 19)
cortisol at 09:00. Cortisol levels less than 50 nmol/L
Facial appearance (moon-like face), with hirsutism
effectively rule out the diagnosis of Cushing’s
and thinning of scalp hair
syndrome
Central obesity and abdominal striae
Low-dose dexamethasone suppression test: give
Thin skin and evidence of bruising
0.5 mg dexamethasone ever 6 h for 2 days (eight doses)
Proximal muscle weakness
and check cortisol levels thereafter, which should be
Hypertension
<50 nmol/L in the absence of Cushing’s syndrome
Fractures (secondary to osteoporosis)
Differentiate between different causes of Cushing’s
Diabetes mellitus or impaired glucose tolerance
syndrome
Increased pigmentation does not occur in adrenal
ACTH levels: these are suppressed in adrenal
disease as ACTH levels are not increased (ACTH produc-
Cushing’s but detectable in pituitary Cushing’s disease
tion is suppressed due to negative feedback)
or cases due to ectopic ACTH production
32
Part 1: Basics
Table 15 Causes of increased aldosterone production.
Angiotensinogen
Renin
+
Renin-dependent
Renin-independent
(produced in
hyperaldosteronism (raised
hyperaldosteronism
the kidneys)
plasma renin activity)
(suppressed plasma renin
Angiotensin I
activity)
-
Renal hypoperfusion (renal
Aldosterone-producing adrenal
artery stenosis, severe heart
adenoma (Conn’s syndrome)
Angiotensin II
failure, cirrhosis)
and rarely carcinoma
Renin-producing tumour (rare) Bilateral adrenal hyperplasia
High-dose dexamethasone suppression test: give
BP
2 mg dexamethasone every 6 h for 2 days. If cortisol is
AG
suppressed to more than 50% of basal value, it suggests
a diagnosis of pituitary Cushing’s disease, ruling out
the possibility of adrenal Cushing’s syndrome
Imaging
Na+ absorption
CT or MRI of the adrenal: shows a mass in adrenal
K+ excretion
Aldosterone
(by renal tubules)
Cushing’s syndrome
-
Signs of malignant adrenal mass include: large size
(>6 cm), heterogeneity (calcification and necrosis) and
local invasion
Figure 20 Control of aldosterone secretion. Decreased
perfusion pressure in the kidneys results in renin secretion and
Treatment
the conversion of angiotensinogen to angiotensin I, and
Surgery for benign adrenal adenoma: prognosis is
subsequently to angiotensin II (ATII) by angiotensin converting
good
enzymes. ATII stimulates the adrenal glands to produce
Surgery followed by adrenolytic treatment (i.e. mito-
aldosterone, which increases sodium reabsorption and
tane) for adrenal carcinoma: prognosis is poor as only
potassium excretion by the renal tubules. Increased plasma
one in five individuals survive for 5 years
sodium results in water retention consequently raising the
blood pressure and switching off renin secretion (negative
Mineralocorticoid excess
feedback). Furthermore, low potassium has a negative
This results from the increased production of aldoste-
feedback effect on aldosterone secretion. AG, adrenal gland;
BP, blood pressure.
rone. Causes of increased aldosterone production are
summarized in Table 15 (see also Fig. 20).
Signs are moderately severe hypertension, particularly
Box 9 Primary hyperaldosteronism
in a younger individual
Primary hyperaldosteronism results in:
Investigations
Renal sodium retention
Hypertension with hypokalaemic alkalosis should be
Potassium and hydrogen ion loss
investigated for the possibility of hyperaldosteronism.
Consequently leading to hypertension, hypokalaemia
Raised aldosterone/renin ratio is highly suggestive of
and metabolic alkalosis
the diagnosis
Imaging
Clinical presentation
CT or MRI of the adrenals: differentiates between
Often asymptomatic and diagnosis is made during
adrenal adenoma and bilateral hyperplasia
investigation for hypertension
Adrenal vein sampling: reserved for difficult cases
Patients sometimes experience symptoms related to
hypokalaemia such as muscle weakness and myopathy,
Treatment
and polyuria
For adrenal adenoma (Conn’s syndrome), surgery is
The adrenal glands
33
the treatment of choice, which cures hypertension in
Surgery
two-thirds of cases
Drugs
For bilateral adrenal hyperplasia, the aldosterone
β-blocker (without previous α-blockade)
antagonist, spironolactone or potassium-sparing diure-
Anaesthetics
tics, amiloride or triamterene, are used
Opiates
Antidepressants
Catecholamine excess
Pheochromocytomas can be part of a syndrome as
Adrenal medullary tumours (pheochromocytomas) can
shown in Table 17.
produce adrenaline and noradrenaline resulting in
hypertension.
Investigations
24 h urine collection for catecholamines: due to the
Clinical presentation
episodic nature of the disease, plasma levels of catechol-
A pheochromocytoma should be suspected in:
amines can be normal and, therefore, measurement of
Severe or resistant hypertension
these hormones in the urine is more reliable (two to three
Hypertension in the young
collections are necessary)
Hypertension associated with the symptoms and signs
Plasma catecholamines: this can be useful if samples
summarized in Table 16
are collected during a crisis
Symptoms are episodic and similar to how you feel
Suppression tests (pentolinium or clonidine suppres-
when you are angry or before sitting an important
sion): rarely needed to make a diagnosis
exam.
Imaging
Patients may present with a hypertensive crisis, which
CT or MRI of adrenals: usually reveal a large adrenal
can be fatal. Factors precipitating a crisis include:
tumour
Exercise
Body MRI: if suspected extra-adrenal tumours
Pressure on the abdomen
(such as abdominal
Meta-iodobenzylguanidine (MIBG) scan: can detect
examination)
two-thirds of pheochromocytomas. Useful for investi-
gating extra-adrenal tumours
Table 16 Symptoms, signs and complications of
Adrenal venous sampling
pheochromocytomas.
Reserved for difficult cases
Presence of bilateral pathology
Symptoms
Signs
Complications
Sweating
Hypertension
Cardiomyopathy
Treatment
Surgery: removal of the tumour is curative
Pallor or flushing
Postural hypotension Heart failure
Patient should be prepared before surgery with ade-
Feeling of
Diabetes (rare)
quate α-blockade to avoid a hypertensive crisis, which
apprehension
may be fatal
Palpitations
Stroke
Medical treatment is not an option
The ‘rule of 10’ should be remembered when consider-
Throbbing headaches
ing pheochromocytomas:
Table 17 Syndrome associated with
MEN II
Von-Hippel Lindau
Neurofibromatosis
pheochromocytomas (MEN, multiple
endocrine neoplasia).
Hyperparathyroidism
Cerebellar and retinal
Multiple neurofibromas
haemangioblastomas
Thyroid carcinoma
Renal cell carcinoma
Café au lait spots
Pheochromocytoma (50%)
Pheochromocytoma
Pheochromocytoma (rare)
(around 20%)
34
Part 1: Basics
10% are malignant
Clinical presentation
10% are extra-adrenal (arising in the sympathetic or
In glucocorticoid deficiency, presentation is similar to
parasympathetic chain)
that described under pituitary failure except for the pres-
10% are familial (in which case screening should be
ence of pigmentation (secondary to high ACTH) and this
preformed)
can be seen in:
Palmar creases
Adrenal failure
Scar tissue
Primary adrenal failure results in glucocorticoid and
Buccal mucosa
mineralocorticoid deficiency
(commonly described as
Aldosterone deficiency, resulting in:
Addison’s disease)
Postural hypotension
Secondary adrenal failure results in glucocorticoid
Hyponatraemia
deficiency only
Hyperkalaemia
Causes of adrenal failure are summarized in Table 18.
Metabolic acidosis
Isolated aldosterone deficiency may be secondary to
Table 18 Causes of adrenal failure.
impaired renin secretion (hyporeninaemic hypoaldoste-
ronism). This is also known as renal tubular acidosis type
Primary adrenal
Secondary adrenal
IV, which can be seen in renal disease (such as diabetic
insufficiency
insufficiency
nephropathy), and it is a condition that is probably
underdiagnosed.
Autoimmune (main cause in
Autoimmune disease:
the Western world)
autoimmune hypophysitis,
Investigations
isolated ACTH deficiency
Disease should be suspected in the presence of:
Infiltrative disease:
Infiltrative disease: sarcoidosis,
Hypotension
haemochromatosis,
histiocytosis,
Hyponatraemia
amyloidosis
haemochromatosis
Hypokalaemic acidosis
Infections: tuberculosis, fungal
Infections: tuberculosis,
Random serum cortisol
infections, opportunistic
pituitary abscess
Undetectable cortisol is diagnostic of adrenal
infections (seen in patients
insufficiency
with AIDS)
ACTH stimulation test (commonly known as synac-
Vascular
Vascular
then test)
Failure of cortisol to rise after ACTH stimulation is
Haemorrhage (anticoagulant
Haemorrhage
therapy, meningococcal
diagnostic of adrenal insufficiency
septicaemia)
Renin and aldosterone
Aldosterone levels are low and renin levels are
Infarction
Infarction
elevated in primary adrenal insufficiency
Adrenoleucodystrophy: an
Radiotherapy
Establish the cause
inherited disease, associated
Adrenal autoantibodies
with quadriplegia
Adrenal imaging
Congenital adrenal
Head trauma
hyperplasia
Treatment
Emergency treatment is required in anyone with sus-
Drug induced
Drug induced
pected adrenal insufficiency.
Ketoconazole ( cortisol
Long-term steroid treatment
Acute treatment
synthesis)
results in suppression of
Intravenous hydrocortisone
ACTH production
Rifampicin ( cortisol
Intravenous fluid
metabolism)
Watch for hypoglycaemia and correct as necessary
Chronic treatment
Malignant disease with
adrenal metastasis
Oral hydrocortisone given in two to three daily doses
(to replace glucocorticoids)
The adrenal glands
35
Oral fludrocortisone
(to replace mineralo-
characterized by a deficiency of one of the enzymes
corticoids)
involved in cortisol biosynthesis (fully discussed in the
Treatment is monitored clinically and by measure-
reproductive section). The commonest is due to 21α-
ment of electrolytes and plasma renin levels
hydroxylase deficiency, which results in:
Patient education: all patients should be warned to
Failure of cortisol and aldosterone synthesis
double the dose of glucocorticoid in mild intercurrent
Increased production of
17-OH-progesterone and
illness and to give parenteral treatment in severe illness
testosterone
or prior to major surgery
The clinical spectrum is very wide and includes:
In severe cases (severe deficiency)
Adrenal tumours
Salt wasting in the neonatal period
(male or
Adrenal tumours can be picked up during routine inves-
female).
tigations for non-adrenal disease and these are often
Virilization of female fetus (affected female subjects
called ‘incidentalomas’.
are sometimes raised as males)
Less severe cases (mild deficiency), these are usually
Box 10 Investigation of adrenal tumours
clinically evident in females:
Hirsutism
Any adrenal tumour should be investigated for the
Acne
possibility of:
Menstrual irregularities
Glucocorticoid production (i.e. Cushing’s syndrome)
Infertility
Cortisol suppression tests
Mineralocorticoid production (i.e. Conn’s syndrome)
Investigations
High aldosterone/renin ratio
High levels of hormones upstream of the enzymatic
Catecholamine production (i.e. pheochromocytoma)
defect (elevated 17-OH-progesterone levels).
High catecholamines on 24-h urine collection
Androgen production (i.e. androgen-secreting tumours)
High plasma androgen levels
Treatment
Cortisol replacement to suppress ACTH production,
Adrenal tumours may be non-functional
(do not
thereby limiting androgen production. Response to
produce any hormones) and these can be simply followed
therapy is monitored:
up by repeated scanning. Adrenalectomy is advised for:
Clinically
Large tumours (>4 cm)
By assessing
17-OH-progesterone levels
(aim to
Tumours with fast growth
suppress to around twofold of normal)
(Large and fast growing tumours are more likely to be
Fludrocortisone in severe cases with salt wasting.
malignant.)
Response to therapy is monitored:
Clinically
Congenital adrenal hyperplasia (CAH)
Renin levels
(aim to suppress into the normal
Congenital adrenal hyperplasia is an inherited disease
range)
The reproductive system
Anatomy
The hypothalamus produces gonadotrophin releasing
The reproductive endocrine organs include the ovaries in
hormones (GnRH) in a pulsatile fashion
females and testes in males.
GnRH stimulate the pituitary to release follicle stimu-
The ovaries are situated in the pelvis on either side of
lating hormone (FSH) and luteinizing hormone (LH)
the uterus as shown in Fig. 21.
FSH results in:
During reproductive life, the ovaries contain follicles
Growth and maturation of ovarian follicles (which
(each containing an oocyte) at different stage of matura-
contain the oocyte)
tion embedded in the ovarian stroma
Stimulation of oestrogen production by follicular
In adults, the testes are found in the scrotum, except in
cells
a minority with testicular mal descent, in which case the
LH results in:
testicles can be in the inguinal canal
Ovulation (a surge in LH production is responsible
In an adult male the testicular size is 15-25 ml (Fig.
for ovulation)
22)
Maintenance of progesterone production by the
Testes are composed of:
corpus luteum
Interstitial or Leydig cells, which produce
Inhibin, secreted by the ovaries and under FSH control,
testosterone
has a negative feedback effect on FSH production
Seminiferous tubules made up of germ (producing
Oestradiol has a negative feedback effect on FSH pro-
sperm) and sertoli cells (producing inhibin)
duction but has a positive effect on LH surge (necessary
Full description of the female and male reproductive
for ovulation)
systems can be found in many other textbooks.
Physiology
Ovaries have two functions
Box 11 The menstrual cycle
Endocrine: production of oestrogen and
The menstrual cycle can be divided into:
progesterone
Follicular phase (day 6-13): around 20 follicles (each
Reproductive: storage and release of oocytes
containing an oocyte) grow under the influence of FSH
Testicles have two functions
and secrete oestradiol
Endocrine: production of testosterone
Ovulation (day 14): a surge in LH results in ovulation
Reproductive: production of sperms
(one oocyte is passed into the fallopian tubes)
This section will mainly concentrate on the endocrine
Luteal phase (day 15-25): after ovulation the corpus
function of these organs.
luteum forms from theca interna cells, which produce
progesterone
Physiology of the female reproductive system
Premenstrual phase (day 25-28): LH levels fall, and
Ovarian function is under the control of the
theca cells lose the ability to maintain adequate
progesterone production
hypothalamic-pituitary axis (Fig. 23)
Menstruation (day 1-6): low progesterone levels lead to
loss of endometrial support, which starts shedding and
Endocrinology and Diabetes: Clinical Cases Uncovered. By R. Ajjan.
menstruation takes place
Published 2009 by Blackwell Publishing, ISBN: 978-1-4051-5726-1
36
The reproductive system
37
Oviduct
Opening of
oviduct
Ovary
Endometrium
Cervix
Body of uterus
Myometrium
Vagina
Figure 21 Anatomy and histology of the ovaries. Relationship of ovaries to the uterus and fallopian tubes. From Holt, RIG &
Hanley, NA (2007) Essential Endocrinology and Diabetes, 5th edition. Blackwell Publishing, Oxford.
(a)
(b)
Vas deferens
Lumen of
seminiferous tubule
Epididymis
Capillary
Septum
Leydig
cell
Sertoli
cell
Basement
Developing
Lobule: seminiferous
membrane
spermatocytes
Rete testis
tubules + Leydig cells
Figure 22 Anatomy and histology of the testes. (a) Testes are usually found within the scrotum. (b) Histology of the testes showing
the seminiferous tubules, within which sertoli cells can be found. Leydig cells are found in the interstitial space. From Holt, RIG &
Hanley, NA (2007) Essential Endocrinology and Diabetes, 5th edition. Blackwell Publishing, Oxford.
38
Part 1: Basics
Hypothlamus
Hypothlamus
-
-/+
GnRH
(pulsatile)
GnRH
+
(pulsatile)
+
Pituitary
-
-
Pituitary
-
-/+
FSH and LH
+
Testis
FSH and LH
+
Ovaries
Inhibin
Testosterone
(Sertoli cells)
(Leydig cells)
Figure 24 The hypothalamic-pituitary-testicular axis.
Gonadotrophin (GnRH) secreted in pulsatile fashion, stimulates
Inhibin
Oestradiol
FSH and LH production by the pituitary. FSH is responsible for
sperm maturation and also controls inhibin production. LH
stimulates testosterone secretion by Leydig cells. Inhibin has a
Progesteron
negative feedback effect on FSH production, whereas
testosterone mainly suppresses LH production.
Figure 23 The hypothalamic-pituitary-ovarian axis.
Gonadotrophin (GnRH), secreted in pulsatile fashion, stimulate
follicle stimulating hormone (FSH) and luteinizing hormone
Hypothalamic GnRH, secreted in pulses, regulate FSH
(LH) production by the pituitary. FSH results in follicular growth
and LH secretion by the pituitary
and maturation in the ovary associated with oestradiol
FSH is important for sperm maturation
production. LH results in ovulation and subsequently maintains
LH is important for testosterone production by Leydig
progesterone production by corpus luteum theca cells. Inhibin
cells
secretion by the ovaries, stimulated by FSH, has a negative
Inhibin, produced by sertoli testicular cells and under
feedback effect on pituitary FSH production. Oestradiol has a
FSH control, has a negative feedback effect on FSH
negative feedback effect on FSH production but it facilitates
production
the LH surge necessary for ovulation.
Testosterone has a negative feedback effect, which
Effects of hormonal changes on the uterine endome-
mainly affects LH production
trium include:
Repair and proliferation of the endometrium
Pathophysiology of the endocrine
(oestradiol)
reproductive system
Increase endometrial thickness and preparation for
Abnormalities of the female reproductive system:
implantation (progesterone)
Menstrual abnormalities
Premature ovarian failure
Physiology of the male reproductive system
Polycystic ovary syndrome (PCOS)
Similarly to the ovaries, testicular function is under the
Congenital adrenal hyperplasia
control of the hypothalamic-pituitary axis (Fig. 24)
Virilizing tumours
The reproductive system
39
Infertility
Rule out physiological causes
Abnormalities of the male reproductive system:
Establish growth and development of the child
Hypogonadism
(particularly in primary amenorrhoea)
Gynaecomastia
Details of previous menstrual cycle (if any)
Testicular tumours
Any recent stress/weight loss
Infertility
Past or present illness
History of radiation or chemotherapy
Menstrual abnormalities
Previous pelvic operation or pelvic inflammatory
This topic is covered in detail in other textbooks and is
disease
only briefly discussed here. Menstrual abnormalities can
Review drug history as some medications can cause
be divided into:
amenorrhoea, e.g. previous use of oral contraceptive
pills
Physiological
Presence of galactorrhoea
Prepubertal
Pregnancy
Investigations
Lactation
Only hormonal investigations will be listed here. These
Menopause
include plasma levels of oestradiol, FSH, LH and prolac-
tin and sex hormone binding globulin (SHBG).
Pathological
Low ovarian hormones and raised FSH/LH indicates
Primary amenorrhoea: the failure to reach menarche by
primary ovarian failure
(menopause or premature
the age of 16. This may be due to:
ovarian failure)
Structural abnormality (such as imperforated hymen,
Low ovarian hormones with low FSH/LH indicates
congenital absence of the uterus)
pituitary or hypothalamic disease
Genetic disorders (such as Turner’s syndrome)
Prolactin should always be checked as raised levels
Testicular feminization syndrome: the individual is
result in suppression of GnRH production and sub-
genetically a male (XY chromosome) but phenotypically
sequent menstrual irregularities (see pituitary section)
a female due to tissue insensitivity to androgens
Thyroid and/or adrenal abnormalities should be
Causes of secondary amenorrhoea (see below)
excluded in suspicious cases
Secondary amenorrhoea: the cessation of menstrual
Karyotype in suspected conditions: Turner’s syndrome
periods in women who had previously menstruated.
(XO), testicular feminizing syndrome (XY)
Causes include:
Ovarian, e.g. polycystic ovary disease, or premature
Treatment
ovarian failure due to a chromosomal abnormality
Treat the cause
(Turner’s syndrome), autoimmune disease or iatrogenic
Hormone replacement therapy should be considered
cause
(chemo- or radiotherapy, e.g. after cancer
in those with irreversible disease
treatment)
Uterine or fallopian tubes, e.g. adhesion in the uterus/
Premature ovarian failure
fallopian tubes or uterine tumours
This is defined as the development of menopause (low
Pituitary, e.g. hypopituitarism or prolactinoma
oestrogen and raised gonadotrophins) before the age of
Hypothalamic, e.g. excessive exercise (such as profes-
40. Causes include:
sional athletes), severe weight loss, stress (physical or
Chromosomal abnormalities: Turner’s syndrome
psychological), hypothalamic tumours or infiltrative
The commonest X chromosome abnormality in
lesions
females affecting 1 in 2500
General endocrine, these are usually associated with
There is a complete or partial absence of one X
menstrual irregularities rather than amenorrhoea and
chromosome
include thyroid dysfunction and Cushing’s syndrome
Characteristic phenotype
(Fig.
25, colour plate
Clinical presentation
section) comprises short stature, webbed neck, widely
The patient presents with amenorrhoea or menstrual
spaced nipples and poor breast development, cubitus
irregularities. It is important to establish in the history:
valgus and shortened metacarpals
40
Part 1: Basics
Associated clinical abnormalities include aortic
Clinical presentation
coarctation, left-sided heart defects, hypothyroidism
Hirsutism
and lymphoedema
Face, abdomen, back and extremities
Autoimmune disease of the ovary
95% of women presenting to the outpatient clinic
Can be associated with other organ-specific autoim-
with hirsutism have PCOS
mune disease (thyroid, type 1 diabetes, etc.)
Symptoms often begin around puberty
Iatrogenic
Oligo- or amenorrhoea: secondary to unovulation
Chemotherapy
Obesity: the majority of these patients are overweight
Radiotherapy
Complications
Infections
Infertility
Human immunodeficiency virus (HIV)
Endometrial carcinoma: absence of menstruation
Mumps
and regular shedding of the endometrium predisposes
to endometrial carcinoma
Clinical presentation
Investigations
Subjects present before the age of 40 with oligo- or
Plasma levels of the following hormones should be
amenorrhoea
checked:
Symptoms of oestrogen deficiency
Testosterone: usually raised
Hot flushes
Sex hormone binding globulin: usually low
Mood swings
LH/FSH ratio: raised in two-thirds of patients
Fatigue
Imaging
Dyspareunia
Ultrasound of the ovaries shows multiple cysts in the
stroma in the majority of subjects (Fig. 26)
Investigations
Measurement of endometrial thickness is also useful,
The combination of low oestradiol and high FSH/LH
which demonstrates endometrial hyperplasia
confirms the diagnosis
Other tests: due to the association with insulin resis-
Tests should be undertaken to investigate the cause as
tance patients should be screened for:
appropriate
Diabetes
Karyotype
Lipid abnormalities
Pelvic imaging
Complications
Osteoporosis
Increased risk of cardiovascular disease
Treatment
Hormone replacement therapy
Polycystic ovary syndrome (PCOS)
A very common condition
The leading cause of hirsutism in women
Characterized by:
Insulin resistance
Hyperandrogenaemia and low sex hormone binding
globulin (SHBG) (hence, high levels of free testosterone)
Polycystic ovaries in the majority of cases
Figure 26 Ultrasound of the ovaries showing multiple cysts, a
Failure of ovulation
characteristic finding in polycystic ovary syndrome. Courtesy of
Most patients are overweight
Professor S. Atkin.
The reproductive system
41
Treatment
Anti-androgen agents
These are a number of different treatments, which are
Androgen receptor blockers such as spironolactone,
summarized in Table 19.
cyproterone acetate and flutamide (the latter rarely
Weight loss: this reduces insulin resistance and, conse-
used due to risk of hepatic toxicity)
quently, hyperandrogenaemia
5α-reductase inhibitors such as finasteride, which
Suppression of ovarian androgen production
blocks the conversion of testosterone to its more potent
Oral contraceptive pills suppress FSH/LH pro-
androgen, dehydrotestosterone
duction, and increase SHBG and thus reduce free
Insulin sensitizers
testosterone levels
Metformin: can be helpful in some patients and may
induce ovulation
Thiazolidinediones (glitazones): generally inferior to
Table 19 Summary of treatment options for polycystic ovary
metformin
syndrome.
Local cosmetic treatment for hirsutism
Agent
Mode of action
Epilators
Creams
Oral contraceptive pills
Suppression of ovarian stimulation
Electrolysis
Laser therapy
Increase in sex hormone binding
Fertility treatment (to induce ovulation)
globulin
Clomiphene citrate
Spironolactone
Anti-androgen
GnRH preparations
Cyproterone acetate
Laparoscopic ovarian surgery
Flutamide
Congenital adrenal hyperplasia
Finasteride
An autosomal recessive condition, which results in a
defect in one of the enzymes involved in the synthesis of
Metformin
Insulin sensitizers
steroid hormones (Fig. 27)
Thiazolidinedione
The commonest is 21α-hydroxylase (90%) and less
Clomiphene
Induction of ovulation
commonly 11β-hydoxylase (<10%). Other enzyme defi-
ciencies are rare
Gonadotrophins
(Fertility treatment)
Surgery
21α-hydroxylase deficiency (see Fig. 27 )
This leads to:
Epilators, electrolysis, laser
Cosmetic measures
Cortisol deficiency, which may cause an Addisonian
therapy
crisis in severe cases
Cholesterol
Pregnenolone
17OH- Pregnenolone
DHEA
Progesterone
17OH- Progesterone
Androstenedione
21-hydroxylase
21-hydroxylase
DOC
11-Deoxycortisol
Testosterone
11-hydroxylase
11-hydroxylase
Corticosterone
Cortisol
Figure 27 Synthesis of adrenal
hormones. DHEA,
dehydroepiandrosterone; DOC,
Aldosterone
deoxycorticosterone.
42
Part 1: Basics
Aldosterone deficiency, resulting in salt loss and
Elevated ACTH plasma levels (due to defective cortisol
hypotension
production)
Loss of negative feedback on the pituitary results in
Hormonal abnormalities can be minimal in non-
increased ACTH secretion and excessive production of
classical CAH and, therefore, ACTH stimulation test
17-OH-progesterone, subsequently resulting in increased
may be required to make the diagnosis (raised 17-OH-
testosterone production and virilization
progesterone after synacthen test)
Severity of the disease varies and depends on whether
the individual has complete or partial enzyme
Treatment
deficiency
Steroid treatment: this results in ACTH suppression
and subsequently reduction in adrenal androgen
11β-hydroxylase deficiency (see Fig. 27 )
production
This leads to:
Fludrocortisone may be required in those with high
Increased androgen production resulting in
renin levels
virilization
Bilateral adrenalectomy in subjects with severe
Increased corticosterone (DOC) accumulation result-
virilization
ing in hypertension
Virilizing tumours
Clinical presentation
These include tumours of the ovaries or adrenal glands,
Only the common 21α-hydroxylase deficiency will be
which can be benign or malignant.
discussed here, which can present in two forms:
Clinical presentation
Complete 21α-hydroxylase deficiency (classical CAH)
Adrenal or ovarian virilizing tumours should be sus-
Female
pected in:
Virilization
Rapid onset of symptoms and
signs of
Clitoromegaly
hyperandrogenism
Labial fusion
Virilization
(severe hyperandrogenism),
manifested
Renal salt wasting, resulting in hypotension
as:
Males
Clitoromegaly
Renal salt wasting
Increased muscle mass
Reduced fertility due to suppression of gonadotro-
Deepening of voice
phin secretion secondary to high adrenal testosterone
Frontal balding
production
Symptoms related to the tumour
Abdominal mass
Partial 21α-hydroxylase deficiency
Ascites
(non-classical CAH)
Female
Investigations
Hirsutism
Testosterone levels are usually very high
Acne
Imaging of adrenal/pelvis
Menstrual abnormalities
Ultrasound
Infertility
CT or MRI scan
Male
Usually asymptomatic
Treatment
Investigations
Surgery
Raised testosterone plasma levels
Prognosis
Raised 17-OH-progesterone plasma levels
In benign disease: excellent
Elevated renin plasma levels (due to defective produc-
In malignant disease: generally poor with 20-40%
tion of aldosterone)
5-year survival
The reproductive system
43
Male hypogonadism
Tiredness
Defined as the failure of the testes to produce spermato-
Decreased facial and/or body hair
zoa and/or testosterone. Causes of male hypogonadism
Reduced libido
include:
The following can be found on examination:
Increased height: low testosterone in childhood
Primary
results in delayed closure of epiphysis
Kleinefelter’s
syndrome
(hypergonadotrophic
Decreased facial and/or body hair
hypogonadism)
Small penis (usually in prepubertal disease)
Occurs in 1 : 500 births
Small testes
Sex chromosome abnormality (47 chromosomes
Anosmia
XXY)
Gynaecomastia (discussed below)
Individuals are usually tall
Evidence of chronic disease (liver, renal, etc.)
Gynecomastia is common with increased risk of
It should be noted that clinical presentation depends
breast cancer
on:
Intellectual dysfunction is found in around half the
Severity (partial or complete)
individuals
Age of onset
Crytorchidism: failure of testes to migrate into the
Duration of sex hormone deficiency
scrotum (undescended testes)
Orchitis (testicular inflammation)
Investigations
Previous chemotherapy or radiotherapy
Testosterone, FSH, LH and prolactin should be
Testicular trauma
checked
Alcohol excess
Low testosterone with elevated FSH and LH indicates
Chronic illness
primary hypogonadism
Low testosterone with low/normal FSH and LH indi-
Secondary
cates secondary hypogonadism
Hypothalamic
Normal levels of testosterone/FSH/LH and prolactin
Hypothalamic tumours or infiltrative disease
usually rule out hypogonadism secondary to endocrine
Kallman’s syndrome: a genetic disease characterized
abnormality
by hypogonadotrophic hypogonadism and anosmia
Sex hormone binding globulins are usually requested
(impaired sense of smell) in the majority
to enable calculation of free testosterone index, which
Idiopathic hypogonadotrophic hypogonadism:
gives a more accurate measurement of functional or
similar in presentation to Kallman’s syndrome but
active testosterone hormone levels
sense of smell is intact
Severe exercise
Treatment
Severe weight loss
Treat the cause
Stress (physical or emotional)
In cases of irreversible disease:
Systemic illness
Testosterone replacement: transdermally, testoster-
Pituitary
one (as a gel) can be applied to the skin once a day;
Tumours
or by injections, these can be given every few
Infarct
weeks and with the newer preparations every few
Infiltrative disease
months
In cases of secondary irreversible gonadal failure,
Clinical presentation
treatment with GnRH is a possibility in order to restore
Failure of progression through puberty
fertility
Infertility
Sexual dysfunction (unable to maintain or absence
of erection)
Gynaecomastia
Symptoms related to testosterone deficiency
Enlargement of the male breast is a relatively common
Hot flushes
condition. Causes include:
44
Part 1: Basics
Physiological
Treatment
Puberty
Treat the underlying cause
Familial
Surgical treatment in severe cases
(reduction
Drugs
mammoplasty)
Digoxin
Oestrogens
Infertility
Spironolactone
Failure of pregnancy after 1 year of regular unprotected
Opiates
sex is defined as infertility. This can be very complicated
Antipsychotics
to investigate.
Heroin
Alcohol
Hypogonadism (any cause, see above)
Oestrogen or androgen producing tumours
Box 12 Causes of infertility
Chronic illness
Causes of female infertility
Liver cirrhosis
Hypothalamic abnormality
Renal failure
Hypothalamic amenorrhoea is commonly found in
Breast cancer
young athletes who undergo rigorous exercise
Kallman’s syndrome
Clinical presentation
Pituitary abnormality affecting gonadotrophin secretion
Pituitary tumours
Patient presents with breast enlargement. Particular
Infiltrative disease
care should be taken in:
Primary gonadal failure/abnormality
Rapid growth
PCOS
Associated pain or tenderness
Turner’s syndrome
Hypogonadal symptoms (see above)
Primary ovarian failure
Drug history is essential
(including recreational
Chemotherapy or radiotherapy
drugs)
Tubular lesions
Examination
Adhesions due to previous infections
Palpate the breast: universal enlargement, lump,
Endometriosis
look for galactorrhoea
Uterine abnormalities
Palpate the testicles: rule out tumour and measure
Congenital abnormalities
Adhesions due to previous infection
testicular size
Fibroids
Look for evidence of systemic illness
Systemic debilitating disease
Causes of male infertility include:
Investigations
Hypothalamic disease
Blood tests
Pituitary disease
Testosterone
Testicular abnormality
Oestradiol
FSH/LH
Prolactin
Human chorionic gonadotrophin (hCG, raised in
Investigations
some malignant tumours)
This is done in specialized centres. Briefly:
Liver, renal and thyroid function
Imaging
Male partner
Breast mammography if tumour is suspected
Semen analysis
Testicular ultrasound
Normal: investigate female partner
CT abdomen if adrenal lesion is suspected
Abnormal
Biopsy
Endocrine tests: normal testosterone, FSH, LH and
Breast tissue biopsy if tumour is suspected
prolactin usually rules out an endocrine cause; low
The reproductive system
45
testosterone with high LH indicates primary gonadal
Precocious puberty
failure; low testosterone with low/normal LH suggests
Onset of puberty before the age of 8 in girls and 9 in
secondary hypogonadism
boys. Causes include:
karyotyping in suspected Kleinfelter’s syndrome
Familial or idiopathic
Testicular ultrasound may show features of inflam-
Intracranial tumours
mation or testicular tumour
Rare genetic defects resulting in sex hormone pro-
duction independent of central control
Female partner
Endocrine tests
Delayed puberty
Normal testosterone, oestradiol, FSH, LH, TFTs and
Failure to progress to puberty after the age of 14 in girls
prolactin makes an endocrine cause for the infertility
and 16 in boys. Causes include:
unlikely
Constitutional delay: fortunately the commonest
Abnormalities with the above hormones should be
cause
investigated as discussed earlier
Chronic disease during the childhood period
Investigate for structural tubular and uterine
Hypergonadotrophic hypogonadism: Kleinfelter’s
abnormalities
syndrome and Turner’s syndrome
Hypogonadotrophic hypogonadism: Kallman’s
Treatment
syndrome and disorders of the hypothalamus and
Treat the cause
pituitary
Ovulation induction
In some cases no cause for infertility is found and
Investigations
intrauterine insemination
(IUI) or IVF may be
Blood tests
considered
Testosterone
Oestradiol
Puberty
FSH/LH
Average age of onset in girls is 10-12 years and boys
Prolactin
12-13 years
Karyotype
Puberty involves breast development (the first sign),
Imaging of:
appearance of pubic hair and menarche in girls, whereas
Ovaries
in boys it includes testicular and penile enlargement,
Testes
appearance of pubic hair, voice change and increase in
Pituitary/hypothalamus
facial hair
Puberty has five different stages (Tanner’s stages) in
girls and boys with specific measures applied to the
Treatment
parameters outlined above
Treat the cause
The pancreas
Anatomy
Around 2-3 million individuals in the UK have diabe-
The pancreas is situated behind the posterior wall of
tes but only half of them are diagnosed with the disease;
the abdomen (Fig. 28)
in the rest the condition is clinically silent
It can be divided into:
Head: located within the duodenal curve
Classification of diabetes
Body connected to the head through a slight con-
Type 1 diabetes (5-15% of cases)
striction (neck)
This is an autoimmune condition, resulting in destruc-
Tail representing tapering of the body as it extends
tion of pancreatic β-cells
to the left approaching the gastric surface of the
Subjects are often young (children or young adults)
spleen
but the older age group can also be affected
The anterior surface of the pancreas is covered by the
Latent autoimmune diabetes of adults (LADA) is also
stomach, whereas the posterior surface is in contact with
due to autoimmune β-cell destruction but the process is
the large vessels
(aorta, inferior vena cava and renal
slower than classical T1DM and occurs in an older age
vessels)
group
Physiology
The pancreas has an:
Box 13 Cells responsible for the endocrine
Exocrine function: secretes hormones into the gastro-
function of the pancreas
intestinal system to help with food digestion
This function of the pancreas is discussed in the
β-cells, producing insulin:
gastroenterology book of this series
Main hormone that maintains glucose homeostasis
Endocrine function: the pancreas is the main organ
Secretion is triggered by high plasma glucose
Composed of two polypeptide chains linked by
that regulates blood glucose levels.
disulphide bridges
Derived from proinsulin, which is packaged in the
Pathophysiology
Golgi system of β-cells and transformed to the active
Defects in insulin secretion and action result in the
form, insulin, by cleavage of C peptide by protease
development of diabetes mellitus
enzymes. Endogenous insulin production is associated
Insulin deficiency is the pathophysiological abnormal-
with detectable C peptide levels in contrast to
ity in type 1 diabetes (T1DM)
administering exogenous insulin, which can be useful in
Insulin resistance with consequent β-cell dysfunction
differentiating criminal or self-harm cases of insulin
are the pathophysiological abnormalities in most cases of
administration (raised insulin with undetectable C
type 2 diabetes (T2DM)
peptide levels) from endogenous insulin production
(raised insulin and C peptide levels)
α-cells, producing glucagons (discussed in the
Diabetes mellitus
neuroendocrine section)
This common disease is characterized by raised blood
δ-cells, producing somatostatin (discussed in the
glucose
neuroendocrine section)
PP cells, producing pancreatic polypeptide (discussed in
Endocrinology and Diabetes: Clinical Cases Uncovered. By R. Ajjan.
the neuroendocrine section)
Published 2009 by Blackwell Publishing, ISBN: 978-1-4051-5726-1
46
The pancreas
47
Splenic artery
Mitochondrial mutations
Hepatic artery
Left
adrenal
Genetic defect in insulin action:
Bile duct
Aorta
gland
Right
Resulting in severe insulin resistance (very rare)
adrenal
Spleen
gland
Drug induced (<5%)
Glucocorticoid treatment: particularly in those receiv-
ing high dose of steroids
Thiazides
Pancreas
Gestational diabetes (<5%)
Right
Left
Diabetes that occurs during pregnancy
kidney
kidney
Inferior
Resolves spontaneously after giving birth
Duodenum
vena cava
Associated with increased risk of macrosomic (large)
Figure 28 Anatomy of the pancreas.
babies
Affected women are at high lifetime risk of developing
T2DM (up to one-third)
From the practical point of view, it is important to
Type 2 diabetes (75-85% of cases)
distinguish between type 1 and type 2 diabetes as failure
This is due to a combination of:
to initiate insulin in a type 1 diabetes patient may result
Insulin resistance (usually as a result of obesity)
in death. This is fully discussed below.
β-cell dysfunction: insulin resistance is compensated
for by an increase in insulin production by pancreatic β-
Clinical presentation
cells. Eventually, these cells get
‘worn out’ and their
There are a wide range of symptoms, including:
insulin production decreases to a level that is unable to
Polyuria secondary to osmotic diuresis
compensate for insulin resistance, leading to high plasma
Polydipsia or increased thirst
glucose and the development of diabetes
Visual disturbances: due to changes in the lens, sec-
Type 2 diabetes is on the increase, mainly due to the
ondary to high glucose levels
increased prevalence of obesity and sedentary lifestyle
Repeated skin infections
Vaginal candidiasis
(thrush) is common in female
Secondary causes of diabetes (<5%)
subjects
Pancreatic destruction
Tiredness
Pancreatitis
Weight loss (usually in type 1 diabetes)
Trauma
The patient may present with associated complications
Pancreatic cancer
such as:
Cystic fibrosis
Myocardial infarction
Haemachromatosis (infiltration of the pancreas with
Stroke
iron)
Renal disease
Endocrine disease
In type 2 diabetes, the disease is commonly clinically
Acromegaly
silent and is discovered during investigations for other
Cushing’s syndrome
pathologies
During assessment of patients, it is important to dif-
Genetic defects (<5%)
ferentiate T1DM from T2DM as management of these
Genetic defect in insulin secretion:
conditions is entirely different (see Table 20). The follow-
Maturity onset diabetes of the young (MODY)
ing should be taken into account:
An autosomal dominant condition
Detailed history: patients with T1DM present with
A number of different types have been described, the
short history of symptoms (days to weeks), in contrast to
commonest are due to mutation in genes for hepatic
individuals with T2DM (months to years)
nuclear factor (HNF)1α, known as MODY 3, and glu-
The presence of weight loss, particularly in a younger
cokinase, known as MODY 2
individual, suggests T1DM
48
Part 1: Basics
Table 20 Summary of the main features of type 1 diabetes (T1DM) and type 2 diabetes (T2DM).
T1DM
T2DM
Aetiology
Autoimmune
Insulin resistance and β-cell dysfunction
(β-cell destruction)
Peak age
12 years (can occur at any age)
60 years (increasingly seen at a young age due to obesity)
Prevalence
0.3%
Around 6%
Presentation Osmotic symptoms (days to weeks), weight loss, DKA
Osmotic symptoms (months to years), diabetic
complications
Patient usually slim
Patient usually obese
Treatment
Diet and insulin
Diet, exercise (weight loss), oral hypoglycaemic agents,
insulin later
DKA, diabetic ketoacidosis.
Individuals with T1DM tend to be thin (but not
Diabetic hyperosmolar non-ketotic hyperglycaemia
always) and those with T2DM tend to be overweight (but
(HONK)
not always)
A complication of T2DM, usually seen in the elderly
Although rare, causes of secondary diabetes should be
Characterized by high glucose levels and severe
kept in mind and appropriate investigations should be
dehydration
arranged if necessary
Acidosis is usually absent
The presence of a family history suggestive of an auto-
somal dominant condition, particularly in those with
Hypoglycaemia
diabetes at a young age, should alert to the possibility of
This is discussed under the treatment of diabetes
MODY
below
All patients
(particularly older subjects) should be
assessed for the presence of complications (macrovascu-
lar and microvascular complications, detailed below)
Chronic complications
Macrovascular (large vessel) complications
Complications of diabetes
Patients with diabetes are at high risk of cardiovascular
Acute complications
disease including:
Diabetic ketoacidosis (DKA)
Ischaemic heart disease (IHD): all newly diagnosed
DKA is due to the absence of insulin and, therefore, it
diabetes patients should be assessed for the possibility of
is mainly seen in patients with T1DM
IHD
Not uncommonly, DKA is the first presentation of
History: chest pain or shortness of breath on
T1DM
exertion
The absence of insulin results in switching from glucose
ECG: previous myocardial infarction, ischaemic
to fat metabolism (in order to provide energy), a pathway
changes
that is associated with accumulation of ketone bodies,
More sophisticated tests if in doubt (exercise test,
which cause metabolic acidosis
angiogram)
Therefore, ketoacidosis is characterized by the pres-
Cerebrovascular disease:
ence of:
History of weakness or slurred speech should alert
Acidosis (due to the accumulation of ketone bodies)
to the possibility of this diagnosis
Dehydration (due to osmotic diuresis and vomiting,
Any neurological signs will warrant further
see below)
investigations
The pancreas
49
Peripheral vascular disease
Random plasma glucose
History of pain in the legs on exertion
Levels above 11.0 mmol/L in the presence of symp-
Feel the foot pulses
toms confirm the diagnosis
Oral glucose tolerance test
Microvascular (small vessel) complications
This should be performed in unclear cases
These include retinopathy, nephropathy and
Subjects are given 75 g glucose and plasma glucose
neuropathy.
is assessed at 0 min and 120 min
Retinopathy: the following changes can be observed:
Individuals with 2-h glucose <7.8 mmol/L: diabetes
Background changes (minor changes): microaneu-
ruled out
rysms, small intraretinal haemorrhages
(dots) and
Individuals with 2-h glucose >11.1 mmol/L: diabetes
hard exudates due to the leakage of lipids
is confirmed
Preproliferative changes
(serious changes, need
Individuals with
2-h glucose
>7.8 and
<11.1:
attention): soft exudates
(areas of infarction), also
impaired glucose tolerance is present (a prediabetic
known as cotton wool spots, and intraretinal micro-
condition-risk of future diabetes is high)
vascular abnormalities (IRMA), tortuous and dilated
2. Differentiate between types of diabetes
looking vessels occurring as a result of retinal
A careful history is probably the most important tool
ischemia
to differentiate between different types of diabetes
Proliferative changes
(very serious changes, need
immediate attention): new vessel formation
Nephropathy
Microalbuminuria: excretion of small amounts of
Box 14 How to assess a newly diagnosed
albumin in the urine. This is an early stage of diabetic
diabetes patient
nephropathy, which can be reversible
Take a careful history
Macroalbuminuria: excretion of large amounts of
Onset of symptoms: sudden (days/weeks) or gradual
albumin in the urine. This is seen in more advanced
(months or years)
stages
Any history of weight loss
Raised urea and creatinine: indicates renal
Any family history
failure
Autoimmunity: a personal or family history of
Neuropathy
autoimmunity (thyroid disease, vitiligo, coeliac disease,
Peripheral: altered sensation in the feet, which pre-
etc.), should raise the possibility of T1DM
disposes to foot ulcers; Charcot’s osteoarthropathy,
Diabetes at a young age in an autosomal dominant
results in bone fractures and deformity and can be
fashion: should raise the possibility of MODY
difficult to diagnose and treat (Fig. 29, colour plate
Check weight and body mass index (BMI): overweight
subjects are more likely to have T2DM
section); and neuropathy can also involve a main nerve
Look for signs of secondary diabetes (Cushing’s
or a group of nerves (third nerve palsy for example),
syndrome, acromegaly, etc.)
causing sensory or motor abnormalities
Check for the presence of complications (particularly in
Autonomic: can result in orthostatic hypotension,
those with suspected T2DM)
gastrointestinal symptoms (vomiting, diarrhoea), or
Macrovascular: chest pain or breathlessness on
erectile dysfunction
exertion; history of cerebrovascular accident (CVA) or
transient ischaemic attacks (TIA) (slurred speech, limb
Investigations
weakness); history of leg pain after exercise is suggestive
1. Confirm the diagnosis of diabetes:
of peripheral vascular disease; need to do a thorough
Fasting plasma glucose
cardiovascular examination
Levels above 7.0 mmol/L in the presence of symp-
Microvascular: history of foot ulcers/swelling of the
joints (examine the feet and check sensation using
toms or two tests above 7.0 mmol/L in the absence of
monofilament test); history of visual abnormalities
symptoms confirm the diagnosis of diabetes
(examine the fundi for diabetic retinopathy); renal
Subjects with fasting glucose >6.0 but 7.0 mmol/L are
disease is clinically silent in the early stages of the
labelled as having impaired fasting glucose and should
disease (check urine for microalbuminuria)
undergo a glucose tolerance test (see below)
50
Part 1: Basics
Urine dipstick: this is an essential test in all diabetes
patients. The presence of heavy ketonuria is indicative of
T1DM. Ketonuria may also occur after prolonged
fasting
Laboratory tests can be useful in difficult cases:
Anti-glutamic acid decarboxylase (GAD) and anti-
tyrosine phosphatase
(IA-2) antibodies: antibodies
0
2
4
6
8
10
12
14
16
18
20
22
24
against one or both molecules are present in around
Figure 30 Duration of insulin cover after actrapid injection (h).
80% of patients with T1DM. Their absence does not
rule out the diagnosis of T1DM
Genetic testing: in suspected MODY commonest are
mutations in HNF1α (MODY 3) and glucokinase
(MODY 2) genes
Cases with suspected secondary cause: ferritin levels
(haemachromatosis), CT abdomen in pancreatic
cancer, Cushing’s syndrome and acromegaly
3. Investigate for the presence of complications
0
2
4
6
8
10
12
14
16
18
20
22
24
Acute
Figure 31 Duration of insulin cover after insulatard injection
If in doubt whether the patient has early DKA,
(h).
you can measure venous pH and bicarbonate (bicar-
bonate
<15 mmol/L with or without low pH is
working in 30 min and peaks at 2-4 h after injection,
diagnostic)
covers up to 6-8 h post injection (Fig. 30)
Do not miss the diagnosis of acute diabetic ketoaci-
Intermediate acting insulin or NPH insulin (Insula-
dosis, which may be fatal if not treated appropriately
tard): starts working in 2 h and peaks 8 h post injec-
Chronic
tion, covers for 16-20 h (Fig. 31)
ECG: this should be done in all newly diagnosed
Mixtures: short and intermediate acting with varied
diabetes subjects as silent myocardial infarction is
proportions; humulin M1 (10% short and 90% inter-
common in this group of patients
mediate acting), humulin M3 (30% short and 70%
CT head and carotid Doppler: in case of history of
intermediate acting) (Fig. 32)
TIA
Analogue insulin preparations
Doppler of peripheral arteries: in case of history or
Ultra-short
(or ultra-fast) acting insulins
(lispro,
examination suggesting peripheral vascular disease
aspart, glulisine): start working almost immediately
Urinary microalbumin and U&Es to rule out
and peak at 1-2 h post injection and cover for around
nephropathy
4 h post injection
Nerve conduction tests: in the presence of atypical
Long acting insulins (glargine, detemir): relatively
neuropathic changes
flat profile (minimal peak, thus less chance of hypogly-
caemia), start working in 2 h and last 20-24 h post
injection
Treatment
Mixtures: ultra-short acting analogues with interme-
Treatment of type 1 diabetes (T1DM)
diate insulin. There are no mixtures with long acting
Patients with T1DM should be treated with insulin.
insulin analogues
There are different preparations of insulin, which are
briefly discussed here. At present, the main insulin prepa-
How to give insulin injections
rations in use are human insulin and insulin analogues.
There are a number of regimes that can be used in T1DM
Animal insulin preparations (bovine and pork) are very
but the most widely adopted are:
rarely used these days.
Twice daily injections with mixture of insulins (i.e.
Types of insulin:
Novomix 30, Humulin M3, Humalog mix 25) (Fig. 33)
Human insulin preparations
Four daily injections of insulin: also called basal bolus
Short
(or fast) acting insulin
(Actrapid): starts
regime (Fig. 34)
The pancreas
51
10/90 mix
20/80 mix
0
2
4
6
8
10
12
14
16
18
20
22
24
0
2
4
6
8
10
12
14
16
18
20
22
24
30/70 mix
40/60 mix
0
2
4
6
8
10
12
14
16
18
20
22
24
0
2
4
6
8
10
12
14
16
18
20
22
24
50/50 mix
0
2
4
6
8
10
12
14
16
18
20
22
24
Figure 32 Duration of insulin cover after injection of different insulin mixtures (h).
6
7
8
9
10
11
12
13
14
15
16
17
18
19
20
21
22
23
0
1
2
3
4
5
Hours
Breakfast
Lunch
Evening
Sleep
Figure 33 Twice daily insulin injection regime for T1DM.
One injection of intermediate or long acting insulin
In T2DM patients:
(to cover basal insulin)
Single injection of intermediate or long acting insulin
Three injections of short acting or ultra-short acting
can be added to existing oral hypoglycaemic agents
insulin with meals (bolus insulin)
Above regimes (same as T1DM) can also be used if one
Basal bolus regime gives better flexibility and has a
injection of insulin is not controlling plasma glucose
lower risk of hypoglycaemic episodes
levels
52
Part 1: Basics
Long acting
insulin
6
7
8
9
10
11
12
13
14
15
16
17
18
19
20
21
22
23
0
1
2
3
4
5
Hours
Breakfast
Lunch
Evening
Sleep
Figure 34 Four daily injections of insulin for T1DM.
How to start a newly diagnosed T1DM patient
Lipoatrophy and lipohypertrophy at insulin injec-
on insulin?
tion sites: the former is rarely seen now but the latter
Newly diagnosed patients can be started on two or four
can still occur (Fig. 35, colour plate section). Further
injections of insulin after appropriate education, which
injection into affected areas should be avoided
is best done by the diabetes nurse specialist.
Starting total 24-h dose of insulin is usually around
Other than daily injections, are there any other modes
0.3-0.6 unit/kg, divided into:
of delivery for insulin?
Half to two-thirds of the dose as intermediate/long
Insulin can be delivered by an insulin pump, using a
acting insulin
cannula placed in the abdomen (changed every 2-3
One-third to half the dose as short acting insulin
days)
A newly diagnosed patient weighing 70 kg can be
Insulin is continuously infused with bolus doses
started on:
given with meals
Mixture of insulin 12 units morning and 8 units
Inhaled insulin
evening
An insulin preparation, which can be given through
Basal bolus: 8-10 units of intermediate/long acting
inhalation. It only substitutes short acting insulin and
and 2-6 units of fast/ultra-fast acting with meals
the patient still needs to inject the long or intermediate
It is good practice to teach insulin-treated patients car-
acting insulin
bohydrate counting (assessment of carbohydrate in each
meal) to adjust the doses of insulin injections according
What to do with a T1DM patient during
to meal size
clinic reviews?
Assess diabetes control
What are the complications of insulin treatment?
Review blood glucose diary, paying particular atten-
Hypoglycaemia
tion to highs and troughs
All patients should be warned about the symptoms
Is there any particular pattern for the sugar readings?
of hypoglycaemia, which can be very unpleasant
For example, high fasting sugar indicates the need for
including: tremor, sweating, nausea and feeling
higher doses of intermediate or long acting insulin;
hungry
high post meal sugars indicate the need for higher
All patients should be properly educated to learn
short acting insulin before the meal
how to manage a hypoglycaemic episode (detailed
Check HbA1c, which gives an indication of the
below)
average diabetes control over the previous 6 weeks
The pancreas
53
Check for associated complications
in the UK. These agents lower blood glucose by stimu-
Cardiovascular (particularly in older patients); check
lating pancreatic insulin secretion. Side effects include
blood pressure in all patients
hypoglycaemia and weight gain
Nephropathy (check urine for albumin excretion)
Meglitinides: natiglinide and repaglinide are the
Retinopathy
(regular retinal examination or
most widely used agents. These increase insulin secre-
photography)
tion by the pancreas, an effect that is more pronounced
Foot examination: pulses and peripheral sensation
after a meal. They are less commonly associated with
hypoglycaemia and weight gain compared with sul-
phonylureas. In practice, they are often less effective
Treatment of type 2 diabetes (T2DM)
at
reducing glucose levels compared with
Education of patients is very important
sulphonylureas
The importance of diet and exercise should be
Insulin sensitizers
emphasized
Thiazolidinediones
(also known as glitazones),
All patients should be reviewed by a dietician for
pioglitazone and rosiglitazone, are stimulators
appropriate advice
of
the
peroxisome
proliferators
nuclear
Measures to induce weight loss (most of these patients
receptor
(PPAR)-γ, which results in improve-
are overweight) should be encouraged/implemented
ment in insulin resistance and decrease in blood
In the initial phases of the disease diet and exercise may
sugar
be enough to maintain good diabetes control
These agents have cardiovascular protective
Patients will eventually need medical intervention with
features
oral hypoglycaemic agents or insulin
Recent analysis, however, indicates that rosiglitazone
has a neutral effect on cardiovascular events, whereas
Oral hypoglycaemic agents
pioglitazone may reduce the risk
These have proliferated in the past decade or so and we
These agents can cause fluid retention and, there-
now have a number of different treatment options. These
fore, they are contraindicated in subjects with heart
include:
failure
Biguanides (main agent is metformin)
These agents induce weight gain
The first-line agent in obese T2DM patients
Drugs that interfere with glucose absorption
Metformin lowers blood glucose levels by: reducing
α-Glucosidase inhibitors (acarbose is perhaps the
hepatic glucose output (decrease in glycogenolysis),
most widely used agent in this group)
reducing glucose absorption and mildly reducing
Use of these agents in the UK is limited due to a
insulin resistance
modest blood glucose lowering effect and gastrointes-
Effective, cheap and safe
tinal side effects, mainly bloating, which are very
Use of this agent is associated with weight loss, which
common
is welcome in subjects with diabetes
Agents working on the glucagon-like peptide-1
Side effects are mainly gastrointestinal
(nausea,
(GLP-1) system
bloating) and these are minimized by a gradual
New agents introduced in the UK in 2007
increase in drug dose or use of long acting
GLP-1 is a natural hormone secreted by the gastro-
preparations
intestinal tract in response to meals
Contraindications include: renal failure - if creati-
GLP-1 stimulates insulin secretion by the pancreas
nine is >150 μmol/L, the drug should be discontinued
and inhibits glucagon production, thereby lowering
(or not started) due to fears of inducing lactic acidosis;
plasma glucose levels
advanced heart or liver failure - again there is a risk of
GLP-1 has a very short half-life as it is metabolized
inducing lactic acidosis. Use of metformin in mild
by dipeptidyl peptidase (DPP)-4 enzymes and quickly
heart failure or minor derangement of liver function is
cleared from the circulation
perfectly safe
GLP-1 analogues (exenatide)
Insulin secretagogues
Work similarly to native GLP-1 but are slowly
Sulphonylureas: gliclazide, glibenclamide and
metabolized by DPP-4 enzymes, resulting in a longer
glimepiride are probably the most widely used agents
half-life in the circulation
54
Part 1: Basics
Table 21 The main features of the new hypoglycaemic
Rimonabant is a new agent for weight reduction
agents, DPP-4 and glucagon-like peptide-1 (GLP-1) analogues.
licensed for use in the UK in 2007. It is a cannabinoid
receptor type 1 (CB1) blocker and works at multiple
DPP-4
GLP-1 analogues
levels (see Table 22). It has a role in the management
of multiple cardiometabolic factors, e.g. improvement
Administration
Oral
Injections
in lipid profile, improvement in glycaemic control and
Efficacy
++
+++
reduction in central obesity. Side effects include
depression in up to 15% of treated individuals, and,
Weight
Neutral
Weight loss
therefore, this agent is contraindicated in those with a
Side effects
Little
15-20% (gastrointestinal)
history of depression or during treatment with
antidepressants
The mode of action and contraindications of each of
the antidiabetic agents is summarized in Fig. 36 and
Table 22 Site and mechanism of action of the cannabinoid
Table 23.
receptor blocker, rimonabant.
Site
Mechanism
When do we need to move patients from oral
hypoglycaemic treatment to insulin?
Hypothalamus
Decreases appetite
Failure of oral hypoglycaemic agents to maintain ade-
quate glucose levels
(metformin is usually continued
Muscle
Increases glucose uptake
with insulin treatment)
Gastrointestinal tract
Increases satiety signals
Pregnancy: insulin is safe to use during pregnancy and,
therefore, pregnant women with diabetes are usually
treated with insulin only
Injected s.c. twice a day resulting in a reduction in
Severe illness or operation requiring hospital admis-
blood sugar levels and weight loss. Side effects includ-
sion: oral hypoglycaemic agents are temporarily stopped
ing gastrointestinal symptoms in relatively large
number of patients (around one-fifth), which may
Management of diabetic complications
improve with continued use of the drug
Acute complications
DPP-4 inhibitors (sitagliptine/vildagliptine)
Diabetic ketoacidosis
Inhibition of DPP-4 results in slower breakdown of
A medical emergency with a death rate of 3-5%
‘native’ GLP-1 and consequently an increase in plasma
Is due to the lack of insulin and subsequent switch
levels
from glucose to fatty acid metabolism, which results in
These agents are less effective at reducing blood
the production of ketone bodies:
glucose levels compared with GLP-1 analogues
Acetoacetic acid
DPP-4 inhibitors are weight neutral
Hydoxybutyric acid
They have advantages over GLP-1 analogues in that
Acetone
(giving DKA patients acetone-smelling
they are given orally (no injections are needed) and
breath)
side effects are minimal
Subjects with DKA have three fundamental
Table
21 summarizes the main characteristics of
abnormalities
GLP-1 analogues and DPP-4 inhibitors.
Metabolic acidosis, which causes abdominal pain
Slimming tablets
and vomiting, and compensatory hyperventilation
Orlistat inhibits intestinal lipase thereby reducing fat
(Kussmaul respiration): blowing off CO2 results in
absorption. It is important to comply with a low-fat
respiratory alkalosis, trying to compensate for the
diet whilst on treatment with this agent, otherwise it
metabolic acidosis
may cause an oily (pretty unpleasant) diarrhoea
Dehydration, secondary to osmotic diuresis (high
Sibutramine is central appetite suppressant, which
glucose levels) and vomiting
may induce tachycardia and high blood pressure
Electrolyte imbalance, including hyperkalaemia, sec-
(regular monitoring is mandatory). This can be a
ondary to metabolic acidosis, hyponatraemia and
problem in hypertensive patients with diabetes
‘relative’ hypokalaemia due to vomiting
The pancreas
55
Biguanides
(metformin)
Muscle
Thiazolidinediones
Figure 36 Mode of action of different
oral hypoglycaemic agents. The
Adipose
biguanide metformin reduces hepatic
tissue
glucose uptake and has a mild insulin
DPP-4
Liver
sensitizing effect. Thiazolidinediones
inhibitors
reduce insulin resistance, making
Inactive
Pancreas
GLP-1
insulin more effective at reducing
blood sugar. Sulphonylureas and
DPP-4
Insulin
Stomach
meglitinides stimulate pancreatic
insulin secretion. GLP-1 analogues
have a similar effect to GLP-1 (increase
Glucose
insulin and reduce glucagon secretion),
GLP-1
but are slowly degraded by DPP-4
enzymes. DPP-4 inhibitors interfere
with GLP-1 degradation resulting in
Sulphonylureas and
increased levels of this hormone. α-
GLP-1
meglitinides
analogues
Glucosidase inhibitors reduce glucose
Gut
α-glucosidase
absorption.
inhibitors
Table 23 Main side effects and contraindications of agents used to control blood sugar levels in individuals with T2DM. Pregnant
women with diabetes should be treated with insulin and oral hypoglycaemic agents are generally contraindicated.
Agent
Side effects
Contraindication
Metformin
Gastrointestinal (bloating, diarrhoea)
Renal failure (even if mild)
Advanced heart and liver failure
Pregnancy (relative contraindication)
Sulphonylureas
Hypoglycaemia
Pregnancy
Weight gain
Thiazolidinediones
Fluid retention
Heart failure
Weight gain
Pregnancy
GLP-1 analogues
Gastrointestinal
Pregnancy
DPP-4 inhibitors
Rare
Pregnancy
Orlistat
Diarrhoea
Pregnancy
Sibutramine
Hypertension
Uncontrolled hypertension
Tachycardia
Pregnancy
Rimonabant
Depression
History or current treatment of depression
Pregnancy
56
Part 1: Basics
Causes of DKA:
Fluid
New diagnosis of T1DM in 20% of cases
Fluid replacement usually starts with normal saline
In a known diabetic patient, DKA can be due to:
(0.9%): 1 L over the first h, 1 L over 2 h, then 1 L every
infection in 35% of cases, non-compliance with insulin
4-6 h, with careful monitoring of the patient and
injection in 30%, and errors in insulin administration
adjustment of fluid replacement accordingly. Normal
and dose calculation in 15%. In older patients, DKA
saline should be substituted with 5% dextrose infusion
may be precipitated by an ischaemic event such as
once plasma glucose drops below
12-15 mmol/L
myocardial infarction
(different protocols use different cut-offs)
Potassium
Clinical presentation of DKA
Failure to replace potassium can result in severe
DKA subjects can present with a variety of symptoms:
hypokalaemia, which may cause cardiac arrhythmias,
Gastrointestinal
potentially resulting in death. Serum potassium is
Nausea
usually elevated on initial presentation due to the pres-
Vomiting
ence of acidosis, despite low total body potassium.
Abdominal pain
Potassium levels quickly drop after initiation of DKA
Generally feeling unwell
treatment, as both insulin replacement and correction
Coma in advanced cases
of acidosis shift the potassium from the extracellular
In those with known diabetes, DKA should be
space into the cells. As a rough guide, 20 mmol/L
suspected in anyone who is not feeling or looking
potassium should be added to the fluid in patients with
well
normokalaemia, 40 mmol/L to those with hypokalae-
In subjects with suspected DKA who are not known to
mia and no potassium should be given to those with
have diabetes, a proper history is paramount to make the
hyperkalaemia. Monitoring potassium levels
(every
correct diagnosis
2-4 h) during treatment is extremely important
Insulin
Investigations in suspected DKA
Insulin is started as an i.v. infusion at around 0.1 u/
These consist of:
kg/h and adjusted according to a sliding scale insulin
1. Confirm the diagnosis
(see Table 24). Capillary glucose should be checked
Raised glucose levels: glucose can be only slightly
hourly and i.v. insulin should only be stopped once the
elevated
urine is ketone-free and the patient is clinically well
Reduced plasma bicarbonate levels with or without
Bicarbonate
low pH (bicarbonate <15 mmol/L confirm the diag-
This is very rarely given; only in cases of severe aci-
nosis of DKA)
dosis not responding to conventional treatment. Bicar-
Presence of ketonuria
bonate administration should only be done in an
2. Rule out precipitating cause:
intensive care setting and after the involvement of a
Chest X-ray (? infection)
senior colleague with expertise in DKA management
Check urine for the possibility of infection
ECG (? myocardial infarction)
Table 24 An example of sliding scale insulin. This is only a
Take blood and urine samples for culture
guide and different sliding scales can be used as some
Note that a high white cell count may occur in sub-
individuals require higher doses of insulin whereas others need
jects with DKA in the absence of infection
less.
Treatment of DKA
Capillary glucose
Insulin dose
This should be promptly started and consists of fluid and
insulin replacement as well as management of electrolyte
<4.0 mmol/L
0.5 units/h (with i.v. dextrose): review
imbalance. In addition, treatment should be directed to
4.1-10.0 mmol/L
2 unit/h
the precipitating cause (if any). Monitoring of patients
after initial treatment is very important and local hospital
10.0-16.0 mmol/L
4 units/h
guidelines for the management of these patients should
>16.0 mmol/L
6 units/h: review
be strictly followed.
The pancreas
57
Type of fluid
Amount of fluid
Venous potassium
Potassium in i.v. fluid
Initially normal saline,
give 1 L over 1 hour, 1L over 2 hours,
>5.5 mmol/L
No potassium in IV
Once glucose <12-15 mmol/L switch
1L over 4 hours, then 1L 6 hourly,
3.5-5.5 mmol/L
20 mmol potassium/L
to 5% desxtrose
Modify according to clinical response
<3.5 mmol/L
40 mmol potassium/L
Bicarbonate is not usually used in DKA.
ELECTROLYTES It may be needed if acidosis not responding.
FLUID
DO NOT give bicarbonate without discussion
with a specialist with an expertise in DKA
management
Culture blood, urine and sputum
Only for high risk patients
DKA
?PRECIPITANT
CXR and ECG (and review!)
ANTICOAGULATION
MANAGEMENT
(Clexane 40 mg s.c.)
Broad spectrum antibiotics
(if infection cannot be ruled out)
MONITORING
INSULIN
Check venous glucose, bicarbonate and potassium at:
Capillary glucose (BM)
Insulin
0, 2, 4, 8 and 24 hours
<4.0 mmol/L
0.5 units/hr (plus i.v. dex.)
Review the results and the patient and act!
4.1-10.0 mmol/L
2.0 units/hr
Modify monitoring according to results and
10.1-16.0 mmol/L
4.0 units/hr
clinical condition.
> 16.0 mmol/L
6.0 units/hr
Please note that above tables are only guidelines and insulin infusion rate may need
modification. Continue i.v. insulin/dextrose until urine is ketone-free.
Figure 37 Summary of the management of diabetic ketoacidosis.
Precipitating cause(s)
Mortality is very high approaching
50% in these
Around two-thirds of DKA cases are due to
patients
newly diagnosed type
1 diabetes or compliance
Causes include:
problems/errors in insulin administration in known
Omission of oral hypoglycaemic agents or insulin
diabetic patients. In around one-third, DKA is due
(rarely it can be the first presentation of T2DM)
to other causes such as infection or myocardial
Infection
infarction and these conditions should be treated
Vascular events such as myocardial infarction and
appropriately
stroke
Other measures
Some recommend low-dose heparin to prevent
Clinical presentation of HONK
thromboembolism, but there is no clear evidence to
Insidious onset of symptoms with ill health for weeks
support this practice, which is perhaps unnecessary
History of osmotic symptoms
unless other risk factors exist (prolonged immobility)
Symptoms of precipitating cause
A nasogastric tube may need to be inserted in those
Coma
with severe vomiting or in those with impaired con-
scious level
Investigations in HONK
Monitoring
Glucose levels: these are usually very high
Capillary glucose should be checked hourly and
U&Es, this usually shows high urea and creatinine
blood samples should be taken every 2-4 h for U&Es,
levels, with a relatively larger impairment in urea (pre-
bicarbonate and venous glucose. The clinical condition
renal renal failure)
of the patient should be regularly assessed
There is no acidosis in these patients (unless it is due
The management of DKA is summarized in Fig. 37.
to the precipitating cause)
Investigations for causes of HONK are mandatory
Hyperosmolar non-ketotic hyperglycaemia (HONK)
(CXR, ECG, urinalysis, CT head if necessary)
This is characterized by the gradual development of
Blood and urine cultures should be requested in all
hyperglycaemia
patients with HONK
58
Part 1: Basics
Treatment of HONK
Patients with hypoglycaemic symptoms should have
Treatment of HONK is broadly similar to that of DKA,
their capillary glucose checked to confirm the diagnosis
but with some differences:
before initiating treatment
Fluid
Patients with frequent hypoglycaemic episodes may
Fluid replacement should be more gentle in HONK
lose their warning symptoms, in which case plasma
compared with DKA as these are older patients, who
glucose should be kept slightly elevated for 2-3 weeks in
are more prone to heart failure with aggressive fluid
order to regain the hypoglycaemic symptoms
replacement. In difficult cases, a central line should be
inserted to help guide the appropriate fluid
Treatment of hypoglycaemia
replacement
Patient conscious:
Potassium
Oral glucose or sucrose (any fluid high in sugar
In uncomplicated HONK, potassium levels do not
content would do, such as Lucozade)
drop particularly quickly due to the absence of acido-
Patient unconscious
sis, but this should still be carefully monitored
Intravenous glucose
Insulin
Intramuscular or s.c. glucagons (this loses its effect
Despite the very high glucose levels in these patients,
with repeated dosing)
insulin requirements in HONK are modest and, there-
fore, insulin should be given at 0.5-2 units/h aiming
Chronic complications
for a gradual drop in blood sugar
(around
Investigations for chronic complications have been dis-
5 mmol/L/h)
cussed above and only treatment is covered here.
Bicarbonate
This is not needed in uncomplicated hyperosmolar
Treatment of microvascular disease
hyperglycaemia as the patient is not usually acidotic
Retinopathy
Precipitating cause(s)
Ensure good glucose control
Infection is the most common precipitating cause
Ensure good blood pressure control
and, therefore, antibiotic cover must be started after
Laser therapy in advanced stages
appropriate cultures
Nephropathy
Other measures
Early nephropathy (microalbuminuria): angiotensin
Due to high osmolarity and dehydration, throm-
converting enzyme inhibitors or angiotensin receptor
botic complications are very common and, therefore,
blockers (sometimes a combination of the two) can be
all patients should be covered with prophylactic
used to delay/prevent further deterioration in renal
unfractionated heparin
function. Also, need to ensure good glycaemia and
Monitoring
blood pressure control
This should be done regularly with blood samples
Advanced nephropathy
(macroalbuminuria or
taken every 2 h in the first 6-8 h to assess response to
raised creatinine): similar measures to those above can
treatment
be used. Potassium and renal function should be fre-
quently monitored and referral to a renal physician
Hypoglycaemia
considered
This is fully discussed in the neuroendocrine section of
End-stage renal disease: dialysis and renal
this book and, therefore, it will only be addressed here in
transplant
relation to diabetes.
Neuropathy
Hypoglycaemia in diabetes patients may be secondary
Painless peripheral neuropathy: repeated foot exam-
to oral hypoglycaemic agents (usually sulphonylurea) or
ination by patient and/or cohabiting relative and
insulin
regular chiropody
All patients with diabetes should be warned regarding
Charcot’s arthopathy: immobilization of the joint is
hypoglycaemic symptoms:
important to prevent further damage, and bisphos-
Tremor and sweating
phonate may be of help
Nausea
Painful peripheral neuropathy: difficult to treat and
Hunger
most only have a partial response. Some of the agents
The pancreas
59
used include: tricyclic antidepressants, capsaicin, anti-
T2DM diabetes is similar to those without diabetes and
convulsants (phenytoin, gabapentin) and opiates
a previous cardiac event
Autonomic neuropathy: postural drop in blood
Patients with diabetes and established cardiovascular
pressure can be treated with mechanical measures
disease are treated similarly to high risk non-diabetic
(wearing support stockings, sleeping with the head
individuals with known cardiovascular disease (discussed
elevated) and fludrocortisone (monitor for hyperten-
in the cardiovascular book of this series)
sion and hypernatraemia). Gastrointestinal symptoms
Diabetes patients should, therefore, be treated with:
such as vomiting can be treated with metoclopramide,
Lipid lowering agents such as statins (simvastatin,
domperidone and erythromycin, and diarrhoea with
atorvastatin, rosuvastatin) to lower cholesterol levels
loperamide
and reduce cardiovascular events.
Sexual dysfunction: rule out an endocrine cause.
Other agents can also be used such as ezetimibe,
Phosphodiesterase inhibitors, such as sildenafil, may
which inhibits cholesterol absorption in the gut, and
help. Ensure good diabetes and blood pressure
fibrates, which lower cholesterol levels but their main
control
effect is on triglycerides
Angiotensin converting enzyme inhibitors (ACEI)
Treatment of macrovascular complications
Antiplatelet agents (aspirin or clopidogrel)
The majority of patients with diabetes die of cardio-
Agents to maintain strict blood pressure control
vascular disease
Treat microalbuminuria, which is a cardiovascular
The risk of myocardial infarction in subjects with
risk
Lipid abnormalities and obesity
Lipid abnormalities
Diabetes mellitus: mainly affects triglyceride levels
The two main lipid molecules in the plasma are triglyc-
(increase) and HDL levels (decrease), particularly in
erides and cholesterol
those with poor glucose control
To make these lipid particles water-soluble, they are
Hypothyroidism: affects LDL levels (increase)
bound to phospholipids and lipoproteins in plasma
Renal failure: affects LDL levels
(increase), HDL
Lipid measurements are best performed on a fasting
(decrease) and triglycerides (increase)
sample. The following measurements can be done:
Liver disease: obstructive liver lesions affect LDL
Total cholesterol (TC) high levels are atherogenic.
levels (increase)
TC is composed of low-density lipoprotein cholesterol
Drugs: a number of drugs can affect lipid levels
(LDLc) - high levels are associated with increased risk
including β-blockers, thiazide diuretics, steroids, pro-
of vascular disease - and high-density lipoprotein cho-
tease inhibitors and alcohol
lesterol
(HDLc) - low levels are associated with
increased risk of vascular disease, whereas high levels
Clinical presentation
are protective
Patients may be asymptomatic and hyperlipidaemia is
Triglycerides: high levels are atherogenic and can
picked up during routine testing
also result in pancreatitis
Others can present with complications of hyperlipi-
daemia including:
Hyperlipidaemia
Atherothrombotic disease
(myocardial infarction,
There are a number of different types of hyperlipidae-
stroke)
mias, including:
Pancreatitis
Isolated raised cholesterol
Individuals with secondary hyperlipidaemia present
Polygenic hypercholesterolaemia: probably the com-
with symptoms of original disease
monest cause of isolated hypercholesterolaemia
Familial hypercholesterolaemia: an autosomal dom-
Treatment
inant condition affecting 1 : 500 people
Lifestyle changes are important
(diet, exercise and
Isolated raised triglycerides
stopping smoking) as these simple measures can lower
Autosomal dominant affecting around 1 : 300 people,
LDL and increased HDL levels
characterized by eruptive xanthomas and pancreatitis
For primary prevention (individuals with no previous
Raised cholesterol and triglycerides
complications due to hyperlipidaemia): there are special
Familial combined hyperlipidaemia: occurs in 1 : 250
risk factor engines that calculate future cardiovascular
people
risk and hyperlipidaemic agents are usually used in those
Secondary causes of hyperlipidaemia
with more than 20% risk over a 10-year period
Diet excessive in fat
Individuals with a previous vascular event or high-risk
subjects (for example diabetics), are treated with hyper-
lipidaemic agents even in the presence of normal lipid
profile
Endocrinology and Diabetes: Clinical Cases Uncovered. By R. Ajjan.
Agents used include statins, such as simvastatin, ator-
Published 2009 by Blackwell Publishing, ISBN: 978-1-4051-5726-1
vastatin and rosuvastatin
60
Lipid abnormalities and obesity
61
Most widely used hyperlipidaemic agents due to
Cardiovascular disease
their undoubted clinical benefits
Hypertension
Mode of action is related to decreased synthesis of
Mechanical joint pain and osteoarthritis
cholesterol in the liver [inhibition of
3-hydroxy, 3-
Sleep apnoea
methylglutaryl coenzyme A (HMG CoA)]
Increased risk of cancers
Effective at reducing LDL levels, minor effect on
Reproductive abnormalities
(PCOS, impaired
HDL and triglycerides
fertility)
Side effects are rare and include muscular aches and
pains, derangement in liver function and rhabdomy-
Clinical presentation
olysis, a potentially life-threatening complication but
Concerns over body image secondary to obesity
fortunately very rare
Complications (e.g. diabetes, cardiovascular disease)
All diabetes patients above the age of 40 are prescribed
a statin (regardless of cholesterol levels), to reduce the
Box 15 Obesity and BMI
risk of future vascular events
Ezetimibe
Obesity is assessed using body mass index (BMI),
This agent reduces cholesterol absorption
calculated by the formula: weight (kg)/[height(m)]2
Effective in combination with a statin but less
BMI = 18.5-24.9: healthy
impressive when used alone
BMI = 25-30: overweight
Fibrates
BMI >30: obese
Effective at reducing triglycerides and, to a lesser
extent, LDL levels. Also, they raise HDL levels
Investigations
Their role in reducing cardiovascular risk is not as
Fasting glucose (rule out diabetes)
clear as statins
Fasting lipid profile
Usually used as second- or third-line treatment,
Thyroid function tests
except in those with isolated hypertriglyceridaemia,
ECG
when fibrates are used as first-line treatment
Specific tests in case of clinical suspicion
(such as
Nicotinic acid
Cushing’s syndrome for example)
Very effective at increasing HDL levels
The role of this agent in cardiovascular protection is
Treatment
unknown
Lifestyle changes
Use is limited by side effects (severe flushing)
Diet: it is always useful to arrange an appointment
Omega-3 fatty acids
with a dietician as minor changes in dietary habits can
Effective at reducing triglyceride levels
have a major influence on weight reduction
Increase exercise activity
Obesity
More severe dietary restrictions: those with severe
A major health problem in the developed world and it
obesity are sometimes admitted to hospital to initiate a
is on the increase
very low calorie diet under clinical supervision
Related largely to increased food intake and sedentary
Drug treatment
lifestyle
Orlistat inhibits gut lipase activity and reduces fat
Genetic factors play a role as some individuals are
absorption. Patient should comply with low-fat diet.
more susceptible to developing obesity
Side effects include diarrhoea (oily diarrhoea is char-
There are some rare cases of obesity that have a clear
acteristic), often in those who do not comply with
genetic basis (monogenic obesity), including:
reduction in fat intake
Leptin and leptin receptor deficiency
Sibutramine is a centrally acting appetite suppres-
Prader-Willi syndrome
sant. Side effects include hypertension and increased
Laurence-Moon-Biedl syndrome
heart rate, which limit its use
Complications of obesity include:
Rimonabant is a newer agent which acts on the
Insulin resistance and diabetes mellitus
cannabinoid receptor blocker resulting in reduced
Lipid abnormalities
appetite, increased feeling of satiety, positive effect on
62
Part 1: Basics
plasma glucose and lipid profile and helps to quit
Surgery
smoking. Side effects include depression in up to 1 in
Gastric bypass surgery is an effective treatment but
7 patients, individuals on antidepressants should not
reserved for those with severe obesity who are not
be prescribed this agent. Individuals should be observed
responding to lifestyle changes and/or medical
closely for the development of this complication
treatment
The neuroendocrine system
Neuroendocrine cells are found in the gastrointestinal
72-h fast: this may be necessary in suspicious cases
tract
Patient is admitted to hospital and fasted
Benign and malignant tumours of the neuroendocrine
Blood glucose and patient symptoms are regularly
system are rare and result in excess hormone production.
monitored
Clinical manifestations differ according to the nature of
The presence of low glucose (<2.5 mmol/L) together
the secreted hormone
with elevated insulin and C peptide confirms the
It should be noted that some of these tumours may
diagnosis
also secrete pure endocrine hormones such as ACTH,
It is important to measure C peptide to rule out
PTH and GHRH resulting in Cushing’s syndrome,
exogenous administration of insulin (injected insulin
hypercalcaemia and acromegaly respectively
has no C peptide, whereas endogenous insulin produc-
The characteristics of neuroendocrine tumours are
tion is associated with detectable plasma levels of C
summarized in Table 25
peptide)
Localizing the tumour
Insulinomas
This can be difficult as tumours are often small
Insulinomas result in hypoglycaemia through exces-
CT or MRI of the pancreas
sive secretion of insulin
Endoscopic ultrasound of the pancreas
These are usually benign and only around 10% show
Radiolabelled octreotide scanning: the majority of
evidence of malignancy
these tumours take up octreotide
May be part of MEN-1 (see below)
Treatment
Clinical presentation
Surgical removal is the treatment of choice
Symptoms of hypoglycaemia
(usually relieved by
Octreotide and/or diazoxide can be useful to reduce
eating)
insulin secretion
Tremor and sweating
Malignant tumours with metastases: palliative treat-
Nausea
ment with streptozotocin or 5-fluorouracil
Hunger
Weight gain secondary to frequent snacking
(to
Carcinoid tumours
avoid/treat hypoglycaemia)
The majority of these tumours develop in the gut but
a minority can be found in the lungs and rarely other
Investigations
organs
Fasting tests
These tumours produce mainly serotonin (which is
16-h fast: the absence of hypoglycaemia after 16-h fast
metabolized to 5-hydroxyindolacetic acid, 5HIAA)
makes the diagnosis of insulinoma unlikely (test to be
These tumours also have the ability to produce a large
repeated three times)
number of other hormones and proteins including
ACTH, PTH, histamine and prostaglandin
Carcinoid tumours are malignant but usually slow-
Endocrinology and Diabetes: Clinical Cases Uncovered. By R. Ajjan.
growing and some patients live 20-30 years after the
Published 2009 by Blackwell Publishing, ISBN: 978-1-4051-5726-1
diagnosis
63
64
Part 1: Basics
Table 25 Characteristics of the neuroendocrine tumours.
Treatment
Surgical removal
Excess hormone
Produced by
Clinical disease
Somatostatin analogues for residual disease or if
surgery is contraindicated
Insulin
Pancreatic β-cells
Insulinoma
Palliative therapy
Serotonin, kinins
Intestine
Carcinoid
Hepatic
embolization:
using
angiography
syndrome
techniques
Stomach
Chemotherapy: streptozotocin and 5-fluorouracil
Pancreas
Immunotherapy: α-interferon: useful in controlling
Gastrin
Pancreatic G cells
Gastrinoma
symptoms and can be combined with octreotide
(Zollinger Ellison
Stomach
syndrome)
Gastrinomas
Small intestine
Two-thirds of these tumours are malignant
Glucagon
Pancreatic α-cells
Glucagonoma
Excessive gastrin secretion results in increased acid
secretion by the stomach
Somatostatin
Pancreatic δ-cells
Somatostatinoma
Stomach
Clinical presentation
Recurrent peptic ulcer disease that is refractory to
Small intestine
treatment
Vasoactive intestinal
Pancreatic VIP cells
VIPoma
Malabsorption and diarrhoea
peptide (VIP)
Investigations
Clinical presentation
Inappropriately elevated gastrin levels in the presence
Flushing, can be precipitated by:
of increased stomach acid secretion confirms the
Alcohol
diagnosis
Spicy food
Imaging
Exercise
As described under insulinoma
Carcinoid flush usually affects the face and upper
thorax and is shown in Fig. 38 (colour plate section)
Treatment
Diarrhoea
Surgical removal is the treatment of choice
Asthma
Proton pump inhibitors (omeprazole, lansoprazole)
Right valvular heart lesions: fibrosis of right heart
for those with residual tumour or in whom surgery is
valves, which may be due to serotonin
contraindicated
Pellagra-like skin lesions (excessive tryptophan metab-
olism results in nicotinamide deficiency)
Glucagonomas
It should be noted that individuals with carcinoid syn-
Two-thirds of these tumours are malignant but they
drome due to gastrointestinal tumours will only be
are slow-growing
symptomatic if they have liver metastases, in contrast to
bronchial carcinoid subjects, who develop the symptoms
Clinical presentation
before metastases have taken place.
Skin rash: necrolytic migratory erythema, can precede
the diagnosis by many years
Investigations
Glucose intolerance and diabetes: due to excess gluca-
Urinary 5-HIAA levels are elevated in the majority of
gon secretion
patients and have a high specificity
Mucous membrane involvement: stomatitis and
Plasma chromogranin A: higher sensitivity than 5-
glossitis
HIAA but lower specificity
Imaging
Investigations
CT/MRI of chest and/or abdomen
Raised plasma glucagon in the presence of symptoms/
Octreotide scanning
signs is diagnostic
The neuroendocrine system
65
Localization
Table 26 Summary of the clinical presentation of different
CT or MRI of the abdomen
neuroendocrine tumours.
Octreotide scanning: particularly useful to evaluate
Neuroendocrine tumour
Clinical presentation
the extent of metastases
Insulinoma
Symptoms of hypoglycaemia
Treatment
Surgery: cure rate is unfortunately very low (<10%)
Weight gain
Octreotide or long acting somatostatin analogues can
Carcinoid syndrome
Flushing
be very useful to control symptoms
Diarrhoea
Palliative
treatment with streptozotocin or
5-fluorouacil
Asthma
Right-sided heart lesions
Somatostatinomas
Pellagra-like skin lesions
These are very rare with an incidence of
1 in 40
million
Gastrinoma (Zollinger Ellison
Recurrent and refractory peptic
Characterized by:
syndrome)
ulcer disease
Glucose intolerance and diabetes mellitus
Glucagonoma
Typical skin rash (necrolytic
Gall stones
migratory erythema)
Diarrhoea and malabsorption
Glucose intolerance
VIPoma
Somatostatinoma
Glucose intolerance
Very rare and characterized by:
Gall stones
Watery diarrhoea
Hypokalaemia
Diarrhoea and malabsorption
VIPoma
Watery diarrhoea
Neuroendocrine syndromes
These describe the association of a number of neuroen-
docrine abnormalities and include:
Some patients may have mucosal neuromas and
Multiple endocrine neoplasia type 1
a marfanoid habitus and these are classified as MEN
An autosomal dominant condition (gene on chro-
2B.
mosome 11, menin gene is affected), with a prevalence
Von Hippel-Lindau disease
of 1 in 10 000
An autosomal dominant condition
Tumours occur in two or more endocrine glands:
Clinical manifestations include: pheochromocy-
parathyroid
(hyperplasia or adenoma), almost all
toma, bilateral in half the patients; pancreatic neuro-
cases; pancreas
(insulinoma, gastrinoma),
70% of
endocrine tumours; retinal and central nervous system
patients; pituitary (prolactinoma, acromegaly), 30% of
hemangiomas; and renal cell carcinoma (the usual
patients. The easiest way to remember this is PPP
cause of death in these patients)
(parathyroid, pancreas, pituitary)
Neurofibromatosis (NF)
Multiple endocrine neoplasia type 2
NF1 is an autosomal dominant condition character-
An autosomal dominant condition (gene of chro-
ized by: multiple neurofibromas, café-au-lait spots, iris
mosome 10, ret protooncogene is affected)
lisch nodule, pheochromocytoma and gut endocrine
Around one-third of gene carriers do not manifest
tumours
clinically significant disease
NF1 should be differentiated from NF2, which is
Tumours occur in two or more endocrine glands:
characterized by: disorders of the central nervous
thyroid gland (medullary thyroid cancer), often the
system (meningiomas), cranial nerve tumours (usually
presenting feature; adrenal glands (pheochromocyto-
optic glioma); NF2 is not associated with endocrine
mas), in 50% of patients; parathyroid glands, in 30%
abnormalities
of patients. The way to remember this is TAP (thyroid,
The main clinical characteristics of the neuroendo-
adrenal, parathyroid)
crine tumours are summarized in Table 26.
Case 1
A 19-year-old with abdominal pain
and vomiting
Kathryn, a 19-year-old student, who is usually fit and well, is
What clinical features are associated
admitted to accident and emergency (A&E) with a 2-day
with weight loss?
history of abdominal pain, vomiting and feeling generally
Chronic infections and infestations: particularly in
unwell. She has lost 5 kg in weight over the past 3 weeks
individuals with a deranged immune system, such as
for no clear reason. There is no significant past medical
patients with AIDS
history of note except for three episodes of urinary tract
Malignancy
infection (UTI) over the past 6 months.
Diabetes mellitus
Hyperthyroidism
What are the differential diagnoses of
Malnutrition: uncommon in Western countries
abdominal pain and vomiting?
Degenerative neurological and muscular diseases
Intra-abdominal pathology
It is impossible to give an accurate diagnosis at this
Peptic ulcer disease
stage and a more detailed history and careful physical
Pancreatitis
examination is of paramount importance in order to
Cholecystitis and gall stones
establish the correct diagnosis.
Appendicitis
Ectopic pregnancy
What questions will you ask?
Intestinal obstruction
Has the pain and vomiting started recently or has it
Renal calculi and pyelonephritis
been occurring for weeks, months or years?
Was the onset of pain sudden or gradual?
Other conditions associated with abdominal
What is the pain like and how severe is it?
pain but less likely to cause vomiting
Where is the pain localized?
Dysmenorrhea
Does anything relieve the pain?
Pelvic inflammatory disease
Has a new treatment been introduced recently?
Inflammatory bowel disease
Are there any associated symptoms
(review of
Intra-abdominal arterial and venous thrombosis
systems)?
Ruptured aortic aneurysm (in older individuals)
On further questioning, Kathryn tells you that the abdominal
Endocrine causes of abdominal pain and
pain was gradual in onset over 4-6 h, generalized and
vomiting
cramp-like, with severity varying between 2/10 and 4/10.
Diabetes mellitus
complicated by diabetic
Nausea and vomiting preceded the abdominal pain by 6 h or
ketoacidosis
so. Kathryn has been on oral contraceptive pills (OCP) for 18
Hypoadrenalism
months and her last withdrawal bleed was 1 week ago.
Hypercalcaemia
Does this help with the diagnosis?
The gradual onset of pain and low severity (although
Endocrinology and Diabetes: Clinical Cases Uncovered. By R. Ajjan.
this is subjective) make a surgical cause for the pain less
Published 2009 by Blackwell Publishing, ISBN: 978-1-4051-5726-1
likely, but do not fully rule it out. For example, appendi-
66
Case 1
67
citis may initially present with gradual and cramp-like
Nausea
abdominal pain. Also, it should be noted that in some
Vomiting
cases of acute abdomen, the symptoms may be relatively
Abdominal pain
mild and this can be seen in older patients or in individu-
Associated symptoms: a few days/weeks history of
als who are on steroid treatment
polyuria and polydipsia (known as osmotic symp-
The combination of OCP use and a recent normal
toms), and weight loss
withdrawal bleeding rules out dysmenorrhea and ectopic
The family history of autoimmunity further supports
pregnancy as causes for Kathryn’s pain
this diagnosis, as it suggests a genetic predisposition to
autoimmune disease in Kathryn
During review of systems and on further questioning,
Kathryn tells you that she had polyuria up to 15 times/day
Kathryn deteriorates and becomes slightly confused, with a
and nocturia 6 times/night for 2 weeks prior to her current
drop in her Glasgow Coma Scale (GCS) from 15/15 to 13/15
presentation. She is a non-smoker and drinks up to 20 units
(E3, V5, M5). On examination, she is tired, dehydrated,
of alcohol per week. Family history includes pernicious
tachycardic at 112 beats/min, has a temperature of 36.6°C,
anaemia in her uncle and hypothyroidism in her mother.
blood pressure of 115/70 mmHg, with a postural drop of
20/10 mmHg, and respiratory rate of 32/min, with otherwise
Does this help with the diagnosis?
normal chest examination. Cardiac auscultation is normal,
Polyuria can be secondary to a number of causes. In this
and abdominal palpation reveals minimal generalized
case, a urinary tract infection may have caused the
tenderness with no rigidity, guarding or rebound tenderness.
abdominal pain and polyuria, which is usually associated
with dysuria, and only small amounts of urine are passed
How do her clinical findings help with
on each occasion.
the diagnosis?
Kathryn is clinically dehydrated with tachycardia and
a postural drop in blood pressure indicating significant
Box 16 Other causes of polyuria
fluid loss
The tachypnea is a matter of concern and could be due
Electrolyte abnormalities such as hypercalcaemia
Chronic renal disease
to a primary lung pathology or secondary causes. A
Endocrine disease such as diabetes insipidus (lack of, or
primary lung pathology in this patient may be:
ineffective, antidiuretic hormone)
Chest infection may result in tachypnea and pneu-
Osmotic diuresis due to high plasma glucose levels
monia is a recognized cause of abdominal pain.
(diabetes mellitus)
However, she is apyrexial and chest auscultation is
The use of drugs such as lithium and demeclocycline
unremarkable, making this an unlikely diagnosis
Pulmonary embolus causes tachypnea and hypoten-
In this patient:
sion, and OCP use is a known risk factor for thrombo-
Hypercalcaemia can indeed cause abdominal pain and
embolism. However, Kathyrn has no chest pain,
polyuria but this condition is infrequently seen in a
whereas abdominal pain and vomiting are not usually
young person. Nevertheless, it should be checked out
features of pulmonary embolism
There is no indication that this patient has chronic
Metabolic acidosis: the respiratory system compen-
renal disease but this should certainly be excluded. The
sates for metabolic acidosis by increasing the respira-
fact that she had three UTIs in 6 months may indicate a
tory rate to blow off CO2, resulting in respiratory
pathology in the urinary tract. However, urinary infec-
alkalosis, which may fully or partially compensate for
tions are common in female individuals, particularly if
the metabolic acidosis. Causes of metabolic acidosis
they are sexually active
are summarized in Table 27 and DKA is one cause,
Diabetes insipidus is a recognized cause of polyuria but
which seems to fit the diagnosis. A distinctive ketotic-
is not associated with abdominal pain or vomiting and
smelling breath can further aid the diagnosis of
therefore this diagnosis is unlikely
DKA.
Type 1 diabetes mellitus (T1DM) is a strong possibil-
Abdominal examination revealed only minimal gener-
ity. Diabetic ketoacidosis
(DKA), a complication of
alized tenderness with no signs of acute abdomen (rigid-
T1DM, classically presents with:
ity, guarding or rebound tenderness), which is reassuring
68
Part 2: Cases
Table 27 Causes of metabolic acidosis.
Metabolic acidosis with increased anion gap
Metabolic acidosis with normal anion gap (imbalance between
(increased acid production/ingestion)
HCO-3 and H+ ions)
Lactic acidosis: increased production of lactate due to
Renal tubular acidosis: loss of HCO3 or excessive absorption of H+
infection, shock or hypoxia
ions
Uraemic acidosis: renal failure
Diarrhoea: loss of HCO3
Ketotic acidosis: diabetes or alcohol
Pancreatic fistula: loss of HCO3
Toxins and drugs: salicylate overdose, ethylene glycol and
Addison’s disease: excessive absorption of H+ ions
methanol ingestion
Drugs: acetozolamide: excessive absorption of H+ ions
and makes a surgical cause for Kathryn’s abdominal pain
attending A&E, and can aid in the diagnosis of anaemia
less likely
(low haemoglobin) and infection (raised white cells). It
is worth bearing in mind that infection may precipitate
What test(s) would you request to
DKA
confirm the diagnosis?
Cultures: DKA can be precipitated by an infection and,
Taken together, the most likely diagnosis here is DKA,
therefore, blood and urine cultures (as well as culture of
which is characterized by:
sputum if respiratory symptoms are present) are usually
Metabolic acidosis
requested on presentation, unless the cause of DKA is
Raised plasma glucose
clear
(non-compliance with insulin injections for
Dehydration
example)
Increased ketone production
Chest X-ray: to rule chest infection as the precipitating
Therefore, the following tests should be requested:
cause. This is perhaps not necessary in newly diagnosed
Venous bicarbonate and pH: bicarbonate falls in DKA
patients with no reason to suspect a respiratory
to <15 mmol/L and can be as low as 1 mmol/L, resulting
pathology
in a variable degree of acidosis. In early DKA, pH can be
Electrocardiogram: this should be requested in patients
normal due to compensated respiratory alkalosis. A
with diabetes particularly in the older age group as silent
common hospital practice is to take an arterial blood
myocardial infarction (myocardial infarction with no
sample for bicarbonate and pH measurement (arterial
chest pain) is common in these patients and may precipi-
blood gas analysis), which is unnecessary unless a primary
tate DKA. A myocardial infarction is unlikely here due to
lung pathology precipitating DKA is suspected
Kathryn’s young age but an ECG may show abnormali-
Plasma glucose: this is elevated in DKA. Capillary
ties and arrhythmias consistent with electrolyte distur-
glucose (finger-prick glucose) can be initially done to
bances (hyperkalaemia for example), which may require
give a quick result but it should always be followed by
urgent attention
plasma glucose measurement
Abdominal X-ray (AXR): this is usually requested in
U&Es: in DKA these show:
patients attending A&E with severe abdominal pain and
High or high-normal urea (due to dehydration)
vomiting to rule out intestinal obstruction and/or perfo-
High or high-normal potassium (due to acidosis)
ration. Opinions will differ, but an AXR is probably not
In advanced or more severe cases, creatinine can be
necessary here as Kathryn has no signs to suggest an acute
elevated (due to pre-renal renal failure)
abdomen
Urine dipstick: detection of large amounts of ketone
bodies in the urine aids the diagnosis of DKA
Blood, urine and radiological tests show the following:
In addition to DKA-specific tests, other blood tests
FBC:
Hb 14.1 g/L
should be requested, including:
WBC 23.3× 109/L (neutrophils 18.2× 109/L)
Full blood count: usually requested in ill individuals
Platelets 380 × 109/L
Case 1
69
U&Es:
Na 131 mmol/L
Box 17 Precipitants of DKA
K 5.4 mmol/L
Urea 10.1 mmol/L
New diagnosis of T1DM in 20% of cases
Creatinine 124 mmol/L
In a known diabetic patient, DKA can be due to:
Bicarbonate 9 mmol/L
Infection in 35% cases
pH 7.16
Non-compliance with insulin injection in 30%
Glucose
22 mmol/L
Errors in insulin administration and dose calculation in
Amylase normal
15%
CXR
clear
In a minority of patients, DKA may be precipitated by
AXR
normal
an ischaemic event such as myocardial infarction
Urine dipstick: ketones +++, glucose +++, RBC , WBC ,
nitrates
How would you manage this patient?
How do you interpret these results?
The management should be directed to:
Kathryn has high WBC with elevated neutrophil counts
Correct the metabolic abnormality by replacing:
suggesting an underlying infection. However, DKA
Fluid
patients may have very high WBC count without
Insulin
associated infection, which normalizes once DKA is ade-
Potassium
quately treated. In some cases, infection is difficult to rule
Treat the precipitating cause (if any)
out, and this is why a septic screen is requested (blood
Monitor the patient carefully during treatment of
and urine cultures, sputum culture if any, CXR), fol-
DKA
lowed by antibiotic cover if the suspicion of infection
Local hospital guidelines should be followed for the
is high
management of patients with DKA
The diagnosis of metabolic acidosis is evident from the
combination of low pH and low bicarbonate
Fluid
The metabolic acidosis together with high plasma
Fluid replacement usually starts with normal saline
glucose and strongly positive urinary ketones confirm the
(0.9%): 1 L over the first h, 1 L over 2 h then 1 L every
diagnosis of DKA
4-6 h, with careful monitoring of the patient clinical
Patients with DKA excrete large amounts of ketones
status and urine output
in their urine due to deranged glucose metabolism and
Fluid replacement should be modified according to the
the production of abnormally high levels of ketone
clinical status of the patient. For example, if the patient
bodies: acetone, acetoacetate, β-hydroxybutarate
has a very low blood pressure at presentation with signs
Urine dip testing methods check only for acetone
of shock, initial fluid replacement should be more
and acetoacetate
aggressive
Other abnormalities include:
Normal saline should continue until the blood glucose
High urea consistent with dehydration
drops below 12-15 mmol/L (different protocols use dif-
High potassium secondary to acidosis, which shifts
ferent cut-offs), when saline should be substituted with
the potassium from the intracellular compartment to
5% glucose. This helps to restore normal energy metabo-
the extracellular space. Potassium falls with successful
lism and clears the blood of ketone bodies, thereby nor-
treatment of DKA and this should be monitored care-
malizing the pH
fully as detailed below
Marginally low sodium is commonly seen in patients
Potassium
with DKA due to high plasma glucose and this normal-
Monitoring of potassium status is very important as
izes with treatment of the condition and the fall in
failure to replace potassium can result in severe hypoka-
blood sugar
laemia, which may cause cardiac arrhythmias, potentially
Urine dipstick is positive for ketones and glucose con-
resulting in death
sistent with the diagnosis of DKA. Of note is the absence
Serum potassium is usually elevated on initial presen-
of pyuria and nitrates on urine dipstick, making the diag-
tation due to the presence of acidosis, but it quickly drops
nosis of a urinary tract infection unlikely
after the initiation of treatment (both insulin replace-
70
Part 2: Cases
Table 28 An example of sliding scale insulin. This is only a
In around one-third, DKA is due to other causes such
guide and different sliding scales can be used as some
as infection or myocardial infarction and these condi-
individuals require higher doses of insulin whereas others need
tions should be treated appropriately
less.
Other measures
Capillary glucose
Insulin dose
Low-dose heparin to prevent thromboembolism is
recommended by some, but there is no clear evidence to
<4.0 mmol/L
0.5 units/h (with i.v. dextrose): review
support this practice, which is unnecessary unless other
4.1-10.0 mmol/L
2 unit/h
risk factors exist (prolonged immobility)
10.0-16.0 mmol/L
4 units/h
A nasogastric tube should be inserted into patients
with protracted vomiting
>16.0 mmol/L
6 units/h: review
Monitoring
ment and correction of acidosis shift the potassium from
Capillary glucose should be checked hourly
the extracellular space into the cells)
Potassium levels should be regularly assessed and this
As a rough guide:
can be done using the following time points as a guide:
20 mmol/L potassium should be added to i.v. fluid
presentation (time 0 h), 2 h, 4 h, 8 h, 16 h and 24 h
in patients with normokalaemia
Both venous bicarbonate and glucose can also be
40 mmol/L to those with hypokalaemia
checked at the same time points as above to assess
No potassium should be given to those with
response to treatment
hyperkalaemia
The above time points can be modified according to
the severity of the DKA and the response to treatment
Insulin
The management of DKA is summarized in Fig. 37
Insulin is started as an i.v. infusion at around
(Part 1, p. 57).
0.1 U/kg/h and adjusted according to a sliding scale
insulin (see Table 28)
What is the prognosis in this case?
Some diabetologists feel that a sliding scale insulin
Prognosis is very good in uncomplicated DKA and the
should be avoided in DKA patients (to avoid too many
mortality rate is less than 3%.
insulin dose adjustments) and the insulin dose should be
regularly reviewed and adjusted by an experienced doctor.
Kathryn improves after initial treatment, her BP normalizes
However, this can be difficult practically and, therefore, a
and her confusion clears. However, she starts feeling very
simple sliding scale is used in most hospitals
weak 12 h after admission and complains of palpitations. An
A common practice is to give a starting dose of 4-6
ECG is shown (Fig. 39).
units of insulin/h and modify the dose according to
plasma glucose levels
What complication has occurred? How
Intravenous insulin should only be stopped when the
should this be treated?
urine is ketone-free and the patient is clinically well
ECG shows changes consistent with hypokalaemia
ST depression
Bicarbonate
Presence of U wave after T wave
This is very rarely given; only in cases of severe acidosis
Plasma potassium should be checked and corrected
not responding to conventional treatment
urgently:
Bicarbonate administration should only be done in an
Supplementation of potassium to i.v. fluid
intensive care setting and after the involvement of senior
A cardiac monitor should be attached to the
clinicians with expertise in DKA management
patient
Precipitating cause(s)
Twenty hours after admission, Kathryn’s blood glucose levels
Around two-thirds of DKA cases are due to newly
fall to 6.8 mmol/L and the acidosis clears. However, Kathryn
diagnosed type 1 diabetes or compliance problems/errors
becomes suddenly confused and agitated and subsequently
in insulin administration in known diabetic patients
GCS drops to 6.
Case 1
71
I
aVR
V1
V4
PRTU
II
aVL
V2
V5
S
III
aVF
V3
V6
Figure 39
What urgent test would you request at
acting and three short acting insulin with meals) or
this stage?
two daily injections with a mixture of short and long
A drop of GCS and neurological signs in a treated DKA
acting insulin preparations. An insulin pump can be
patient should raise the suspicion of cerebral oedema,
used for those with erratic glucose control
which may be secondary to over-enthusiastic fluid
Good diabetes control is important to avoid long-
replacement
term microvascular complications
(retinopathy,
This complication is rarely seen in adults but it is not
nephropathy and neuropathy) as well as macrovascu-
uncommon in children
lar complications (coronary artery disease, cerebrovas-
If cerebral oedema is suspected, urgent CT/MRI of the
cular and peripheral vascular disease)
head should be requested, and if the diagnosis is
Tight glucose control should not be achieved at the
confirmed:
expense of increasing hypoglycaemia, which can be
The patient should be immediately transferred to an
dangerous and sometimes fatal
intensive care unit
Screen for the development of microvascular
Should be treated with mannitol and dexametha-
complications
sone, which may help to reduce the cerebral oedema
Yearly retinal screening
Unfortunately, prognosis is poor once this compli-
Yearly check of urinary microalbumin: usually done
cation occurs, with mortality approaching
90% in
on an early morning urine sample with results
adults
expressed as albumin/creatinine ratio (ACR)
Yearly foot examination to rule out neuropathy:
What does long-term management of a
usually done using monofilament test
type 1 diabetes patient involve?
Watch for/prevent the development of macrovascular
Ensure strict glucose control
complications
Monitored by home sugar readings and glycosylated
Have a role threshold for investigating individuals
haemoglobin levels (HbA1c). The input of the diabetes
with suspected vascular pathology
nurse specialist is important to provide support
Add statin treatment to patients above the age of 40
to patients and help with adjustments of insulin
or earlier in those at high risk
doses
Adding aspirin treatment to high-risk individuals is
Most commonly used insulin injection regimes in
of debatable benefit
type 1 diabetes include four daily injections (one long
Aggressively treat hypertension
CASE REVIEW
Kathryn is a young woman admitted to hospital with short
examination usually help to rule out a surgical cause,
history of abdominal pain, vomiting and feeling unwell.
which should be diagnosed early as delays can have serious
Also, there is a history of significant weight loss over a 3-
consequences. There is nothing in the history or
week period. The differential diagnosis of abdominal pain
examination to suggest an acute abdomen, and, therefore,
is wide and a detailed history together with a full
a surgical cause for this patient’s symptoms is less likely.
Continued
72
Part 2: Cases
Kathryn’s clinical condition subsequently deteriorates and
and inadequate monitoring of her potassium levels, which
her Glasgow Coma Scale (GCS) drops from 15/15 to 13/15.
can fall very rapidly during DKA treatment. She is treated
She is found to be clinically dehydrated, tachycardic
with intravenous fluid containing potassium, which
and tachypneic. Taken together, diabetic ketoacidosis is
stabilizes her condition, but she deteriorates again 24 h
suspected, which is subsequently confirmed by
after admission and her GCS drops to 6. This raises the
demonstrating low plasma pH and bicarbonate levels,
possibility of cerebral oedema and an urgent CT or MRI
raised glucose and significant ketonuria. Appropriate tests
of the head should be requested. If confirmed this
are arranged to rule out an underlying infection (CXR,
complication should be aggressively treated in intensive
blood cultures) and she is treated with intravenous fluid
care settings.
and insulin with initial improvement in her symptoms.
DKA is a common condition and frequently the first
However, 12 h after her admission, she starts complaining
presentation of diabetes. Monitoring is a vital part in the
of palpitations; an ECG shows an abnormal U wave and
management, in order to avoid the development of serious
a depressed ST segment. This was due to hypokalaemia
complications, which may have tragic consequences.
KEY POINTS
Diabetic ketoacidosis (DKA) is a relatively common
Treatments for DKA include:
condition, which can be life-threatening
Intravenous fluid (with adequate potassium
Abdominal pain, vomiting and tachypnea (air hunger) are
replacement)
typical manifestations of this diabetic complication
Intravenous insulin
DKA should be suspected in any type 1 diabetes patient
Treat the precipitating cause
with gastrointestinal symptoms. In those with no history
Monitoring of glucose, bicarbonate and potassium is
of diabetes, DKA should be suspected in individuals,
paramount to assess response to treatment
particularly the young, who are acutely unwell and have
Intravenous fluid and insulin should be continued until
the above symptoms
bicarbonate normalizes and the urine is ketone-free
Around one-third of patients with DKA present as a new
Serious complications of DKA include:
diagnosis of diabetes, one-third are due to errors or
Hypokalaemia (common)
non-compliance with insulin administration in a known
Cerebral oedema (rare)
diabetes patient, and in the final third the DKA arises
Long-term management of type 1 diabetes patients
secondary to infections or an ischaemic event
should include:
A history of osmotic symptoms, with or without weight
Good glucose control
loss, should prompt appropriate investigations to rule out
Screen for the development of complications: for
diabetes as a cause
microvascular disease use retinal screening, urinary
Biochemical abnormalities in DKA include:
microalbumin and regular foot examination; for
Raised blood glucose
macrovascular disease, promptly investigate potential
Low bicarbonate with or without low pH
vascular pathology, initiate statin treatment for
Low pCO2 on arterial blood gas analysis (not necessary
individuals at high risk or those above the age of 40
to make the diagnosis)
and aggressively treat hypertension and
Heavy ketonuria
microalbuminuria
Case 2
A 35-year-old woman with palpitation
and irritability
Andrea is a 35-year-old solicitor who presents with a 10-day
How would this information help you
history of constant palpitations, weakness and irritability.
in the diagnosis?
Heat intolerance, hand tremor, loose bowel motions
What is the differential diagnosis and
and weakness are all classical features of hyperthyroidism
how would you proceed?
(symptoms of hyperthyroidism are summarized in Table
Palpitation is a common complaint and is a perception
8, p. 16), making this diagnosis a real possibility
of ‘increased’ heart action. It can be physiological or
There is a personal history of atopy and a family history
pathological:
of autoimmune diseases, suggesting a genetic predisposi-
Physiological: stressful life events can result in palpita-
tion to autoimmunity in Andrea
tion. A classical example is palpitations experienced by
The family history of ischaemic heart disease is prob-
university students or junior doctors sitting an important
ably irrelevant as it did not occur at a young age and
exam
myocardial infarction is not uncommon in men above
Pathological: cardiac tachyarrhythmias (fast regular or
the age of 70
irregular heart rate) may be due to:
A primary heart problem: in young patients, tachyar-
How would you proceed here?
rhythmias, such as supraventricular tachycardia, com-
Having taken the history, physical examination is the
monly occur with no significant structural cardiac
next step, with special emphasis on the assessment of
abnormality but may sometimes be secondary to
thyroid status. The signs of hyperthyroidism are sum-
serious cardiac abnormalities. However, these are
marized in Table 8, p. 16.
usually intermittent and do not persist for 10 days as
in this case
On examination, Andrea has sweaty palms with a marked
Non-cardiac palpitations: the commonest cause is
hand tremor. Her pulse is regular at 104 beats/min and her BP
hyperthyroidism and, therefore, it is important to rule
is 110/70 with no postural drop. She has a marked lid lag.
this out in our patient
A detailed history and examination is required at this
How would you interpret these findings,
stage.
and what other examination(s) would
you do and why?
On further questioning Andrea tells you that she has had a
Andrea has clinical hyperthyroidism supported by the
number of symptoms for the past week, including: heat
presence of: hand tremor, sweaty palms, tachycardia and
intolerance, hand tremor, generalized weakness, inability to
lid lag. The next step would be directed at establishing
sleep and frequent bowel motions. Her past medical history
the aetiology. Hyperthyroidism is due to Graves’ disease
includes eczema localized to her hands, which is
(GD) in 80% of cases, and, therefore, it is important to
longstanding. Family history includes ischaemic heart disease
look for specific signs of GD, including:
in her father, diagnosed after a myocardial infarction at the
Smooth symmetrical thyroid goitre
age of 72, her mother suffers from vitiligo and her sister has
Graves’ ophthalmopathy (GO)
type 1 diabetes.
Pretibial myxoedema
Andrea has eye signs similar to the patient shown in
Endocrinology and Diabetes: Clinical Cases Uncovered. By R. Ajjan.
Fig. 40 (colour plate section). What abnormality do you
Published 2009 by Blackwell Publishing, ISBN: 978-1-4051-5726-1
see? How would that help in the diagnosis?
73
74
Part 2: Cases
The patient has proptosis, marked periorbital oedema
What other tests can be requested in
and conjunctival injection, indicating the presence
patients with hyperthyroidism?
Graves’ ophthalmopathy
Thyroid peroxidase antibodies are positive in up to
This is pathognomic for GD and is clinically evident in
80% of GD patients
around 50% of patients with the disease
Thyroid stimulating hormone receptor (TSHR) anti-
bodies are positive in around 99% of patients if sensitive
methods are used. However, these are not routinely
How would you clinically assess the
requested and are reserved for difficult cases and for
severity of the eye condition? What
pregnant patients
associated condition would you look
for?
What is the treatment of Graves’
The following signs are indications for urgent ophthal-
disease?
mology review:
There are three treatment options for patients with
Failure of full eye closure
Graves’ disease: medical treatment, radioactive iodine
Significant ophthalmoplegia
and surgery.
Evidence of optic nerve compression
The presence of a skin condition called pretibial myxo-
Medical treatment
edema in a patient with hyperthyroidism is also diagnos-
This is used to:
tic of GD but this is found in less than 10% of GD
Control thyroid function
patients and is almost always associated with clinically
Induce remission
detectable GO.
The most commonly used antithyroid drugs are
carbimazole and propylthiouracil. These can be given
What would you like to do next?
as:
A neck examination; this usually reveals a smooth and
A ‘block and replace regime’: a high dose of the drug
symmetrical goitre in patients with GD.
is used and once the hyperthyroidism is brought under
control, L-thyroxine
(T4) is added to avoid
hypothyroidism
What blood tests would you request?
Titration regime: once euthyroidism is achieved by
This patient should have her thyroid function (TFTs)
high-dose antithyroid drug, a low maintenance dose is
checked.
given to keep the patient euthyroid
Her TFTs show the following (normal ranges):
In the block and replace regime, treatment continues
Free T4 24.2 pmol/L (10.0-25.0)
for 6-12 months, whereas 18 months is usually required
TSH <0.05 mIU/L (0.2-6.0)
using the titration regime. Once the treatment is stopped,
the chances of disease remission are around 50%.
How do you interpret these results?
What would you do?
This patient has a normal FT4, suggesting euthy-
!RED FLAG
roidism
An important side effect, albeit rare, of antithyroid drugs
TSH is suppressed, consistent with hyperthyroidism
is agranulocytosis and all patients are advised to seek
The most likely explanation for these results is T3 toxi-
IMMEDIATE medical attention if they develop a sore
cosis, where the thyroid produces excess T3 without sig-
throat, mouth ulcers or high temperature to rule out this
nificant increase in T4 production
serious, and potentially, fatal complication of treatment.
Therefore, T3 levels should be requested
Her free T3 is 17.5 pmol/L (normal range 3.4-7.2).
Radioactive iodine (RAI)
What conclusion would you make?
This is administered orally, usually as a capsule, and it
This patient has hyperthyroidism due to T3 toxicosis
controls hyperthyroidism in the majority of patients. A
secondary to GD
minority of patients need a second, and rarely a third,
Case 2
75
dose. A large proportion of patients become hypothyroid
Her TFTs show:
with radioactive iodine therapy and will, therefore,
FT4 39 pmol/L
require thyroxine replacement therapy for life. RAI is
TSH <0.05 mIU/L
contraindicated in:
TSHR-antibodies negative
Children
Andrea is concerned about the possibility of thyrotoxicosis
Pregnant and lactating women
due to Graves’ disease.
Patients with urinary incontinence
Patients who cannot comply with the safety precau-
Do you agree with her? Why? What
tions following RAI treatment
other blood test would you request?
This treatment is best avoided in patients with moder-
This patient is unlikely to have Graves’ disease, because:
ate to severe active GO, particularly smokers, as RAI
She has pain and severe tenderness in her neck, which
may worsen the eye condition. In patients with active eye
are not usual features of Graves’ disease
signs in whom RAI treatment cannot be avoided, it is
She does not seem to have a goitre
advisable to cover with steroids starting just before RAI
She has no eye signs to suggest Graves’ ophthalmopa-
treatment and continuing with a reducing dosing regime
thy, nor does she have pretibial myxoedema
for around 6 weeks
Her symptoms followed a recent viral/bacterial
infection
Her TSHR antibodies are negative (see p. 15)
Surgery
Taken together (recent infection, thyrotoxicosis, severe
This is reserved for the following patients:
neck tenderness), the likely diagnosis is De Quervain’s
Failed or contraindicated medical therapy
thyroiditis, which usually occurs after upper respiratory
Unwilling to have RAI or in whom it is
infections, and thyrotoxicosis is due to thyroid destruc-
contraindicated
tion with consequent release of thyroid hormones, and
Graves’ disease and a suspicious thyroid nodule
not thyroid hormone overproduction by thyroid cells. In
Graves’ disease and compressive symptoms
thyroiditis, inflammatory markers are raised and, there-
Patient preference
fore, C-reactive protein (CRP) should be requested.
Early complications of thyroid surgery include:
Recurrent laryngeal nerve damage
What test would you request to confirm
Hypocalcaemia
the diagnosis?
Local haemorrhage
Thyroid uptake scan would confirm the diagnosis. In
Wound infection
Graves’ disease there is a uniform increased uptake in the
Thyroid storm in patients who are poorly prepared for
thyroid gland due to over-activity of the thyroid cells,
surgery
whereas in thyroiditis there is absent uptake due to
Late complications include:
thyroid destruction, as shown in Fig. 41.
Hoarse voice (secondary to recurrent laryngeal nerve
damage)
How would you treat this patient?
Hypothyroidism
Treatment is supportive:
Hypocalcaemia
β-blockers can be given to control the symptoms of
thyrotoxicosis
Three months later, Andrea brings her friend to see you,
Pain killers and non-steroidal anti-inflammatory
who has had classical symptoms of hyperthyroidism,
agents are given to control the pain
including hand tremor, increased sweating and palpitations,
Rarely, in severe cases, short courses of oral steroids
for a week. All these symptoms started following a flu-like
may be necessary
illness. On examination, she has signs suggestive of
The inflammation is self-limiting and the thyroid
hyperthyroidism, including a marked tremor and tachycardia,
gland usually recovers with or without a brief period of
but she has no eye signs or skin abnormality. No goitre is
hypothyroidism.
seen on neck inspection; neck palpation is difficult due to
exquisite tenderness in the area of the thyroid gland but no
What are the causes of thyrotoxicosis?
clear goitre is palpated.
The causes of thyrotoxicosis are listed in Table 29.
76
Part 2: Cases
(a)
(b)
Figure 41 Thyroid uptake scan showing (a) increased uptake in Graves’ disease and (b) decreased uptake in a case of thyroiditis.
Table 29 The aetiology of thyrotoxicosis.
Associated with increased thyroid hormone production
Not associated with increased thyroid hormone production
by thyroid cells
by thyroid cells
Graves’ disease (80% of cases of thyrotoxicosis)
Thyroiditis (de Quervain’s, postpartum thyroiditis, following
amiodarone treatment)
Toxic nodule or toxic multinodular goitre
TSH-secreting pituitary tumour
Exogenous thyroid hormone use
Trophoblastic tumour secreting human chorionic gonadotrophin
Production of thyroid hormones from ectopic thyroid tissue
(hCG) with TSH-like activity
(Struma ovarii: a teratoma in the ovary producing thyroid
hormones)
Pituitary thyroid hormone resistance (lack of negative feedback
on TSH secretion)
What are the main points in relation to
Box 18 Graves’ disease during pregnancy
long-term treatment of this condition?
Remission of Graves’ disease is frequently seen in
Achieve euthyroidism
pregnancy and the dose of antithyroid drugs can be
Medical treatment: a course of 6-18 months of anti-
reduced and often stopped altogether (usually in the
thyroid treatment can be given, which results in remis-
third trimester)
sion of 50% of Graves’ disease patients. Antithyroid
Propylthiouracil, rather than carbimazole, is used during
drugs do not induce disease remission in hyperthy-
pregnancy, as some reports suggest a link between
roidism due to other causes
carbimazole use in pregnancy and rare congenital
Radioactive iodine treatment is an effective treat-
defects in the newborn, although this is not fully
ment but commonly results in hypothyroidism. RAI
proven
should not be given to patients with active eye disease
Case 2
77
Surgery is a treatment option, particularly in indi-
Patients with Graves’ disease should be monitored for
viduals with large goitres or in those who could not
the occurrence of extrathyroidal complications
tolerate medical treatment
Box 19 Treatment of thyroid storm
A thyroid storm is a rare but life-threatening complication
Multisystem failure (heart, lung, kidney and liver)
of severe hyperthyroidism
Management
It can be precipitated by:
Patient should be transferred to intensive care and a
Infection
senior endocrinologist should be involved in the
Surgery
management
Radiographic contrast agents
Treat dehydration, arrhythmias and infection
Withdrawal of antithyroid treatment
Give high-dose propylthiouracil via nasogastric tube
Clinical manifestations include:
Give β-blockers (preferably propranolol) as an infusion
Confusion
Cover with high-dose steroids
High fever
Potassium iodide may be used, after starting anti-
Signs of severe hyperthyroidism (including
thyroid drugs, to inhibit thyroid hormone release
tachyarrhythmias)
CASE REVIEW
Andrea, who is 35 years old, consults her doctor with a
Treatment of Graves’ disease includes antithyroid drugs,
short history of palpitations, weakness and irritability. On
radioactive iodine and surgery, and these options should
further questioning, it becomes apparent that she has a
be discussed with the patients. A rare, and potentially fatal,
number of symptoms suggestive of hyperthyroidism
side effect of antithyroid drugs is agranulocytosis, and all
including heat intolerance, hand tremor and insomnia. On
patients should be warned of the possibility of this
examination, she is found to have a hand tremor, sweaty
complication.
palms and lid lag further suggesting hyperthyroidism. A
Jill, Andrea’s friend, also has classical symptoms of
raised T3 (requested after finding normal T4 levels) with
thyrotoxicosis with no clear goitre or eye signs but the neck
suppressed TSH confirms hyperthyroidism. A diagnosis of
is tender to palpation. Also, her symptoms started following
Graves’ disease is made, supported by the presence of a
a viral illness, raising the possibility of thyroiditis as a
smooth goitre and eye changes (Graves’ ophthalmopathy
cause. This is further supported by negative thyrotropin
or thyroid-associated ophthalmopathy). In unclear cases,
antibodies and the diagnosis is confirmed by demonstrating
thyrotropin receptor antibodies can be requested, which
the absence of technetium uptake on thyroid scan.
are positive in more than 95% of Graves’ patients, whereas
Treatment of this condition is symptomatic (β-blockers,
thyroid peroxidase antibodies are positive in around 80%.
pain killers and rarely a short course of steroids).
KEY POINTS
Hyperthyroidism is a common condition, affecting mainly
The commonest symptoms of hyperthyroidism include:
the female population, and Graves’ disease is the
heat intolerance, hand tremor, palpitations, frequent
underlying aetiology in 80% of cases. Other causes of
bowel motions, irritability and weight loss despite an
hyperthyroidism include toxic nodule or toxic multinodular
increase in appetite. However, some individuals,
goitre and thyroiditis
particularly the elderly, can present with non-specific
Hyperthyroidism due to Graves’ disease can be associated
symptoms (apathetic hyperthyroidism)
with extrathyroidal complications, including Graves’
Treatments for hyperthyroidism include:
ophthalmopathy and pretibial myxoedema
Antithyroid drugs: result in remission of hyperthyroidism
Continued
78
Part 2: Cases
due to Graves’ disease in around half of individuals.
Surgery: reserved for those with personal preference,
Hyperthyroidism due to other causes usually relapses
active eye disease with intolerance to medical
after stopping antithyroid drugs. Antithyroid drugs may
treatment, large disfiguring goitres and fears of
cause agranulocytosis and patients should be warned
malignancy
about the possibility of developing this serious, but
A prompt referral to an ophthalmology assessment is
fortunately rare, complication
required in patients with GO if they experience:
Radioactive iodine: one dose of radioactive iodine is
Decrease in visual acuity
effective at controlling hyperthyroidism in the majority
Problems with colour vision
of patients, but hypothyroidism and long-term thyroxine
Inability to fully close the eyelids (leaving the sclerae
replacement is a likely complication. This treatment
exposed)
should be avoided in pregnant women, individuals with
Sudden ophthalmoplegia
urinary incontinence, patients with active eye disease
and children (the latter is not an absolute
contraindication)
Case 3
A 61-year-old man with polyuria,
polydipsia, cough and weight loss
John, who is a 61-year-old postman, presents to his GP with
regular, and respiratory rate 22 breaths/min. A 3.5-cm mass
a 6-week history of increasing tiredness, polyuria and
can be felt in the left clavicular fossa. Chest examination
polydipsia. He also noticed weight loss over the past 6
shows dullness to percussion, reduced breath sounds and
months (around 5 kg) and an irritating cough that seems to
vocal fremitus on the left side. Abdominal examination is
have coincided with him stopping smoking around 5 months
unremarkable.
ago. His past medical history includes a partial gastrectomy
for a gastric ulcer 26 years ago, which according to John
Does this help in making the diagnosis?
was related to heavy alcohol intake. He was a heavy smoker
The patient has signs compatible with a left pleural effu-
(40/day) for 38 years, but stopped 5 months ago due to
sion, a common finding in lung malignancy. Also, there
increasing shortness of breath.
is a large mass in the supraclavicular fossa, which may be
due to lymph node metastasis from a primary lung
What is the differential diagnosis at this
cancer.
stage? What is the next step?
John presents with a 6-week history of polyuria and poly-
What tests would you request in
dipsia, the differential diagnosis of which includes:
this patient?
Electrolyte abnormalities such as hypercalcaemia
FBC: anaemia is commonly associated with malignant
Chronic renal disease
conditions. Pancytopenia can be seen with marrow inva-
Diabetes insipidus
sion by the tumour (advanced metastatic stage)
Osmotic diuresis due to high plasma glucose levels
U&Es: the patient is clinically dehydrated and hyper-
(diabetes mellitus)
calcaemia of malignancy can impair renal function. Also,
The use of drugs such as lithium and demeclocycline
U&Es should be checked in patients with polyuria and
The rest of the medical history includes:
polydipsia
Recent weight loss
Calcium: malignancy can be associated with hypercal-
Cough and increasing breathlessness
caemia and its presence would explain the polyuria and
A history of previous heavy smoking
polydipsia in this patient
The history of smoking, respiratory symptoms and
Glucose: this should be checked in any patient with
weight loss, should raise the suspicion of lung malig-
polyuria and polydipsia to rule out the possibility of
nancy. Polyuria and polydipsia may be due to hypercal-
diabetes
caemia, which can be associated with malignancy
LFTs: abnormal LFTs may indicate liver metastasis in
(hypercalcaemia of malignancy). The next step is full
a patient with suspected malignancy. However, normal
examination, with special emphasis on the respiratory
liver function does not rule out liver metastasis
system.
CXR: to investigate the abnormal physical findings
On examination, John looks dehydrated, has a temperature
Tests showed:
of 36.6°C, blood pressure of 130/75, pulse 92 beats/min
FBC: Hb 9.8 g/L, WBC 6.7× 109/L, platelets 365× 109/L
U&Es: Na 145 mmol/L, K 3.6 mmol/L, urea 15.1 mmol/L,
creatinine 192 μmol/L
Endocrinology and Diabetes: Clinical Cases Uncovered. By R. Ajjan.
Calcium: 2.8 mmol/L, corrected 3.2 mmol/L (normal range
Published 2009 by Blackwell Publishing, ISBN: 978-1-4051-5726-1
2.1-2.6)
79
80
Part 2: Cases
What are the causes of hypercalcaemia?
What is the aetiology of hypercalcaemia
of malignancy?
Causes of hypercalcaemia include:
1 Excessive bone resorption
Primary hyperparathyroidism: relatively common
condition, diagnosed by the presence of hyper-
calcaemia with elevated plasma PTH levels
Hypercalcaemia of malignancy without bony metas-
tasis: due to the production PTH-related peptides
(which mimic the action of PTH)
Hypercalcaemia of malignancy with bony metasta-
sis: bone destruction resulting in hypercalcaemia
Hyperthyroidism: a rare cause of
‘mild’ hyper-
calcaemia
Figure 42
2 Excessive gastrointestinal calcium absorption
Milk-alkali syndrome
Vitamin D toxicity
Random glucose: 5.6 mmol/L
Granulomatous disorders such as sarcoidosis
LFTs (normal ranges):
3 Increased renal reabsorption of calcium
ALT 110 U/L (8-40)
Thiazide diuretic use: these are associated with
AP 465 U/L (36-200)
hypocalciuria, which may result in mild hyper-
Bilirubin 34 umol/L (4-18)
calcaemia
Albumin 25 g/L (30-50)
Familial hypocalciuric hypercalcaemia: an autoso-
CXR (see Fig. 42)
mal dominant condition due to a mutation in the
calcium-sensing receptor. Diagnosis is made by dem-
Comment on these results. What other
onstrating low urinary calcium excretion (in primary
blood test would you request?
hyperparathyroidism, urinary calcium excretion is
These results show:
high)
FBC: mild anaemia, which is commonly seen in malig-
4 Uncertain mechanisms
nant conditions
Addison’s disease
U&Es: deranged renal function probably related, at
The mechanisms of hypercalcaemia of malignancy
least
in part, to dehydration secondary to
include:
hypercalcaemia
Tumour secretion of PTH-related peptide (PTHrP)
Corrected calcium is elevated. It should be noted that
that mimics the action of PTH
calcium levels should always be adjusted according to
Bony destruction due to metastases to the bone,
albumin levels, as uncorrected calcium may underesti-
consequently releasing the calcium into the blood
mate hypercalcaemia in subjects with low albumin
stream
Deranged LFTs suggest liver metastasis and this should
be further investigated with appropriate imaging
techniques
How would you treat John’s
CXR: a left pleural effusion consistent with the clinical
endocrine problem?
findings
Fluid replacement is the first step in the treatment of
Parathyroid hormone (PTH) plasma levels should be
hypercalcaemia due to any cause, and usually large
requested in any individual with raised plasma calcium
amounts are required (3-6 L of 0.9% saline over the first
to rule out the possibility of primary hyperparathyroid-
24 h)
ism. In hypercalcaemia of malignancy, PTH is undetect-
Once the patient is volume replete, intravenous
able, whereas in primary hyperparathyroidism PTH is
bisphosphonates can be used, which are very effective at
high or in the high normal range.
correcting clinically significant hypercalcaemia. Pami-
Case 3
81
dronate is most frequently used at doses of 30-90 mg
High-dose steroids may be necessary to treat some
diluted in 0.9% saline
resistant cases of hypercalcaemia of malignancy
Calcitonin can also be used to treat hypercalcaemia,
which is usually reserved for patients not responding to
bisphosphonate
CASE REVIEW
John, a 61-year-old postman, presents with a few weeks’
malignancy is a strong probability and the osmotic
history of tiredness, polyuria and polydipsia. Other
symptoms may be due to hypercalcaemia of malignancy,
complaints include significant weight loss and a cough that
frequently seen with advanced cancers. Blood tests confirm
coincided with him stopping smoking (used to smoke 40/
hypercalcaemia, in addition to abnormal liver function,
day for almost 40 years) around 5 months ago due to
which may be due to metastatic disease. From the endocrine
shortness of breath. The differential diagnosis of polyuria
point of view, John will need to be rehydrated first and
and polydipsia should be kept in mind and an appropriate
then treated with bisphosphonate infusion to control his
history taken. Weight loss and cough in a smoker should
hypercalcaemia. Longer term, appropriate management of
always raise the suspicion of lung malignancy. On
the lung condition should help to correct his hypercalcaemia
examination, John was dehydrated and a mass was palpable
but repeated bisphosphonate and even steroid therapy
in the supraclavicular fossa. Chest auscultation was
may also be required.
consistent with a left pleural effusion. Taken together, lung
KEY POINTS
Hypercalcaemia is a common condition and should be
Hypocalciuric hypocalcaemia (should be suspected in the
suspected in individuals with:
presence of family history of hypercalcaemia)
Osmotic symptoms (polyuria, polydipsia)
Treatments for hypercalcaemia include:
Abdominal pain
Treat the cause
Constipation
In severe symptomatic hypercalcaemia (commonly due to
Causes of hypercalcaemia include:
malignancy), management includes: intravenous
Primary hyperparathyroidism (common) and tertiary
rehydration, intravenous bisphosphonate and sometimes
hyperparathyroidism (renal failure patients, rare)
steroids (the latter can be effective in resistant
Hypercalcaemia of malignancy
hypercalcaemia of malignancy)
Dietary (milk alkali syndrome)
Complications of long-term untreated hypercalcaemia
Drugs (thiazides)
include renal calculi, nephrocalcinosis and renal failure
Case 4
A 44-year-old woman with
visual problems
Debra, a 44-year-old woman, is seen by her optician for
Excessive secretion of ACTH (Cushing’s disease):
recent deterioration of her vision. The optician performs a
Weight gain
visual field test, results of which are shown in Fig. 43.
Easy bruising
Proximal muscle weakness
What abnormality can you see?
Mood disturbance
The visual field test shows bilateral hemianopia.
Menstrual irregularities
Low libido and impotence
Where is the lesion?
Recurrent infections
The lesion is at the optic chiasm. The different types of
Symptoms of diabetes
visual field defects are shown in Fig. 44.
Excessive production of TSH (TSH-oma): symptoms
of hyperthyroidism as detailed in Case 2
What pathology does the above lesion
Excessive production of FSH or LH: these are rare and
suggest?
usually present in the same manner as a non-functioning
This suggests a pathology in the pituitary gland such as
pituitary adenoma
a pituitary tumour growing outside the pituitary fossa
If none of the above symptoms is present, then we are
and causing compression of the optic chiasm.
probably dealing with a non-functioning adenoma,
which, if large enough, may cause compression of healthy
What questions would you ask?
pituitary tissue resulting in a variable degree of pituitary
A pituitary tumour may be associated with increased
insufficiency. Therefore, symptoms of hormonal defi-
production of a pituitary hormone or may be a non-
ciency should be considered including:
functioning tumour. Large tumours may result in reduced
Growth hormone deficiency
production of one or multiple hormones due to com-
Tiredness
pressive effects on normal pituitary cells, and may also
Impaired psychological well-being
result in cranial nerve palsies due to invasion of the cav-
ACTH
ernous sinus. Therefore, the questions to ask would
Symptoms of primary hypoadrenalism, except for
concern the following symptoms:
the lack of pigmentation
Excessive production of prolactin (prolactinoma):
TSH
Galactorrhoea (90% of women, 10% of men)
Symptoms of primary hypothyroidism
Menstrual irregularities
FSH/LH
Low libido and impotence
Menstrual irregularities
Excessive secretion of growth hormone (acromegaly):
Reduced libido and erectile dysfunction in men
Change in glove or shoe size
Excessive sweating
What signs would you look for during
Arthralgia, headaches
examination of Debra?
Symptoms of diabetes
Prolactinoma
Galactorrhoea
Endocrinology and Diabetes: Clinical Cases Uncovered. By R. Ajjan.
Acromegaly
Published 2009 by Blackwell Publishing, ISBN: 978-1-4051-5726-1
Coarse facial appearance (prognathism, increased
82
Case 4
83
Figure 43 Results of visual field
testing.
Visual field defects
L
R
A
A. Unilateral visual loss
B
C
B. Bitemporal hemianopia
D
C. Homonymous hemianopia
Figure 44 Visual field defect. A. lesion in the optic
nerve causes loss of vision in the corresponding eye.
B. lesion in the optic chiasm results in a bitemporal
D. Upper homonymous
field defect. C,D. lesion distal to the optic chiasm
quadrantanopia
may result in contralateral homonymous hemi- or
quadrantanopia.
dental separation, frontal bossing, oily skin, tongue
Fig. 45 is an MRI of Debra’s brain. It shows a large
enlargement
pituitary tumour causing compression of the optic
Deep voice
chiasm, and, hence, the visual field defect.
Enlargement of hands and feet
Soft tissue swelling
(may result in carpal tunnel
syndrome)
What tests would you request to rule
Organomegaly
out hormonal excess or deficiencies?
Hypertension
Tests for hormonal excess include:
Cushing’s disease:
Prolactinoma: plasma prolactin levels
Facial appearance (round and plethoric face, acne
Acromegaly: glucose tolerance test. Administration of
and hirsutism)
glucose suppresses growth hormone production and
Truncal obesity with thin extremities
failure of this suppression is strongly suggestive of
Thin and fragile skin
acromegaly
Hypertension
Cushing’s disease: dexamethasone suppression test.
Osteoporosis (may cause vertebral fracture)
Administration of dexamethasone results in suppression
TSH-secreting tumour
of cortisol production. Failure of suppression indicates
Signs of hyperthyroidism
Cushing’s syndrome, which may be due to Cushing’s
84
Part 2: Cases
disease, ectopic ACTH production or excessive produc-
Debra has no symptoms or signs of hormonal excess or
tion of cortisol by an adrenal tumour
deficiency.
TSH-secreting tumour: TFTs. Thyroid hormones will
be elevated and, in contrast to primary hyperthyroidism,
What one endocrine blood test would
TSH will also be elevated
you request that may have important
Tests for pituitary hormone deficiency include:
implications for the clinical
TFTs (assess thyroid hormone status)
management in this case?
Glucagon stimulation test or insulin stress test (assess
It is important to exclude a prolactinoma (raised pro-
growth hormone and pituitary adrenal axis)
lactin levels), which may be clinically silent (except for
Tests for hormonal deficiencies are not required if the
the local effect), particularly in men
patient is scheduled for emergency surgery. The patient
The management of prolactinomas is usually medical,
will be covered with steroid during the operation and
and, therefore, patients can be spared surgical
tests for hormonal deficiencies will be done routinely
intervention
after the surgical procedure as surgery itself may result in
However, large non-functioning tumours may cause
damage to normal pituitary tissue, consequently result-
raised prolactin due to stalk compression, but prolactin in
ing in hypopituitarism.
this case is usually less than 6000 mU/L. In contrast, in
large pituitary macroprolactinoma
(the term ‘macro’
defines tumours >1 cm in diameter) prolactin levels are
usually >10 000 mU/L and can even exceed 100 000 mU/L
Provided all her hormonal tests are
normal, what is the diagnosis and best
treatment option?
The most likely diagnosis here is a large non-function-
ing pituitary adenoma
The best treatment option is surgical removal (usually
transphenoidal surgery)
One day after the initial assessment, Debra attends A&E with
severe headaches, double vision, dizziness and vomiting.
What does Fig. 46 show and what is the
diagnosis?
Fig. 46 shows complete right ptosis, abduction of the
right eye with mydriasis (dilated pupil)
Figure 45
The diagnosis is right third nerve palsy
Figure 46
Figure 5 Typical acromegalic facial features.
Courtesy of Dr Steve Orme.
(a)
Figure 9 Extrathyroidal manifestations of
Graves’ disease. (a) Graves’ ophthalmopathy
showing proptosis and previous
tarsorrhaphy in a patient with inactive
disease. (b) Quiescent myxoedema,
secondary to the accumulation of
glycosaminoglycans and associated
inflammatory infiltrate. (c) Graves’
acropachy, which looks similar to clubbing
and is due to subperiostal new bone
formation.
(b)
(c)
Facing p. 84
Figure 29 Deformed, red and hot ankle joint in a diabetes
subject should raise the suspicion of Charcot’s
osteoarthropathy. Diagnosis can be confirmed by X-rays (these
Figure 25 Characteristics of Turner’s syndrome; short stature,
can be normal in the early stages), magnetic resonance
webbed neck, poor breast development and widely spaced
imaging and isotope bone scans. Courtesy of Dr Carol Amery.
nipples. Courtesy of Dr Paul Belchetz.
Figure 38 Carcinoid flush affecting
(a)
(b)
the face. Courtesy of Dr Paul
Figure 35 Potential complications at insulin injection sites. (a) Lipoatrophy
Belchetz.
and (b) lipohypertrophy at insulin injection sites
Figure 40 Periorbital oedema, conjunctival infection, chemosis
and proptosis in an individual with active Graves’
ophthalmopathy. Courtesy of Mr Bernard Chang.
Figure 48 Courtesy of Dr Paul Belchetz.
(a)
(b)
(c)
Figure 53 (b) Courtesy of Dr D.A. Burns, Leicester.
Figure 47 Cushing’s disease (a), (b) before treatment, (c) after
treatment. Courtesy of Dr Dinesh Nagi.
(a)
Figure 56
(b)
Figure 60 (a, b) Paget’s disease of the skull.
Figure 62 Courtesy of Dr D.A. Burns, Leicester.
Case 4
85
What complication has occurred? How
Therefore, urgent medical treatment with intravenous
would the patient present and what
cortisol is required in patients with suspected pituitary
test would you request?
apoplexy. Low levels of other hormones are less critical
The most likely diagnosis is pituitary apoplexy (pitu-
short term, and these can be replaced later
itary infarction)
Neurosurgeons should be informed as these patients
The patient typically presents with sudden onset head-
are usually treated with early surgical intervention;
ache, vomiting, visual disturbances and cranial nerve
however, some are managed conservatively if there are
palsies
no visual or neurological symptoms
Diagnosis is based on typical MRI findings, which
should be requested urgently
How and where should patients with
previous pituitary tumours be followed
What urgent medical management does
up?
she need? What other treatment can be
Individuals with pituitary pathology should always be
offered?
followed up in specialized centres to monitor:
Pituitary apoplexy is usually associated with cessation
Recurrence of the disease
of pituitary hormone production due to infarction in the
Check for the development of complete/partial pitu-
pituitary tissue. This results in low ACTH levels, conse-
itary failure
(particularly those who had pituitary
quently leading to inadequate cortisol production by the
radiotherapy)
adrenal gland, which may result in a hypoadrenal crisis.
Ensure adequate hormonal replacement
CASE REVIEW
Debra, a middle-aged woman, is seen by her optician for
Debra’s endocrine tests are normal suggesting she has a
recent deterioration of her vision. Her visual field testing
non-functioning pituitary adenoma, for which surgical
shows bitemporal hemianopia suggesting a lesion in the
intervention is the best treatment option. Debra suddenly
optic chiasm, possibly secondary to a pituitary pathology.
develops severe headaches and third nerve palsy, associated
An appropriate history focussing on excess or deficient
with dizziness and vomiting. In view of the large pituitary
pituitary hormone production should be taken in any
tumour, pituitary apoplexy
(infarction) is a strong
individual with suspected pituitary tumour. In this case,
possibility, which is a known complication of pituitary
there are no clear indications for pituitary hormone excess
tumours. Neurosurgeons should be informed of the latest
or deficiency and Debra undergoes an MRI scan, which
development, an urgent MRI requested and the patient
shows a large pituitary tumour compressing the optic
should be covered with steroids (due to potential loss of
chiasm. It is important to rule out the possibility of
ACTH-secreting cells).
prolactinoma
(which responds to medical treatment);
KEY POINTS
Pituitary tumours may be non-functioning or functional
prolactinoma, which is unique amongst the pituitary
secreting one or more of pituitary hormones
tumours as it can be treated medically without the need
Individuals with bitemporal hemianopia should be
for surgery in the majority of patients
suspected as having a pituitary tumour. On the other
Following surgery, patients should be checked for pituitary
hand, pituitary tumours may occur without visual field
hormonal deficiencies using appropriate tests
defects if the tumour is not compressing the optic
Long-term monitoring for pituitary failure is warranted for
chiasm
patients who have had pituitary radiotherapy (for
Individuals with pituitary tumours should be assessed both
recurrent or incompletely resected pituitary tumours)
clinically and biochemically for pituitary hormone excess.
All pituitary patients should be followed up long term in
They should also be assessed for pituitary hormone
specialized centres to monitor disease recurrence,
deficiency and possible cranial nerve pathologies
development of pituitary failure and ensure adequate
The commonest functional pituitary tumour is a
hormone replacement
Case 5
A 20-year-old man with recent
diagnosis of diabetes
Richard, a 20-year-old man, is referred to the diabetes clinic
Proximal muscle weakness (hence inability to stand
with newly diagnosed diabetes (fasting glucose on two
from a squatting position).
occasions >10 mmol/L). He has had osmotic symptoms
Complications of the disease:
(polyuria and polydipsia) for at least 6 months and his
Diabetes
weight has recently increased by around 14 kg. Fig. 47a,b
Hypertension
(colour plate section) shows Richard’s face and abdomen.
The clinical features of Cushing’s syndrome are sum-
marized in Table 30.
What do you see?
The face appears round (moon-like)
What are the aetiologies of Cushing’s
There is truncal obesity and abdominal striae
syndrome and what tests would you
request here to confirm the diagnosis?
What questions would you like to
Cushing’s syndrome may be due to:
ask Richard? What signs would you
1. Pituitary adenoma secreting excessive ACTH
look for?
(Cushing’s disease)
Questions should be asked to look for evidence of:
2. Ectopic ACTH secretion due to a malignant tumour
Easy bruising
secreting ACTH
Muscle weakness
3. Adrenal tumour secreting excessive cortisol
Mood disturbances
4. Cushing’s syndrome secondary to exogenous steroid
Low libido and impotence
use (patients treated with steroids for, e.g. respiratory
Signs to look for include:
problems and rheumatoid arthritis)
Thin skin with easy bruising
Tests to diagnose Cushing’s syndrome include (detailed
Proximal muscle weakness
in Part 1, p. 31):
Hypertension
Midnight cortisol
Overnight dexamethasone suppression test
Richard is unable to stand from a squatting position, has
Low-dose dexamethasone suppression test
very thin skin with multiple bruising and is hypertensive at
24-h urinary cortisol
160/95 mmHg.
Once a diagnosis of Cushing’s syndrome has been made,
further tests should be done to establish the aetiology.
Does Richard have type 1 or type 2
A good clinical history can quickly exclude Cushing’s
diabetes? Why?
syndrome secondary to exogenous steroids
Neither. Richard has a secondary form of diabetes as a
To differentiate between 1, 2 and 3, ACTH should be
complication of Cushing’s syndrome. Richard has classi-
measured which is elevated in 1 and 2, whereas it is
cal features of Cushing’s syndrome including:
undetectable in 3 (excessive cortisol results in suppres-
Facial appearance (round ‘moon-like’ face)
sion of pituitary ACTH secretion)
Truncal obesity and abdominal striae
Differentiating between 1 and 2 can be more difficult:
Thin skin and easy bruising
A low potassium is suggestive of ectopic ACTH pro-
duction (hypokalaemia is found in 90%), but it is not
Endocrinology and Diabetes: Clinical Cases Uncovered. By R. Ajjan.
diagnostic as 10% of Cushing’s disease patients may
Published 2009 by Blackwell Publishing, ISBN: 978-1-4051-5726-1
have hypokalaemia
86
Case 5
87
Table 30 Symptoms and signs of Cushing’s syndrome.
Radiotherapy can be used as a second-line treatment
following:
Symptoms
Signs
Relapsed disease
Unsuccessful surgery
Weight gain
Facial appearance: round face, acne
Medical treatment is indicated in:
and hirsutism
The perioperative period
Truncal obesity
Central obesity and abdominal striae
Patients in whom surgery is contraindicated
While awaiting radiotherapy to take effect (which
Easy bruising
Thin skin and easy bruising
may take months to years)
Muscle weakness
Proximal muscle weakness
Drugs used include: metyrapone and ketoconazole
Mood disturbances
Hypertension
Adrenalectomy: this is usually reserved for cases not
(depression, psychosis)
responsive to the above treatment measures
Menstrual irregularities Impaired glucose tolerance or diabetes
What is cyclical Cushing’s?
Low libido and
Fractures due to osteoporosis
A minority of Cushing’s syndrome patients have
impotence
intermittent cortisol secretion, which can make the life of
Recurrent infections
Vascular disease
the investigating endocrinologist very difficult indeed.
The results of the dynamic tests can only be accurately
interpreted when the disease is clinically active, and,
High-dose dexamethasone suppression test: in pitu-
therefore, repeated investigations are required.
itary Cushing’s cortisol levels fall by >50% of basal
values. However, in a minority of ectopic ACTH-
secreting tumours, cortisol can be also suppressed by
more than 50% of basal values
Box 20 Complications of Cushing’s disease
Inferior petrosal sampling may be necessary to
measure levels of ACTH after corticotrophin releasing
Hypertension
hormone stimulation in the inferior petrosal sinus.
Impaired glucose tolerance/diabetes
High central
(in the petrosal sinus) to peripheral
Osteoporosis
(peripheral vein) ACTH levels are diagnostic of Cush-
Susceptibility to infection
ing’s disease
Easy bruising due to thin and fragile skin
If a diagnosis of ectopic ACTH is made, investigations
Other complications vary according to the aetiology of
should focus on finding the tumour secreting excessive
the disease. For example, patients with Cushing’s disease
ACTH.
may have complications related to the pituitary mass
(visual field defect, pituitary apoplexy). Patients with
ectopic ACTH secretion may develop complications
What are the treatment options for
secondary to the presence of a malignant tumour (e.g.
Cushing’s disease?
haemoptysis from a primary lung tumour or complications
Transphenoidal surgery is the best treatment but is not
arising secondary to metastasis).
always successful
CASE REVIEW
Richard is a young man who presents with new diagnosis
bruising, hypertension and proximal muscle weakness.
of diabetes and significant weight gain. He has evidence of
Cushing’s syndrome can be due to increased ACTH
excessive steroid hormone production
(Cushing’s
production by a pituitary tumour or by non-pituitary
syndrome) manifested as a round and ‘moon-like’ face,
malignant tissue (ectopic ACTH secretion), both of which
central obesity with abdominal striae, thin skin, easy
are associated with increased plasma ACTH levels. In
Continued
88
Part 2: Cases
contrast, Cushing’s syndrome due to an adrenal adenoma
syndrome can be difficult and requires specialist input.
or exogenous steroid administration are associated with
Complications of Cushing’s syndrome include hyper-
suppression of ACTH production.
tension, diabetes, increased susceptibility to infections and
Tests to diagnose Cushing’s syndrome include midnight
osteoporosis. Treatment of Cushing’s syndrome depends
cortisol, overnight dexamethasone suppression test, low-
on the aetiology of the condition and medical therapy with
dose dexamethasone suppression test and 24-h urinary
metyrapone or ketoconazole can be considered until
cortisol. Differentiation between causes of Cushing’s
definitive treatment measures are used.
KEY POINTS
Cushing’s syndrome is a condition that arises secondary to
Proximal muscle weakness
increased plasma cortisol levels
Hypertension and vascular disease
Cushing’s syndrome can be:
Diabetes
ACTH-dependent, e.g. pituitary tumours and ectopic
Increased susceptibility to infections
ACTH production by malignant tissue
Fractures due to osteoporosis
Non-ACTH dependent, e.g. adrenal adenoma and
Biochemical tests for the diagnosis of Cushing’s syndrome
prolonged, high-dose steroid treatment (for asthma,
include:
rheumatoid arthritis, etc.)
Midnight cortisol
The main clinical features of Cushing’s syndrome include:
Overnight or low-dose dexamethasone suppression test
Round, ‘moon-like’ face
Urinary cortisol (collection over 24 h)
Weight gain, central obesity and abdominal striae
Treatment of Cushing’s syndrome differs according to the
Thin skin and easy bruising
aetiology of the condition
Case 6
Tiredness and weight gain in a
30-year-old woman with diabetes
Iwona is a 30-year-old woman with known type 1 diabetes
How does this information help you?
for 12 years. She visits her GP complaining of tiredness and
The above suggests that Iwona has an organic cause for
weight gain. Her HbA1c levels over the past 7 years have
her tiredness that is probably not directly related to her
ranged between 6.4 and 7.1%, but her most recent test
diabetes
showed an HbA1c of 8.3%.
Her symptoms should be further explored keeping in
mind the association of type 1 diabetes with other endo-
What would you do at this stage?
crine autoimmune conditions such as hypothyroidism
This is a young woman with type 1 diabetes that has been
and hypoadrenalism
well controlled indicating that she is a reliable and a
Hypothyroidism is a common disease, particularly in
compliant patient. Her diabetes control has deteriorated
type 1 diabetes and questions regarding specific symp-
recently, which may be related to:
toms of hypothyroidism (Table 9, p. 18) should be asked
Weight gain
at this stage
Change in the dose of insulin
The possibility of hypoadrenalism (Addison’s disease)
Compliance issues
is less likely as this is usually associated with weight loss
Problems with
the
injection
sites
(i.e.
and hypoglycaemia (or reduced insulin requirements),
lipohypertrophy)
which are not seen here
Weight gain may be due to:
Change in lifestyle (different diet, less exercise)
On further questioning, Iwona tells you that her skin is
Endocrine problems
(Cushing’s syndrome,
getting very dry, is feeling constantly cold and her hair is
hypothyroidism)
becoming coarse and brittle. Also, she has had recent
Depression (some individuals with depression tend
problems with menstrual irregularities and has been
to eat more)
constipated.
Tiredness is a non-specific symptom (reviewed else-
where). In this particular patient, it may simply be due
What would you do now?
to deterioration in diabetes control
Iwona’s symptoms are consistent with hypothyroidism,
A more detailed history at this stage is essential, in
and, therefore, examination of her thyroid status should
particular addressing any change in diet, lifestyle or dose
be the next step (Part 1, p. 34).
of insulin.
On examination, Iwona indeed has dry skin and inspection
Iwona tells you that her diet has not changed but she is
of her face reveals periorbital puffiness. Her pulse is slow at
undertaking less exercise due to extreme fatigue. The dose
52 beats/min regular and she has slow relaxing reflexes.
of her insulin has not changed and she continues in her
Neck palpation reveals no goitre.
current job as a teacher and has no family problems or
social issues of note.
What is your diagnosis so far and what
tests would you request?
Iwona has classical signs of hypothyroidism (summa-
Endocrinology and Diabetes: Clinical Cases Uncovered. By R. Ajjan.
rized in Table 9, p. 18). Therefore, the likely diagnosis is
Published 2009 by Blackwell Publishing, ISBN: 978-1-4051-5726-1
autoimmune hypothyroidism (AH), which may occur:
89
90
Part 2: Cases
In the presence of a thyroid goitre, a goitrous form
What do these results indicate?
or Hashimoto’s thyroiditis
The patient seems to be well replaced with thyroxine
In the absence of a thyroid goitre, the atrophic form
as both her FT4 and TSH are in the normal range
or primary myxoedema
It is advisable to have the TSH between
0.2 and
The diagnosis can be confirmed by checking thyroid
2.0 mU/L in patients having thyroxine replacement
function tests
(TFTs) and thyroid peroxidase (TPO)
therapy, which is the case in this patient
antibodies
TFTs are expected to show low thyroid hormones
Iwona comes to see you 2 years later complaining of
and raised TSH
tiredness, muscle cramps, aches and weight gain.
TPO antibodies are usually positive in patients with
AH
What would you do?
Iwona’s TFTs should be checked as her symptoms are
Iwona’s tests show a FT4 of 6.1 mmol/L and TSH of
consistent with under-replacement with thyroxine.
81 mIU/L with positive TPO antibodies.
Her TFTs showed:
How would you manage this patient
FT4
22.1 pmol/L
now?
TSH
15.8 mU/L
These tests are consistent with AH and the patient will
need T4 replacement therapy
How do you explain these findings?
Thyroxine treatment can be started in a young patient
This is a relatively common finding in patients on
at a full replacement dose. In the older age group, in
thyroxine replacement and is usually indicative of
those with cardiac problems, and in longstanding hypo-
non-compliance
thyroidism, an initial small dose is advised with gradual
The patient is not taking thyroxine regularly causing
titration to an appropriate maintenance dose
an elevation of TSH. However, the patient takes the thy-
TSH should be rechecked around 6 weeks after starting
roxine before the blood test resulting in normal FT4 but
treatment or after modifying the dose of T4
TSH remains high
The maintenance dose of T4 is around 1.4 mcg/kg
It takes TSH a few weeks to normalize in patients
having thyroxine replacement and this is why TFTs
Iwona tells you that she is planning a pregnancy in the next
should not be repeated less than 4-6 weeks following
year or so.
initiation or change in treatment
What advice would you give her?
Iwona admits to having some difficulties at work resulting in
Pregnant hypothyroid women usually need a 30-50%
non-compliance. These issues are subsequently resolved and
increase of T4 dose and this should be fully explained to
her TFTs normalize 3 months later.
patients with hypothyroidism of child-bearing age
She comes to see you again with a skin condition, as
Iwona should inform her endocrinologist once she
shown in Fig. 48 (colour plate section).
becomes pregnant, in order to increase the dose of T4
and make appropriate arrangements to monitor TFTs
What is the diagnosis?
during pregnancy
Iwona’s skin shows areas of decreased pigmentation
The diagnosis is vitiligo
Iwona’s symptoms completely disappear on 100 mcg of T4,
which is further increased to 150 mcg when she becomes
Is Iwona’s skin condition related to her
pregnant 9 months later. She goes through an uneventful
thyroid disease?
pregnancy and the dose of T4 is decreased after delivery to
Vitiligo is an autoimmune condition that can be asso-
100 mcg/day. Twelve months after delivery her TFTs showed
ciated with autoimmune disorders, particularly autoim-
a FT4 of 18.6 pmol/L and TSH 1.2 mU/L on 100 mcg T4.
mune thyroid disease
Case 6
91
Box 21 Causes of hypothyroidism
Autoimmune thyroid disease
Congenital development and hereditary biosynthetic
Non-goitrous: atrophic hypothyroidism
defects
Goitrous: Hashimoto’s thyroiditis
Iodine deficiency
Thyroiditis (including postpartum thyroiditis):
Thyroid surgery
hypothyroidism is usually preceded by a brief period of
Secondary (lesion in the pituitary gland or hypothalamus)
hyperthyroidism. It is a self-limiting disease and thyroid
Thyroid hormone resistance (peripheral tissue fails to
function usually normalizes, with or without a brief period
respond to thyroxine)
of thyroxine treatment
See Table 9, p. 18 for symptoms and signs of
Drug-induced: amiodarone, lithium
hypothyroidism.
Post radiation or following treatment with radioactive
iodine
CASE REVIEW
Iwona is a young woman with known type 1 diabetes. She
treated with thyroid hormone replacement. Subsequently,
presents to her GP with classical symptoms of
she becomes pregnant necessitating an increase in the dose
hypothyroidism including tiredness, weight gain, dry skin,
of thyroxine, due to increased requirement of this hormone
cold intolerance, brittle hair and menstrual irregularities,
during pregnancy. Two years after giving birth, her blood
in addition to deterioration in her glucose control. A
test shows raised TSH with normal FT4, which turns out
diagnosis of autoimmune hypothyroidism is made by
to be secondary to non-compliance, the commonest cause
demonstrating low plasma FT4 with increased TSH and
of such a blood abnormality in patients having thyroxine
positive TPO antibodies. Her condition is successfully
replacement.
KEY POINTS
Hypothyroidism is a common condition, particularly in the
Bradycardia
presence of personal or family history of autoimmunity,
Slow-relaxing reflexes
and it affects mainly the female population
Biochemical abnormalities in hypothyroidism include a
The commonest aetiology is related to thyroid
raised TSH with low or low-normal FT4. Thyroid
autoimmunity. Other causes include thyroiditis, medical
peroxidase antibodies are usually positive in
treatment (amiodarone, lithium) and iodine deficiency
hypothyroidism secondary to autoimmunity
Hypothyroidism can present with a wide range of clinical
Treatment of hypothyroidism is simple and involves
symptoms/signs, the commonest being:
thyroid hormone replacement, usually in the form of L-
Tiredness
thyroxine (T4)
Cold intolerance
Dose of thyroxine usually needs adjustment during
Dry skin, brittle hair and puffy face
pregnancy or after weight gain/loss
Weight gain
Constipation
Case 7
Acute confusion in an 82-year-old with
known type 2 diabetes
Brian, an 82-year-old gentleman with known type 2
Nutritional deficiencies
diabetes, is brought to A&E with general deterioration and
B12
acute confusion.
Thiamine (particularly in alcoholics)
The first step is to take a proper history to narrow
What differential diagnoses would you
down the differential diagnosis. Questions asked should
consider and what would you do?
include:
Older people commonly present to hospital with acute
Onset of confusion
and subacute confusional states. The differential diagno-
Acute
sis is wide and includes:
Acute on chronic
Infection:
Associated symptoms/previous history
Urinary tract infections
(UTIs), which are very
Urinary symptoms or incontinence (UTI)
common, particularly in women
Cough or shortness of breath (chest infection)
Chest infections
Weakness in arms or legs or slurred speech
Encephalitis and meningitis (rare)
(stroke/TIA)
Drugs and alcohol
Any falls (even mild head bumps may result in a
Intoxication (opiates, sedatives, anticholinergics)
subdural haematoma, particularly in patients treated
Withdrawal
with warfarin)
Hypoxia
History of alcohol abuse
Central (sedatives)
Detailed drug history
Pulmonary (infection)
Metabolic
It is not possible to take a history from Brian as he is
Uraemia
confused and agitated. Members of his family tell you that
Liver failure
they saw Brian a week ago when he was absolutely fine.
Hypoglycaemia
They stress that he is usually in good health and does his
Hypercalcaemia
shopping and cooking and has been managing alone for 5
Vascular
years after the death of his wife. Apart from diabetes and
Stroke
‘mild’ hypertension, both diagnosed 10 years ago, he has
Transient ischaemic attack (TIA)
never had any problems with his health. He drinks
Intracranial lesion:
occasionally (1-4 units/month). His medications include:
Raised intracranial pressure (due to a brain tumour
Metformin 850 mg b.d.
for example)
Gliclazide 40 mg b.d.
Subdural haematoma
Aspirin 75 mg o.d.
Epilepsy
Atorvastatin 10 mg o.d.
Temporal lobe epilepsy
Bendrofluazide 2.5 mg o.d.
Post ictal states
Does this help you to rule out any of
differential diagnoses mentioned
Endocrinology and Diabetes: Clinical Cases Uncovered. By R. Ajjan.
above? What would you do next?
Published 2009 by Blackwell Publishing, ISBN: 978-1-4051-5726-1
Although limited, the history from the family estab-
92
Case 7
93
lishes that this is an acute confusional state in an elderly
What would you do next?
gentleman who is managing to live alone with no appar-
The normal heart and abdominal examination make
ent problems
primary pathology in the cardiovascular and gastrointes-
The list of Brian’s medications does not include any
tinal system unlikely, although do not completely rule it
sedatives or opiates and his alcohol intake is minimal,
out
ruling out drugs/alcohol as a cause of his confusion.
Although full neurological examination was difficult,
However, he is on gliclazide, which may cause hypogly-
the ‘brief version’ described above indicates that it is
caemia. At this stage, an urgent test of capillary glucose
unlikely a major neurological pathology is causing the
is required
above abnormalities. However, this examination does
not conclusively rule out a neurological condition. For
The nurse performs a set of initial assessments and these
example, a subdural haematoma does not necessarily
reveal:
cause any weakness and may manifest as unexplained
Glasgow Coma Scale (GCS) 13/15 (E3, V5, M5)
confusion
Blood pressure 100/58
The respiratory examination is consistent with lung
Pulse 110/min regular
consolidation, making a diagnosis of pneumonia a strong
Temperature 37.4°C
possibility
Respiratory rate 30/min
ABG analysis shows:
O2 saturation 89%
Hypoxia
Capillary glucose ‘high’
Mild metabolic acidosis
Secondary hypocapnia (trying to correct the meta-
What would you do next?
bolic acidosis)
The patient is hypotensive and tachycardic with a drop
It is essential to test the urine for ketonuria in any
in GCS
diabetes patient with high blood glucose, particularly in
He has low-grade temperature
the presence of acidosis, to rule out the possibility of
His oxygen saturation is low
diabetic ketoacidosis.
His capillary glucose is ‘high’ indicating blood sugar
probably in excess of 30 mmol/L (most capillary glucose
Brian’s urine dipstick shows:
meters fail to accurately measure very high glucose levels
Glucose +++
and simply refer to these as ‘high’)
Ketones +
As Brian is tachypneic with low oxygen saturation, he
WBC negative
should:
Nitrates negative
Undergo a full physical examination with special
emphasis on the respiratory system
His arterial blood gases should be checked
Does Brian have DKA? What other
He should also be started on oxygen therapy
conclusions can be made from the urine
dipstick results?
On examination, Brian is clinically dehydrated although
Brian’s urine dipstick results are not compatible with
cardiovascular and abdominal examination are both normal.
DKA due to the absence of heavy ketonuria. Mild keto-
Chest auscultation indicates decreased percussion note on
nuria can be frequently seen, particularly in fasted
the right with increased vocal fremitus and bronchial
individuals
breathing. Neurological examination is difficult as the
The urine dipstick fails to show white cells or nitrates
patient is uncooperative, but it is noted that he is:
in the urine making a UTI an unlikely diagnosis
confused and agitated, moving all four limbs, his pupils are
normal in size with a normal light reflex and plantars are
down going.
What other tests would you request in
Brian’s arterial blood gas (ABG) analysis showed:
this patient?
PO2 7.1 kPa
FBC: looking for raised white cells
(infection),
PCO2 2.3 kPa
anaemia
HCO3 16 mmol/L
Blood cultures: raised temperature, likely chest
pH 7.32
infection
94
Part 2: Cases
U&Es: checking kidney function, particularly in view
What is the anion gap? What are the
of the dehydration and acidosis
potential causes of his acidosis?
Glucose: in view of the history of diabetes and raised
Anion gap = (sodium + potassium)
capillary glucose
- (chloride + bicarbonate)
CXR: in view of the positive findings on examination
ECG: all acutely unwell individuals, particularly diabe-
Anion gap = (148 + 4.0) (111 + 15)
tes patients, should have an ECG done to rule out silent
= 26 (normal 12-20)
myocardial infarction and cardiac arrhythmias
His anion gap is high. Causes of high anion gap meta-
To complete the confusion screen, the following should
bolic acidosis are outlined in Case 1.
be checked:
LFTs
What is his calculated plasma
Calcium
osmolarity?
B12 plasma levels
Plasma osmolarity can be calculated from the
formula:
What is the diagnosis so far and what
would you do while awaiting the results
2 (sodium + potassium) + urea + glucose
of the above tests?
2 (148 + 4.0) + 30 + 54 = 388 (normal 285-295)
In complicated cases, it is advisable to make a list of
the abnormalities, which usually helps in organizing
His calculated plasma osmolarity is greatly increased.
further investigations and reaching the correct diagnosis.
The abnormalities in this case thus far:
What does his X-ray show? (Fig. 49)
Acute confusion
The X-ray shows right middle lobe pneumonia.
Signs of dehydration
Signs of chest infection
What test would you request next?
Hyperglycaemia
Brian has an infection and metabolic acidosis; there-
Hypoxia
fore, lactic acid levels should be requested. Metformin
Metabolic acidosis (mild, partially compensated)
can also cause lactic acidosis particularly in the presence
Taken together, the most likely diagnosis is pneumonia
of renal failure and this is another reason to check lactate
complicated by hyperglycaemia, dehydration and meta-
levels.
bolic acidosis, resulting in confusion and reduced
GCS.
Brian’s lactic acid levels are 5.8 mmol/L (1.0-2.4).
The patient should be started on i.v. fluid (0.9% saline)
due to dehydration and low blood pressure, as well as
What are the diagnoses?
broad spectrum i.v. antibiotics (after taking appropriate
Chest infection associated with hypoxia and
cultures) for his chest infection. Oxygen treatment should
hypotension
continue.
Hyperosmolar non-ketotic hyperglycaemia resulting
in dehydration and contributing to low blood pressure
Brian’s condition quickly deteriorates and his GCS falls to
Metabolic acidosis due to:
8/15 (E2, M4, V2). His blood tests show:
Raised lactic acid (secondary to infection, hypoxia
FBC Hb 15.7 g/L
and possibly metformin treatment)
WBC 28.3 (neutrophils 25.2)
¥ 109/L
Deranged renal function may have also contributed
Platelets 293
¥ 109/L
to the metabolic acidosis
U&Es: Na 148 mmol/L
K 4.0 mmol/L
How would you treat Brian?
Cl 111 mmol/L
Brian requires treatment for:
Urea 30.1 mmol/L
Chest infection
Creatinine 223 μmol/L
Hyperosmolar non-ketotic hyperglycaemia
Bicarbonate 15 mmol/L
Treatment of the infection and normalization of renal
Glucose 54 mmol/L
function will correct Brian’s metabolic acidosis.
Case 7
95
Box 23 Treatment of hyperosmolar non-ketotic
hyperglycaemia
Broadly similar to that of DKA, but with some differences:
Fluid: Fluid replacement should be more gentle in
hyperosmolar hyperglycaemia compared with DKA as
these are older patients and more prone to heart failure
with aggressive fluid replacement. In difficult cases, a
central line should be inserted that should guide
appropriate fluid replacement, to avoid sending the
patient into heart failure
Insulin: Despite the very high glucose levels in these
patients, insulin requirements in non-ketotic
hyperosmolar hyperglycaemia are modest and, therefore,
Figure 49 Courtesy of the Radiology Department, University
insulin should be given at 0.5-2 units/h to achieve a
of Leeds.
gradual drop in blood sugar (around 5 mmol/L/h)
Potassium: In uncomplicated hyperosmolar non-ketotic
hyperglycaemia, potassium levels do not drop that
quickly with treatment due to the absence of acidosis.
Box 22 Treatment of chest infection
However, the patient may have acidosis due to other
causes (as in the present case) and potassium should,
Intravenous administration of broad spectrum antibiotics
therefore, be monitored carefully
Respiratory support as necessary
Bicarbonate: This is not needed in uncomplicated
hyperosmolar hyperglycaemia as the patient is not
usually acidotic
Precipitating cause(s): Infection is the most common
What is the prognosis in this case?
precipitating cause and, therefore, antibiotic cover must
The prognosis of hyperosmolar non-ketotic hypergly-
be started after appropriate cultures
caemia is unfortunately poor
Other measures: Due to high osmolarity and
More than a third of patients die, commonly from
dehydration, thrombotic disease is very common in
thromboembolic disease
these patients and, therefore, all should be covered
with prophylactic unfractionated heparin (unless
Brian is treated with i.v. antibiotics, fluid and insulin and
haemorrhage is suspected)
Monitoring: This should be done regularly with blood
makes a very good recovery, with all his blood parameters
samples taken every 2 h in the first 6-8 h to assess
returning to normal 48 h after admission. However, 60 h
response to treatment
after his admission, he complains of sudden onset shortness
of breath. He denies chest pain.
On examination, blood pressure is 110/65 (145/85 earlier
What is the most likely differential
that day), pulse 104 beats/min regular, O2 saturation 90%
diagnosis?
(98% earlier in the day) and respiratory rate 32 breaths/min.
The most likely differential diagnosis here is:
Cardiovascular auscultation reveals an additional S3 gallop
Pulmonary embolism
heart sound. Chest auscultation shows bilateral basal
Relapsed/partially treated chest infection
crepitations.
Myocardial infarction
Are these findings compatible with a
What would you do?
pulmonary embolus (PE) and why?
A physical examination, concentrating on the cardio-
Although PE is a strong possibility, the clinical findings
vascular and respiratory system
do not fit this diagnosis. In large PE, S3, due to right
ECG
ventricular dysfunction, may be heard but bilateral basal
CXR
crepitations are not a feature and these are usually found
Routine bloods
in left ventricular failure
96
Part 2: Cases
l I
l aVR
l
IV1
l
V4
l II
l aVL
l
V2
l
V5
l III
l aVF
l
V3
l
V6
Figure 50
Table 31 Main features and management of hyperosmolar non-ketotic hyperglycaemic (HONK)
Age group
Older people with or without a history of diabetes (this could be the first presentation of diabetes)
affected
Glucose
Usually very high (30-80 mmol/L)
Acidosis
Not a feature unless complicated by metabolic acidosis due to other causes (i.e. infection or myocardial infarction)
Serum
Very high (>350 mmol)
osmolarity
Ketonuria
There is an absence of severe ketonuria but mild to moderate ketonuria is common (starvation/vomiting)
Precipitating
Common: suspect an infection or a vascular event
factor
Management Gentle i.v. fluid
Gentle i.v. insulin
Prophylactic heparin is mandatory (unless a bleed is suspected)
Aggressive use of i.v. antibiotics is encouraged
The most likely diagnosis here is left ventricular dys-
What is the diagnosis? How would you
function resulting in pulmonary oedema
explain the absence of chest pain?
The diagnosis is acute myocardial infarction causing
Brian’s ECG on admission did not show major abnormalities,
left ventricular dysfunction
his repeat ECG is shown in Fig. 50.
Silent MI (no chest pain) is common in diabetes
patients, and this should be taken into account when
What does the repeat ECG show?
assessing these individuals
The ECG shows ST elevation in V1-V4 indicating acute
The main features of hyperosmolar non-ketotic hyper-
anterior-septal myocardial infarction.
glycaemia are summarized in Table 31.
CASE REVIEW
Brian is an older gentleman who is brought into hospital
lesion or metabolic/nutritional derangements. An
with general deterioration and acute confusion, a common
appropriate history, taken usually from relatives or friends,
clinical presentation in this age group. The differential
is important to give some clues to the cause of the
diagnosis is wide and includes infection, intoxication with
confusion. Physical examination in this gentleman is con-
drugs or alcohol, hypoxia, vascular event, intracranial
sistent with dehydration and chest infection. Subsequent
Continued
Case 7
97
tests show biochemical evidence of dehydration with raised
However, 2 days later he complains of sudden onset
glucose, together with hypoxia and lactic acidosis secondary
breathlessness without chest pain, which was due to heart
to chest infection. The diagnosis is hyperosmolar non-
failure secondary to silent myocardial infarction. The latter
ketotic hyperglycaemia precipitated by a chest infection.
is common in patients with diabetes and should be
Brian is treated with intravenous antibiotics, fluid and low-
considered in those with recent history of shortness of
dose insulin, subsequently making a good recovery.
breath.
KEY POINTS
Hyperosmolar non-ketotic hyperglycaemia (HONK) is a rare
Acidosis is not a feature of HONK, unless it is due to the
complication of type 2 diabetes and usually affects the
associated condition, and there is no heavy ketonuria
older patient
(mild ketonuria may be present due to starvation)
HONK carries a poor prognosis as mortality rates can be
Patients with HONK should be treated with intravenous:
as high as 50%
Fluid (careful not to overload and precipitate heart
In more than half the cases, HONK occurs in patients who
failure)
are not known to have diabetes
Insulin (only small doses are required)
HONK is frequently precipitated by an infection or
Antibiotics after appropriate cultures (infection is a
vascular event and patients are severely dehydrated with
common precipitating cause)
impaired kidney function and very high plasma glucose
Prophylactic heparin (vascular thrombosis is a common
levels
cause of death in patients)
Case 8
A 42-year-old man with headaches,
increased sweating and
sexual dysfunction
Adrian, who is 42 years old, is seen by his GP for a 5-month
The patient tells you that his:
history of headaches, increased sweating, tiredness and
Headaches are constant, partially relieved by pain killers,
sexual dysfunction.
generalized but more prominent in the frontal region,
have no clear associated symptoms, and he is not
How would you proceed?
suffering increased stress at work or home
Unfortunately, each of the above symptoms is encoun-
Sweating is constant, can be very severe (he sometimes
tered very frequently and at this stage it is unclear
changes his shirt three times/day), and is nocturnal as well
whether:
as during the day
Symptoms are caused by the same pathology
Sexual dysfunction, he can achieve partial erection but this is
Symptoms are the result of a number of independent
insufficient for sexual intercourse, and he has had
pathologies
decreased libido, particularly in the past 3 months
Therefore, more information should be obtained on
Tiredness is severe, particularly in the past 4 weeks, and is
each of the above symptoms.
associated with musculoskeletal aches and pains
Headache
Adrian says that all the above symptoms started around 5
Duration
months ago.
Frequency
Location
Does the above help in the diagnosis?
Associated symptoms
The constant headache for 5 months rules out causes of
Provoking and relieving factors
acute and of recurrent headaches.
Social circumstances (e.g. increased stress at work,
family problems)
Box 24 Causes of acute and recurrent headaches
Increased sweating
Increased sweating and hot flushes can be features
Acute headache
Subarachnoid haemorrhage
of hypogonadism in men
Meningitis and encephalitis
Episodes of increased sweating can occur in endo-
Acute sinusitis (but chronic sinusitis is a possibility)
crine diseases such as phaeochromocytoma, acromeg-
Dental caries
aly and carcinoid syndrome
Recurrent headaches
May be psychological
Migraine
May be idiopathic
Cluster headache
Tiredness
Trigeminal neuralgia
A non-specific symptom, the differential diagnosis
Glaucoma
of which is discussed elsewhere (Case 10)
Sexual dysfunction
Subacute headache due to temporal arteritis is unlikely
This is discussed in detail elsewhere (Case 21)
as this occurs after the age of 50 and is commonly associ-
ated with scalp tenderness. Therefore, the cause of Adri-
Endocrinology and Diabetes: Clinical Cases Uncovered. By R. Ajjan.
an’s headache is probably related to one of the following
Published 2009 by Blackwell Publishing, ISBN: 978-1-4051-5726-1
diagnoses:
98
Case 8
99
Tension or psychogenic headache
The feeling of pins and needles in the hands may be
This is a diagnosis of exclusion
due to carpal tunnel syndrome, which can be associated
Increased intracranial pressure
with acromegaly secondary to soft tissue swelling and
Usually associated with focal neurological signs
compression of the median nerve
Papilloedema on funduscopy
Signs to look for include:
Pituitary tumours
Facial appearance (an old photo of the patient is
These can cause a headache, which is usually related
helpful to look for changes): coarse features with
to tumour size
prominent supraorbital ridges, increased dental sepa-
In acromegaly the headache is independent of
ration, prognathism (protrusion of the lower jaw),
tumour size
enlarged nose, lips and tongue and deep voice
Enlarged hands (spade-like hands)
A full neurological examination and funduscopy are both
Thick, oily skin
normal. Formal visual field testing is normal.
Hypertension
Signs of carpal tunnel syndrome
Goitre and organomegalies
Would you make a diagnosis of tension
headache at this stage?
Figure 51 (and Fig. 5 in colour plate section) shows a picture
A normal neurological examination and the absence
of Adrian. He has a deep voice, greasy skin and his blood
of papilloedema on funduscopy make the diagnosis of
pressure is 170/95.
raised intracranial pressure less likely
Normal visual field testing does not rule out a pituitary
adenoma
How does this help you in making
The above information is not enough to make a diag-
a diagnosis?
nosis of tension headache
Adrian has prominent supraorbital ridges and coarse
A more detailed history is required asking specifically
facial features
for symptoms of pituitary hormone excess
Deep voice, greasy skin and hypertension are classic
features of acromegaly
Adrian’s main symptoms are headache and sweating.
Therefore, the patient is very likely to have
acromegaly
Which pituitary hormone excess should
What biochemical and what radiological
be ruled out first and what specific
tests would you request to confirm the
questions would you ask?
diagnosis?
Increased sweating and headache are common symptoms
Oral glucose tolerance test: in a normal person, plasma
of patients with acromegaly. Therefore, the questions to
growth hormone levels are suppressed after an oral
be asked should include:
glucose tolerance (OGT) test. In acromegaly there is a
Changes in glove or shoe size
failure of growth hormone suppression after OGT
Changes in facial appearance
MRI of the pituitary looking for a pituitary tumour
Arthralgia
Symptoms of diabetes
Adrian fails to suppress plasma growth hormone after OGT
and his MRI shows a pituitary tumour measuring 1.5 cm,
Adrian tells you that he has been unable to take off his
with no optic nerve compression.
wedding ring recently and he has had generalized pain in
his joints as well as a sensation of pins and needles in his
What is the best treatment option for
hands.
this patient?
Surgical intervention, usually through a transphenoi-
What signs would you look for?
dal approach, is the preferred treatment
Inability to take off the wedding ring and joint
Radiotherapy is reserved for patients who fail surgical
pain are consistent with the suspected diagnosis of
treatment or have contraindication to surgical
acromegaly
treatment
100
Part 2: Cases
What medical treatments are there for
Table 32 Main symptoms, signs and complications of growth
hormone excess and deficiency.
patients with acromegaly?
Somatostatin analogues (octreotide, lanreotide)
Growth hormone excess
Growth hormone deficiency
Dopamine agonists (bromocriptine, cabergoline)
Growth hormone receptor antagonist (pegvisomant)
Symptoms
Symptoms
Radiotherapy
Fast growth (in children)
Failure of growth (in children)
Headaches (independent of
Tiredness
local tumour effect)
Depression
Increased sweating
Box 25 What are the complications of
Decreased body mass
acromegaly?
Musculoskeletal pains
Change in glove/ring and
Hypertension
shoe size
Diabetes or impaired glucose tolerance
Obstructive sleep apnea
Signs
Signs
Increased risk of colonic polyps and colonic carcinoma;
Facial appearance (see text)
Failure of growth and thin
therefore, routine colonoscopy is recommended
skin in children
Soft tissue and skeletal
Ischaemic heart disease, cerebrovascular disease and
changes
No specific signs in adults
heart failure
Organomegaly
Visual field defect
Deficiency of other pituitary
Can clinical and biochemical acromegaly
hormones
occur in the absence of a primary
pituitary pathology?
Complications
Complications
Yes, very rarely acromegaly may occur secondary to
Hypertension
Short stature in untreated
children
excessive secretion of growth hormone releasing hormone
Diabetes
(GHRH)
Hypoglycaemia (mainly in
Colonic polyps and colonic
Increased secretion from the hypothalamus
children)
carcinoma
Ectopic secretion from a tumour (such as carcinoid)
Osteoporosis in adults
can result in acromegaly without a primary pituitary
Obstructive sleep apnoea
pathology but this is rare.
CASE REVIEW
Adrian, a middle-aged man, presents with a few months’
presence of an adenoma, indicating that the patient’s
history of headaches, increased sweating, tiredness and
symptoms are due to a growth hormone secreting pituitary
sexual dysfunction. Further questioning reveals changes in
tumour. Treatment of this condition includes surgery and
hand size and symptoms compatible with carpal tunnel
radiotherapy; medical treatment is only partially effective
syndrome. On examination, Adrian has facial features of
at controlling growth hormone production. Complications
acromegaly together with greasy skin and hypertension.
of acromegaly include hypertension, diabetes, obstructive
His neurological examination is normal and he has no
sleep apnea, increased risk of colonic carcinoma and
visual field defects. An OGT test fails to suppress growth
vascular disease
hormone production and a pituitary MRI confirms the
Case 8
101
KEY POINTS
Acromegaly is a rare condition that usually results from
Surgery is the best treatment option for this condition but
excessive pituitary growth hormone production. If excessive
radiotherapy and medical treatment are considered for
growth hormone is produced during childhood, it causes
incomplete resection of the tumour, disease relapse or for
gigantism, whereas increased hormone production in
individuals not fit for surgery
adulthood leads to acromegaly
Complications of acromegaly include:
The main clinical manifestations of acromegaly include:
Hypertension
Headaches
Cardiovascular disease
Increased sweating
Diabetes mellitus
Tiredness
Obstructive sleep apnea
Change in ring, glove or shoe size
Increased risk of colonic carcinoma
Typical facial appearances and deep voice
Hypopituitarism and visual field defect
Entrapment neuropathies due to soft tissue swelling
(carpal tunnel syndrome)
Case 9
Amenorrhoea in an 18-year-old
Sutapa, who is 18, attends her GP clinic complaining of
stress, or hypothalamic tumours or infiltrative lesions]
amenorrhoea.
or general endocrine (these are usually associated with
menstrual irregularities rather than amenorrhoea
What is the differential diagnosis and
and include thyroid dysfunction and Cushing’s
how would you proceed with this
syndrome)
patient?
A full menstrual history is required in order to narrow
Causes of amenorrhoea can be (Fig. 51):
down the differential diagnosis.
Physiological
Pregnancy: occasionally some patients do not con-
Sutapa tells you that her menarche occurred at the age of
sider this as a possibility, and rarely the physician fails
13 and her periods have been regular until 9 months prior
to rule this out initially, ending up requesting compli-
to her presentation, when they became less frequent (every
cated, expensive and unnecessary tests
6-7 weeks) and subsequently stopped altogether more than
Lactation
5 months ago. She had three pregnancy tests (most recent a
Menopause
week ago) and they were all negative.
Pathological
Primary amenorrhoea: the failure to reach menarche
How does this information help you and
by the age of 16. This may be due to: structural abnor-
what would you do next?
mality
(such as imperforated hymen, congenital
This history of normal menarche and initial regular
absence of the uterus), genetic disorders
(such as
menses rules out primary causes of amenorrhoea. Preg-
Turner’s syndrome), testicular feminization syndrome
nancy is ruled out by three negative tests.
(the individual is genetically a male, with the XY chro-
At this stage specific questions should be asked tar-
mosome, but phenotypically a female due to tissue
geted at secondary causes of amenorrhoea:
insensitivity to androgens) and causes of secondary
Ovarian and uterine
amenorrhoea (see below)
Hirsutism, obesity (polycystic ovary syndrome)
Secondary amenorrhoea: the cessation of menstrual
Symptoms of oestrogen deficiency: hot flushes,
periods in women who had previously menstruated.
sweating, mood swings (premature ovarian failure)
Causes can be ovarian [such as polycystic ovary disease,
History of gynaecological procedures or pelvic infec-
see Case 13, or premature ovarian failure occurring
tions (uterine adhesions)
due to chromosomal abnormality (Turner’s syndrome),
Pituitary
gene mutation in gonadotrophin receptors or autoim-
Symptoms of pituitary failure: growth hormone
mune disease, or iatrogenic premature ovarian failure
deficiency, ACTH deficiency
(hypoadrenalism) and
(chemo- or radiotherapy)], uterine (adhesion in the
TSH deficiency (hypothyroidism)
uterus), pituitary (hypopituitarism or prolactinoma),
Symptoms of prolactin excess: galactorrhoea (breast
hypothalamic [excessive exercise (as in professional
milk production)
athletes), severe weight loss, physical or psychological
Hypothalamic
History of excessive exercise
Endocrinology and Diabetes: Clinical Cases Uncovered. By R. Ajjan.
History of recent stress
Published 2009 by Blackwell Publishing, ISBN: 978-1-4051-5726-1
History of weight loss
102
Case 9
103
Pregnancy
Lactation
Menopause
Physiological
Amenorrhoea
Pathological
Primary
Secondary
Genetic
disorders
Structural
Androgen
abnormalities
insensitivity
Ovary
Uterus
Hypothalamus
Pituitary
Figure 51 Causes of amenorrhoea.
Sutapa denies any change in lifestyle or weight, and she is
Table 33 Presentation of prolactinomas.
not having any major stress in her life. However, she
mentions galactorrhoea that is becoming an embarrassing
Symptoms related to excess
Symptoms related to the
prolactin
mass effect
problem.
Galactorrhoea (90% of women,
Headaches
What signs would you look for?
10% of men)
Galactorrhoea: this can be confirmed on physical
examination
Menstrual disturbances in the
Visual field defects
Visual field defects: abnormalities suggest a large pitu-
majority
itary tumour
Reduced libido in both women
Hypopituitarism
Rule out clinical pituitary insufficiency (see Case 4)
and men
Erectile dysfunction in men
Cranial nerve palsies
Sutapa has normal visual fields and no signs to suggest
(invasion of the cavernous
pituitary failure. Galactorrhoea is confirmed on physical
sinus or pituitary apoplexy)
examination.
What is the most likely diagnosis and
Diagnosis can be confirmed by measuring plasma pro-
how would you confirm this?
lactin levels
The most likely diagnosis is a prolactinoma (Table
33)
Plasma prolactin is 14 600 mU/L (normal range <600 mU/L).
104
Part 2: Cases
Does this confirm the diagnosis?
What is the best treatment option?
Yes, this confirms the diagnosis of a prolactinoma. Dif-
Dopamine agonists (bromocriptine and cabergoline)
ficulties can arise when prolactin levels are less than
do not only reduce plasma prolactin levels but also result
6000 mU/L, which may be due to a prolactinoma but
in shrinkage of prolactinomas
may equally be due to a large pituitary tumour with a
Surgery is usually the best treatment for most pituitary
“stalk effect”, consequently resulting in raised prolactin
tumours except for prolactinomas, where medical
levels.
treatment is first line and surgery is reserved for non-
responders to dopamine agonists
What is the differential diagnosis of
Successful treatment is associated with a drop in pro-
raised plasma prolactin?
lactin levels, reduction in tumour size and normalization
Physiological
of menstrual cycles
Pregnancy
Nipple stimulation
Sexual intercourse
Box 26 Difference between micro- and
macroprolactinomas
Stress (taking a blood sample from some individuals
can be a stressful experience and may result in modest
Microprolactinomas are tumours measuring 1 cm in
elevation of plasma prolactin)
diameter
Drug treatment, there is an extensive list of drugs that
Macroprolactinomas are tumours measuring >1 cm in
can result in raised prolactin including:
diameter
Dopamine receptor antagonists (metoclopramide)
Neuroleptics (such as chlorpromazine, haloperidol)
Antidepressants
Patients started on dopamine agonists should be coun-
Opiates
selled regarding pregnancy
Antiretroviral treatment
Endocrine causes
Sutapa comes to see you 8 months after starting cabergoline
Hypothyroidism (raised TRH stimulates prolactin
treatment to tell you that she is now pregnant.
secretion)
Metabolic
What would you do with her treatment?
Renal failure (decreased excretion of prolactin)
In patients with microprolactinomas, it is usually safe
Hypothalamic
to stop the treatment as tumour expansion is very rare
Mass compressing the stalk
In patients with macroadenomas, management
Infiltrative disease
remains controversial and should be individualized. A
Pituitary
large number of patients continue on dopamine agonists,
Prolactinoma
which, particularly bromocriptine, seem to be safe in
Large pituitary tumour causing stalk compression
pregnancy
This patient should be referred to an endocrinologist
MRI of the pituitary shows a large pituitary adenoma
with experience in managing pituitary pathologies
measuring 1.8 cm in diameter with no optic nerve
Symptoms and signs of pituitary prolactinoma are
compression.
summarized in Table 33.
CASE REVIEW
Sutapa is a young woman who presents with secondary
raised plasma prolactin levels, an abnormality that can be
amenorrhoea and galactorrhoea. Pregnancy is ruled out as
due to a large number of reasons including physiological
the cause of her amenorrhoea, an important step in
causes, certain medications, endocrine and metabolic
investigating amenorrhoea in order to avoid unnecessary
conditions. Further investigations confirm a pituitary
investigations. Apart from galactorrhoea, her physical
macroadenoma as the cause of her raised prolactin.
examination is unremarkable and she does not have signs
Treatment of prolactinomas is usually medical and surgery
of pituitary hormone deficiency. Subsequent tests confirm
is rarely required.
Case 9
105
KEY POINTS
Secondary amenorrhoea is a common condition and
Physiological (pregnancy, sexual intercourse, nipple
pregnancy should always be ruled out as a cause before
stimulation, stress)
embarking on expensive investigations
Pituitary tumours (prolactinomas or other tumours with
Causes of secondary amenorrhoea include abnormalities
stalk compression)
in:
Drugs (metoclopramide, antipsychotics and
Hypothalamus (increased stress)
antidepressants, opiates, HIV treatment)
Pituitary (pituitary tumours in general, prolactinomas in
Metabolic and endocrine (hypothyroidism, polycystic
particular)
ovary syndrome, chronic renal failure)
Ovary (polycystic ovary disease)
Treatment of hyperprolactinaemia should be directed at
Uterus (uterine adhesions following infections)
the cause. In the case of a pituitary prolactinoma, the
In addition to menstrual irregularities, raised prolactin may
treatment is usually medical, and not surgical, using
cause galactorrhoea
dopamine agonists (bromocriptine, cabergoline)
There is an extensive list for the causes of
hyperprolactinaemia, including:
Case 10
A 28-year-old with tiredness and
abnormal thyroid function postpartum
One week after her 28th birthday and 6 weeks after giving
Thyroid status: looking for signs of hypothyroidism
birth, Nicola consults her GP, with a 3-4-week history of
Visual field: pituitary failure may occur in the pres-
extreme tiredness. Her GP checks her TFTs and results show:
ence of a large pituitary adenoma
FT4 5.1 pmol/L (normal range 10.0-25.0 pmol/L)
TSH 2.1 mIU/L (normal range 0.2-6.0 mIU/L)
What investigations would you request?
Investigation of pituitary insufficiency involves (Fig. 52):
What questions would you ask?
Hormonal tests
Results show a low FT4 with inappropriately “normal”
Basal hormone levels: 9:00 am cortisol, very low
TSH, indicating secondary hypothyroidism. Questions
levels can confirm hypoadrenalism (primary or sec-
should be directed towards symptoms of pituitary
ondary). However, levels in the low-normal range are
failure:
non-diagnostic and dynamic tests are required (see
Tiredness
(hypothyroidism, hypoadrenalism and
below); prolactin, during the period of breast feeding,
growth hormone deficiency)
prolactin levels are high and low levels early in the
Gastrointestinal
symptoms,
weight
loss
postpartum period are suggestive of hypopituitarism;
(hypoadrenalism)
sex hormones [oestrogen (in females), testosterone (in
Dizziness
due to
low
blood
pressure
males), FSH and LH] are usually requested in individ-
(hypoadrenalism)
uals with suspected pituitary failure. However, sex
Failure of lactation (prolactin deficiency)
hormone levels can be difficult to interpret shortly
after giving birth
Nicola tells you that she managed to breast feed for only a
Dynamic tests: glucagon stimulation test
(GST),
week post delivery. She has had dizziness for at least 3
insulin stress test
(IST), low-dose synacthen test
weeks and has been feeling very weak with reduced
(LDST)
appetite and rapid weight loss.
GST and IST assess cortisol and growth hormone
reserve, whereas LDST determines cortisol reserve only
What signs would you look for?
(Fig. 52)
Dizziness, weakness, reduced appetite and failure to
Imaging
lactate are strongly suggestive of pituitary failure
MRI of the pituitary gland
The following should be assessed in individuals with
suspected pituitary failure:
Nicola’s blood tests show low prolactin with inadequate
Blood pressure: low blood pressure is seen in ACTH
cortisol and growth hormone response to IST.
deficiency but it can be in the normal range in patients
with pituitary failure due to preserved aldosterone pro-
What is your diagnosis?
duction by the adrenal gland (which is mainly con-
Abnormal IST, low prolactin and low FT4 with normal
trolled by the renin-angiotensin system)
TSH is diagnostic of pituitary failure
Remember that in early pituitary failure an individual
may only have one or two hormonal deficiencies, and as
Endocrinology and Diabetes: Clinical Cases Uncovered. By R. Ajjan.
the condition progresses lack of other hormones becomes
Published 2009 by Blackwell Publishing, ISBN: 978-1-4051-5726-1
evident
106
Case 10
107
MRI
gland secondary to hypotension, resulting in hypopitu-
itarism and is called Sheehan’s syndrome. Fortunately,
improved obstetric care in the developed world has made
Imaging
this a rare complication
The differential diagnosis for the causes of hypopitu-
Investigations for
itarism include:
suspected pituitary
failure
Tumours affecting the pituitary gland
Radiotherapy of the head
Pituitary apoplexy
Endocrine tests
Infiltrative disease (sarcoidosis, haemachromatosis)
Pituitary infection (abscess of the pituitary gland)
Head trauma
Basal tests
Dynamic tests
How would you manage this patient?
An MRI of the pituitary should be requested to rule
TFTs
PRL
GST
IST
out other causes of pituitary pathology
She will need hormonal replacements to cover her
E2, Test
LDST
FSH, LH
multiple hormone deficiencies
Figure 52 Investigations for suspected pituitary failure. E2,
Which hormone should be
oestradiol; FSH, follicle stimulating hormone; GST, glucagon
stimulation test; IST, insulin stress test; LDST, low-dose
replaced first?
synacthen test; LH, luteinizing hormone; MRI, magnetic
Cortisol should be the first hormone to get replaced.
resonance imaging; PRL, prolactin; Test, testosterone; TFTs,
If thyroxine is given before adequate cortisol replacement
thyroid function tests.
it may precipitate a hypoadrenal crisis. Therefore, in pan-
hypopituitarism, cortisol is replaced first followed 48 h
Nicola tells you that she had a difficult labour and lost large
later by thyroid hormone replacement
amounts of blood.
Female hormones should be replaced using an ade-
quate combination of oestrogen and progesterone
What is the most likely aetiology of her
In men with hypopituitarism, testosterone should be
pituitary failure and what is the
replaced
differential diagnosis?
Certain criteria are needed to replace growth hormone
Severe blood loss can cause infarction in the pituitary
and this is best left to an expert in this field
CASE REVIEW
Six weeks after giving birth, Nicola seeks medical advice
imaging is essential to rule out a pituitary pathology.
for
3-4 weeks’ history of extreme tiredness. An initial
Severe blood loss during delivery may cause pituitary
blood test shows low FT4 with inappropriately normal
failure through infarction, known as Sheehan’s syndrome,
TSH, indicating a diagnosis of secondary hypothyroidism.
which is a possible diagnosis in this case. In individuals
Specific questioning directed at pituitary hormone
with multiple pituitary hormone deficiency, cortisol should
deficiency strongly suggests pituitary failure manifested as
be replaced first, as early replacement with thyroxine may
inability to breast feed (absence of prolactin) and symptoms
precipitate an adrenal crisis. Cortisol replacement is
consistent with steroid hormone deficiency (absence of
important in any individual with suspected adrenal or
ACTH). Further investigations confirm the deficiency of
pituitary failure, with the treatment started even before
several pituitary hormones and a diagnosis of pituitary
initiating investigations in case the patient is acutely
failure is made. In patients with pituitary failure MRI
unwell.
108
Part 2: Cases
KEY POINTS
Low thyroid hormones with inappropriately low or normal
Polyuria and polydipsia (ADH deficiency): only occurs if
TSH should raise the suspicion of secondary
the posterior pituitary is involved in the pathological
hypothyroidism
process
Individuals with secondary hypothyroidism should be
Individuals with pituitary failure should be investigated for
investigated for other pituitary hormone deficiency using
the aetiology of the condition, including:
static and stimulatory hormonal tests
Pituitary or parapituitary tumours
Clinical features of hypopituitarism include:
Pituitary infarction
Growth arrest in children and tiredness in adults (GH
Infiltrative disease (histiocytosis, haemachromatosis,
deficiency)
sarcoidosis)
Amenorrhoea in women (FSH and LH deficiency) and
Previous radiotherapy
erectile dysfunction in men (LH deficiency)
Trauma (following head injury)
Weight loss and tiredness (ACTH deficiency)
Management of pituitary failure requires replacement of
Symptoms of hypothyroidism (TSH deficiency)
the deficient hormone(s) and treatment of the cause
Failure of lactation (prolactin deficiency)
Case 11
A 33-year-old man with polyuria
and polydipsia
Peter, aged 33, is referred by his GP following a 2-month
Lethargy
history of polyuria and polydipsia. The patient says that the
Confusion
problem got worse recently and he can pass up to 6 L of
Depression
urine per day.
Chronic renal failure
Previous history of renal injury
What is the differential diagnosis at
Anorexia
this stage?
Lethargy
The differential diagnosis of polyuria and polydipsia
Itching
includes:
Anaemia
Diabetes (type 1 or type 2)
Oedema
Hypercalcaemia
Hypertension
Chronic renal failure
Diabetes insipidus (DI)
Diabetes insipidus
History of head injury
Diuretic abuse
Intracranial tumours
Psychogenic
Chronic inflammatory conditions (tuberculosis, his-
It is important to note that some patients complain of
tiocytosis, sarcoidosis)
polyuria without an actual increase in their urine output
Use of drugs (lithium, demeclocycline)
and urine volumes should be measured over 24 h to
confirm the diagnosis of polyuria in uncertain cases.
What tests would you request at
this stage?
What symptoms and signs would you be
Plasma glucose
looking for to narrow down the
Plasma calcium
differential diagnosis?
U&Es
Diabetes
Plasma and urine osmolarities
Skin infections
Blurred vision
Blood tests show:
Overweight (T2DM)
Fasting glucose
5.1 mmol/L
History of weight loss (T1DM)
Corrected calcium
2.35 mmol/L
Tiredness
Sodium
147 mmol/L
Hypercalcaemia
Potassium
3.9 mmol/L
Anorexia
Urea
5.5 mmol/L
Vomiting
Creatinine
88 mmol/L
Abdominal pain
Plasma osmolarity
300 mOsm/kg
Constipation
Urine osmolarity
155 mOsm/kg
What do these results indicate?
Endocrinology and Diabetes: Clinical Cases Uncovered. By R. Ajjan.
The above results rule out the following as the cause of
Published 2009 by Blackwell Publishing, ISBN: 978-1-4051-5726-1
this patient’s symptoms:
109
110
Part 2: Cases
Table 34 Results of water deprivation test.
Time
Plasma osmolarity Urine osmolarity
0 h
300
158
4 h
306
154
5 h (i.m. vasopressin
308
155
given)
8 h
295
835
Diabetes
Hypercalcaemia
Chronic renal failure
The high plasma sodium and increased plasma osmo-
larity with inappropriately low urine osmolarity suggest
a diagnosis of diabetes insipidus.
What test would you request to confirm
Figure 53 (a) Courtesy of the Radiology Department,
the diagnosis?
University of Leeds.
A water deprivation test with desmopressin
administration.
The results of Peter’s water deprivation test are shown
in Table 34.
Infections
Bacterial (Streptococcus, tuberculosis)
How would you interpret these results?
Viral
These results confirm DI, as the patient fails to con-
Fungal
centrate his urine despite increasing plasma osmolarity
Inflammatory bowel disease
Concentration of urine after vasopressin administra-
Malignancy
tion, together with a fall in plasma osmolarity, is consis-
Sarcoidosis
tent with a diagnosis of cranial DI (lack of ADH)
Drugs
(oral
contraceptives, sulphonamides,
penicillin)
The patient had an X-ray done a week ago (Fig. 53a) due
Taken together, the most likely unifying diagnosis is
to persistent cough. He is also complaining of painful and
sarcoidosis causing DI, secondary to the inflammatory
red skin eruptions on his shins, which he has had for
infiltrate in the posterior pituitary. Another remote pos-
3 days (Fig. 53b, colour plate section).
sibility is tuberculosis.
What abnormality can you see on the
What imaging would you request in
CXR? What is the skin lesion? Can you
this patient?
give a unifying diagnosis?
MRI of the pituitary.
The CXR (Fig. 53a) shows bilateral hilar enlargement.
Hilar lymphadenopathy can be seen in:
What is the treatment of cranial DI?
Sarcoidosis
Treat the cause
Infection (tuberculosis)
Desmopressin can be given:
Malignancy (lymphoma)
Orally
The red and painful skin lesions are characteristic of
Intranasally
erythema nodosum, which can be seen in:
Injections
Case 11
111
Drug treatment (only partially effective) includes:
Box 27 Causes of diabetes insipidus
Thiazide diuretics
Prostaglandin synthase inhibitors such as
These can be divided into:
indomethacin
Cranial
Familial
Acquired: head injury, pituitary tumours, infiltrative
disease (tuberculosis, sarcoidosis, histiocytosis),
What is the main difference in serum
infections (meningitis or encephalitis), vascular events,
and urine osmolarities comparing DI
idiopathic (no cause found)
with psychogenic polydipsia?
Nephrogenic
In both DI and psychogenic polydipsia the urine
Familial
osmolarity is low
Acquired: drugs (lithium, demeclocycline), electrolyte
Plasma osmolarity is high in DI but low or low-normal
abnormalities (hypokalaemia, hypercalcaemia), chronic
in psychogenic polydipsia
renal disease
In difficult cases, a water deprivation test should
be performed to differentiate between these two
What is the treatment of
conditions
nephrogenic DI?
Table 35 summarizes the main features of cranial and
Treat the cause and maintain adequate fluid intake
nephrogenic DI as well as psychogenic polydipsia.
Table 35 Main features of cranial and
Cranial DI
Nephrogenic DI
Psychogenic polydipsia
nephrogenic DI and psychogenic
polydipsia.
Posmo
High-normal or high
High-normal or high
Low-normal or low
Uosmo Low
Low
Low
After water deprivation test
Posmo
High
High
Normal
Uosmo Low
Low
Normal
After desmopressin administration
Posmo
Normal
High
Normal
Uosmo Increase (normalizes)
Low
Normal
Posmo, plasma osmolarity; uosmo, urine osmolarity.
CASE REVIEW
Peter consults his doctor with a
2-month history of
ADH), rather than nephrogenic DI (reduced responsiveness
polyuria and polydipsia. Diuretic abuse, diabetes,
of the kidneys to ADH). The patient has an abnormal CXR
hypercalcaemia and chronic renal failure are ruled out as
with bilateral hilar enlargement and a skin lesion consistent
a cause for his symptoms. Diabetes insipidus is suspected,
with erythema nodosum. This strongly suggests a diagnosis
which is confirmed following a water deprivation test. An
of sarcoidosis as a cause for this patient’s DI. Treatment of
improvement in urine osmolarity after vasopressin
cranial DI is replacement with desmopressin, which can be
injection indicates a diagnosis of cranial DI (deficiency of
given orally, intranasally or subcutaneously.
112
Part 2: Cases
KEY POINTS
Diabetes insipidus is a known cause of passing large
Nephrogenic: drugs (lithium, demeclocycline), chronic
amounts of diluted urine. It should be differentiated from
renal disease, electrolyte imbalance (hypercalcaemia,
other causes of polyuria, including:
hypokalaemia), or familial disorders
Diabetes
Investigations for DI include:
Hypercalcaemia
Water deprivation test
Psychogenic polydipsia
Pituitary imaging
Causes of DI include:
Treatment includes:
Cranial: infiltrative disease (histiocytosis,
Desmopressin with adequate fluid intake
haemachromatosis, sarcoidosis), pituitary or
Treat the cause
parapituitary tumours, pituitary infarction, trauma
(following head injury), or rare familial disorders
Case 12
A 62-year-old man with tiredness
and hyponatraemia
Max, a 62-year-old gentleman, is seen by his GP with a few
Max tells you that he started to feel increasingly tired 6
days’ history of severe tiredness. Routine blood tests are
weeks ago, lost his appetite and this was associated with
requested and these show:
weight loss. Past medical history includes temporal lobe
Na+
119 mmol/L
epilepsy for which he has been on treatment for more than
K+
3.4 mmol/L
15 years. On examination he looks well hydrated and his
Urea
4.2 mmol/L
physical examination is unremarkable.
Creatinine
65 μmol/L.
How would the above information help
What differential diagnosis would you
in establishing a diagnosis?
think of at this stage?
Max is clinically well hydrated
Hyponatraemia is a common finding and not infre-
The urea is not elevated further suggesting that the
quently mismanaged. A large proportion of patients with
patient is well hydrated
hyponatraemia are initially diagnosed as syndrome of
It should be noted that in malnourished individuals
inappropriate ADH secretion (SIADH) without proper
(such as alcoholics) urea can be very low, and, there-
assessment. Hyponatraemia may be due to (Fig. 54):
fore, urea in the normal range does not rule out
Sodium loss and dehydration (in which case the patient
dehydration
is hypovolaemic):
Taken together, hyponatraemia secondary to dehydra-
Use of diuretics (a very common cause)
tion is unlikely here
Diarrhoea
A normal physical examination further rules out:
Vomiting
Cirrhosis
Renal disease and salt wasting
Cardiac failure
Mineralocorticoid deficiency: hypoaldosteronism or
Nephrotic syndrome
Addison’s disease
Hypothyroidism
(but in some case individuals
Water excess with euvolaemia:
with hypothyroidism may have very few clinical
SIADH
signs)
Glucocorticoid deficiency
Hypothyroidism
What is the likely diagnosis from the
Water excess with hypervolaemia:
clinical evidence given above?
Cirrhosis
The likely diagnosis is SIADH
Cardiac failure
Glucocorticoid deficiency and hypothyroidism remain
Nephrotic syndrome
two possibilities and these should be ruled out
What would you like to do at
this stage?
What tests would you request to
A full medical history, including a review of current
confirm the diagnosis?
medications
Plasma osmolarity
Clinical examination
Urine osmolarity with urinary electrolytes
TFTs (to rule out the possibility of hypothyroidism)
Endocrinology and Diabetes: Clinical Cases Uncovered. By R. Ajjan.
A random cortisol with or without a synacthen test (to
Published 2009 by Blackwell Publishing, ISBN: 978-1-4051-5726-1
rule out glucocorticoid deficiency)
113
114
Part 2: Cases
Renal
GI
skin
Addison’s
Aldosterone
disease
deficiency
Fluid loss
Mineralocorticoid
deficiency
Hypovolaemia
Hyponatraemia
Euvolaemia
Hypervolaemia
GC
Heart
Nephrotic
SIADH
deficiency
Failure
syndrome
Figure 54 Causes of hyponatraemia.
GC, glucocorticoid; SIADH, syndrome
Hypothyroidism
Cirrhosis
of inappropriate antidiuretic hormone
secretion.
Before the blood is taken for the above
A previous U&Es result would help in differentiating
tests, what would you like to know?
hyponatraemia due to drug use
A review of current medications is essential (particularly
the use of diuretics and thyroxine).
Max had his U&Es checked 6 months ago which showed:
Sodium
136 mmol/L
The list of medications includes carbamazepine and
Potassium
4.3 mmol/L
paracetamol as required.
Urea
3.8 mmol/L
Max’s tests showed:
Creatinine
67 μmol/L.
Plasma osmolarity
243 mOsm
Urine osmolarity
487 mOsm
Urine sodium
52 mmol/L
Random cortisol
620 nmol/L
Box 28 Causes of SIADH
FT4
21 pmol/L
TSH
1.8 mIU/L.
Malignancy
Chest infections (particularly atypical pneumonia)
What is the diagnosis?
Abnormalities in the central nervous system
Max has a normal thyroid function and no evidence of
Infections
glucocorticoid deficiency
Head injuries
Max has:
Vascular disorders
Low plasma osmolarity
Metabolic
Inappropriately high urine osmolarity
Porphyria
High urinary sodium excretion
Drugs
Chemotherapy
The diagnosis is, therefore, SIADH.
Psychiatric drugs
Anti-epileptics (carbamazepine)
What is the aetiology of SIADH in
Antidiabetics (chlorpropamide)
this case?
Idiopathic
A possibility is carbamazepine use
Case 12
115
If the above tests are negative, whole body CT should
be considered
The patient’s abdominal ultrasound is shown in Fig.
55.
What does this section of the liver
show? What is the likely diagnosis?
The ultrasound shows multiple hepatic metastases, and,
therefore, the likely cause of SIADH is metastatic
malignancy.
Figure 55 Courtesy of the Radiology Department, University of
How would you manage Max’s
Leeds.
hyponatraemia?
Fluid restriction (750-1500 mL/day)
Treat the underlying cause if possible
What do these results indicate?
In long-term cases, demeclocycline can be tried (which
The normal sodium 6 months earlier makes SIADH
induces nephrogenic diabetes insipidus)
secondary to carbamazepine use unlikely.
!RED FLAG
What would you do now?
Malignancy as a cause of SIADH should be ruled out,
Hyponatraemia should be corrected slowly (0.5 mmol/h
particularly in view of the recent history of weight loss
and less than 10 mmol/24 h) to avoid the rare
A number of tests can be requested at this stage
complication of central pontine myelinolysis.
including:
In severe hyponatraemia with neurological signs (seizures),
CXR
rapid correction of the low sodium to a ‘safe level’ may
Abdominal ultrasound
be necessary and can be achieved by infusion of
Prostate specific antigen (PSA; rule out cancer of the
hypertonic saline.
prostate)
CASE REVIEW
Max, who is 62 years old, consults his doctor with a history
Max is on long-term treatment with carbamazepine.
of tiredness. Initial investigations show significant
However, his plasma sodium levels 6 months earlier, when
hyponatraemia and he is, therefore, admitted to hospital
he was on the same treatment, were normal casting doubts
for further management. Hyponatraemia can occur in
about this diagnosis. He is subsequently investigated for
the presence of hypovolaemia, euvolaemia or hypervolaemia.
the possibility of malignancy as a cause of his SIADH and
Assessing hydration status of the patient is important
liver ultrasound confirms the presence of multiple
to plan appropriate investigations. Max appears well
metastases. Max is subsequently managed by fluid
hydrated and on further questioning it became apparent
restriction and treatment of the underlying pathology is
that he lost significant weight recently due to poor appetite.
considered. Correction of hyponatraemia should be done
His clinical presentation, plasma and urine osmolarities
gradually, particularly if sodium levels are very low, to
and urinary electrolytes are consistent with SIADH. The
avoid the rare but serious complication of central pontine
possibility of drug-induced SIADH is considered as
myelinolysis.
116
Part 2: Cases
KEY POINTS
Hyponatraemia is a common condition in hospitalized
With hypervolaemia: cirrhosis, heart failure and renal
patients
failure
It is very important to assess fluid status in individuals
Diagnosis of SIADH is confirmed by demonstrating low
with hyponatraemia (hypovolaemic, euvolaemic,
plasma osmolarity in the presence of high urine osmolarity
hypervolaemic) together with a review of their
and high urinary electrolyte concentration
medications (diuretics and inappropriate fluid replacement
Treatment of SIADH includes:
probably remain the commonest cause of hyponatraemia
Treat the cause
in hospitalized patients)
Fluid restriction
Causes of hyponatraemia include:
Medical treatment is considered in difficult cases
With hypovolaemia: renal salt loss (diuretics, tubular
Treatment of other causes of hyponatraemia should be
defect, mineralocorticoid deficiency) and gastrointestinal
directed at the cause
loss (vomiting, diarrhoea)
With euvolaemia: SIADH, glucocorticoid deficiency and
hypothyroidism
Case 13
Excess hair in a 29-year-old woman
Julie is 29 years old and is complaining of excess hair on her
mented hair occurring in areas where men normally
neck, chin and body.
develop hair growth
Androgen-independent hair growth: excess vellus
What differential diagnosis would you
hair over face and trunk
consider at this stage?
Excess hair or hirsutism is a common complaint affecting
What signs would you be looking for?
more than 10% of women.
Assess severity and distribution of hirsutism (special
Causes include:
tables can be used for this)
Polycystic ovary syndrome (PCOS): common
Distinguish between androgen-dependent and
Familial or racial (for example Mediterranean origin):
androgen-independent hair growth
common
Look for signs of virilization: these are usually associ-
Drugs (phenytoin, corticosteroids, cyclosporine, ana-
ated with an androgen-producing tumour
bolic steroids, minoxidil)
Abdominal examination: looking for abdominal
Congenital adrenal hyperplasia: rare
masses (ovarian tumour)
Cushing’s syndrome: rare
Signs of Cushing’s syndrome
Ovarian and adrenal tumours: rare
Acanthosis nigricans: this can be associated with
Idiopathic
insulin resistance suggesting a diagnosis of PCOS
Julie tells you she always had excess body hair but this
What questions would you ask to try to
started to worsen in the past 18 months or so. Her periods
establish a diagnosis?
have been irregular for the past 8 years occurring every 2-3
Onset: long history or recent problem; a recent history
months. There is a family history of T2DM but there is no
of hirsutism, particularly in severe disease, warrants
history of hirsutism.
prompt and full investigations
On examination, she is overweight with a BMI of
Family history of hirsutism (familial hirsutism)
32.4 kg/m2. She has a lesion in her axilla, shown in Fig. 56
Menstrual history: normal periods effectively rule out
(colour plate section). She has excess hair on her chest, chin,
significant hyperandrogenism
(ovarian and adrenal
abdomen and inner thighs (noted as “mild”). There are no
tumours unlikely)
signs of virilization.
History of virilism
Change of voice
Given the information above, what is
Clitoromegaly
the most likely diagnosis?
Frontal bolding
The most likely diagnosis is PCOS supported by:
Increased muscle mass
Long history of hirsutism and oligomenorrhoea
Distinguish between:
Obesity
Androgen-dependent hair growth: coarse and pig-
Excessive hair in androgenic distribution
Absence of signs of virilization
Endocrinology and Diabetes: Clinical Cases Uncovered. By R. Ajjan.
Presence of acanthosis nigricans suggesting increased
Published 2009 by Blackwell Publishing, ISBN: 978-1-4051-5726-1
insulin resistance, which is a feature of PCOS
117
118
Part 2: Cases
Can you rule out an androgen secreting
Raised LH/FSH ratio
tumour from history and examination?
Impaired fasting glucose
An androgen secreting tumour in this patient is highly
Taken together, the most likely diagnosis is PCOS. The
unlikely due to:
failure to detect polycystic ovaries on abdominal ultra-
Slow onset of symptoms
sound does not rule out the diagnosis, particularly as this
Absence of virilization
has a lower sensitivity compared with transvaginal
ultrasound.
What tests would you request to
What other blood test(s) would
confirm the diagnosis?
you request?
Testosterone: normal or mildly elevated in PCOS
This patient should undergo a glucose tolerance test due
(<5 nmol/L)
to her impaired fasting glucose.
SHBG low in more than half of PCOS women, conse-
quently resulting in high free androgens
What are the treatment options
FSH and LH: high LH/FSH ratio in two-thirds of
for Julie?
PCOS
Weight loss, this is important as it results in:
Ovarian ultrasound: typical ovarian morphology can
Reduction in hyperandrogenism
be seen in the majority of PCOS patients (particularly
Increase in insulin sensitivity
using transvaginal ultrasound)
A minor reduction of weight by 5% can result in 50%
Other tests:
improvement in hirsutism.
Fasting glucose
(diabetes in
10% and impaired
Oral contraceptive pills (OCP), the most commonly
glucose tolerance in 40% of PCOS patients)
used is Dianette which contains:
Lipids
Oestrogen (ethinyl oestradiol)
Androgen receptor blocker (cyproterone acetate)
Julie’s blood tests show the following (normal ranges):
Insulin sensitizers
Testosterone 2.8 nmol/L (1-2.5)
Metformin: has been successfully used to restore
SHBG 9 nmol/L (22-120)
ovulation and induce fertility in PCOS
FSH 2.4 U/L
Thiazolidinediones: rosiglitazone and pioglitazone
LH 5.3 U/L
are less frequently used compared with metformin
Fasting glucose 6.8 mmol/L
Anti-androgen treatment
Triglyceride 2.1 mmol/L
Cyproterone acetate: increases hepatic androgen
Cholesterol 4.8 mmol/L
clearance
Prolactin 445 (<600 mU/L)
Spironolactrone: weak anti-androgen
17-OH progesterone 12 nmol/L (1-20)
Flutamide: strong anti-androgen
Abdominal ultrasound: Normal appearance of the ovaries
5α-reductase inhibitors
Finasteride: inhibits the conversion of testosterone
to the potent androgen dihydrotestosterone
Why did Julie have her prolactin and
It should be noted that women with PCOS receiving
17-OH progesterone checked?
pharmacological treatment should be given appropriate
Prolactin was checked as a routine test investigating
contraceptive advice.
her irregular periods (see Cases 4 & 10)
17-OH progesterone was checked to rule out the pos-
Julie is very keen on starting a family.
sibility of congenital adrenal hyperplasia (discussed in
What advice would you give her and
Case 13)
would you start her on any
pharmacological treatment?
What is the diagnosis?
The importance of weight loss should be emphasized
Julie has:
Metformin treatment has been shown to restore ovula-
Mildly elevated testosterone
tion in some patients with PCOS and therapy with this
Low SHBG (consequently increasing the levels of
agent is a possibility. Other treatment options are sum-
unbound/free testosterone)
marized in Table 19, p. 41.
Case 13
119
CASE REVIEW
Julie is a young woman who presents with hirsutism and
acanthosis nigricans, a condition associated with insulin
menstrual irregularities. More than 10% of women have
resistance, together with excess hair in androgenic
varying degrees of hirsutism, with the commonest causes
distribution with no signs of virilism. The most likely
being polycystic ovary syndrome and familial or racial
diagnosis is polycystic ovary syndrome, which is further
predisposition. It is important to take a full history
supported by biochemical tests showing mildly raised
including onset of symptoms, menstrual history and
testosterone with low SHBG and elevated LH/FSH ratio
markers of virilism. Severity of the condition should be
together with impaired fasting glucose. Treatment for this
assessed and causes of secondary hirsutism should be
condition includes weight reduction, oral contraceptive
looked for. Julie is overweight and is found to have
pills, insulin sensitizers and anti-androgen treatment.
KEY POINTS
Hirsutism in women is a common complaint
Clinical presentation
Causes of hirsutism include:
Low sex hormone binding globulin (SHBG) with or
Common: polycystic ovary syndrome (PCOS), and
without mildly elevated testosterone levels
familial or racial
Raised LH/FSH ratio
Rare: adrenal or ovarian tumours, congenital adrenal
Cystic ovaries on ultrasound (transvaginal is more
hyperplasia and Cushing’s syndrome
accurate than transabdominal)
Rapid onset of symptoms, particularly in the presence of
Treatment of PCOS includes:
virilization, warrants prompt and full investigations
Weight loss
The majority of women with hirsutism have PCOS, which
Oral contraceptive pills (result in elevated SHBG and,
is further characterized by:
hence, lower free testosterone)
Menstrual irregularities
Insulin sensitizers (metformin is the most commonly
Obesity
used agent)
Insulin resistance (hyperinsulinaemia with or without
Androgen receptor blockers (cyproterone acetate,
high glucose levels)
spironolactone)
PCOS is diagnosed by:
5α-reductase inhibitors (finasteride)
Case 14
A 52-year-old woman with paroxysmal
atrial fibrillation and abnormal
thyroid function
Bridget, who is 52 years old with a past medical history of
In the first 3 months of amiodarone use, TSH may
paroxysmal atrial fibrillation, was referred with the following
be elevated (lack of full negative feedback on the pitu-
thyroid function results:
itary due to low T3 levels)
FT4
30.1 pmol/L (10.0-25.0)
After 3 months, the pituitary seems to adjust to the
FT3
4.7 pmol/L (3.4-7.2)
low normal T3 concentrations and TSH normalizes
TSH
3.5 mIU/L (0.2-4.5)
The combination of high FT4, normal T3 and TSH in
TPO antibodies negative
an individual taking amiodarone is not uncommon and
Bridget is asymptomatic.
does not require any medical intervention at this stage,
but will require regular monitoring.
What would you like to ask the patient?
The patient has unusual TFT results with elevated FT4,
Bridget’s repeat TFTs in 3 months show:
normal T3 without suppression of TSH
FT4
25.1 pmol/L
Drug history should be established (is the patient on
FT3
4.1 pmol/L
thyroxine or other treatment that may alter thyroid
TSH
3.8 mIU/L
function?)
What would you do now?
Bridget denies ever taking any thyroxine. Her medication
Nothing
includes: amiodarone (started 3 months ago), warfarin,
Keep monitoring TFTs every 3-6 months
ibuprofen and paracetamol
Bridget is referred 18 months later by her GP with clear
Can you explain her abnormal
symptoms and signs of hyperthyroidism and TFTs showing:
TFTs now?
FT4
67 pmol/L
Amiodarone use can impair thyroid function in a
FT3
19.1 pmol/L
number of ways
TSH
<0.05 mIU/L
It can result in abnormal TFTs, which have no clini-
cal significance
What do these results indicate?
It may cause clinical hyperthyroidism
Bridget now has frank primary hyperthyroidism as shown
It may result in clinical hypothyroidism
by:
Due to suppression of T4 to T3 conversion, amioda-
High thyroid hormones (both T3 and T4)
rone use can be associated with:
Suppressed TSH
High FT4 concentrations, without clinical hyperthy-
Bridget has amiodarone-induced hyperthyroidism.
roidism (therefore, it is essential to check T3 in patients
receiving amiodarone treatment)
What other tests would you request?
Low FT3 concentrations
(usually low normal
Thyroid autoantibodies
levels)
CRP
Thyroid ultrasound (Doppler studies)
In type 1 AIT:
Endocrinology and Diabetes: Clinical Cases Uncovered. By R. Ajjan.
Thyroid antibodies are positive in the majority
Published 2009 by Blackwell Publishing, ISBN: 978-1-4051-5726-1
CRP levels are usually normal
120
Case 14
121
Table 36 Differentiation between type 1 and type 2
How would you treat the patient?
amiodarone-induced thyrotoxicosis (AIT).
The patient’s investigations are consistent with type 2
AIT and the treatment includes:
Type 1 AIT
Type 2 AIT
Discontinuation of amiodarone if possible
High-dose oral steroids (prednisone 40 mg daily.)
Clinical
Goitre
Usually present
Absent
The cardiologists are keen to continue amiodarone
treatment and Bridget is started on steroids for 4 weeks
Blood tests
followed by a reducing dose. Her TFTs 3 months later (off
Thyroid autoantibodies
Positive (majority)
Negative
steroids) showed:
Plasma CRP
Normal
Elevated
FT4
21.1 pmol/L
FT3
4.6 pmol/L
Doppler studies and RAI uptake
TSH
3.9 mIU/L.
Vascularity
Increased
Decreased
Radioiodine uptake
Normal or reduced
Absent
What do these results indicate?
The patient is now euthyroid and no action is required
Treatment
other than regular monitoring of her TFTs.
Antithyroid drugs
Yes
No
Steroids
No
Yes
Bridget is seen 3 months later with the following TFTs:
FT4
12.1 pmol/L
FT3
3.8 pmol/L
TSH
10.1 mIU/L
Box 29 Amiodarone-induced hyperthyroidism
She is clinically asymptomatic.
Type 1 amiodarone-induced thyrotoxicosis (AIT): similar
to autoimmune hyperthyroidism (increased production
What would you do?
of thyroid hormones)
The blood tests are consistent with subclinical
Type 2 AIT: similar to thyroiditis (thyroid destruction and
hypothyroidism
release of thyroid hormones)
The patient is likely to become clinically hypothyroid
It is important to differentiate between the two types as
(which is not uncommon after type 2 AIT, due to thyroid
they are treated differently (see Table 36). Some patients
may have a mixed type, in which case they should be
destruction)
treated for both type 1 and type 2 AIT.
As she is clinically asymptomatic, no treatment is nec-
essary at this stage (just in case her thyroid recovers) and
she can be simply followed up with repeat TFTs
Thyroid ultrasound
(Doppler) shows increased
vascularity
She is seen 3 months later with TFTs:
In type 2 AIT:
FT4
6.7 pmol/L
Thyroid antibodies are usually negative
FT3
1.6 pmol/L
CRP levels are elevated
TSH
39.2 mIU/L
Thyroid Doppler shows decreased vascularity
Clinically, she has classical symptoms of hypothyroidism.
Bridget’s tests showed
TPOAb negative
What would you do?
CRP 56 mg/L
Bridget’s TFTs are consistent with hypothyroidism and
Ultrasound Doppler decreased vascularity
she should, therefore, start treatment on thyroxine.
122
Part 2: Cases
Box 30 Amiodarone facts
Amiodarone has a high iodine concentration (40% of
Amiodarone-induced hyperthyroidism can develop a few
weight), which contributes to the abnormal TFTs seen
months after stopping amiodarone due to the long half-
during treatment with this drug
life of the drug
More than a quarter of patients taking amiodarone
Differentiating type 1 from type 2 AIT can be difficult, in
develop abnormalities in thyroid function
which case the patient should be treated for both with
Up to 5% of patients develop clinical hyperthyroidism and
high-dose antithyroid drugs and steroids
up to 15% develop clinical hypothyroidism
CASE REVIEW
Bridget, who is on amiodarone treatment for paroxysmal
TSH). The differential diagnosis is between type
1
atrial fibrillation, has abnormal thyroid function with
amiodarone-induced
thyrotoxicosis
(similar
to
elevated levels of T4, whereas T3 and TSH are normal.
autoimmune hyperthyroidism) or type
2
(similar to
Amiodarone can affect the thyroid gland and cause
thyroiditis). Her investigations showed negative thyroid
hypothyroidism, hyperthyroidism and may also result in
antibodies, raised CRP and reduced thyroid gland
mildly abnormal TFTs, which have no clinical significance.
vascularity consistent with type 2 amiodarone-induced
Due to inhibition of T4 to T3 conversion in the periphery,
thyrotoxicosis. She was treated with steroids and her
amiodarone resulted in the initial abnormal thyroid
thyroid function normalized after 3 months. Subsequently,
function seen here, and this has no clinical significance.
she became hypothyroid
(low FT4 and raised TSH)
However, 18 months later Bridget developed symptoms of
secondary to thyroid tissue destruction following her
hyperthyroidism and her thyroid function was consistent
thyroiditis and was started on treatment with thyroxine.
with frank hyperthyroidism (raised T3, T4 and suppressed
KEY POINTS
Amiodarone, which has a high iodine content, can cause
AIT type 2: similar to thyroiditis and usually
hypothyroidism (around 15% of patients), hyperthyroidism
characterized by no goitre, raised CRP levels, decreased
(around 5% of patients) and can result in abnormal
vascularity of the thyroid gland and response to steroid
thyroid function (high T4 with or without high TSH and
treatment
normal T3), which has no clinical significance
Mixed AIT: a mixture of type 1 and type 2 and is treated
Amiodarone-induced thyrotoxicosis (AIT) can be divided
by both steroid and antithyroid agents
into
It is preferable to stop amiodarone in AIT if at all possible
AIT type 1: similar to autoimmune hyperthyroidism and
Amiodarone-induced hypothyroidism is simply treated
usually characterized by presence of a goitre, positive
with thyroxine replacement
thyroid antibodies, increased vascularity of the thyroid
Patients on amiodarone treatment should be regularly
gland on Doppler examination and response to
monitored for the development of thyroid dysfunction
antithyroid drugs
Case 15
A 22-year-old man with hypertension
Stuart, who is 22 years old, is found to be hypertensive at
Sweating and heat intolerance
188/105 during routine blood pressure measurement taken
Episodes of pallor
after registering with a new GP. The patient is asymptomatic
Headaches
and there is no previous history of note. Two repeat blood
Primary aldosteronism (symptoms of hypokalaemia)
pressure measurements in the next few days showed
Weakness
readings of 185/103 and 182/101.
Paraesthasia
Cramps
What is the differential diagnosis at
Cushing’s syndrome
this stage?
Weight gain
The patient has high blood pressure. This can be divided
Easy bruising
into:
Abdominal striae
Essential hypertension: majority of cases
Visual problems (in case of pituitary adenoma)
Secondary hypertension: this should be considered
Acromegaly
here due to the patient’s young age. Causes of secondary
Headaches
hypertension include:
Increased sweating
Renal disease: parenchymal or vascular (renal artery
Change in shoe and glove sizes
stenosis)
Visual problems
Coarctation of the aorta
Endocrine abnormalities: pheochromocytoma,
What signs would you look for during
primary aldosteronism, Cushing’s syndrome, acro-
physical examination?
megaly, associated with primary hyperparathyroidism
Signs of renal disease
Abdominal bruits (renal artery stenosis)
What would be your next step?
Radio-femoral delay; weak or absent femoral pulses;
A thorough history
cardiac murmurs (coarctation of the aorta)
Physical examination
Abdominal masses (pheochromocytoma)
Signs of Cushing’s syndrome
What symptoms would you look for?
Signs of acromegaly
Chronic renal disease
Signs of hypertensive retinopathy
Fatigue
Muscle cramps
The patient has a normal physical examination except for
Itchy skin
grade II hypertensive changes on funduscopy. His ECG is
Coarctation of the aorta
shown in Fig. 57.
Symptoms of heart failure
Pheochromocytoma
What is the significance of this ECG?
Episodes of palpitations
It shows left ventricular hypertrophy, indicating long-
standing and severe hypertension
Endocrinology and Diabetes: Clinical Cases Uncovered. By R. Ajjan.
LVH on ECG and retinal changes indicate end organ
Published 2009 by Blackwell Publishing, ISBN: 978-1-4051-5726-1
damage secondary to hypertension
123
124
Part 2: Cases
What do these results indicate?
These results rule out pheochromocytoma and chronic
V1
renal disease as the cause of this patient’s hypertension
I
Normal renal artery Doppler makes a diagnosis of
V2
renal artery stenosis highly unlikely although it does not
conclusively rule out this diagnosis, which may require
MRA in suspicious cases
II
V3
The combination of low potassium and high bicarbon-
ate
(hypokalaemic alkalosis) is strongly suggestive of
primary aldosteronism as a cause of the hypertension in
III
this patient
V4
Hypokalaemic alkalosis and hypertension may also
occur in Cushing’s syndrome and this should be ruled
aVR
out if there is any clinical suspicion
What test would you request next?
V5
Potassium should be normalized with supplements
aVL
followed by blood tests to check:
Plasma renin activity (PRA)
aVF
Aldosterone
25.0
mm/sec
V6
The patient should not be on any hypertensive
Figure 57
medications that may interfere with PRA and aldoste-
What investigations would Stuart need
rone, such as:
at this stage?
ACEI
U&Es, to rule out chronic renal disease
β-blockers
Calcium profile
spironolactone
CXR: may show signs of coarctation of the aorta
If treatment of the patient is necessary while the above
Renal ultrasound and renal artery Doppler or mag-
tests are carried out, α-blockers
(i.e. doxazosin) or
netic resonance angiography (MRA) to rule out renal
calcium channel-blockers can be used
artery stenosis
Supine measurements of aldosterone and PRA are carried
Urinary
catecholamines
(rule
out
pheo-
out and these show:
chromocytoma)
Aldosterone
1220 nmol/L (100-500)
If there is any suspicion of Cushing’s syndrome
PRA
0.2 pmol/ml/hr (1.1-2.7)
24 h urinary cortisol
Overnight or low-dose dexamethasone suppression
What do these results indicate?
test
The diagnosis is probable primary hyperaldosteronism
If there is any suspicion of acromegaly
resulting in hypertension and this is supported by:
Glucose tolerance test with GH measurement
High aldosterone levels
Thoracic MRI or CT should be arranged if there is
Low PRA
suspicion of coarctation of the aorta
A ratio of aldosterone/PRA >2000 is diagnostic of
primary aldosteronism.
Stuart’s tests showed:
Renal ultrasound and Doppler normal
What radiological test would you
Urinary catecholamines: normal on three occasions
request and why?
Na
144 mmol/L
MRI of the adrenal glands
K
3.0 mmol/L
Primary aldosteronism may be due to:
U
5.4 mmol/L
Adrenal adenoma
(usually benign): this can be
HCO3
29 mmol/L
treated surgically or medically
Creatinine
76 μmol/L
Adrenal hyperplasia: no role for surgical interven-
Calcium
2.3 mmol/L
tion and treatment is medical
Case 15
125
MRI shows a nodule in the left adrenal gland measuring
Hypertension is cured in only two-thirds of patients with
2.4 cm in diameter.
Conn’s syndrome following surgical removal of the
tumour and this should be made clear to the patient.
What is the most likely diagnosis?
The most likely diagnosis is Conn’s syndrome (adrenal
What medical treatment can you offer?
adenoma secreting excessive aldosterone).
Spironolactone (aldosterone antagonist): side effects
include gynaecomastia and impotence in men, menstrual
What treatment would you advise?
irregularities in women
Surgical removal of the adrenal adenoma is advised
Potassium-sparing diuretic
It should be noted that large tumours (more than 4 cm
Amiloride
in size) have a greater potential of being malignant and
Triamterene
these should always be surgically removed
The patient asks you ‘Would my high blood pressure be
cured after having surgery?’
CASE REVIEW
Stuart is a young man who is found to be hypertensive
alkalosis suggests hyperaldosteronism as a cause of
during routine examination. Subsequent measurements of
hypertension. This suspicion is confirmed by demonstrating
blood pressure confirm a diagnosis of hypertension. Due
a raised aldosterone:renin ratio and MRI imaging shows
to his young age, secondary hypertension is suspected.
an adrenal tumour consistent with a diagnosis of Conn’s
Apart from retinal hypertensive changes, his clinical
syndrome. Surgical treatment is the best option for this
examination is unremarkable. His ECG shows left
condition although it does not normalize blood pressure
ventricular hypertrophy indicating end organ damage
in all patients. Medical treatment includes the aldosterone
despite his young age. Initial investigations rule out renal
antagonist spironolactone and potassium-sparing diuretics
artery stenosis and pheochromocytoma, but hypokalaemic
(amiloride and triamterene).
KEY POINTS
Secondary causes of hypertension should be suspected in
confirmed by demonstrating raised aldosterone/renin
young individuals and those with severe disease
activity (provided the patient is not on treatment with
Secondary causes of hypertension include:
diuretics or agents that affect the renin-aldosterone
Kidney disease: vascular (renal artery stenosis) or
system)
parenchymal (chronic renal failure)
Primary aldosteronism may be due to:
Endocrine disease: pheochromocytoma, primary
Adrenal adenoma (Conn’s syndrome): usually treated
aldosteronism, Cushing’s syndrome, acromegaly
with surgery
Cardiovascular disease: coarctation of the aorta
Adrenal hyperplasia: treated medically
Primary hyperaldosteronism should be suspected in
Medical treatment of primary hyperaldosteronism includes:
individuals with hypertension and hypokalaemic alkalosis
Aldosterone antagonists: spironolactone
Diagnosis of primary hyperaldosteronism is usually
Potassium-sparing diuretics: amiloride and triamterene
Case 16
A 20-year-old woman with polyuria
and polydipsia
Ivy, a 29-year-old woman, presents with a short history of
Family history of diabetes
polyuria and polydipsia.
Family history of T1DM or autoimmunity
(i.e.
thyroid disease, pernicious anaemia) suggests a genetic
What differential diagnosis would you
predisposition to T1DM
be thinking of?
Family history of diabetes at young age not requiring
The differential diagnosis includes:
insulin or diabetes inherited in an autosomal domi-
Diabetes mellitus
nant manner is suggestive of Maturity Onset Diabetes
Hypercalcaemia
of the Young (MODY)
Chronic renal failure
Diabetes insipidus
What test would you ask the nurse to
Psychogenic polydipsia
perform that may help to differentiate
between T1DM and T2DM?
Her blood tests done by her GP earlier showed:
Urine dipstick for ketones
Glucose
20 mmol/L
Heavy ketonuria is consistent with T1DM
Sodium
131 mmol/L
Absence of ketonuria does not rule out T1DM
Potassium
4.4 mmol/L
Urea
5.4 mmol/L
What else would you like to know?
Creatinine
76 mmol/L
The weight/BMI of the patient
Calcium
2.34 mmol/L
Box 31 Diabetes and BMI
What questions would you ask
this patient?
An overweight patient with diabetes is more likely to
Ivy has high plasma glucose levels indicating a diagnosis
have T2DM
of diabetes. It is important at this stage to differentiate
A thin patient with diabetes is more likely to have
between type 1 diabetes (T1DM) and T2DM. Questions
T1DM
to ask:
However, T1DM can occur in obese individuals and
How long have the symptoms of polyuria and polydip-
T2DM may be seen in thin subjects
sia been present?
A short history of symptoms (days to weeks) is sug-
gestive of T1DM
Ivy tells you that she had osmotic symptoms for 7-10 days,
A long history of symptoms (months) or no symp-
associated with 4 kg weight loss. Her sister has vitiligo but
toms is suggestive of T2DM
there is no other family history of note. Her BMI is 22 kg/m2.
History of rapid weight loss is strongly suggestive of
Her urine dipstick shows:
T1DM
Glucose 3+
Ketone 3+
Endocrinology and Diabetes: Clinical Cases Uncovered. By R. Ajjan.
Nitrate negative
Published 2009 by Blackwell Publishing, ISBN: 978-1-4051-5726-1
WBC negative
126
Case 16
127
What is the most likely diagnosis?
control was initially very good on these doses of insulin, but
The most likely diagnosis is T1DM supported by:
4 months after diagnosis she had to drastically reduce the
Short history of symptoms
dose of her insulin to 8 and 4 units due to recurring
Significant weight loss
hypoglycaemic attacks.
Thin patient
Family history of autoimmunity (vitiligo)
Why did this happen?
The pancreas of patients with T1DM may partially
In unclear cases, can you do a blood test
recover after the initial diagnosis resulting in decreased
to help to differentiate between T1DM
insulin requirement. This is known as the honeymoon
and T2DM?
period.
Glutamic acid decarboxylase
(GAD) and islet cell
antibodies are positive in the majority of T1DM patients
What do you need to ensure with any
(around 80%)
diabetes patient during a routine
A negative antibody test does not rule out the diagnosis
review?
of T1DM
Ensure adequate control of blood sugar
Measure HbA1c levels and aim for <6.5%
What treatment would you start?
Check glucose diary
Ivy should be immediately started on insulin.
Look for signs of microvascular disease:
Most patients can be managed on an out-patient
Eyes: retinopathy (retinal screening once a year)
basis.
Kidney: check for microalbuminuria
(request
Admission should be considered for patients who look
urinary albumin/creatinine ratio once a year)
unwell or in the presence of abdominal pain/vomiting to
Feet: examine for neuropathy (monofilament test
rule out the possibility of diabetic ketoacidosis
and vibration sense once a year)
Ensure prevention/treatment of macrovascular
complications:
Box 32 Insulin regimes
Treat hypertension
There are a number of insulin regimes, and the most
Treat hyperlipidaemia: patients with diabetes above
commonly used are:
the age of 40 are usually started on lipid lowering treat-
Two injections a day with a mixture of insulin (short
ment with a statin no matter what their plasma
acting and long acting) such as:
cholesterol levels are
Insulin aspart and aspart protamine (Novorapid 30)
Antiplatelet treatment (aspirin or clopidogrel) in
Insulin lispro and lispro protamine (Humalog 25)
high risk subjects
Insulin actrapid and isophane (M3)
Aggressive measures for prevention/treatment from
(the number indicates the percentage of short acting
macrovascular disease should be implemented in the
insulin in the mixture)
presence of microvascular complications
Four injections a day, to include:
One injection of long acting insulin such as: insulin
isophane (Insulatard), insulin glargine (Lantus) or
What are the types of diabetes?
insulin detemir (Levemir)
The two main types of diabetes are shown in Table 20,
Three injections of short acting insulin that can be
p. 48. Traditionally, young patients with diabetes were
taken with meals: insulin aspart (Novorapid) or insulin
more likely to have T1DM. However, due to the recent
lispro (Humalog)
problem of obesity, T2DM can be now seen at a very
Although the second regime includes more injections, it
young age (even children).
gives more flexibility and is usually preferred for
Other types of diabetes include:
younger patients with diabetes
Maturity Onset Diabetes of the Young (MODY, up to
3% of T2DM)
Ivy is started on Novomix 30, 12 units in the morning and 6
This is a monogenic form of diabetes (due to a single
units in the evening and the dose is gradually titrated up to
gene defect)
24 and 14 units over a period of 6 weeks. Her glycaemic
Has an autosomal dominant mode of inheritance
128
Part 2: Cases
Patients are usually young and can be misdiagnosed
What is the likely diagnosis?
as having T1DM
This gentleman has T2DM supported by:
Latent Autoimmune Diabetes of Adults (LADA)
High fasting plasma glucose (more than 7.0 mmol/L
An autoimmune form of diabetes occurring at an
on two occasions)
older age
Overweight
Patients are usually slim
Absence of symptoms
Patients are initially managed by oral hypoglycaemic
Urine dipstick negative for ketones
agents but usually require insulin early after diagnosis
Rarely, some patients are misdiagnosed as having
(LADA is commonly a retrospective diagnosis)
T2DM, when they have a secondary form of diabetes,
Gestational diabetes
and, therefore, the above list of causes of secondary dia-
Occurs during pregnancy
betes should be kept in mind when assessing a new
Disappears after giving birth
patient with suspected T2DM.
Subjects with a history of gestational diabetes are at
increased risk of T2DM in the future
What medical treatment would you
Secondary diabetes
initiate to control his blood
Destruction of the pancreas: pancreatitis, pancreatic
glucose levels?
tumour, infiltrative disease (haemochromatosis)
None. Instead, advise the patient to:
Endocrine abnormalities: acromegaly, Cushing’s
Change to a healthy diet
disease, pheochromocytoma, hyperthyroidism (rare)
Regular exercise
Associated with genetic syndromes
Try to lose weight
Down’s syndrome
Turner’s syndrome
Andrew implements your suggestions and is reviewed 3
Lawrence-Moon-Biedl syndrome
months later. He has lost 4 kg in weight and his HbA1c is
Prader-Willi syndrome
now 6.8%.
Drug-induced
Steroids
What would you do?
Ivy’s father, Andrew aged 62, presents a few months later to
Congratulate Andrew and encourage him to continue
his GP, stating that his daughter checked his blood sugar
with his programme of:
with her glucose meter and found it to be elevated. He is
Diet
asymptomatic and overweight with a BMI of 29.9. He has
Exercise
no past medical history of note and his urine dipstick shows:
Glucose 3+
He is reviewed 18 months later. Despite continuing with diet
Ketone trace
and exercise, his HbA1c has risen to 8.1%.
Nitrate negative
WBC negative
What would you do to control his
What test would you request?
blood sugar?
Fasting plasma glucose on two occasions.
Andrew needs to be started on antidiabetic treatment.
Diabetes is usually confirmed by checking fasting
The preferred first-line agent in overweight T2DM
glucose twice, particularly in individuals who are
patients is metformin (Glucophage).
asymptomatic
In subjects with classical symptoms, one glucose
Andrew is well on metformin for 2 years but his diabetes
sample is enough to confirm the diagnosis
control subsequently deteriorates and his HbA1c rises to
8.9%.
His blood tests showed:
Fasting glucose: 10.3 and 11.6 mmol/L
What would you do now?
HbA1c: 8.6%
Andrew can be started on one of the following drugs:
U&Es normal
A sulphonylurea: this group of drugs stimulate insulin
Case 16
129
Box 33 Common side effects of metformin
Treatment of
type 2 diabetes
The most common side effects of metformin are
gastrointestinal and include:
Nausea
Vomiting
Bloating
1st line
Diet and exercise
Diarrhoea
These side effects can usually be avoided by initiating a
small dose of the drug and gradually titrating to higher
2nd line
Metformin
doses. A long acting preparation of metformin
(Glucophage SR) seems to be associated with fewer side
effects.
The most serious side effect of metformin is lactic
Suplhonylureas or
acidosis. This occurs in the presence of:
3rd line
thiazolidinediones
Renal failure
Advanced heart failure
Septicaemia
Suplhonylureas or
Therefore, metformin should be avoided in patients with
4th line
thiazolidinediones
a creatinine above 150 μmol/L or in those with
advanced heart failure. Metformin should also be
stopped in patients who become septic.
5th line
Insulin
secretion by the pancreas. Commonly used drugs
include:
Can be used at any
Gliclazide
of the above stages
Slimming tablets
Glimepiride
A thiazolidinedione: this group of drugs act as insulin
Figure 58 Treatment of type 2 diabetes. Metformin is usually
sensitizers and seem to have cardioprotective properties.
the first-line agent except in those with contraindication or
Commonly used drugs include:
intolerance. Oral hypoglycaemic agents can be used in
Rosiglitazone
combination therapy (even as triple therapy). Insulin can be
used in combination with metformin or a sulphonylurea and
Pioglitazone
even pioglitazone. New agents that have been recently
What are the two main drawbacks
released include glucagon-like peptide analogues (injections)
of sulphonylureas?
and DPP-4 inhibitors (oral), which can be used as second- to
fourth-line agents.
Weight gain due to stimulation of insulin secretion
Hypoglycaemia
Pregnancy,
as
oral
hypoglycaemics
are
What is the main contraindication for
contraindicated
the use of thiazolidinediones?
Heart failure is the main contraindication as these
Apart from antidiabetic agents, what
agents may cause fluid retention, thereby worsening
other drugs are available that may help
existing heart failure
to control blood sugar?
When do you use insulin in type
Slimming tablets such as:
2 diabetes?
Sibutramine
Failure of oral therapy
Orlistat
Hospital admission
Rimonabant
Infection
Acarbose which inhibits glucose absorption
Myocardial infarction
Treatment of type 2 diabetes is summarized in Fig. 58.
130
Part 2: Cases
CASE REVIEW
Ivy is a young woman presenting with a short history of
found to be elevated confirming a diagnosis of diabetes.
polyuria and polydipsia. Her blood tests are consistent
Andrew is overweight, asymptomatic with no ketonuria
with a new diagnosis of diabetes. A detailed history is taken
consistent with a diagnosis of type 2 diabetes. Andrew
to establish the type of diabetes. A short history of
initially manages to control his diabetes with diet, exercise
symptoms in a lean patient, weight loss, a family history of
and weight loss. His diabetes control deteriorates
18
autoimmunity and ketonuria all suggest a diagnosis of type
months later and he is started on metformin treatment,
1 diabetes. In unclear cases, autoantibody measurement
which is the first-line agent in overweight T2DM patients.
and insulin levels can be helpful to distinguish between
His diabetes control deteriorates again and, traditionally,
different types of diabetes. Ivy is started on insulin
either a sulphonylurea or a thiazolidinedione can be given
treatment, which controls her diabetes well. However, her
at this stage. Newer agents, including GLP-1 analogues and
insulin requirements subsequently decrease due to partial
DDP-4 inhibitors, can also be used as second-, third- or
recovery of the pancreas, often known as the honeymoon
fourth-line treatments.
period, which is a temporary phenomenon.
It should be remembered that the majority of diabetes
Andrew, Ivy’s father, measures his blood sugar using his
patients develop vascular complications and it is important
daughter’s glucose meter and his capillary glucose is found
to treat a cluster of risk factors, rather than blood sugar
to be elevated. He is asymptomatic, overweight and his
alone, in these patients to prevent long-term complications.
urine test shows absence of ketonuria. His fasting glucose
These risk factors include hypertension, microalbuminuria,
is checked on two occasions (as he is asymptomatic) and
dyslipidaemia and increased thrombosis potential.
KEY POINTS
Diabetes mellitus, a common condition, is characterized
Type 2 diabetes is characterized by:
by high blood glucose
Occurrence in the older population (peak age 60 years
Diagnosis of diabetes
but can occur at any age) and usually in individuals
Elevated fasting glucose (>7 mmol/L) or postprandial
who are overweight
glucose (>11.1 mmol/L) twice in the absence of
Long history of symptoms (months and even years) or
symptoms or once in the presence of symptoms
no symptoms at all
Glucose tolerance test is warranted in unclear cases and
No history of weight loss and no ketonuria
in those with impaired fasting glucose (>6 mmol/L)
Autoantibody testing (positive in the majority of type 1
Most cases of diabetes (around 80-90%) are due to
diabetes) and fasting insulin levels (low or undetectable in
type 2 diabetes, usually secondary to increased insulin
type 1 diabetes) can help to distinguish type 1 from type
resistance, a condition closely associated with obesity
2 diabetes
In a minority, diabetes is due to specific autoimmune
Treatment
destruction of β cells (type 1 diabetes)
Type 1 diabetes is treated with insulin and the most
Less common causes of diabetes include generalized
common regimes include: twice daily injections of
pancreatic destruction (chronic inflammation, alcohol,
mixed insulin (short acting and intermediate acting),
trauma), endocrine conditions (acromegaly, Cushing’s
four daily injections of long acting insulin (once) and
syndrome) and other rare conditions
short acting insulin with meals, insulin pump
Type 1 diabetes is characterized by:
(continuous insulin infusion)
Occurrence in the younger population (peak age 12
Type 2 insulin is treated in stepwise manner: step 1, diet
years but can occur at any age) and usually in
and exercise; step 2, start metformin treatment; step 3,
individuals who are not overweight
add in a sulphonylurea or thiazolidinedione; step 4,
Short history of osmotic symptoms (days to weeks)
triple oral hypoglycaemic agent therapy; step 5, add in
Rapid weight loss
or switch to insulin treatment. Others: new agents
Ketonuria
(GLP-1 analogues and DDP-4 inhibitors can be used in
Case 16
131
steps 2-4); consider weight-reducing agents (any step)
complications involves the management of a cluster of risk
A large proportion of diabetes individuals develop:
factors:
Microvascular complications (nephropathy, neuropathy
Hypertension (antihypertensive agents)
and retinopathy)
Dyslipidaemia (statins)
Macrovascular complications (cardiovascular disease):
Microalbuminuria (ACE inhibitors and angiotensin
major cause of mortality in diabetes
receptor blockers (ARBs))
In addition to glucose control, prevention of diabetic
Increased coagulation potential (antiplatelet agents)
Case 17
A 78-year-old man with pain in
the leg and knee
Graham, aged 78, has been undergoing investigations for
How would you rule out osteomalacia
pain in his right hip and knee. Routine blood tests showed:
in this patient?
Hb
13.4 g/L
Osteomalacia is associated with:
WBC
8.7 × 109/L
Low or low-normal calcium
Plats
387 × 109/L
Low vitamin D
Na
139 mmol/L
High PTH
K
4.4 mmol/L
High AP
Urea
4.5 mmol/L
Therefore, calcium profile, vitamin D and PTH should
Creat
77 μmol/L
be requested in this patient.
Glu.
5.4 mmol/L
ALT
28 U/L
Calcium profile, vitamin D and PTH are all in the normal
AP
1370 U/L
range.
GGT
31 U/L
Bil
16 μmol/L
What imaging would you request in this
patient?
How do you interpret these results and
X-ray of the hip and knee
what would you do next?
Isotope bone scan
The only abnormality here is a high alkaline phospha-
tase (AP) with otherwise normal LFTs
Figure 59 shows a pelvic X-ray and bone uptake scan.
This suggests that raised AP is from a bony and not
a liver origin
What abnormalities can you identify
If in doubt:
and what is the diagnosis?
AP isoenzymes can be requested that can differenti-
The X-ray shows mixed lytic and sclerotic bone lesions
ate between AP of liver and bony origin
in the pelvic bone (particularly on the right)
A raised AP with normal gamma glutamyl transpep-
The bone scan shows multiple areas of increased uptake
tidase (GGT) further suggests that it is from a bony
The likely diagnosis here is Paget’s disease.
origin
What are the common symptoms of
What are the causes of raised bony AP?
Paget’s disease?
Osteomalacia
Up to 90% of individuals are asymptomatic and the
Fractures
disease is picked up during routine investigations for
Bony metastasis
another pathology
Paget’s disease
Symptoms include:
Growing children and adolescents
Bone pain
Bone deformity: bowing of the tibia is a classical
Endocrinology and Diabetes: Clinical Cases Uncovered. By R. Ajjan.
sign
Published 2009 by Blackwell Publishing, ISBN: 978-1-4051-5726-1
Nerve compression (may cause deafness)
132
Case 17
133
What are the signs of Paget’s disease?
Bone deformity (cranial and tibial bone deformity are
classical features) (see Fig. 60, colour plate section, and
Fig. 15)
Warm bones due to the formation of collateral
circulation
A bruit can be heard over the bone secondary to extra
blood flow
Deafness (nerve compression)
Cranial nerve palsies at the base of the skull
Spinal cord compression
Fractures
Osteogenic sarcoma: rare and serious complication
(a)
Would you treat Graham and why?
Graham needs to be treated as he is:
Symptomatic
Has a high AP
What is the best treatment for this
condition?
Bisphosphonates are currently the best treatment option
for this condition.
How do you monitor response
to treatment?
Improvement in symptoms
Reduction in AP levels
What happens to calcium levels in
patients with Paget’s disease?
Calcium levels are usually normal in uncomplicated
Paget’s disease.
(b)
Figure 59 (a) X-ray of the hip lesions. (b) Bone uptake scan.
What is the long-term management of
this condition?
Monitor for the development of complications
Repeat bisphosphonate treatment as necessary
CASE REVIEW
Graham, who is 78 years old, presents with pain in his knee
bony destruction with new bone formation in the pelvis,
and leg. Initial investigations show a raised alkaline
whereas increased uptake in multiple bones is demonstrated
phosphatase with otherwise normal liver function tests.
on an uptake scan. These findings are consistent with a
Calcium profile, vitamin D and parathyroid hormone
diagnosis of Paget’s disease. The best treatment for
levels are normal ruling out osteomalacia as a cause for
symptomatic Paget’s disease is bisphosphonate that can be
raised alkaline phosphatase. Further investigations show
given orally or intravenously.
134
Part 2: Cases
KEY POINTS
Paget’s disease develops secondary to enhanced activity of
X-ray findings consistent with bone resorption and new
osteoclasts, leading to increased osteoblast activity and
bone formation in a disorganized manner
disorganized new bone formation
Bone uptake scan showing increased activity in affected
The commonest bones to be affected are:
areas
Skull
Complications of the disease include:
Tibia
Fractures
Pelvis
Deafness
Vertebrae
Spinal cord compression
Clinically, Paget’s disease is characterized by:
Development of osteogenic sarcoma: fortunately very
Bony pain
rare
Bone deformity
Hypercalcaemia
The majority of patients are asymptomatic and the
Monitoring treatment
disease is picked up during routine investigations
Clinical symptoms
Diagnosis of Paget’s disease is made by demonstrating:
Alkaline phosphatase levels
Increased plasma levels of bony alkaline phosphatase
Case 18
A 32-year-old woman with a lump in
the neck
A GP refers Sharon with the following letter:
Malignant thyroid nodules are usually cold
This 32-year-old woman has noticed a lump in her neck
(non-functioning)
that moves with swallowing. An ultrasound examination
confirmed this to be a thyroid nodule measuring 2 × 3 cm.
What would be your next step?
I would be most grateful for your advice regarding the
Physical examination
management of this patient’s thyroid condition.
What features would you be looking for
What specific questions would you like
during the physical examination?
to ask the patient?
Establish the patient’s thyroid status
Questions asked should concentrate on trying to differ-
Neck examination to determine the characteristics of
entiate between a benign and malignant thyroid nodule.
the nodule and feel for lymphadenopathy. Findings sug-
These should include the following:
gestive of malignancy include:
How long has the thyroid mass been there?
Firm or hard nodule
A long history
(years) makes it unlikely to be
Fixation to adjacent tissue
malignant
Presence of lymphadenopathy
Has it been growing?
A mass that has not grown over the years is more
Sharon tells you that she noticed the nodule around 12
likely to be benign
months ago and thinks that it may have grown in size in the
Any associated symptoms?
past 2 months or so. On examination, the patient is clinically
Hoarseness and/or dysphagia may indicate a malig-
euthyroid and neck palpation reveals a firm solitary thyroid
nant condition spreading beyond the thyroid gland
nodule with no fixation to adjacent tissue and she has no
Previous history of radiation
(particularly in
palpable lymph nodes. TFTs showed:
childhood)
FT4
19.3 pmol/L
May raise the suspicion of a malignant condition
TSH
1.8 mU/L
Family history of thyroid cancer increases the risk of
malignancy in a thyroid nodule
What would be your next step?
Symptoms of hyperthyroidism
Fine needle aspiration of the nodule.
If the patient is thyrotoxic, the nodule may be hot
(i.e. overactive), making it less likely to be malignant
Cytology of the fine needle aspiration of the thyroid is
It is worth noting:
consistent with a papillary carcinoma.
Thyroid nodules are more common in women but
more likely to be malignant in men
What is the best treatment option for
Thyroid nodules are more likely to be malignant if
this patient?
the patient is younger than 20 or older than 60
Surgery (total thyroidectomy)
The rate of malignancy in thyroid nodules is usually
Radioiodine treatment is usually arranged after surgery
low (less than 10%)
for the ablation of any possible thyroid remnants
The patient undergoes thyroidectomy followed by
Endocrinology and Diabetes: Clinical Cases Uncovered. By R. Ajjan.
radioiodine ablation. There is no evidence of tumour
Published 2009 by Blackwell Publishing, ISBN: 978-1-4051-5726-1
spreading beyond the thyroid gland.
135
136
Part 2: Cases
Therefore, the dose of thyroxine replacement should
Box 34 Different types of thyroid cancer
be increased to suppress TSH levels, without causing sig-
nificant clinical thyrotoxicosis
Papillary carcinoma
TSH suppression is not necessary in patients with
Commonest (70-80%)
Age at presentation: usually 30-50
medullary thyroid cancer or those with lymphoma, as the
Prognosis: good
malignant cells are not TSH responsive (they are not
Treatment: thyroidectomy and radioiodine remnant
thyroid follicular cells)
ablation
Follicular carcinoma
How would you monitor patients with
Less common (15%)
treated thyroid cancer?
Age at presentation: usually 40-50
Regular physical examination
Prognosis: good
Thyroglobulin measurement in the plasma:
Treatment: thyroidectomy and radioiodine remnant
Detection of thyroglobulin in a patient who had pre-
ablation
vious thyroidectomy and ablation therapy indicates the
Anaplastic carcinoma
presence of thyroid tissue
Rare (5%)
Age at presentation: usually 60-80
This is particularly important if thyroglobulin
Prognosis: poor
becomes measurable following a period when levels of
Treatment: surgery and chemotherapy
this protein were undetectable
Lymphoma
Uncommon
If our patient was both clinically and
Age at presentation: usually in women >40, with a
biochemically thyrotoxic on
background of Hashimoto’s thyroiditis
presentation, what would you have
Prognosis: variable
done?
Treatment: radiotherapy/chemotherapy
In a patient who is thyrotoxic and with a thyroid
Medullary thyroid carcinoma
nodule, we need to establish whether:
Rare: can be familial
The patient has a toxic (hot) nodule causing thyro-
Age at presentation: any age (may even occur in
children particularly as part of multiple endocrine
toxicosis (unlikely to be malignant)
neoplasia type 2)
The patient has an overactive thyroid
(Graves’
Prognosis: variable
disease or toxic goitre) with a cold nodule
(cold
Treatment: surgery
What is the prognosis in this case?
Prognosis is excellent with more than 95% cure rate in
developed countries.
After surgery and radioiodine ablation, Sharon is treated
with thyroxine replacement with a dose of 150 mcg/day. Her
TFTs showed:
FT4
20.3 pmol/L
TSH
1.2 mIU/L
What would you do?
TSH stimulates the growth of benign as well as malig-
nant thyroid cells
Therefore, in patients post-thyroidectomy for papil-
lary or follicular cancer, thyroxine replacement should
aim to suppress TSH without inducing clinical
thyrotoxicosis
Figure 61 Thyroid uptake scan.
Case 18
137
nodules are more likely to be malignant in patients
What are the complications
with Graves’ disease)
of thyroidectomy?
The patient should, therefore, undergo a thyroid uptake
Hypothyroidism
scan to differentiate between the two above possibilities
Hypocalcaemia (secondary to damage of the parathy-
roid glands)
Figure 61 is a thyroid uptake scan from a thyrotoxic patient.
Recurrent laryngeal nerve damage
(resulting in a
hoarse voice)
What does it show?
Local haemorrhage
A hot nodule (increased uptake), in the right lobe of
Wound infection
the thyroid
Keloid formation
CASE REVIEW
Sharon, a young woman, is referred by her GP for advice
scan is helpful to distinguish between cold (non-functional)
regarding the management of a thyroid nodule. This is a
and hot (producing excess thyroid hormones) nodules in
common condition and fortunately most thyroid nodules
individuals who are thyrotoxic. Sharon was found to be
are benign. Clinical features suggestive of malignancy
euthyroid and, therefore, she has undergone a fine needle
include a fast growing and hard nodule, hoarseness of voice
aspiration of the thyroid nodule, which was consistent with
or dysphagia and the presence of cervical lymphadenopathy.
a papillary thyroid carcinoma. Consequently, she was
Special care should be taken in individuals with a previous
referred for surgery followed by radioiodine ablation,
history of irradiation or a family history of thyroid cancer.
which is a standard treatment regime for papillary thyroid
Thyroid nodules can be functional, secreting excess thyroid
carcinoma. The prognosis for this type of thyroid cancer,
hormone, which are very rarely malignant, whereas non-
which is the commonest, is very good with a cure rate
functional nodules may be malignant. A thyroid uptake
approaching 95%.
KEY POINTS
Thyroid nodules are common, with a prevalence rate of
(producing excess thyroid hormones). A toxic nodule is
5-30% according to the population studied (prevalence in
usually benign, whereas cold nodule in an individual with
the UK is around 10%)
Graves’ disease carries a significant risk of malignancy
Only a minority of thyroid nodules are cancerous and
Thyroid cancers include:
these are usually non-functional
Papillary (75%)
Toxic thyroid nodules are very unlikely to be malignant
Follicular (10%)
Special care should be taken in individuals with a previous
Medullary (5%)
history of irradiation or a family history of thyroid cancer
Anaplastic (5%)
(malignancy more likely)
Lymphoma (5%)
Clinical features of malignant thyroid nodules include:
The prognosis of papillary and follicular thyroid cancers is
Fast growing, hard nodules and skin fixation over the
usually good, whereas anaplastic cancers carry a very poor
nodule
prognosis. Medullary cancers and lymphomas have a
Presence of cervical lymphadenopathy
variable prognosis
Hoarseness of voice
Treatment of thyroid cancers involves:
Dysphagia
Surgery and radioiodine ablation therapy (papillary and
In euthryoid individuals, the first-line investigation of a
follicular)
thyroid nodule is fine needle aspiration
Surgery (medullary)
In an individual with thyrotoxicosis and thyroid nodule(s),
Radiotherapy and chemotherapy (lymphoma)
thyroid uptake scan should be arranged to investigate
Palliative radiotherapy (anaplastic)
whether the nodule(s) are cold (non-functional) or hot
It important to give a high dose of thyroxine replacement
Continued
138
Part 2: Cases
to suppress TSH in patients with previous papillary and
Thyroglobulin measurement (increased levels in disease
follicular thyroid cancers, without rendering them clinically
relapse)
thyrotoxic
Imaging in cases of clinical suspicion: thyroid uptake
Patients with previous thyroid cancer should have lifelong
scan, neck ultrasound, magnetic resonance imaging
monitoring in specialist centres, using:
Clinical examination
Case 19
A 26-year-old with headaches
and hypertension
Omar, who is 26 years old, is seen by his GP because of
Abdominal MRI: sensitive at detecting these tumours
recurrent headaches and generally feeling unwell. His blood
CT: less sensitive at detecting adrenal tumours
pressure was found to be elevated at 195/100.
Radiolabelled meta-iodobenzylguanidine
(MIBG)
scan: this is positive in around three-quarters of pheo-
What would you do next?
chromcytomas and may detect tumours not visualized
Secondary causes of hypertension should be sought
by MRI
(detailed in Case 15) and appropriate history and physi-
cal examination should be undertaken.
The patient is found to have a large left adrenal tumour
measuring 5 cm in diameter. The surgeon would like to
The patient tells you that for the past 6 months, he has
immediately remove the tumour.
been suffering from increased sweating and heat
intolerance, severe headaches and episodes of palpitations
Do you agree?
associated with pallor.
No, because a surgical procedure in a patient with a
pheochromocytoma may precipitate a hypertensive
From the history, what would you like
crisis
to rule out as a cause for this patient’s
The patient should be prepared for surgery with appro-
hypertension?
priate antihypertensive therapy
Given the:
Symptoms
What antihypertensive is used to treat
Young age
these patients?
Severe hypertension
Patients are usually treated with α-blockers
A pheochromocytoma should be ruled out.
(phenoxybenzamine)
Before surgery, i.v. phenoxybenzamine is used for 3
What test would you request?
days to ensure complete α-blockade
Three
24-h urine collections for catecholamine and
β-blockers can only be used once the patient is fully
metanephrine measurement (see Part 1, p. 33).
α-blocked
Urinary catecholamines were found to be two- to threefold
The patient undergoes surgery and the tumour is
above the upper end of normal in all three 24-h urinary
successfully removed with restoration of normotension off
collections.
any antihypertensive treatment. However, during routine
tests, the patient’s calcium is found to be elevated at
What would you do next?
3.2 mmol/L.
The results are highly suggestive of a pheochromocy-
toma. The next step should be directed at localizing the
What test would you request next?
tumour. Imaging techniques include:
Any patient with hypercalcaemia should have their PTH
levels checked.
Endocrinology and Diabetes: Clinical Cases Uncovered. By R. Ajjan.
The patient’s PTH levels are elevated at 14.3 pmol/L (normal
Published 2009 by Blackwell Publishing, ISBN: 978-1-4051-5726-1
range 1-6.1 pmol/L).
139
140
Part 2: Cases
What is the cause of hypercalcaemia?
Multiple neurofibromas
(Fig.
62, colour plate
Raised calcium with raised PTH is indicative of primary
section)
hyperparathyroidism.
Café au lait spots
Iris Lisch nodules
What diagnosis would you suspect in
Endocrine abnormalities including pheochromo-
this case?
cytoma
The combination of pheochromocytoma and primary
hyperparathyroidism should raise the suspicion of mul-
Does a pheochromocytoma tumour
tiple endocrine neoplasia type II (MEN II).
always occur in the adrenal gland?
No, 10% can be extra-adrenal.
What other endocrine organ would you
like to examine in this patient and why?
Can a pheochromocytoma occur in both
The thyroid gland should be examined in this patient as
adrenal glands?
MEN II includes:
Yes, 10% are seen in both adrenal glands.
Medullary thyroid cancer (MTC): often the initial pre-
sentation of this familial condition and it occurs in all
Is pheochromocytoma a benign tumour?
individuals involved
Most are, but 10% can be malignant.
Pheochromocytoma: usually occurs later in up to half
Box 35 Rule of 10 when dealing with a
the affected individuals
pheochromocytoma
Hyperparathyroidism: occurs in around a quarter of
the affected subjects
The rule of 10 refers to the fact that approximately 10%
of pheochromocytomas are:
The patient is found to have a large thyroid nodule.
Familial
Extra-adrenal
What blood test(s) may help you to
Bilateral
confirm your suspicion?
Malignant
The patient probably has a medullary thyroid
carcinoma
What is MEN I?
Serum calcitonin can be requested, which is usually
MEN I is also an autosomal dominant condition second-
elevated in patients with MTC
ary to a mutation in the menin gene. It involves the
association of:
What is the aetiology of MEN II?
Parathyroid hyperplasia
MEN II is due to a mutation in the ret proto-oncogene,
Pancreatic endocrine tumours, usually:
which is a transmembrane receptor
Gastrinoma
Any patient who is suspected to have MEN II should
Insulinoma
be screened for a ret proto-oncogene mutation
Pituitary adenomas, usually:
As MEN is an autosomal dominant condition, other
Prolactinoma
family members should also be screened for the mutation
Acromegaly
MEN I is best remembered by PPP:
Are all pheochromocytomas familial?
Parathyroid
No, only 10% are familial (some would argue up to 20%
Pancreas
are familial). Familial pheochromocytomas can be seen
Pituitary
in:
MEN II is best remembered by TAP:
MEN II
Thyroid
Von Hippel-Lindau disease, which includes:
Adrenal
CNS and retinal haemangioblastomas
Parathyroid
Pheochromocytoma
Patients and their relatives with MEN I or MEN II
Renal cysts and carcinoma
should be managed in specialized endocrine clinics with
Neurofibromatosis type I, which includes:
the help of a geneticist.
Case 19
141
CASE REVIEW
Omar, a young man, visits his doctor with episodes of
avoid a hypertensive crisis, which may occur during the
headaches, increased sweating, palpitations and is found to
operation. After appropriate medical management, Omar
have significant hypertension. These symptoms raise the
undergoes successful adrenal surgery but his calcium and
possibility of pheochromocytoma as the cause of his
PTH levels are found to be elevated after the operation,
hypertension (secondary hypertension is discussed in Case
indicating a diagnosis of primary hyperparathyroidism.
15). Further investigations demonstrate high levels of
The association of pheochromocytoma and primary
urinary catecholamines, indicating a diagnosis of
hyperparathyroidism should raise the suspicion of multiple
pheochromocytoma. Imaging of the adrenal shows a 5-cm
endocrine neoplasia type II. This diagnosis is further
adrenal mass and the surgeon wants to undertake
supported by the presence of a thyroid nodule, likely to be
immediate surgery. However, Omar should be treated with
a medullary carcinoma, in which case plasma calcitonin
α-blockers prior to surgery to lower the blood pressure and
levels are elevated.
KEY POINTS
Individuals with severe hypertension, particularly the
Headaches and visual disturbances
young, should be investigated for the possibility of
The rule of 10 in pheochromocytoma:
secondary hypertension (see Case 15)
Bilateral in 10%
Pheochromocytomas are rare medullary adrenal tumours
Extra-adrenal in 10%
and a cause of hypertension in less than 0.1% of cases
Malignant in 10%
Pheochromocytomas may be an isolated condition or part
Diagnosis of pheochromocytoma is made by
of/associated with:
demonstrating raised urinary catecholamines and/or
MEN type II
metanephrines in the urine
Neurofibromatosis type I
Surgical treatment is curative in the majority of patients.
Von Hippel-Lindau syndrome
Blood pressure should be controlled before surgery and
Clinical presentation of pheochromocytoma includes:
α-blockers are given prior to the introduction of other
Hypertension sometimes with postural hypotension
antihypertensive agents. Intravenous phenoxybenzamine
Episodes of palpitations and sweating
is given for 3 days before surgery to ensure full
Flushing or pallor
α-blockade
Case 20
Sweating, nausea and hand tremor in
a 24-year-old woman
Eleanor, a thin 24 year old woman, is seen at clinic with a
What would you do next?
few weeks’ history of episodes of sweating, sometimes
This suggests that the cause of this patient’s symptoms is
associated with hand tremor and feeling nauseous. The
hypoglycaemia, which may be due to:
symptoms are particularly pronounced in the morning,
Insulinoma
frequently occur after exercise and are always relieved by
Usually a pancreatic tumour secreting insulin
eating.
(benign in 85% of cases)
Diagnosis is made by demonstrating raised insulin
What differential diagnosis would you
levels and C peptide in the presence of
think of at this stage?
hypoglycaemia
Sweating can be due to a number of conditions
Drug-induced
including:
Insulin: injection of insulin in a non-diabetic indi-
Thyrotoxicosis
vidual results in hypoglycaemia, in which case plasma
Pheochromocytoma
levels of insulin will be high but C peptide will be
Carcinoid syndrome
undetectable, in contrast to a patient with insulinoma.
Menopause
Insulin injection can be seen in: self-injection, patients
Hypoglycaemia
with psychiatric problems
(attention seeking and
Psychological
trying to commit suicide); and injection by others
(possible criminal intent)
What would you do next?
Sulphonylurea
(both insulin and C peptide are
A detailed history is essential concentrating on symptoms
elevated)
of:
Alcohol (due to impairment of gluconeogenesis)
Thyrotoxicosis (see Case 2)
Hormonal deficiencies:
Pheochromocytoma (see Case 19)
Hypopituitarism
Carcinoid syndrome (see Case 24)
Addison’s disease: tiredness, history of pigmentation
Early menopause: if periods are regular, this diagnosis
(due to excessive ACTH secretion), weight loss and
is unlikely
gastrointestinal symptoms
(nausea and vomiting,
Hypoglycaemia: the fact the symptoms are relieved by
abdominal pain, diarrhoea, dizziness and postural
eating suggests that this may be a possibility
hypotension)
Psychological: this is only considered once the above
In severely ill patients:
possibilities are ruled out
Organ failure (acute liver failure)
Infection (septicaemia, malaria)
Eleanor further tells you that her brother has type 1 diabetes
Postprandial
and in view of this her GP checked her fasting blood
Usually postgastrectomy
(dumping syndrome):
glucose, which was found to be low at 2.2 mmol/L.
rapid glucose absorption due to fast gastric emptying
leads to excessive insulin secretion resulting in hypo-
Endocrinology and Diabetes: Clinical Cases Uncovered. By R. Ajjan.
glycaemia 1-3 h after eating
Published 2009 by Blackwell Publishing, ISBN: 978-1-4051-5726-1
Rare causes
142
Case 20
143
Autoimmune: insulin receptor activating antibodies
There is a family history of autoimmunity, and,
Mesenchymal tumours secreting insulin-like growth
therefore, Addison’s disease is a possibility
factors
Blood tests performed by the GP 10 days earlier showed:
What are the symptoms of
Sodium
131 mmol/L
hypoglycaemia and what are
Potassium
5.2 mmol/L
they due to?
Bicarbonate
19 mmol/L
These can be divided into:
Urea
4.6 mmol/L
Adrenergic
Creatinine
76 μmol/L
Sweating
Calcium
2.62 mmol/L
Tremor
Palpitation
What is a possible diagnosis in this
Pallor
patient? What signs would you
Neuroglycopenic (more likely to occur with prolonged
look for?
hypoglycaemia)
The patient has the following symptoms:
Poor concentration
Tiredness
Confusion
Weight loss
Irritability and uncharacteristic violent behaviours
Reduced appetite and nausea
Seizures and coma in severe and more prolonged
Probably postural hypotension
cases
Blood tests show:
Fasting hypoglycaemia
On further questioning, Eleanor categorically denies insulin
Hyponatraemia
injection or sulphonylurea (SU) ingestion and her alcohol
Hyperkalaemia
intake is less than 5 units a week. She has been feeling very
Low bicarbonate
tired for a few weeks, and has lost a stone in weight due to
Mild hypercalcaemia
reduced appetite and nausea. She has also been feeling
A possible diagnosis is Addison’s disease, which can be
dizzy, particularly when getting out of bed first thing in the
associated with the symptoms and the electrolyte abnor-
morning.
malities listed above.
Signs to look for include:
How would this information help you in
Postural hypotension
the diagnosis?
Pigmentation: seen only in primary hypoadrenalism
Insulinoma is less likely as this condition is associated
(due to raised ACTH):
with weight gain; high insulin levels and hypoglycaemia
Areas exposed to pressure (elbows, knees and under
result in frequent snacking.
bras)
Drugs
Palmar creases
There is nothing in the history to suggest insulin/SU/
Scar tissue
alcohol as a cause of her hyperglycaemia
Mucosa
However, most patients injecting insulin/taking SU
deny ever doing so, often making the diagnosis a chal-
On checking the blood pressure, a postural drop from
lenging task
110/70 to 90/55 is noted. Eleanor has clear pigmentation
The patient is not severely ill (which would suggest
in the oral mucosa and palmar creases.
organ failure or septicaemia as the cause for her
symptoms)
Hypoadrenalism
(primary or secondary) is a
What tests would you request next?
possibility:
Short synacthen test: serum cortisol is checked at
Dizziness first thing in the morning is suggestive of
0 min and 30 min
postural hypotension, which is not infrequently seen
ACTH levels
in hypoadrenalism (more common in primary hypo-
Plasma renin activity
adrenalism due to defective secretion of aldosterone)
Aldosterone
144
Part 2: Cases
Her blood tests showed:
What tests can be requested to
Synacthen test:
establish the aetiology of this
0 min cortisol
76 nmol/L
patient’s hypoadrenalism?
30 min cortisol
110 nmol/L
Serological
ACTH 330 ng/L (normal range 10-80 ng/L)
Adrenal autoantibodies, which are positive in most
Aldosterone
40 pmol/L (supine 100-500 pmol/L)
patients with autoimmune hypoadrenalism
PRA
11.1 pmol/mL/h (supine 1.1-2.7 pmol/mL/h)
Radiological
CT or MRI of the adrenals
What is the diagnosis?
Adrenal enlargement can be seen in tuberculosis,
The patient has a highly subnormal cortisol response
infiltrative or metastatic disease
to short synacthen test
Adrenal atrophy is seen in autoimmune
High ACTH
hypoadrenalism
Low aldosterone
Elevated PRA
Adrenal antibodies are positive and a diagnosis of
The diagnosis is, therefore, primary hypoadrenalism.
autoimmune hypoadrenalism is made.
What treatment would you start and
what precautions would you give
the patient?
Glucocorticoid replacement
Cortisol replacement divided into two or three daily
doses
Box 36 Causes of primary hypoadrenalism
Mineralocorticoid replacement
Fludrocortisone
Autoimmune: the commonest cause in Western society
The patient should be advised to double the dose of
(more than two-third of cases)
steroids for a few days in case of a mild illness (i.e. cold
Vascular event: infarction or haemorrhage into the
adrenal glands:
or flu)
Antiphospholipid syndrome
If the patient is more severely ill, i.m. or i.v. steroids
Warfarin treatment
may be required
Meningococcal septicaemia (Waterhouse-Friedrichson
The patient should be given an ampoule of hydrocor-
syndrome)
tisone to inject in cases of emergency (i.e. unable to take
Infection
oral steroids due to vomiting)
Tuberculosis
Fungal: histoplasmosis, cryptococcosis
The patient is admitted 3 months later with:
Opportunistic infections: particularly in patients with
Abdominal pain
AIDS
Diarrhoea
Malignant metastatic disease
Severe dizziness and low blood pressure at 60/40 mmHg.
Congenital adrenal hyperplasia
Inherited disorders of fatty acid metabolism
(adrenoleucodystrophy)
What would you do?
Iatrogenic
The likely diagnosis is acute adrenal insufficiency. The
Prolonged use of steroids followed by sudden
patient should immediately be given:
withdrawal (adrenal glands may take some time to
Intravenous steroids
recover after suppression of function due to external
Intravenous fluids
steroids)
In clinical practice, any patient with unexplained
Ketoconazole: can suppress cortisol production (this
hypotension should be given a dose of hydrocortisone
drug is used to treat Cushing’s syndrome)
after taking a blood sample for random cortisol
Surgical adrenalectomy
measurement.
Case 20
145
CASE REVIEW
Eleanor is a young woman with a few weeks’ history of
confirmed by demonstrating an abnormal synacthen test,
episodes of sweating, hand tremor and feeling nauseous.
low aldosterone, together with elevated ACTH and plasma
These symptoms seem to occur in the morning or after
renin activity. Positive adrenal antibodies indicate
exercise and are always relieved by eating, raising the
autoimmune hypoadrenalism as the cause of adrenal
possibility of hypoglycaemia as the cause. Hypoglycaemia
failure, and this fits with a family history of autoimmunity.
may be due to insulinoma, which is usually associated with
The patient should be treated with a combination of
weight gain due to frequent snacking, excess alcohol, drugs
glucocorticoid and mineralocorticoid replacement and
(sulphonylurea
abuse,
insulin
injections)
and
special precautions to double the dose of steroids in case
hypoadrenalism. The latter diagnosis is suspected due to
of a mild illness and to give intravenous or intramuscular
symptoms of tiredness and weight loss as well as typical
steroids in case of severe illness or if unable to take steroids
electrolyte abnormalities (hyponatraemia, hyperkalaemia
orally (i.e. vomiting). Eleanor is admitted 3 months later
with mildly low bicarbonate). In addition to confirming
with postural hypotension and gastrointestinal symptoms,
morning hypoglycaemia, Eleanor is found to have increased
suggesting acute adrenal insufficiency, also known as an
pigmentation in oral mucosa and palmar creases as well as
adrenal crisis, which should be treated urgently with
postural hypotension, making the diagnosis of primary
intravenous fluid and glucocorticoid.
adrenal failure a strong possibility. This suspicion is
KEY POINTS
Hypoglycaemia should be suspected in individuals with
Postural hypotension
episodes of nausea, hunger, sweating and tremor,
Pigmentation of skin and buccal mucosa (primary
particularly if symptoms are relieved by eating
hypoadrenalism)
Causes of hypoglycaemia include:
Causes of primary hypoadrenalism include
Alcohol-induced
Autoimmune (majority of cases in the Western world)
Insulinoma
Long-term steroid treatment
Hypoadrenalism (primary or secondary)
Infection (tuberculosis, fungal): particularly in the
Drug-induced: insulin or sulphonylurea
immunocompromised)
Reactive hypoglycaemia (occurs post meal, usually after
Vascular event (adrenal infarction or haemorrhage)
gastric surgery)
Infiltrative disease
Severe illness (septicaemia, liver failure)
Metastatic malignancy
Autoimmune (insulin or insulin receptor antibodies)
Congenital adrenal hyperplasia
Investigations of hypoglycaemia include:
Biochemical abnormalities in hypoadrenalism include:
Liver function tests
Hyponatraemia
Ethanol concentration if alcohol abuse is suspected
Hyperkalaemia
Prolonged fasts (up to 72 h) with measurement of
Mild metabolic acidosis
glucose, insulin and C peptide
Anaemia and eosinophilia
Rule out adrenal insufficiency
Mild hypercalcaemia
Treatment of hypoglycaemia:
Diagnosis of primary adrenal failure is confirmed by
Acute: conscious, oral glucose; unconscious, intravenous
demonstrating:
glucose and intramuscular glucagon
Subnormal cortisol response to short synacthen test
Chronic: treat the cause
Raised ACTH levels
Hypoadrenalism should be suspected in individuals with:
Low aldosterone with elevated plasma renin activity
Episodes of hypoglycaemia
Individuals with primary adrenal failure should be
Weight loss
investigated for the cause
Case 21
A 19-year-old man with
sexual dysfunction
Alex consults his GP, at the age of 19, with a history of
His testicles are small measuring 4-5 mL
sexual dysfunction.
He has a small penis measuring 3 cm in length and less than
2 cm in width
What do you want to know at this
He has bilateral gynaecomastia
stage?
Sexual dysfunction is a very common complaint, particu-
What do these findings suggest and
larly in older men. Our patient is a young man and at
what blood tests would you request at
this stage we need to establish:
this stage?
Is this an intermittent problem? The commonest cause
The patient is taller than expected (looking at the
of sexual dysfunction is non-organic (psychological) and
height of his parents)
an intermittent nature may be suggestive of this
He has signs of delayed puberty
diagnosis
Small testicles
Are there any associated stressful life events?
Underdeveloped penis
In a young patient, history of pubertal development is
Little facial hair
essential:
Blood tests to be requested next include
Testicular size
Testosterone
Pubic hair
FSH and LH
Body hair
Voice change
His blood tests show:
Penile development
Testosterone
5 nmol/L (normal range 10-40 nmol/L)
Previous history
FSH
45 U/L (normal range 0.5-5 U/L)
Systemic illness (mumps can cause primary gonadal
LH
38 U/L (normal range 3-8 U/L)
failure)
Testicular trauma
What do these results suggest?
History of neurological disease: spinal cord disease,
Primary gonadal failure supported by:
multiple sclerosis
Low testosterone
History of vascular problems
(usually in older
Raised gonadotrophin
patients)
Chemotherapy
What other questions would you like to
Radiotherapy
ask Alex at this stage?
History of testicular trauma
On examination:
History of testicular infection or systemic illness as
Height is 1.95 m (mother 1.67 m, father 1.76 m)
detailed above
He has normal pubic and axillary hair distribution
He has little facial hair (shaves once every 5 days)
There is no significant previous medical history.
What test would you like to request
Endocrinology and Diabetes: Clinical Cases Uncovered. By R. Ajjan.
and why?
Published 2009 by Blackwell Publishing, ISBN: 978-1-4051-5726-1
Chromosomal analysis
146
Case 21
147
Table 37 Differences in testicular failure occurring before and
The patient’s brother, Phil aged 41, with a 3-year history of
after puberty.
T2DM presents with 10 months’ history of erectile
dysfunction. His blood tests show:
Before puberty
After puberty
HbA1c
6.8%
ALT
140 (normal range <40 IU/L)
Testicular volume
<5 mL
<15 mL, soft
AP
630 (normal range 100-300 IU/L)
Penile length
<5 cm
Normal
Bilirubin
14 μmol/L
Sodium
143 mmol/L
Bone age
Delayed
Normal
Potassium
4.3 mmol/L
Body hair
Greatly reduced
Some reduction
Creatinine
78 μmol/L
Voice
High pitched
Normal
Urea
4.3 mmol/L
What would you do now?
A possible diagnosis here is Klinefelter’s syndrome,
Phil has good diabetes control supported by HbA1c
which is the commonest cause of congenital primary
<7%
hypogonadism, affecting around 1 : 500 men
He has abnormal liver function manifested as raised
ALT and AP
What are the differences between
A detailed history is essential, including:
testicular failure occurring before and
Onset and severity: How long has he had the problem
after puberty?
for? Is it intermittent? Can he achieve partial erection?
These are summarized in Table 37. It should be noted that
An abrupt onset of erectile dysfunction, particularly
in some cases the distinction between these two entities
one that is intermittent is often psychogenic in origin
is not that clear.
Presence of night or morning erection: the absence
of morning erection indicates an organic cause rather
Alex’s chromosomal analysis shows that the patient has an
than a psychological problem
extra sex chromosome in XXY pattern.
History of recent stress: which can be associated with
erectile problems and decreased libido
What is the diagnosis?
Associated symptoms of androgen deficiency:
The diagnosis is Kleinfelter’s syndrome. The main clini-
reduced libido, reduced muscle strength, generally
cal features of this syndrome include:
unwell and tired
Clinical
Sexual dysfunction
Phil tells you that he has had the problem for around a year
Reduced testicular volume
but can still achieve partial erection. Also, he noticed a
Gynaecomastia
decrease in libido in the past few months, which he thought
Female type body composition (eunucoidism)
might be related to his age. His medications include:
Intellectual dysfunction in around half the patients
Metformin 850 mg twice daily
Biochemical
Simvastatin 40 mg daily
Low testosterone with high gonadotrophins
Aspirin 75 mg daily
XXY karyotype
He undergoes a full examination, which is recorded as
Treatment
normal.
Androgen replacement: testosterone injections, tes-
tosterone gel, buccal testosterone, testosterone implants
What would you do now?
(rarely used now)
A history of partial erection is very common in diabe-
tes-related erectile dysfunction
How do you advise this patient
A decrease in libido may indicate reduced testosterone
regarding fertility?
levels but may also occur during stressful life events
Most Klinefelter’s patients are infertile
None of his drugs are associated with erectile dysfunc-
A minority can produce enough sperm to conceive,
tion. The drugs commonly associated with erectile dys-
but this is rare
function include:
148
Part 2: Cases
Antihypertensives
Table 38 Results of insulin stress test.
Antidepressants
Glucose
Cortisol
Growth hormone
Tranquillizers
(mmol/L)
(nmol/L)
(U/L)
Steroid hormones
Alcohol
0 min
5.1
360
1.2
Heroin and marijuana
Digoxin
30 min
5.3
410
1.5
Anti-androgens
60 min
2.8
530
5.9
Statin treatment may be responsible for the abnormal
90 min
1.6
740
12.1
liver function and consideration should be given to stop-
ping this treatment or reducing the dose
120 min
1.8
810
25.6
Metformin treatment can also result in abnormal LFTs
150 min
4.1
820
26.2
but this is less common
180 min
5.2
790
26.3
What tests in relation to erectile
dysfunction would you request?
How do you interpret these results?
The following tests should be requested:
Insulin stress test shows adequate hypoglycaemia and
Prolactin
appropriate elevation of cortisol and growth hormone
Testosterone
(more than 580 nmol/L and 20 U/L respectively) ruling
FSH and LH
out ACTH and GH deficiency
TFTs are normal, ruling out TSH deficiency
Blood tests show the following:
Taken together, Phil has isolated FSH/LH deficiency
Prolactin
211 mU/L (normal <600 mU/L)
as the cause of his hypogonadism
Testosterone
5.6 nmol/L (normal range 10-40 nmol/L)
FSH
2.1 U/L (normal range 0.5-5 U/L)
Would you request any imaging in
LH
0.8 U/L (normal range 3-8 U/L)
this patient?
In view of secondary hypogonadism, pituitary MRI
What do these tests show?
should be requested to rule out a primary pituitary
Phil has hypogonadotrophic hypogonadism,
sup-
pathology causing hypogonadotrophic hypogonadism.
ported by:
Testosterone levels are very low
MRI of the pituitary is normal. The GP requests one blood
FSH and LH levels are low
test that uncovers the aetiology of Phil’s hypogonadism.
What is the aetiology?
Are these results consistent with
Phil has:
Klinefelter’s syndrome?
Diabetes
No, in Klinefelter’s gonadotrophins are high.
Abnormal LFTs
Hypogonadotrophic hypogonadism
What other tests would you
Therefore, haemochromatosis should be ruled out and
request here?
ferritin levels should be checked.
Full assessment of pituitary function to rule out defi-
ciency of other pituitary hormones (see pp. 3 & 4).
Phil’s ferritin was very high at 1260 ng/mL (normal range
20-200 ng/mL).
Phil undergoes an insulin stress test. The results are shown
in Table 38.
What is the differential diagnosis of
TFTs show:
raised ferritin and what would you do
FT4
16.7 pmol/L
for this patient?
TSH
2.4 mIU/L
The differential diagnosis for raised ferritin includes:
Case 21
149
Haemochromatosis
Haemochromatosis
Liver disease (including alcoholism)
Histiocytosis
Cancers
Kallman’s syndrome
Autoimmune conditions
A genetic disorder
The patient should be referred to the haematologist
Associated with anosmia in the majority of patients
for:
Idiopathic hypogonadotrophic hypogonadism (IHH)
Full assessment
Functional
Treatment
(usually venesection for haemochroma-
Excessive exercise
tosis)
Stress
Follow-up
Recreational drugs
Systemic illness
The haematologists confirm a diagnosis of
Severe weight changes
haemochromatosis.
How would you treat Phil’s endocrine
problem?
Box 37 Causes of sexual dysfunction
If fertility is not an issue, simple testosterone replace-
ment should be started
Endocrine causes
If the patient wants to start a family, the treatment is
Hypogonadism
more complicated with:
Primary
Human chorionic gonadotrophin (hCG; mimics
Secondary
LH action)
Thyroid dysfunction
Diabetes
FSH
Hyperprolactinaemia
However, this therapy will not be successful if haemo-
chromatosis has also affected the testicles (iron infiltra-
Non-endocrine causes
tion), in which case the patient will have a mixture of
Drugs
primary and secondary hypogonadism
Neurological disorders
Spinal cord disease
Multiple sclerosis
What are the causes of secondary
Vascular disease (generalized atherosclerosis)
hypogonadism (i.e. associated with low
Penile abnormalities
gonadotrophin levels)?
Psychogenic
Infiltrative disease
CASE REVIEW
Alex, aged
19, is complaining of sexual dysfunction.
karyotype. Testosterone replacement should be started in
Although this is a common problem with advancing age,
this patient and issues with fertility discussed as almost all
it is relatively rare in this age group. A careful history and
individuals with Klinefelter’s are infertile.
appropriate physical examination is important to help
The patient’s brother, Phil, a middle-aged gentleman
reach a correct diagnosis. On examination, Alex is found
with T2DM, also presents with a history of erectile
to be tall with little facial hair and small testicles and penis,
dysfunction. This is a common problem in individuals
together with bilateral gynaecomastia. His blood tests are
with diabetes secondary to neuropathic changes and
consistent with primary testicular failure. Taken together,
vascular disease. A detailed history does not give specific
a likely diagnosis is Klinefelter’s syndrome, which is
clues and his medications do not seem to be the cause. His
confirmed on chromosomal analysis showing XXY
blood tests are consistent with secondary gonadal failure
Continued
150
Part 2: Cases
(low testosterone and inappropriately low/low normal
normal pituitary MRI but elevated ferritin levels. This
gonadotrophins). His liver function is abnormal, which
suggests haemochromatosis as a unifying diagnosis for this
may be due to his treatment (statin) but can also be
patient’s hypogonadotrophic hypogonadism
(pituitary
secondary to other pathologies. He undergoes investigations
deposition of iron), abnormal liver function
(liver
of the pituitary axis, and this shows normal cortisol and
deposition) and diabetes
(pancreatic deposition). In
growth hormone response, whereas his prolactin levels and
addition to venesection, Phil will require testosterone or
thyroid functions are normal, indicating isolated
gonadotrophin replacement (the latter is only used when
gonadotrophin deficiency. Further investigations show
fertility is an issue).
KEY POINTS
Erectile dysfunction is common, particularly in older
Tall stature
individuals, and may be caused by neurological damage,
Intellectual dysfunction in up to half the patients
vascular pathology, androgen deficiency and certain
Reduced testicular volume
medications. It can also be psychological.
Gynaecomastia
Sexual history is important to establish the correct
Abnormal blood tests in KS include:
diagnosis, including:
Low testosterone and raised gonadotrophins (primary
Onset and progression (sudden onset erectile
gonadal failure)
dysfunction is usually psychological)
Karyotype: usually XXY
Extent of the problem (partial or complete)
Other causes of primary hypogonadism include:
Presence of morning erections
Testicular trauma or inflammation (mumps)
Associated medical conditions (vascular disease,
Chemotherapy or radiotherapy
diabetes, spinal cord injuries)
Alcohol excess
Medications
Certain drugs
Social history including recent stress, alcohol,
Cryptorchidism
recreational drugs
Chronic illnesses (renal failure, liver cirrhosis)
Examination should include:
Causes of secondary hypogonadism:
Hair distribution
Kallman’s syndrome (frequently associated with
External genitalia
anosmia)
Evidence of associated endocrine or other medical
Idiopathic hypogonadotrophic hypogonadism
conditions
Functional (stress, exercise, weight loss, acute systemic
Initial investigations for erectile dysfunction should
illness)
include:
Any pituitary pathology
Testosterone, FSH and LH
Treatment of hypogonadism
Prolactin
Primary: testosterone replacement
Thyroid function tests
Secondary: testosterone replacement (fertility is not a
Fasting glucose
concern); gonadotrophin replacement (to restore
Renal and liver function tests
fertility)
Klinefelter’s syndrome (KS) is the commonest congenital
cause of primary hypogonadism and is characterized by:
Case 22
A 38-year-old woman with muscular
aches and weakness
Ayesha is a 38-year-old Asian woman who presents with
A detailed history of ‘pins and needles’ to include:
a 2-month history of generalized muscular aches and
Recent activity (painting and decorating; see above)
weakness, in addition to feelings of pins and needles in her
Does it follow a particular root or nerve distribution?
hands. Also, she is complaining of difficulties in standing up
For example, in median nerve lesions sensation in
from a squatting position.
the palmar aspect of lateral
3½ fingers are affected,
whereas in ulnar nerve lesions the medial 1½ fingers are
What would you do?
affected
Generalized aches and pain is a non-specific symptom
and can be due to:
There is no previous history of note and she is not on any
Muscle strain after exertion
medications. She tells you that the feeling of ‘pins and
Myositis
needles’ affects the tips of her fingers and can sometimes
Metabolic abnormalities
be felt around the mouth. Physical examination reveals:
Pins and needles in the hand may be due to:
No gross neurological abnormality in her upper limbs:
Neurological problem: cervical spondylosis, carpal
normal power, normal sensation to pinprick and touch,
tunnel syndrome
normal reflexes
Metabolic abnormality
Positive Trousseau sign
Difficulty in standing up from a squatting position
Negative Chvostek’s sign
suggests proximal myopathy, which is found in:
Muscular disorders
What does this suggest?
Neurological disorders
The history and the positive Trousseau sign are sugges-
Endocrine disorders: hyperthyroidism, Cushing’s
tive of hypocalcaemia.
syndrome, hypocalcaemia
A careful history and examination is important
What test would you request at
A history of exertion (painting and decorating clas-
this stage?
sically causes pain in the shoulder muscles and may
Calcium profile
cause nerve injury resulting in the sensation of pins
U&Es
and needles)
Has the patient been started on any drugs? Statins
Her blood test shows:
can cause muscular pains
Sodium
138 mmol/L
Metabolic abnormalities associated with muscular
Potassium
3.9 mmol/L
pains include:
Creatinine
76 mmol/L
Hyperglycaemia
Urea
3.8 mmol/L
Uraemia
Calcium
1.84 mmol/L (normal range 2.2-2.6 mmol/L)
Hypocalcaemia
AP
640 U/L (normal range 100-300 U/L)
Hypomagnesaemia
What do these results suggest?
Endocrinology and Diabetes: Clinical Cases Uncovered. By R. Ajjan.
These results suggest that the cause of Ayesha’s symp-
Published 2009 by Blackwell Publishing, ISBN: 978-1-4051-5726-1
toms is hypocalcaemia.
151
152
Part 2: Cases
What would you do next?
The cause of her hypocalcaemia should be
established
Low calcium and raised AP suggests osteomolacia as
the cause
The following tests should be requested:
PTH
Vitamin D levels
Her blood tests showed:
PTH
15.6 pmol/L (normal range 1-6.1 pmol/L)
Vit D3
8 ng/L (normal range >60 ng/L)
Figure 63
What is the aetiology of this patient’s
hypocalcaemia?
This patient’s hypocalcaemia is related to vitamin D
deficiency
What do these results indicate?
Vitamin D deficiency is common in Asian individuals
The tests show normalization of her calcium levels and
due to:
PTH, with near normal AP
Diet: low in vitamin D and calcium
She should continue on her current treatment and
Asian women tend to get less exposure to sunlight
calcium levels should be reassessed in 2-3 months
as the body is covered with clothes
However, we should not automatically assume that
What is the indication for intravenous
hypocalcaemia here is simply due to dietary/cultural
calcium administration?
reasons, particularly in view of the severe disease, and
Intravenous calcium should be avoided if possible as
further investigations may be required
extravasation into the interstitium may cause tissue
necrosis.
Figure 63 is an X-ray of the pelvis/left femur.
Definite indication for i.v. calcium:
What does the X-ray show?
The X-ray shows Looser zone or pseudofracture (right
femur), which is pathognomic of osteomalacia.
Box 38 Causes of hypocalcaemia
Primary hypoparathyroidism
What other condition would you like to
Congenital
rule out in this patient?
Autoimmune
Coeliac disease should be ruled out:
Surgical
Antibodies against transglutaminase (tg) should be
Radiation-related
requested, which are positive in the majority of patients
Osteomalacia
with coeliac disease
Vitamin D deficiency
Vitamin D resistance
Ayesha’s tg antibodies are positive and coeliac disease is
Malabsorption
further confirmed by endoscopy and duodenal biopsy. She is
Hypomagnesaemia
started on vitamin D and calcium supplements (Calcichew
Acute pancreatitis
Multiple blood transfusion (complexing of calcium with
D3) and a gluten-free diet.
citrate)
She continues on treatment and her calcium profile 8 weeks
Increased uptake of calcium into the bone
later shows:
Osteoblastic metastasis (such as prostatic metastasis)
Calcium
2.32 mmol/L
Hungry bone syndrome (following parathyroid/thyroid
PTH
4.5 pmol/L
surgery)
AP
320 U/L
Case 22
153
Seizures or tetany in patients with hypocalcaemia
What is a common cause for refractory
Cardiac arrhythmias/arrest with associated
hypocalcaemia failing to respond to
hypocalcaemia
calcium replacement therapy?
Relative indications
This may be due to magnesium deficiency
Severely symptomatic patient
In hypocalcaemic cases, particularly those that are not
Care should be taken to make the infusion into a large
responding to treatment, magnesium levels should be
vein to avoid extravasation.
checked and replacement started as appropriate
CASE REVIEW
Ayesha, a
38-year-old Asian woman, presents with
2
and raised PTH compatible with a diagnosis of osteomalacia.
months’ history of generalized muscular aches and
An X-ray (pelvis and femur) shows changes compatible
weakness, and pins and needles in her hands, as well as
with Looser zones or pseudofractures, a pathognomic
difficulties in standing up from a squatting position. There
finding in osteomalacia. Although vitamin D-poor diet
is no previous medical history and a detailed history reveals
and lack of sun exposure are common causes of vitamin D
that the sensation of pins and needles can sometimes be
deficiency, all individuals should be investigated for coeliac
felt around the mouth in addition to the finger tips. Her
disease, and it turns out that Ayesha has this condition.
examination shows a positive Trousseau but a negative
Treatment of this patient with a gluten-free diet, together
Chvostek’s sign. The history and examination is suggestive
with vitamin D and calcium supplements, resulted in
of hypocalcaemia, which is confirmed biochemically,
resolution of her symptoms and normalization of her
together with raised alkaline phosphatase, low vitamin D
abnormal biochemistry.
KEY POINTS
Osteomalacia, a common condition, occurs as a result of
Osteomalacia
inadequate mineralization of mature bone. Rickets is a
Hypomagnesaemia
similar condition but occurs in the growing skeleton
Acute pancreatitis
Clinical features of osteomalacia include:
Multiple blood transfusions (complexing of calcium with
Bony and muscular aches and pains
citrate)
Fractures
Increased uptake of calcium into the bone: osteoblastic
Proximal myopathy
metastasis (such as prostatic metastasis); hungry bone
Symptoms of hypocalcaemia
syndrome (following parathyroid/thyroid surgery)
Biochemical findings in osteomalacia include:
Treatment of osteomalacia includes:
Low vitamin D
Vitamin D and calcium replacement
Low/low-normal calcium
Treat the cause
Raised alkaline phosphatase
Calcium should be given orally and intravenous calcium is
Raised PTH
only indicated in:
Causes of low vitamin D levels include:
Seizures or tetany in patients with hypocalcaemia
Poor sunlight exposure
Cardiac arrhythmias/arrest with associated
Poor diet
hypocalcaemia
Malabsorption
Severe symptoms associated with significant
Causes of hypocalcaemia include:
hypocalcaemia
Primary hypoparathyroidism
Case 23
A wrist fracture in a 56-year-old
woman
Christine, aged 56, suffers a fracture of her wrist after a
Christine had a premature menopause at 39. Her diet is well
minor fall. Her past medical history is unremarkable and she
balanced but she thinks she has hyperthyroidism due to
is not on any regular medication.
episodes of anxiety and palpitations. She is a lifelong
smoker.
What do you need to rule out in this
patient?
How would this help you in
As the fracture happened after a minor fall, the possibility
the diagnosis?
of osteoporosis should be ruled out.
She has a number of risk factors for osteoporosis:
Early menopause
What questions would you ask this
Smoking
patient, which may help to diagnose
Possible hyperthyroidism
osteoporosis?
History of previous fractures
How can you confirm the presence
Age at menopause (natural or surgical): earlier age
of osteoporosis?
of menopause is more likely to result in premature
The most widely used technique is dual energy X-ray
osteoporosis
absorptiometry (DEXA)
Diet: a low calcium/vitamin D diet predisposes an
Two sites are usually examined:
individual to premature osteoporosis
Femoral neck
Exercise: minimal activity is associated with premature
Vertebral body
osteoporosis
Results are expressed as T score
Smoking: this predisposes to osteoporosis
Alcohol:
excessive
alcohol
predisposes
to
Box 39 T score and osteoporosis
osteoporosis
A history of steroid use (asthma, inflammatory bowel
T score below 2.5 is indicative of osteoporosis
disease, rheumatoid arthritis): prolonged steroid use
T score between 1.0 and 2.5 is indicative of
results in osteoporosis
osteopenia (low bone density but not severe enough to
A history of height loss suggests vertebral crush frac-
be called osteoporosis)
tures, secondary to osteoporosis
T score higher than 1.0 is regarded as normal
Previous medical history is important. For example,
the following conditions predispose to osteoporosis:
The DEXA test shows severe osteoporosis in the femoral neck
Hyperthyroidism
(T = −4.4) with normal vertebral bone density (T = +1.2).
Hyperparathyroidism
Cushing’s syndrome
What would you do?
Chronic inflammatory conditions
This discrepancy in bone density may indicate a col-
Gastrointestinal disorders
lapsed fracture of the vertebral body, artificially increas-
ing bone density. Therefore, an X-ray of the back should
be performed.
Endocrinology and Diabetes: Clinical Cases Uncovered. By R. Ajjan.
Published 2009 by Blackwell Publishing, ISBN: 978-1-4051-5726-1
Figure 64 is an X-ray of the back.
154
Case 23
155
Hyperparathyroidism
Growth hormone deficiency
Gastrointestinal
disorders
associated
with
malabsorption
Coeliac disease
Crohn’s disease
Neoplastic conditions
Multiple myeloma
Inflammatory conditions
Rheumatoid arthritis
Drugs
Steroids
Heparin
Cyclosporine
Hereditary disorders
Osteogenesis imperfecta
What blood tests would you request
in this patient?
FBC: anaemia may be a sign of malabsorption
Plasma viscosity: raised plasma viscosity is found in
multiple myeloma
U&Es: renal failure may cause osteoporosis
Calcium profile
TFTs
Christine’s blood tests showed:
Hb
13.2 g/L
WBC
6.7× 109/L
Plat
330× 109/L
PV
1.71
Sodium
137 mmo/L
Potassium
3.8 mmol/L
Figure 64
Urea
4.1 mmol/L
Creatinine
71 μmol/L
What does the X-ray show?
Calcium
2.33 mmol/L
The X-ray shows a fracture of the vertebral body,
AP
107 U/L
explaining the artificially increased bone density in the
FT4
18.4 pmol/L
back.
TSH
1.1 mIU/L
What are the causes of osteoporosis?
Endocrine disorders
What do these results suggest?
Hypogonadism: in women, early menopause,
All her blood tests are within normal range, suggesting
anorexia nervosa, athletic amenorrhoea, Turner’s syn-
that Christine’s osteoporosis is simply due to:
drome; in men, hypogonadism due to any cause (see
Premature menopause
Case 9)
Lifestyle
Cushing’s syndrome
Smoking
Hyperthyroidism
Alcohol
156
Part 2: Cases
Her normal TFTs rule out the possibility of
Calcitonin: can be given as injections or intranasally
hyperthyroidism.
for a short period particularly in the presence of painful
vertebral crush fractures
How would you treat this patient?
Calcitriol
(vitamin 1, 25 dihydroxycholcalciferol):
In view of her fractures (wrist fracture and vertebral
can be effective but strict monitoring of calcium is
collapse) and the reduced bone density, this patient needs
required as it may induce hypercalcaemia
to start treatment for osteoporosis
PTH analogue: an effective but expensive treatment
Bisphosphonate with vitamin D3 and calcium supple-
and can only be given as injections
ments remain the mainstay of treatment. Response to
treatment should be initially monitored by yearly
Box 40 Main side effect of bisphosphonate
densitometry
Other treatments for osteoporosis include:
Gastrointestinal disturbances, particularly oesophagitis
Hormone replacement therapy: this is falling out of
Patients are advised to take the tablet on an empty
favour due to increased risk of breast cancer but is still
stomach with a lot of water and should stay upright for
used in younger patients, with a history of early
at least 2 h after ingestion
These drugs are usually prescribed once a week
menopause
together with calcium and vitamin D supplements daily
Strontium: is effective in treating osteoporosis but
Bisphosphonate preparations can be given intravenously,
can make monitoring the response to treatment prob-
with a newer agent given once a year, thereby
lematic (the drug is incorporated into the bone making
simplifying treatment of this condition
DEXA scanning difficult to interpret)
CASE REVIEW
Christine, a woman in her mid-fifties, suffers a wrist
suggests a vertebral collapse, falsely elevating her vertebral
fracture after minor trauma. Due to the circumstances of
density score, a suspicion confirmed on back X-ray which
her fracture (mild trauma), osteoporosis is suspected and
shows collapsed vertebral body, also known as a crush
a careful history is obtained. Christine has a number of risk
fracture. Subsequent investigations show normal blood
factors for osteoporosis including premature menopause,
tests and the possibility of hyperthyroidism is ruled out.
smoking and a history compatible with hyperthyroidism.
Christine is started on bisphosphonate together with
Her dual energy X-ray absorptiometry (DEXA) shows
vitamin D and calcium supplements, which remains the
significant osteoporosis in the femoral neck but an
first-line
treatment for this condition unless
increased bone density in the vertebrae. This discrepancy
contraindicated.
Case 23
157
KEY POINTS
Osteoporosis, a common condition, is associated with
trauma) or vertebral fractures (sudden onset back pain
both quantitative and qualitative changes in bone
and gradual loss of height)
structure
Diagnosis
Causes of osteoporosis include:
Bone densitometry
Endocrine disorders: hypogonadism (in women: early
Routine investigations should be done to rule out
menopause, anorexia nervosa, athletic amenorrhoea,
secondary causes of osteoporosis (particularly in the
Turner’s syndrome; in men: hypogonadism due to any
young)
cause), Cushing’s syndrome, hyperthyroidism,
Treatment of osteoporosis includes:
hyperparathyroidism, growth hormone deficiency
Bisphosphonate with Vitamin D3 and calcium
Gastrointestinal disorders associated with
supplements: first choice
malabsorption
Hormone replacement therapy
Multiple myeloma
Strontium
Inflammatory conditions (rheumatoid arthritis)
Calcitonin
Drugs: steroids, heparin, cyclosporine
Calcitriol
Hereditary disorders (osteogenesis imperfecta)
PTH analogue
Clinical presentation
Monitoring response to treatment
The disease is usually clinically silent until the occurrence
Regular DEXA scans
of a fracture: peripheral fractures (typically after minor
Occurrence of further fractures
Case 24
A 37-year-old woman with
recurrent flushing
Valerie, aged 37, presents with a 2-month history of
Severity and frequency
recurrent flushing. She has also developed a watery
Predisposing factors
diarrhoea over the past 3-4 weeks, opening her bowels up
Associated symptoms
to six times/day.
Diarrhoea
What else would you like to know?
Valerie tells you that she can experience flushing up to three
Main causes of flushing include:
times a day and each episode can last from 10-60 min and
Menopause
is associated with redness in the face. These episodes can
Pheochromocytoma
occur at any time of the day, but particularly after alcohol
Carcinoid syndrome
and Indian food.
Psychological
Causes of diarrhoea include:
What diagnosis would you suspect?
Infections and infestations
Valerie is describing classical symptoms of carcinoid syn-
Viruses
drome including:
Bacteria
Flushing and redness in the face, particularly after:
Parasites
Alcohol
Fungi
(particularly in immunocompromised
Spicy food
individuals)
Diarrhoea
Malabsorption
Coeliac disease
Pancreatic
disorders
(tumours,
chronic
What is the cause of the carcinoid
pancreatitis)
syndrome?
Gut resection
Carcinoid syndrome is caused by neuroendocrine
Inflammatory bowel conditions
tumours secreting serotonin and tachykinins leading to
Overflow diarrhoea (which may occur in the presence
the above symptoms. The presence of symptoms usually
of constipation, particularly in the elderly)
indicates hepatic metastasis
Endocrine causes:
Additional symptoms include:
Neuroendocrine tumours
Bronchospasms
Hyperthyroidism
Right ventricular failure (excess serotonin may cause
Diabetes complicated by autonomic neuropathy
right-sided valvular lesions)
Irritable bowel syndrome
Pellagra-like skin lesions (may develop secondary to
It is important to take a detailed history, concentrating
tryptophan depletion; see below)
on one symptom at a time.
These tumours can secrete a large number of other
Flushing
hormones including:
ACTH
Endocrinology and Diabetes: Clinical Cases Uncovered. By R. Ajjan.
PTH
Published 2009 by Blackwell Publishing, ISBN: 978-1-4051-5726-1
Tumour location
158
Case 24
159
The vast majority of these tumours are found in the
Radiolabelled meta-iodobenzylguanidine (MIBG)
gastrointestinal tract and they are clinically silent until
they metastasize to the liver
What are the treatment options for
A minority of these tumours originate in the lung
carcinoid tumours?
Other organ involvement is very rare
In localized disease, surgical treatment may be
curative
What tests would you do to rule out
Somatostatin analogues (octreotide) can be very effec-
this condition?
tive at controlling the patient’s symptoms
24-h urinary 5 hydroxyindole acetic acid (5-HIAA):
Hepatic embolization: usually palliative
serotonin is synthesized from 5 hydroxytryptophan and
Interferon therapy
is metabolized to 5-HIAA. Patients should be on a special
Around half the patients respond to this therapy, but
diet to minimize the possibility of false-positive results.
experience with the use of this agent remains limited
For example, banana and chocolate may increase urinary
Chemotherapy and radiotherapy only have a transient
5-HIAA
effect
Plasma chromogranin A: a very sensitive marker of the
disease
What is the prognosis of these tumours?
Carcinoid syndrome is usually ruled out in patients
Survival of patients with no hepatic metastasis for 5
with normal urinary 5-HIAA and plasma chromogranin
years ranges from 75-90%
A
Around a third of patients with hepatic metastasis
survive for 5 years
This patient’s biochemical tests are consistent with a
Liver transplantation increases 5-year survival to two-
diagnosis of carcinoid syndrome.
thirds in patients with hepatic metastasis
Patients with hepatic metastasis may survive for as
long as 20, or even 30, years
What would you do next?
The tumour needs to be localized, which can be done
What other types of neuroendocrine
by:
tumours are there?
Imaging
These are summarized in Table 39.
Ultrasound
CT
What should patients with carcinoid
MRI
syndrome have before surgery?
Radionucleotide scanning
Octreotide injections are recommended in the periopera-
Radiolabelled octreotide scan as most of these
tive period to reduce the risk of hypotension and
tumours have octreotide receptors
bronchospasm.
Table 39 Neuroendocrine tumour
Tumour
Organs involved Secreted hormone Main symptom(s)
types.
Insulinoma
Pancreas
Insulin
Hypoglycaemia
Gastrinoma
Pancreas
Gastrin
Severe peptic ulcer
disease
Stomach
Intestine
Continued
160
Part 2: Cases
Table 39 Continued
Tumour
Organs involved
Secreted hormone
Main symptom(s)
Glucagonoma
Pancreas
Glucagon
Characteristic skin
rash
Mucous
membrane
involvement
Glucose
intolerance
Diabetes
VIPoma
Pancreas
Vasoactive intestinal
Watery diarrhoea
peptide (VIP)
Somatostatinoma
Pancreas
Somatostatin
Glucose
intolerance
Stomach
Diabetes
Intestine
Diarrhoea
CASE REVIEW
Valerie, who is 37 years old, presents with a short history
This suspicion is confirmed by demonstrating raised 24-h
of flushing and watery diarrhoea. Episodes of flushing can
urinary 5 hydroxyindole acetic acid (5-HIAA) as well as
occur up to three times/day and are associated with redness
plasma chromogranin A. Imaging techniques are necessary
in the face and there seems to be an association with
to localize the tumour and arrange for appropriate
alcohol and spicy food. The history raises the possibility of
treatment.
carcinoid syndrome as a cause for this patient’s symptoms.
KEY POINTS
Carcinoid is a rare neuroendocrine tumour, which should
Hepatic embolization: usually palliative
be suspected in individuals with a history of:
Immunotherapy: interferon
Flushing: particularly after alcohol or spicy food
Chemotherapy and radiotherapy only have a transient
Diarrhoea
effect
Bronchospasms (can be mistaken for asthma)
Prognosis is variable with survival up to 90% for 5 years
Right-sided heart lesions
in those with no hepatic metastasis
Diagnosis is made by demonstrating:
Other neuroendocrine tumours (all exceedingly rare)
Raised 24-h urinary 5 hydroxyindole acetic acid (5-HIAA)
include:
Raised plasma chromogranin A
Insulinoma: results in hypoglycaemia
Tumour localization is done by:
Gastrinoma: results in severe peptic ulcer disease
Imaging (ultrasound/CT/MRI)
Glucagonoma
Radionucleotide scanning (octreotide, MIBG)
VIPoma
Treatment of carcinoid tumours includes:
Somatostatinoma
Surgery may be curative in localized disease
Somatostatin analogues (octreotide) can be effective at
controlling symptoms
Case 25
A 46-year-old man with an abnormal
lipid profile
Oliver, aged 46, is found to have an abnormal lipid profile,
The patient tells you that:
during routine tests prior to employment abroad.
He smokes 10/day
Total cholesterol (TC)
6.5 mmol/L
His father died of myocardial infarction aged 55
Low density lipoprotein cholesterol (LDL)
5.1 mmol/L
He plays football once a week (but not always)
High density lipoprotein cholesterol (HDL)
0.9 mmol/L
He drinks up to 50 units of alcohol a week
Triglycerides
1.4 mmol/L
On examination, his weight is 89 kg (BMI 27) and blood pres-
The patient is asymptomatic.
sure is 169/90 mmHg, and urine dipstick shows protein -,
RBC -, WBC -, Glu -, Nit -.
What would you do?
This patient has high TC, high LDL, high TG and low
What do these results suggest?
HDL.
The patient has multiple risk factors for coronary artery
High TC and LDL predispose to coronary artery
disease, including:
disease
Family history
Low HDL also predisposes to coronary artery disease
Smoking
The role of high triglycerides in atherosclerotic disease
Overweight
is less defined but high triglycerides are probably also a
Little physical activity
risk factor, particularly as they are associated with low
Hypertension
HDL
Excess alcohol
Very high triglyceride levels can cause pancreatitis
Any associated risk factors should be clarified in this
What tests would you request at
patient, including:
this stage?
Diabetes mellitus
Fasting glucose to rule out the possibility of diabetes
Smoking
TFTs
(hypothyroidism is associated with raised
Hypertension
cholesterol)
Previous history of atherothrombotic disease
U&Es (renal disease is associated with lipid abnormali-
Family history of ischaemic heart disease
ties, usually low HDL and raised triglyceride)
Lifestyle issues:
LFTs (cholestatic disease is associated with raised
Obesity
cholesterol)
Lack of exercise
ECG (rule out previous cardiac event or the presence
Alcohol consumption
of left ventricular hypertrophy)
Oliver’s tests show:
Fasting glucose
5.1 mmol/L
Endocrinology and Diabetes: Clinical Cases Uncovered. By R. Ajjan.
FT4
16.7 pmol/L
Published 2009 by Blackwell Publishing, ISBN: 978-1-4051-5726-1
TSH
1.9 mU/L
161
162
Part 2: Cases
LFTs
normal
tion,
thereby increasing hepatic cholesterol
U&Es
normal
requirements
ECG
normal
Very rarely used these days
The decision to start medical treatment for hyperlipi-
What would you do now?
daemia can be guided by special tables and computer
This patient has multiple risk factors for coronary artery
programs, that take into account associated risk factors.
disease and the following issues need to be addressed:
Dietary advice
Give one renal cause for high
Reduce fat in the diet
cholesterol with normal U&Es
Increase fresh fruit and vegetables
Nephrotic syndrome can result in hyper-
Reduce alcohol
cholesterolaemia
Weight control through:
Urine dipstick should be performed in all patients with
Diet
raised cholesterol
Exercise: this can increase HDL levels, thereby offer-
Table 40 summarizes the most widely used antihyper-
ing protection from atherosclerotic disease
lipidaemic agents.
Modification of other risk factors
Stop smoking: effective at increasing HDL levels
Causes of secondary hyperlipidaemia
Treat high blood pressure (needs more measure-
These are listed in Table 41.
ments to confirm)
Drug therapy
Duncan is known to suffer from hypertriglyceridaemia and
If the above fails to improve the lipid profile, the fol-
is not compliant with his fibrates treatment. His last check
lowing medications can be used:
of his triglycerides was 6 weeks ago, which showed high
Statins
levels at 18 mmol/L. He presents with severe epigastric
Inhibit cholesterol synthesis in the liver and are
abdominal pain.
very effective at lowering LDL and proven to reduce
the risk of coronary artery disease. Most commonly
What is the most likely diagnosis?
used are pravastatin, simvastatin, atorvastatin and
The most likely diagnosis is acute pancreatitis secondary
rosuvastatin
to elevated triglyceride levels.
Ezetimibe
Inhibits cholesterol absorption from the gut, usually
used as an add-on therapy
Fibrates
Table 40 Antihyperlipidaemic agents.
Effective at reducing triglycerides and to a lesser
extent cholesterol; usually used as second line
Agent
Main use
Side effects
Nicotinic acid
Effective at increasing HDL and reducing triglycer-
Statins
Raised cholesterol
Myopathy
ide levels
Liver abnormalities
Bile acid sequestrants
Fibrates
Raised
Myopathy (especially if
Bind to bile acids in the gut inhibiting reabsorp-
triglycerides
used with a statin)
Box 41 Main side effects of statins
Liver abnormalities
Gastrointestinal intolerance
Muscle related:
Simple aches and pains
Ezetimibe Raised cholesterol
Gastrointestinal intolerance
Myositis: diagnosed by symptoms and a significant
(usually as an
increase in creatine kinase
add-on therapy)
Rhabdomyolysis: this is very rare
Nicotinic
Low HDL
Flushing
Liver related
acid
Deranged LFTs (may need to stop statins)
Gastritis
Case 25
163
Table 41 Causes of secondary hyperlipidaemia.
Raised cholesterol
Raised triglycerides
Hypothyroidism
Cushing’s syndrome
Nephrotic syndrome
Chronic renal failure
Drugs (diuretics, steroids)
Drugs (isotretinoin, steroids,
β-blockers)
Poor diet (high in saturated fat) Poor diet and excess alcohol
Pregnancy
Pregnancy
Cholestatic liver disease
Diabetes and insulin resistance
CASE REVIEW
Oliver, a middle-aged asymptomatic man, was found to
according to special tables, which offer risk assessment
have elevated cholesterol (high LDL and low HDL) with
taking into account age, cholesterol levels and associated
normal triglyceride levels during routine tests prior to
risk factors.
employment. Both high LDL and low HDL predispose to
Duncan is another middle-aged gentleman with known
cardiovascular disease and associated risk factors should be
hypertriglyceridaemia treated with fibrates. Unfortunately,
clarified. Other risk factors in this patient include smoking,
he is not compliant with his treatment and a recent check
excess alcohol, obesity, family history of ischaemic heart
of his triglycerides showed high levels at 18 mmol/L. He
disease and mild hypertension. Subsequent tests rule out
presents with severe epigastric abdominal pain, and, given
diabetes and secondary causes of hypercholesterolaemia.
the poorly controlled triglycerides, pancreatitis is suspected,
Lifestyle modifications are important to reduce the risk of
which can be confirmed by measuring plasma amylase
cardiovascular disease, which may improve lipid profile and
levels.
blood pressure. Antihyperlipidaemic agents can be started
KEY POINTS
Hyperlipidaemia is a common condition and can be
Nephrotic syndrome
clinically silent until the development of complications
Drugs
Individuals with raised cholesterol, particularly in the
Pregnancy
presence of low HDL, are at risk of cardiovascular disease
Management of hyperlipidaemia
Individuals with raised triglycerides are at additional risk of
Lifestyle changes are important (stop smoking, reduce
pancreatitis
weight, increase exercise)
Associated risk factors should be addressed in individuals
Medical treatment should be started after appropriate risk
with raised cholesterol including:
assessment. Currently used drugs include:
Diabetes mellitus
Statins: effective at lowering cholesterol and proven to
Smoking
reduce the risk of coronary artery disease
Hypertension
Ezetimibe: usually used as an add-on therapy to reduce
Previous history of atherothrombotic disease
cholesterol levels
Family history of cardiovascular disease
Fibrates: effective at reducing triglycerides and to a
Lifestyle issues (obesity, lack of exercise, excess alcohol)
lesser extent cholesterol
Secondary causes of hyperlipidaemia include:
Nicotinic acid: effective at increasing HDL and reducing
Obstructive liver pathology
triglycerides
MCQs
There are 30 MCQs, each with five answers/statements.
The following statements are true except:
In some cases more than one answer can be correct. You
a. Normal MRI does not rule out the possibility of a
may find some of the questions difficult to answer and
microadenoma
this is deliberate in order to give your brain a chance to
b. Raised prolactin may be due to overtreatment with
do some ‘detective work’, which is an essential compo-
thyroxine
nent of endocrinology.
c. Raised prolactin may be due to treatment with
metoclopramide
d. Raised prolactin may be due to polycystic ovary
1 A 45-year-old woman is referred by her GP with a
syndrome
history of tiredness. Her blood tests show FT4 32.3 pmol/L
e. Pregnancy in this woman should be ruled out as a
(10.0-25.0) and TSH of 9.1 mIU/L (0.2-5.0). She was
cause of her raised prolactin
admitted to hospital 6 weeks earlier with a chest infection,
discharged within 2 days and asked to complete a 5-day
course of antibiotics. She was diagnosed with
3 The following are recognized causes of raised alkaline
hypothyroidism 5 years earlier and has been on treatment
phosphatase of bony origin.
with L-thyroxine 100 mcg/day since. Her TFTs 2 years ago,
whilst on treatment with the same dose of L-thyroxine,
a. Osteoporosis
showed FT4 21.2 pmol/L and TSH 1.8 mIU/L.
b. Coeliac disease
c. Paget’s disease
The most likely cause for the abnormal thyroid result is:
d. Metastatic cancer
a. Poor compliance with thyroxine treatment
e. Renal failure
b. Non-thyroidal illness due to her chest infection
c. Pituitary tumour producing TSH
d. Pituitary thyroid hormone resistance
4 A 37-year-old woman underwent total thyroidectomy
e. Malabsorption due to the development of coeliac
for localized papillary carcinoma measuring 2 cm in
disease
diameter. She presents 1 week later with seizures. Her only
treatment is thyroxine 75 mcg/day and BFZ 2.5 mg as
2 A 35-year-old woman is referred by her GP with 2 years
required for intermittent peripheral oedema.
of amenorrhoea. She has had a long psychiatric history,
Her blood tests show:
but is not currently on any antipsychotics. Her past medical
Hb 13.2, WBC 6.2, Plt 245, Na 136 mmol/L, K 3.7 mmol/L,
history includes autoimmune hypothyroidism and she is
U 5.4 mmol/L, Cr 100 μmol/L, FT4 10.1 pmol/L,
overweight with a BMI of 34. Her prolactin is elevated at
TSH 7.3 mIU/L.
1150 mU/L (normal <600). Her medications include
metoclopramide taken when required and L-thyroxine
The most likely cause for her seizures is:
150 mcg/day. MRI of her pituitary is normal.
a. Hypothyroid encephalopathy due to undertreatment
with thyroxine
b. Cerebral metastasis from her thyroid carcinoma
Endocrinology and Diabetes: Clinical Cases Uncovered. By R. Ajjan.
c. Hypocalcaemia due to parathyroid resection during
Published 2009 by Blackwell Publishing, ISBN: 978-1-4051-5726-1
her thyroidectomy
164
MCQs
165
d. Raised intracranial pressure due to a pituitary
c. Treatment with spironolactone
adenoma as part of MEN I
d. Conn’s syndrome
e. Hypomagnesaemia secondary to BFZ treatment
e. Treatment with angiotensin converting enzyme
inhibitors (ACEI)
5 The following are recognized causes of hypercalcaemia
except:
9 In diabetic ketoacidosis, the following statements
are true:
a. Hyperthyroidism
b. Growth hormone deficiency
a. The two main abnormalities are dehydration and
c. Thiazide diuretics
acidosis
d. Vitamin D intoxication
b. Potassium-containing solutions should be
e. Familial hypocalciuric hypercalcaemia (FHH)
withheld until it is certain that the urine flow is
satisfactory
c. Neurological symptoms or signs during
6 The following are recognized causes of hyponatraemia
treatment of DKA may be due to fluid
except:
over-replacement
d. Gastric dilation and gastroparesis are recognized
a. Hypothyroidism
complications
b. Treatment with chlorpropamide
e. All patients with DKA should be covered with
c. Hypoadrenalism
antibiotics as infection is a common precipitating
d. Chest infection
cause
e. Acromegaly
10 In non-ketotic hyperosmolar hyperglycaemia, the
7 A 37-year-old woman presents with a neck mass that
following statements are true:
has been growing slowly over the past 2-3 years and it
now measures around 2 cm in diameter. On examination,
a. Most patients will require antibiotic cover
there is a 2-cm swelling in the left side of the neck,
b. The condition should be aggressively treated
slightly irregular, relatively hard and it moves with
with i.v. fluid and high doses of intravenous
swallowing. She has no palpable cervical lymph nodes. She
insulin
is both clinically and biochemically euthyroid.
c. The prognosis is better than DKA
d. Anticoagulation is contraindicated
The best course of action is:
e. Acidosis is never seen in these patients
a. Reassure that this is probably a thyroid cyst that will
disappear and arrange to see her again in 2-3 months
b. Arrange for urgent thyroid ultrasound to further
11 In Klinefelter’s syndrome, the following statements are
characterize the mass
true except:
c. Perform a fine needle aspiration of the nodule
d. Arrange for an urgent CT scan of the neck and
a. It is the commonest cause of congenital primary
chest
hypogonadism affecting 1 : 500 people
e. Arrange for an urgent thyroid uptake scan
b. It can be associated with anosmia
c. Intellectual dysfunction is common
d. It is associated with an increased risk of breast
8 The following are associated with increased plasma
carcinoma
renin activity (PRA), except:
e. It is associated with increased height
a. Addison’s disease
b. Congestive cardiac failure (CCF)
166
Part 3: Self-assessment
d. An association with hypercalcaemia usually indicates
12 A 40-year-old man presents with a 3-month history of
reduced calcium excretion secondary to high
weight loss, diarrhoea and reduced libido. His past medical
adrenaline levels
history includes gastro-oesophageal reflux disease and he
e. Once the diagnosis is made, patients should be
is currently being treated with omeprazole. His thyroid
started on β-blockers to reduce the risk of a
function shows: FT4 7.2 pmol/L (10-25), TSH 0.82 mU/L
hypertensive crisis
(0.2-6.0).
The next step is:
16 The following statements are correct in relation to
a. Check thyroid peroxidase (TPO) antibodies
Turner’s syndrome:
b. Start on L-thyroxine treatment
c. Urgently investigate pituitary function
a. The karyotype is XXY
d. Arrange for an urgent ultrasound of the thyroid
b. Patients should be screened for cardiac
e. Urgently investigate his gastrointestinal system for
complications
malabsorption
c. Most patients are tall
d. It is characterized by high gonadotrophins
e. Osteoporosis is a common complication
13 A 27-year-old man, previously fit and well, developed
recurrent headaches for 3 months and a few weeks’
history of increased sweating and weight gain. His GP
17 Polycystic ovary syndrome (PCOS) is associated with all
arranged a CT scan of the head, which shows a mass in
the following except:
the pituitary fossa.
a. Increased body weight
Which of the following conditions is he least likely to
b. Impaired glucose tolerance and lipid profile
have?
c. Anovulation
a. Visual field defects
d. Reduced sex hormone binding globulin
b. Hypocalcaemia
e. Raised FSH/LH ratio
c. Hypertension
d. Hyperpigmentation
e. Cranial nerve palsies
18 The following are recognized causes of diabetes except:
a. Haemochromatosis
b. Cystic fibrosis
14 Acromegaly is associated with all the following except:
c. Chronic alcoholism
d. Conn’s syndrome
a. Diabetes or impaired glucose tolerance
e. Cushing’s syndrome
b. Hypokalaemia
c. Increased risk of colonic cancers
d. Sleep apnea
19 A 56-year-old gentleman with type 2 diabetes for 6
e. Carpal tunnel syndrome
years is admitted with severe shortness of breath that
developed over 12-24 h. His medications include
metformin, pioglitazone, simvastatin, ramipril and aspirin,
15 The following statements in relation to
and he has been on this treatment for more than 2 years.
pheochromocytomas are correct:
There is no history of chest pain. His blood tests show a
normal FBC, U&Es and HbA1c of 7.9%.
a. May result in hyperglycaemia
b. A hypertensive crisis can be precipitated by
The most likely cause for this man’s symptoms is:
abdominal examination
a. Silent myocardial infarction resulting in left
c. Can be extra-adrenal in up to 50% of cases
ventricular failure
MCQs
167
b. Treatment with metformin resulting in lactic
c. Addison’s disease
acidosis and compensatory hyperventilation
d. Treatment of cardiac arrhythmias
c. Treatment with pioglitazone resulting in fluid
e. Pituitary adenoma
retention
d. Simvastatin-induced rhabdomyolysis with
consequent renal failure and metabolic acidosis
24 The following abnormality in gonadal function,
e. Ramipril-induced renal dysfunction secondary to
testosterone 3 nmol/L and LH 0.8 U/L, can be caused by
renal artery stenosis
the following conditions:
a. Haemochromatosis
20 In gestational diabetes:
b. Klinefelter’s syndrome
c. Kallman’s syndrome
a. The risk of developing diabetes in later life is around
d. Previous radiotherapy for intracranial tumours
5%
e. Testicular trauma
b. Patients with gestational diabetes who require insulin
treatment will almost always continue to have
diabetes post delivery
25 In diabetes, the following statements are correct:
c. Congenital abnormalities are more prevalent than in
the infants of non-diabetic women
a. Patients with type 2 diabetes never require insulin
d. Gestational diabetes typically improves in the last 6
treatment
weeks of pregnancy
b. Maturity Onset Diabetes of the Young (MODY) is
e. Thiazolidinediones are probably safer to use than
an autosomal recessive condition
insulin as they reduce the risk of hypoglycaemia
c. The majority of type 2 diabetes patients die from
cardiovascular disease
d. Weight loss is a common symptom of type 1
21 Recognized causes of high anion gap metabolic
diabetes
acidosis include:
e. Recognized endocrine causes of diabetes include
acromegaly and Cushing’s syndrome
a. Renal failure
b. Addison’s disease
c. Cushing’s syndrome
26 The following are recognized presentations of
d. Salicylate overdose
autonomic neuropathy in patients with diabetes:
e. Severe diarrhoea
a. Postural hypotension
b. Gustatory sweating (sweating after tasting
22 Recognized causes of metabolic alkalosis include:
food)
c. Vomiting and/or diarrhoea
a. Treatment with diuretics
d. Resting tachycardia
b. Conn’s syndrome
e. Foot ulcers
c. Primary hyperparathyroidism
d. Severe vomiting
e. Medullary thyroid cancer
27 The following statements are correct in relation to
diabetic nephropathy:
23 The following abnormality in thyroid function, FT4
31 pmol/L and TSH <0.05 mU/L, can be caused by any of
a. The presence of microalbuminuria is associated with
these conditions except:
a reduction of cardiovascular risks in patients with
diabetes
a. Thyroid inflammation
b. Microalbuminuria can be reversed by the use of
b. Pregnancy without concomitant thyroid disease
calcium channel-blockers
168
Part 3: Self-assessment
c. The development of diabetic nephropathy protects
c. Orlistat, sibutramine and rimonabant are agents
from diabetic retinopathy
used for treatment of obesity
d. False-positive microalbuminuria may occur in the
d. Obese individuals are at increased risk of both
presence of urinary tract infection (UTI) or after
cardiovascular disease and cancers
exercise
e. Cushing’s syndrome should be excluded in all
e. Blood pressure improves in the majority after the
individuals with a body mass index >30
development of diabetic nephropathy
30 Which of the following statements in relation to
28 A 27-year-old woman has been complaining of
hyperlipidaemia are correct?
episodes of sweating, tremor and nausea for 6 months,
which frequently occur in the morning (before breakfast)
a. Secondary causes of raised cholesterol include
and are relieved by eating. Her father, who is diabetic and
hypothyroidism and obstructive uropathy
on metformin treatment, checked her capillary blood sugar
b. CoA reductase inhibitors (statins) are the best agents
during one of these episodes, which was low at
to lower LDL cholesterol and their use is associated
1.8 mmol/L.
with reduced mortality in high-risk patients,
including subjects with diabetes
Which of the following are recognized causes for this
c. Fibrates are more effective at reducing triglycerides
patient’s symptoms:
than statins
a. Insulinoma
d. Ezetimibe reduces ileal cholesterol absorption but it
b. Ingestion of metformin
is relatively weak when used alone and is best
c. Addison’s disease
combined with a statin
d. Ingestion of sulphonylurea
e. Nicotinic acid is effective at lowering LDL
e. Hyperthyroidism
cholesterol and is frequently used as second line in
cases of intolerance to statins
29 Which of the following statements regarding obesity
are correct?
a. It is commonly due to a single gene mutation
b. Obese individuals are predisposed to type 1 diabetes
EMQs
1 Thyroid
2 Reproductive endocrinology
a. Toxic multinodular goitre or solitary toxic nodule
a. Deep venous thrombosis
b. Graves’ ophthalmopathy
b. Decreased insulin sensitivity (insulin resistance)
c. Thyroiditis
c. Low plasma oestrogen with elevated FSH and LH
d. Small pituitary tumours
d. Amiodarone treatment
e. Polycystic ovary syndrome
e. Low plasma oestrogen and FSH/LH levels
f. Slow relaxing ankle reflexes
f. Spironolactone
g. Intestinal obstruction
g. Osteoporosis
h. High-dose aspirin
h. Klinefelter’s syndrome
i. Hypothyroidism
i. Frusemide
j. Steroids
j. Addison’s disease
k. Lid lag
k. Premature ovarian failure
l. Agranulocytosis
l. Increased insulin sensitivity
m. Radiation therapy for cancers
m. Turner’s syndrome
n. Graves’ disease
n. TSH and FT4 levels
o. Osteoporosis
o. Polycystic ovary syndrome
For each of the statements below, choose the most likely
For each of the statements below, choose the most likely
answer from the list above. Each answer may be chosen
answer from the list above. Each answer may be chosen
once, more than once or not at all.
once, more than once or not at all.
1. Serious side effects of antithyroid drugs include
1. Polycystic ovary syndrome is commonly asso-
2. Long-term complications of untreated
ciated with
hyperthyroidism include
2. Autoimmunity underpins the aetiology of
3. Hypothermia is a complication of
3. One complication of Turner’s syndrome is
4. Radioactive iodine is the preferred first-line
4. Elevated LH/FSH ratio can be seen in
treatment for
5. Pituitary or hypothalamic causes of amenorrhoea
5. Smoking may cause deterioration in
are characterized by
6. One type of amiodarone-induced thyrotoxicosis
6. Use of oral contraceptive pills increases the
can be treated with
risk of
7. Long-term hypothyroidism can be caused by
7. Menopause is biochemically characterized by
8. The commonest cause of hyperthyroidism is
8. XO karyotype is diagnostic of
9. Thyrotoxicosis with absent uptake of technetium
9. One of the treatment options for polycystic ovary
on thyroid scan is diagnostic of
syndrome is
10. A classical sign of hypothyroidism is
10. In pregnancy, hyperemesis gravidarum can be
associated with
Endocrinology and Diabetes: Clinical Cases Uncovered. By R. Ajjan.
Published 2009 by Blackwell Publishing, ISBN: 978-1-4051-5726-1
169
170
Part 3: Self-assessment
3 Biochemical abnormalities
h. ACTH producing pituitary tumours (Cushing’s disease)
a. Growth hormone producing tumours (acromegaly)
i. Testosterone
b. Primary hypothyroidism
j. Bitemporal hemianopia
c. Hyponatraemia
k. Growth hormone deficiency
d. Primary hypogonadism
l. Dopamine agonists
e. Osteomalacia
m. Radiotherapy
f. Treatment with spironolactone
n. Surgery (usually transphenoidal)
g. Osteoporosis
o. Growth hormone producing pituitary tumours
h. Paget’s disease
(acromegaly)
i. Treatment with acetozolamide
j. Hyperkalaemia
For each of the statements below, choose the most likely
k. Raised bicarbonate (metabolic alkalosis)
answer from the list above. Each answer may be chosen
l. Pituitary tumours producing excess prolactin
once, more than once or not at all.
m. Hypocalcaemia
1. Typical visual field defects with large pituitary
n. Conn’s syndrome
tumours
o. Diabetic ketoacidosis
2. Thin skin is a characteristic feature of
3. An increased risk of colonic malignancy has been
documented with
For each of the statements below, choose the most likely
4. The commonest functioning pituitary tumours are
answer from the list above. Each answer may be chosen
5. Prolactinomas are characterized by good response
once, more than once or not at all.
to medical treatment using
1. Hypercalcaemia with an associated pituitary
6. A known complication of Cushing’s disease is
tumour may be caused by
7. Low blood pressure levels in an individual with a
2. A frequent biochemical abnormality in primary
large pituitary tumour strongly suggests
hypoadrenalism is
8. One hormone that does not require replacement in
3. Hypokalaemia and raised blood pressure are
individuals with complete pituitary failure is
characteristic features of
9. A classical clinical sign in individuals with
4. Very high alkaline phosphatase levels are
Cushing’s syndrome is
characteristics of
10. The best first-line treatment option for non-
5. Secondary causes of raised cholesterol include
functioning pituitary tumours is
6. Cushing’s syndrome can be associated with the
following biochemical abnormality
7. High anion gap metabolic acidosis may be caused by
5 Diabetes mellitus
8. Raised alkaline phosphatase and parathyroid
a. Type 2 diabetes
hormone levels are seen in
b. Cardiovascular disease
9. Syndrome of inappropriate antidiuretic hormone
c. Calcium channel-blockers
secretion is characterized by
d. Hyperosmolar non-ketotic hyperglycaemia
10. Early postoperative complications of thyroid
e. Foot ulcers
surgery include
f. β-blockers
g. Diabetic nephropathy
4 Pituitary
h. Metformin
a. Prolactin producing tumours (prolactinoma)
i. Type 1 diabetes
b. Osteoporosis
j. Heart failure
c. Homonymous hemianopia
k. Pain on walking
d. Metoclopramide
l. Gliclazide
e. Cortisol deficiency
m. Angiotensin converting enzyme inhibitors or
f. Aldosterone
angiotensin receptor blockers
g. Proximal myopathy
n. Maturity onset diabetes of the young (MODY)
EMQs
171
For each of the statements below, choose the most likely
5. In a non-diabetic individual, hypoglycaemia in the
answer from the list above. Each answer may be chosen
presence of high insulin but undetectable C peptide
once, more than once or not at all.
plasma levels is suspicious of
1. A strong family history of diabetes at a young age
6. In a non-diabetic individual, hypoglycaemia in the
is usually found in diabetic individuals with
presence of high insulin and detectable C peptide
2. In addition to osmotic symptoms on presenta-
plasma levels is suspicious of
tion, a short history of weight loss is a recognized
7. Tight glucose control in diabetes is important
feature of
for
3. Obesity is a recognized risk factor for
8. Lactic acidosis is a rare but recognized
4. The first-line medical treatment in overweight type
complication of
2 diabetes patients is
9. A test that helps to distinguish between type 1 and
5. A commonly used medical therapy in diabetes that
type 2 diabetes is
can result in hypoglycaemia is
10. Albumin/creatinine ratio is a useful test for
6. The majority of type 2 diabetes individuals die of
7. The use of thiazolidinedione (glitazone) in diabetes
may result in
7 Endocrine tests
8. The first-line antihypertensive agent to use in
a. Toxic multinodular goitre or solitary toxic nodule
individuals with diabetes is
b. Non-functioning pituitary tumours
9. Peripheral neuropathy increases the risks of
c. High plasma calcium and suppressed PTH levels
10. Angiotensin receptor blockers can be used for the
d. Toxic solitary thyroid adenoma
treatment of
e. Polycystic ovary syndrome
f. Acromegaly
g. High plasma calcium and elevated PTH levels
6 Diabetes mellitus
h. Klinefelter’s syndrome
a. Cushing’s disease
i. Adrenal function
b. Increased endogenous insulin production
j. Thyroiditis
c. Metformin treatment
k. Conn’s syndrome
d. Painful peripheral neuropathy
l. Graves’ ophthalmopathy
e. Prevention from pancreatic cancers
m. Kallman’s syndrome
f. Severe hypoglycaemia in insulin-treated diabetic
n. Cushing’s syndrome
individuals
o. Pituitary function
g. Diagnosis of early diabetic nephropathy
h. Glycosylated haemoglobin levels (HbA1c)
For each of the statements below, choose the most likely
i. Diagnosis of type 1 diabetes
answer from the list above. Each answer may be chosen
j. Exogenous administration of insulin
once, more than once or not at all.
k. Diagnosis of retinopathy
1. Glucagon stimulation and insulin stress test are
l. Prevention from diabetic retinopathy, nephropathy and
used to evaluate
neuropathy
2. Short synacthen test is used to assess
m. Orlistat treatment
3. Raised aldosterone/renin ratio is useful for the diag-
n. Urine dipstick for ketonuria
nosis of
o. Diabetic autonomic neuropathy
4. Glucose tolerance test is used for the diagnosis of
5. Overnight dexamethasone suppression test is used
For each of the statements below, choose the most likely
for the diagnosis of
answer from the list above. Each answer may be chosen
6. Mildly raised prolactin can be found in
once, more than once or not at all.
7. In the presence of normal thyroid function, detection
1. Glucagon injection is an option for
of thyroid stimulating hormone antibodies can be
2. Postural hypotension is a classical finding in
useful for the diagnosis of suspected
3. Testing for GAD antibodies can be used for
8. Hypercalcaemia of malignancy is usually
4. Secondary causes of diabetes include
characterized by
172
Part 3: Self-assessment
9. Low testosterone with low FSH and LH levels and
e. Medullary thyroid cancer
associated anosmia are suggestive of
f. Glucagonoma
10. Low testosterone with elevated FSH and LH levels
g. Von Hippel-Lindau disease
are suggestive of
h. Eating spicy food
i. Ovarian or adrenal virilizing tumours
j. Cushing’s syndrome
8 Medical treatment in diabetes and endocrine
k. Graves’ disease
disease
l. Psychogenic polydipsia
a. Polycystic ovary syndrome
m. Multiple endocrine neoplasia type II (MEN II)
b. Graves’ ophthalmopathy
n. Polycystic ovary disease
c. Painful diabetic peripheral neuropathy
o. Excessive gastrin secretion (gastrinoma)
d. Hypercalcaemia of malignancy
e. Hypercholesterolaemia
f. Conn’s syndrome
For each of the statements below, choose the most likely
g. Growth hormone secreting pituitary tumours
answer from the list above. Each answer may be chosen
h. Renal tubular acidosis
once, more than once or not at all.
i. Hypothyroidism
1. Severe flushing of the face and upper thorax after
j. Excess cortisol production by the adrenal glands
alcohol may be due to
k. Pheochromocytomas
2. The combination of an insulinoma and
l. Gynaecomastia
hyperparathyroidism should raise suspicion of
m. Osteomalacia
3. The combination of pheochromocytoma and
n. Hypertriglyceridaemia
retinal hemangioblastoma suggests a diagnosis of
o. Syndrome of inappropriate antidiuretic hormone
4. Severe peptic ulcer disease that is refractory to
secretion
standard medical treatment should be investigated
for the possibility of
5. The combination of hypercalcaemia and low
For each of the statements below, choose the most likely
urinary calcium excretion is suggestive of
answer from the list above. Each answer may be chosen
6. The combination of rapid weight gain and easy
once, more than once or not at all.
bruising in an individual with newly diagnosed
1. Statins are the best agents to treat
diabetes should raise the suspicion of
2. Fibrates are more effective than statins for the
7. The combination of medullary thyroid cancer and
treatment of
pheochromocytoma suggests a diagnosis of
3. Somatostatin analogues are frequently used for the
8. The combination of a thyroid nodule with raised
treatment of
plasma calcitonin levels suggests a diagnosis of
4. Metyrapone can be used for the treatment of
9. In a euvolemic individual, the combination of
5. Metformin is a treatment option for
hyponatraemia, low plasma osmolarity and high
6. Tricyclic antidepressants are a treatment option for
urine osmolarity suggests a diagnosis of
7. Alpha-blockade is a mandatory treatment for
10. The combination of recent hirsutism, deepning of
8. Fluid restriction is used in the treatment of
voice and clitoromegaly is suggestive of
9. Bisphosphonate is a recognized treatment for
10. Vitamin D is used for the treatment of
9 Combinations
a. Multiple endocrine neoplasia type I (MEN I)
b. Familial hypocalciuric hypercalcaemia
c. Carcinoid syndrome
d. Syndrome of inappropriate antidiuretic hormone
secretion
EMQs
173
2. In a muscular body builder, the finding of low
10 Miscellaneous
testosterone with low normal LH levels is suggestive
a. Primary hypogonadism
of
b. Spironolactone
3. Severe liver failure may result in acute
c. Anabolic steroid abuse
4. In a pregnant woman with Graves’ disease, the best
d. Vertebral crush fractures
treatment option to control thyroid hormone
e. HMG-CoA reductase inhibitors (statins)
levels is
f. Hirsutism
5. One of the best treatment options for diabetic
g. Low dose radioactive iodine
Charcot’s arthropathy is
h. Increased lean body mass and decreased fat mass
6. Hyperaldosteronism secondary to bilateral adrenal
i. Intra-articular steroid injections of the affected joint
hyperplasia is best treated medically with
j. Propylthiouracil
7. A non-diabetic, normocalcaemic individuals with
k. Renal colic due to calculi
polyuria and polydipsia should be investigated for
l. Hypoglycaemia
the possibility of
m. Diabetes insipidus
8. Sudden onset back pain in an individual with
n. Frusemide
known osteoporosis suggests a diagnosis of
o. Immobilization of the affected joint
9. Growth hormone and testosterone hormone
replacement therapy is associated with
For each of the statements below, choose the most likely
10. Late onset congenital adrenal hyperplasia is a
answer from the list above. Each answer may be chosen
recognized cause of
once, more than once or not at all.
1. Rhabdomyolysis is a rare but recognized
complication of treatment with
SAQs
1 A 28-year old-woman presents with classical symptoms
5 An overweight 23-year-old woman (BMI 29) presents
of hyperthyroidism including tremor, heat intolerance,
with 4 months’ history of secondary amenorrhoea.
palpitations, frequent bowel motions, irritability and
weight loss.
a. What basic tests would help you to reach a
diagnosis?
a. What clues in the history or examination would
b. In the presence of hirsutism, what is the commonest
point towards a diagnosis of Graves’ disease?
diagnosis in this group of patients?
b. What are the indications for a thyroid uptake scan in
such individuals?
6 A 51-year-old woman complains of sudden onset severe
back pain. An X-ray of the spine shows collapse of one of
2 A 41-year-old man has been recently diagnosed with
the lumbar vertebrae and general decrease in bone density
diabetes after a few weeks’ history of osmotic symptoms
is noted, suggesting osteoporosis.
and elevated fasting blood glucose at 15 mmol/L.
What are the common causes of early osteoporosis?
In newly diagnosed diabetes, how can you differentiate
type 1 from type 2 diabetes?
7 A 56-year-old man, who was diagnosed with type 2
diabetes 4 years earlier, is admitted to hospital with
3 A 35-year-old woman presents with visual disturbances.
sudden onset chest pain and shortness of breath. An ECG
A CT scan of the head reveals a large pituitary tumour
shows ST elevation in the anterior leads consistent with a
compressing the optic chiasm.
myocardial infarction.
Describe briefly the clinical assessment of this patient.
What is the best strategy to prevent cardiovascular
disease in individuals with type 2 diabetes?
4 A 25-year-old is found to be hypertensive at
190/105 mmHg during routine clinical examination while
8 A 66-year-old woman is complaining of abdominal pain,
registering with a new doctor. Repeat measurement of
constipation and osmotic symptoms. Her fasting glucose
blood pressure shows a value of 193/104 mmHg.
and kidney function are normal but her calcium is found
to be elevated at 3.05 mmol/L.
a. What are the endocrine causes of secondary
hypertension?
a. What are the causes of hypercalcaemia?
b. When should the diagnosis be suspected?
b. In an individual with severe hypercalcaemia, what is
the best medical treatment?
Endocrinology and Diabetes: Clinical Cases Uncovered. By R. Ajjan.
Published 2009 by Blackwell Publishing, ISBN: 978-1-4051-5726-1
174
SAQs
175
9 A 64-year-old man had undergone a CT scan of the
10 A 72-year-old man is admitted to hospital with general
abdomen for suspected abdominal aortic aneurysm. The
deterioration and inability to cope at home. His blood tests
scan rules out this diagnosis but an incidental finding of a
show low sodium at 117 mmol/L with otherwise normal
2-cm adrenal mass is documented.
kidney function. His potassium and calcium are both in the
normal range.
a. What endocrine conditions need to be ruled out?
b. What biochemical tests need to be requested?
What are the causes of hyponatraemia?
MCQs answers
1.
a. This is not an uncommon scenario. Failure to
Total thyroid hormone resistance (both pituitary
comply with thyroxine replacement results in an
and peripheral): again this can produce such a
increase in TSH levels (loss of negative feedback)
biochemical abnormality but the patient is
associated with low FT4. Patients usually
usually clinically hypothyroid (peripheral tissue is
compensate before having their blood test done or
not responding to the effects of thyroxine).
before their review at clinic by taking extra tablets
Normal TFTs 2 years earlier effectively rule out a
of thyroxine, and this results in elevated FT4
diagnosis of thyroid hormone resistance as a
without suppression of TSH as levels of the latter
cause for this patient’s symptoms
require days-weeks to change.
Malabsorption may result in increased thyroxine
Non-thyroidal illness, seen in acutely unwell
requirement in hypothyroid patients, but FT4 is
patients, is characterized by low FT4 with
expected to be low or normal and not elevated. It
inappropriately low/low-normal TSH and can be
should be noted that a large number of
difficult to distinguish from secondary
medications may interfere with thyroxine
hypothyroidism. Therefore, checking TFTs in
absorption including commonly used agents
acutely unwell patients should be avoided unless
such as antacids (Gaviscon) and ferrous
the medical condition is deemed to be
sulphate
thyroid-related.
2.
b. A small pituitary microadenoma producing
A pituitary tumour producing TSH may result in
prolactin without detectable radiological
this biochemical abnormality. However, the patient
abnormalities is always possible (sometimes
is usually hyperthyroid clinically and this picture
the tumour is too small to be visualized by
does not occur in subjects with known
MRI).
hypothyroidism (as the thyroid is unable to
Undertreatment with L-thyroxine may result in
produce thyroid hormones), unless the original
elevated prolactin due to increased secretion of
diagnosis was incorrect.
TRH, which can stimulate prolactin production.
Thyroid hormone resistance is a genetic disease
Therefore, in subjects with raised prolactin, TFTs
due to mutations in the thyroid hormone receptor
should be checked to rule out the possibility of
and can be divided into:
hypothyroidism as a cause for raised prolactin.
Pituitary thyroid hormone resistance: this can
Metoclopramide is a dopamine receptor
produce such a biochemical abnormality due to a
antagonist and can increase prolactin levels. A large
compromised feedback mechanism (thyroid
number of agents can modulate plasma prolactin
hormones are unable to switch off TSH
and these are discussed in the clinical section of
production). The patient is clinically thyrotoxic
this book.
(peripheral tissue responds normally to the
Polycystic ovary disease is a possible diagnosis in
effects of excess thyroid hormones) and there is
this patient, particularly given her weight, which
no history of hypothyroidism
can be associated with mildly raised prolactin
levels. Assessing endometrial thickness helps to
differentiate between microprolactinoma and
Endocrinology and Diabetes: Clinical Cases Uncovered. By R. Ajjan.
PCOS, as the endometrium is thin in the former
Published 2009 by Blackwell Publishing, ISBN: 978-1-4051-5726-1
and thick in the latter.
176
MCQs answers
177
Pregnancy should always be excluded as a cause
FHH is associated with raised plasma calcium,
of raised prolactin levels to avoid unnecessary, and
and it is a benign condition that does not require
often embarrassing, investigations.
any treatment.
3.
b, c, d, e. Osteoporosis does not cause elevation of
6.
e. Conditions a, b and d can all cause
alkaline phosphatase (AP) unless it is complicated
hyponatraemia due to inappropriate ADH
by a fracture.
secretion.
Coeliac disease can result in decreased calcium
Hypoadrenalism results in hyponatraemia
and vitamin D absorption and, hence, osteomalacia
through renal salt-wasting (absence of aldosterone).
with elevation of AP.
Acromegaly is not usually associated with
The hallmark of Paget’s disease is elevation of
hyponatraemia.
AP; clinically silent Paget’s disease is often picked
7.
c. This is a young woman with a mass in the neck,
up during routine tests that show elevated AP
which appears to be thyroid-related (moves with
levels.
swallowing). It may be a thyroid cyst but even this
Metastatic cancers can certainly result in elevated
can be malignant, and, therefore, should be
AP through bone destruction.
investigated.
Renal failure is associated with impaired
An urgent ultrasound can be helpful as there are
hydroxylation of vitamin D to its active form,
some ultrasound criteria that make a lesion
resulting in vitamin D deficiency and consequently
suspicious (increased vascularity, hypoechoic
raised AP.
masses and microcalcification are ultrasound
4.
c. Hypothyroidism does not usually cause seizures,
features of malignancy). However, some benign-
particularly in the presence of relatively mild TFT
looking masses on ultrasound may turn out to be
abnormalities as in this patient. Individuals
malignant.
with severe hypothyroidism, resulting in myxo-
CT scan of the neck and chest may be useful in
edema coma, may develop seizures but this is
subjects with thyroid malignancy to establish the
extremely rare.
extent of the disease and it is also helpful in those
Cerebral metastasis is a possibility but the
with suspected retrosternal goitres. However, it is
original tumour is small and it appears to be
unlikely to be that helpful in making the correct
localized, and, therefore, distant metastases are
diagnosis here.
unlikely.
An uptake scan of the thyroid is helpful in cases
Hypocalcaemia post thyroid surgery is a
of thyrotoxicosis and the presence of thyroid
recognized complication and it is the most likely
nodule to determine whether the nodule is hot
diagnosis here.
(increased uptake), in which case malignancy risk
This patient does not have MEN I, which is
is negligible, or cold (decreased uptake) in which
associated with medullary and not papillary thyroid
case the risk of malignancy is significant. An uptake
cancers.
scan is not that helpful in individuals who are
Severe hypomagnesaemia may cause seizures but
euthyroid.
it is unlikely with intermittent bendrofluazide
The best test in this patient is fine needle
therapy. Also, severe hypomagnesaemia is expected
aspiration (FNA) of the thyroid. Those with a
to result in hypokalaemia, which this patient does
benign cytology should have a repeat FNA in 6
not have.
months. Those with undetermined or suspicious
5.
b. Hyperthyroidism may cause mild
cytology should undergo surgery. In patients with
hypercalcaemia by increasing bone resorption.
high clinical suspicion and benign cytology results,
Growth hormone excess and not deficiency is a
it is perhaps safer to ask the surgeon to intervene as
cause of hypercalcaemia.
thyroid FNA can give false-negative results in 5-8%
Thiazide diuretics increase renal calcium
of cases.
reabsorption, thereby resulting in
8.
d. Addison’s disease is associated with raised PRA
hypercalcaemia.
due to low aldosterone production and
Vitamin D increases calcium absorption from
hypotension, which stimulate renal renin
the gut and decreases renal loss.
secretion.
178
Part 3: Self-assessment
CCF is associated with decreased renal perfusion,
complications develop in a large number of
and, therefore, increased renin production.
individuals.
c and e are both associated with increased renin
Acidosis is not a feature of this condition but
production.
can be seen secondary to the precipitating
In d, there is independent increased production
condition, such as infection or myocardial
of aldosterone, which switches off renin
infarction.
production.
11.
b. Klinefelter’s syndrome (KS) is the commonest
9.
a, c, d. Patients with DKA are dehydrated due to
congenital hypogonadism and is characterized by
osmotic diuresis and vomiting. They are acidotic
low testosterone and high gonadotrophins
due to an abnormal metabolism of fatty acids
(hypergonadotrophic hypogonadism).
giving rise to ketone bodies.
KS is not associated with anosmia, in contrast to
Potassium-containing solutions should not be
Kallman’s syndrome which is characterized by low
withheld if the patient has low plasma potassium as
testosterone and low gonadotrophins
severe hypokalaemia can result in cardiac
(hypogonadotrophic hypogonadism).
arrhythmias and death. Hypokalaemia complicates
Intellectual dysfunction is seen in around half of
treatment of DKA, as correction of acidosis and
KS patients.
insulin treatment both lower plasma potassium by
The risk of breast carcinoma is increased in these
shifting this electrolyte from the extracellular to the
individuals.
intracellular compartment.
It is associated with increased height due to
The development of neurological symptoms
delayed fusion of bone epiphyses (secondary to low
during treatment of DKA may be due to cerebral
testosterone levels).
oedema that can arise secondary to over-
12.
c. Checking this patient’s TPO antibodies is not
enthusiastic fluid replacement.
helpful as his TFT results are not consistent with
Gastroparesis is a known complication of DKA
primary hypothyroidism.
and should be suspected in individuals who
Starting this patient on L-thyroxine may
continue vomiting, in which case the insertion of a
precipitate an adrenal crisis if his cortisol levels are
nasogastric tube may alleviate symptoms.
low, which may be lethal.
The majority of DKA cases are not precipitated
This patient has low FT4 with inappropriately
by infection, and, therefore, routine cover with
low-normal TSH, suggesting secondary
antibiotics for all patients is not warranted.
hypothyroidism. Furthermore, reduced libido
10.
a. In contrast to DKA, a large proportion of
suggests reduced androgen levels. Weight loss and
patients with non-ketotic hyperosmolar
diarrhoea may be due to cortisol deficiency.
hyperglycaemia have an infection, and, therefore,
Therefore, urgent investigations of his pituitary
cover with antibiotics is advised after appropriate
function are required including static hormonal
septic screening.
tests (prolactin, testosterone, SHBG, FSH and LH)
Most of these patients are elderly and frail, and,
as well as stimulating hormonal tests (glucagon
therefore, aggressive treatment with fluid may push
stimulation test or insulin stress test).
these individuals into heart failure. The safest way
Ultrasound of the thyroid has no role here in the
to manage the fluid status of these individuals is via
management of this patient.
the insertion of a central line with regular
Malabsorption may explain the weight loss and
monitoring of the central venous pressure. Despite
diarrhoea but it offers no explanation for the
very high plasma glucose levels, insulin is required
abnormal thyroid function.
in low concentrations in contrast to DKA
13.
b. Visual field defects, classically bitemporal
patients.
hemianopia, are often associated with large
The prognosis is far worse than DKA as
pituitary tumours causing chiasmatic
mortality can reach 50%, compared to less than 5%
compression.
for DKA patients.
Hypercalcaemia can be seen in acromegaly, but
Anticoagulation is advised for these patients, in
hypocalcaemia is not a complication of pituitary
the absence of contraindications, as thrombotic
tumours.
MCQs answers
179
Hypertension may occur in acromegaly or
Chronic anovulation is a key feature of the
Cushing’s disease.
disease.
Hyperpigmentation can be seen in Cushing’s
Reduced sex hormone binding globulin is related
disease.
to insulin resistance and it is one of the
Cranial nerve palsies may occur with large
mechanisms for hirsutism in these individuals as
pituitary tumours or in those complicated by
low SHGB results in higher levels of free androgens
infarction (pituitary apoplexy).
resulting in hirsutism.
14.
b. Acromegaly can result in impaired glucose
LH/FSH ratio are raised in two-thirds of subjects
tolerance and even frank diabetes.
(not FSH/LH ratio).
Hypokalaemia, which may occur in Cushing’s
18.
d. Haemochromatosis results in pancreatic
disease, is not a recognized complication of
infiltration with iron, consequently leading to
acromegaly.
pancreatic β-cell failure and diabetes.
There is an increased risk of colonic cancers in
Cystic fibrosis also results in pancreatic
these patients, and, therefore, periodical
destruction resulting in the development of
colonoscopy is advised for those above the age of 50.
diabetes.
Sleep apnea is a known complication of this
Chronic alcoholism may cause recurrent
condition due to soft tissue enlargement.
pancreatitis, consequently leading to pancreatic
Carpal tunnel syndrome (CTS) is a known
destruction.
complication of acromegaly. The other endocrine
High aldosterone secondary to Conn’s syndrome
condition that results in CTS is hypothyroidism.
is not a recognized cause for diabetes.
15.
a, b. Pheochromocytoma is a very rare cause of
Cushing’s syndrome may result in diabetes
diabetes.
through the diabetogenic action of high steroid
Abdominal palpation can occasionally precipitate
levels.
a hypertensive crisis due to mechanical pressure on
19.
a. The development of symptoms over a short
the tumour.
period of time suggests an acute or semi-acute
Pheochromocytoma is extra-adrenal in only 10%
event. Individuals with diabetes are at increased
of cases.
risk of myocardial infarction (MI), and silent MI
An association with hypercalcaemia suggests
(no chest pain) is commonly seen in this group
MEN 2, which includes pheochromocytoma,
of patients. Therefore, ‘a’ is the most likely
medullary carcinoma of the thyroid and
answer.
hyperparathyroidism.
Metformin is known to cause lactic acidosis, but
Once the diagnosis is made, patients should be
this is a very rare complication and it only seems to
started on an α-blocker before β-blockers can be
occur in subjects with renal failure, very severe
introduced.
heart failure or those who have an underlying
16.
b, d, e. The karyotype is XO (XXY is Klinefelter’s).
infection. Therefore, metformin is contraindicated
Cardiac complications are not uncommon in
in those with renal disease (usual cut-off is a
Turner’s syndrome, including coarctation of the
creatinine >150 μmol/L). It is also contraindicated
aorta, bicuspid aortic valve, aortic root dilatation
in those with severe heart failure or advanced liver
and hypertension.
disease. Lactic acidosis is a possible diagnosis in
Turner’s patients are typically short.
this patient, particularly if he had an infarct, but it
Turner’s is due to a primary ovarian defect, and,
will not be the main cause of his symptoms
therefore, gonadotrophin levels are elevated (loss of
Thiazolidinedione, including pioglitazone,
negative feedback).
treatment is associated with fluid retention and this
Osteoporosis is a common complication due to
class of agents may, therefore, precipitate heart
the loss of protective effects of oestrogen on bones.
failure. However, the symptoms are usually more
17.
e. Most subjects with PCOS are overweight.
gradual (weeks to months). Also, pioglitazone
Impaired glucose metabolism and abnormal lipid
treatment was initiated more than 2 years ago,
profile, secondary to insulin resistance can be
making this treatment as the primary cause of the
found in these subjects.
patient symptoms unlikely. However, pioglitazone
180
Part 3: Self-assessment
treatment may have contributed to the symptoms
Causes of normal anion gap metabolic acidosis
through increased water retention.
include:
Rhabdomyolysis is a very rare complication of
Bicarbonate loss through severe diarrhoea
statin therapy and can indeed result in renal failure.
Renal tubular acidosis
However, there are associated symptoms (severe
Addison’s disease
muscular pain), making this diagnosis unlikely.
Treatment with acetozolamide
Ramipril can indeed result in deterioration in
Therefore, acidosis occurs in b and e but it is of
renal function in those with renal artery stenosis,
normal anion gap.
and this is why U&Es are checked around 1 week
Cushing’s syndrome is characterized by
after starting angiotensin converting enzyme
metabolic alkalosis.
inhibitors to ensure that renal function is stable. In
22.
a, b, d. All diuretics result in metabolic alkalosis
the current case, U&Es were normal, ruling out the
except for acetozolamide (used for the treatment of
possibility of ACEI-induced renal dysfunction as a
glaucoma and high altitude sickness), which results
cause of this patient’s symptoms.
in metabolic acidosis as above.
20.
c. The risk of future diabetes in individuals with
Conn’s syndrome is classically characterized by
a history of gestational diabetes is around
hypokalaemic alkalosis.
30%.
Primary hyperparathyroidism results in
Patients with gestational diabetes frequently
hypercalcaemia without any disturbance in
require treatment with insulin, which is usually
acid-base balance.
stopped after delivery.
Severe vomiting may cause metabolic alkalosis
Congenital abnormalities in children of mothers
through hydrochloric acid loss.
with gestational diabetes are more common than
Medullary thyroid cancers secrete
normoglycaemic mothers and it is one reason why
calcitonin, which has no effect on the acid-base
glycaemic control is kept tight during pregnancy in
balance.
these individuals.
23.
e. Thyroid inflammation (thyroiditis) can be viral
Gestational diabetes does not improve but
or immune (bacterial is very rare), and typically
usually gets worse in the last 6 weeks due to
results in elevated thyroid hormone levels in the
increasing insulin resistance.
blood due to thyroid destruction and release of
Oral hypoglycaemic agents are usually
thyroid hormones into the blood stream (it is not
contraindicated in subjects with gestational
due to excess production). This is usually followed
diabetes due to fears of teratogenicity. Small studies
by a period of hypothyroidism, which may last a
have used metformin and sulphonylureas for the
few weeks to months, after which thyroid function
treatment of these individuals but this is not
recovers. In autoimmune thyroiditis, thyroid
common practice.
function may not recover resulting in permanent
21.
a, d. High anion gap metabolic acidosis essentially
hypothyroidism.
indicates the presence of an extra acid in the blood
In some cases of pregnancy, as well as
that is not normally there. In contrast, normal
trophoblastic tumours, high levels of hCG can
anion gap metabolic acidosis is characterized by a
stimulate the thyroid gland (hCG has similar
disturbance in hydrogen and bicarbonate ion
structure to TSH) causing a thyrotoxic picture
without the presence of ‘an additional’ acid.
usually associated with severe vomiting (known as
Causes of high anion gap metabolic acidosis
hyperemesis gravidarum). Only supportive
include:
treatment is needed and antithyroid drugs are not
Diabetic ketoacidosis
used. This is a self-limiting condition that resolves
Renal failure
spontaneously by week 20 of pregnancy.
Lactic acidosis secondary to: hypoxia, infection,
Addison’s disease can be associated with raised
vascular event (such as myocardial infarction),
TSH levels but not suppressed TSH. The
metformin treatment
mechanism is not entirely clear but is possibly
Poisoning: salicylate, methanol, ethylene
related to deranged thyroid hormone action and
glycol
abnormal feedback on pituitary TSH release.
MCQs answers
181
Cardiac arrhythmias are frequently treated with
Recognized endocrine causes of diabetes include:
amiodarone, an agent that is loved by cardiologists
Acromegaly
and loathed by endocrinologists, and this can cause
Cushing’s syndrome
thyroid dysfunction (hypothyroidism in the
Pheochromocytoma
majority and hyperthyroidism in a minority).
Hyperthyroidism (occurrence of diabetes is rare
Pituitary adenoma is unlikely to cause such an
in this common condition)
abnormality in thyroid function. Pituitary tumours
26.
a, b, c, d. Postural hypotension is a classical
may impair TSH production resulting in low FT4
presentation of autonomic neuropathy, which
with low or inappropriately normal TSH. In TSH
results in dizziness and even syncopy, and this
secreting pituitary tumours, both FT4 and TSH are
typically occurs after standing (patients frequently
elevated.
describe dizzy spells after getting out of bed in the
24.
a, c, d. Low testosterone with low gonadotrophins
morning).
indicate secondary or tertiary hypogonadism
Gustatory sweating is a known complication of
(pituitary or hypothalamic abnormality).
diabetes due to autonomic neuropathy.
Haemochromatosis can result in iron infiltration
Gastrointestinal symptoms due to autonomic
of the pituitary, with subsequent secondary
neuropathy are common and include dysphagia,
hypogonadism. Interestingly, it can also cause
delayed gastric emptying, nausea, vomiting,
primary hypogonadism through testicular iron
diarrhoea and constipation.
infiltration.
Resting tachycardia and fixed heart rate (loss of
Klinefelter’s syndrome is characterized by raised
sinus arrhythmia) can also be related to autonomic
gonadotrophins as the primary defect is in the
neuropathy.
testicles (hypergonadotrophic hypogonadism).
Foot ulcers are usually caused by peripheral
Kallman’s syndrome, a genetic disorder, is
neuropathy and loss of sensation in the feet. It
characterized by failure of gonadotrophin secretion
should be noted that autonomic neuropathy
and is often associated with anosmia.
contributes to foot ulcers through absent
Radiotherapy of the head can cause
sweating, making the feet dry and susceptible to
hypothalamic/pituitary damage resulting in
ulceration.
secondary hypogonadism, commonly associated
Other manifestations of autonomic neuropathy
with impaired production of other pituitary
include:
hormones.
Urinary retention
Testicular trauma may cause primary
Absent sweating
hypogonadism with impaired testosterone
Impotence
production but in this case LH levels are
Defective pupillary reflexes
elevated.
27.
d. The presence of microalbuminuria significantly
25.
c, d, e. Patients with type 2 diabetes are initially
increases the risk of cardiovascular disease.
managed with diet and exercise, with or without
Therefore, these patients should have aggressive
oral hypoglycaemic agents. After varying lengths of
management of their cardiovascular risk factors.
time, the majority of patients end up on insulin
Calcium channel-blockers do not generally affect
treatment secondary to pancreatic β-cell failure.
microalbuminuria, which can be reversed by ACEI
Maturity onset diabetes of the young is a
or ARB, and in some cases a combination of the
monogenic form of diabetes (i.e. caused by a
two agents is used.
mutation in a single gene) and it is an autosomal
Diabetic nephropathy is usually associated with
dominant not recessive condition.
diabetic retinopathy. In fact, microvascular
Cardiovascular disease remains the main cause
complications frequently present together, and,
of mortality in subjects with diabetes, as up to 80%
therefore, any patient with one microvascular
of patients die of this condition.
condition should be carefully examined for other
In addition to osmotic symptoms, type 1
microvascular disease.
diabetes subjects present with a short history of
UTI should be ruled out in any individuals who
weight loss.
have a positive microalbuminuria result. Patients
182
Part 3: Self-assessment
are advised to have the urine collection first thing
Obesity predisposes to type 2 but not type 1
in the morning as exercise can cause
diabetes.
microalbuminuria in the absence of renal disease.
Medical treatment of obesity includes orlistat,
Other causes for false-positive microalbuminuria
which acts locally in the gut by inhibiting lipase
include menstruation, pregnancy, any febrile illness
activity, thereby reducing fat absorption.
and congestive cardiac failure.
Sibutramine is a centrally acting appetite
Blood pressure control tends to worsen after the
suppressant but its use in diabetes is not
development of diabetic nephropathy, and this
widespread as it may result in the development of
further results in worsening in renal function
hypertension. Rimonabant is a cannabinoid
creating a vicious cycle. Therefore, tight blood
receptor-blocker, which results in decreased
pressure control in individuals with diabetic
appetite and a feeling of satiety after a meal. This
nephropathy is of paramount importance to avoid
latter agent is very effective at reducing weight but
further deterioration of renal function.
is associated with the development of depression in
28.
a, c, d. Insulinoma typically presents with episodes
around one in seven patients.
of hypoglycaemia, particularly after prolonged
The risk of cardiovascular disease, cancers and
fasting. Patients usually put on weight as they
respiratory conditions is increased in obese
frequently snack to avoid hypoglycaemic attacks.
individuals.
Mode of action of metformin includes inhibition
Obese patients should only be investigated for
of glucose release from the liver, interference with
Cushing’s syndrome in the presence of strong
gut glucose absorption, in addition to a mild
clinical suspicion of this condition.
insulin sensitizing effect. Therefore, it is not usually
30.
b, c, d. Obstructive uropathy does not cause raised
associated with hypoglycaemia as it does not result
cholesterol. Causes of secondary hyperlipidaemia
in increased insulin production.
include hypothyroidism, obstructive liver disease
Addison’s disease is a recognized cause of
and nephrotic syndrome.
hypoglycaemia due to the absence of
Statins have revolutionized treatment of
corticosteroids.
cardiovascular disease and they are used for both
Agents in the sulphonylurea group increase
primary and secondary prevention.
insulin production by pancreatic β-cells, and,
Fibrates are used first line in individuals with
therefore, can result in hypoglycaemia.
elevated triglycerides and can be combined with a
Hyperthyroidism can result in hyper- not
statin in individuals with combined
hypoglycaemia.
hyperlipidaemia.
29.
c, d. Obesity may be due to single gene mutations
The efficacy of ezetemibe is modest when used
(such as mutation in leptin or its receptor), but
alone and best effects are seen when combined with
these cases are very rare. The main cause of obesity
a statin.
is related to the lifestyle: too little exercise and too
Nicotinic acid is the best agent at raising HDL
much food.
levels, but it is not effective at reducing LDL.
EMQs answers
1
4
7
1.
l
1.
j
1.
o
2.
o
2.
h
2.
i
3.
i
3.
o
3.
k
4.
a
4.
a
4.
f
5.
b
5.
l
5.
n
6.
j
6.
b
6.
b, e
7.
m
7.
e
7.
l
8.
n
8.
f
8.
c
9.
c
9.
g
9.
m
10.
f
10.
n
10.
h
2
5
8
1.
b
1.
n
1.
e
2.
k
2.
i
2.
n
3.
g
3.
a
3.
g
4.
o
4.
h
4.
j
5.
e
5.
l
5.
a
6.
a
6.
b
6.
c
7.
c
7.
j
7.
k
8.
m
8.
m
8.
o
9.
f
9.
e
9.
d
10.
n
10.
g
10.
m
3
6
9
1.
a
1.
f
1.
c
2.
j
2.
o
2.
a
3.
n
3.
i
3.
g
4.
h
4.
a
4.
o
5.
b
5.
j
5.
b
6.
k
6.
b
6.
j
7.
o
7.
l
7.
m
8.
e
8.
c
8.
e
9.
c
9.
n
9.
d
10.
m
10.
g
10.
i
Endocrinology and Diabetes: Clinical Cases Uncovered. By R. Ajjan.
Published 2009 by Blackwell Publishing, ISBN: 978-1-4051-5726-1
183
184
Part 3: Self-assessment
10
4. j
8. d
1. e
5. o
9. h
2. c
6. b
10. f
3. l
7. m
SAQs answers
1
symptoms of pituitary hormone deficiency, which may
a. Important clues for the diagnosis of Graves’ disease
occur in large functional or non-functional tumours
include: a personal or family history of
(pressure effects on normal pituitary tissue), e.g.
autoimmunity, the presence of a diffuse,
thyroid: tiredness, dry skin, cold intolerance; adrenal:
symmetrical and smooth goitre, and the presence of
tiredness, weight loss, gastrointestinal symptoms, low
extrathyroidal manifestation of the disease including
blood pressure; and sex hormones: reduced libido,
Graves’ ophthalmopathy (GO), pretibial myxoedema
sexual dysfunction in men, menstrual abnormalities in
(usually in combination with GO) or thyroid
women. Fully examine the patient for signs of excess or
acropachy (rare).
reduced hormonal secretion (see section on pituitary
b. Thyroid uptake scan in thyrotoxicosis can be
tumours and pituitary failure).
requested in suspicion of thyroiditis, thyroid
nodule(s) [hot nodule (increased uptake) is very
4
rarely malignant whereas a cold nodule on a
a. Endocrine causes of hypertension include: Cushing’s
background of Graves’ disease carries a significant
syndrome, acromegaly, Conn’s syndrome,
risk of malignancy], and unclear cases
pheochromocytoma and renal artery stenosis
(thyrotoxicosis in the absence of a goitre).
(consequently resulting in increased renin and
aldosterone production).
2
b. Diagnosis should be suspected in those with:
Important indicators of type 1 diabetes (T1DM) are:
hypertension at a young age, severe and resistant
young age at diagnosis (but T1DM may occur at any
hypertension and the presence of symptoms and/or
age), absence of obesity (but T1DM may occur in obese
signs suggestive of a secondary pathology.
individuals), personal or family history of
autoimmunity, short history of symptoms (days-few
5
weeks), history of weight loss, and presence of ketones
a. Pregnancy test, prolactin, sex hormone levels:
on urine dipstick. Autoantibody tests can be useful in
oestradiol, testosterone, SHBG, FSH and LH, and
difficult cases, but can be false-negative in up to 20% of
pelvic ultrasound, which is helpful to establish
T1DM individuals.
ovarian and endometrial pathology and endometrial
thickness.
3
b. The commonest diagnosis in a non-pregnant young
Examine visual fields for defects and request formal
overweight woman with hirsutism and secondary
visual field testing. Establish in the history any
amenorrhoea is polycystic ovary syndrome. It should
symptoms of hormonal excess in case the tumour is
be stressed that taking a proper history before
functional, e.g. prolactin: galactorrhoea; growth
requesting the blood tests is extremely important to
hormone: changing glove and shoe size, headaches,
establish the correct diagnosis.
increased sweating; and steroids: increase in weight,
easy bruising, proximal myopathy. Establish any
6
Gonadal abnormalities, premature menopause or
Endocrinology and Diabetes: Clinical Cases Uncovered. By R. Ajjan.
prolonged amenorrhoea or hypogonadism in men.
Published 2009 by Blackwell Publishing, ISBN: 978-1-4051-5726-1
Endocrine disease: Cushing’s syndrome,
185
186
Part 3: Self-assessment
hyperparathyroidism, untreated hyperthyroidism and
9
growth hormone deficiency. Gastrointestinal
a and b. Pheochromocytoma: 24 urine collection for
conditions: inflammatory bowel disease and
catecholamines (usually 3 collections are required).
malabsorption due to any cause (for example coeliac
Conn’s disease: U&Es (to rule out hypokalaemia),
disease). Neoplastic disease: multiple myeloma. Chronic
aldosterone/renin ratio.
inflammatory conditions: rheumatoid arthritis. Long-
Cushing’s syndrome: one or two of the tests below can
term steroid use.
be requested: overnight or low-dose dexamethasone sup-
pression test, midnight cortisol, 24-h urinary cortisol
7
measurements.
The risk of myocardial infarction (MI) in a diabetic
individual with no known cardiovascular disease is
10
similar to that of a non-diabetic with a previous MI.
The aetiology varies according to the clinical status of
It is important to treat a cluster of risk factors in these
the patient.
individuals in order to reduce the risk of future ischemic
Hypovolaemic patient (dehydrated)
events:
Renal salt loss
Hyperglycaemia, hypoglycaemic agents
(oral or
Drugs (diuretics)
insulin) to optimize glycaemic control
Renal disease (recovery phase of acute renal
Dyslipidaemia, cholesterol-lowering agents
(usually
failure, relief of bilateral ureteric obstruction, salt
HMG-CoA reductase inhibitors, statins are used)
wasting nephropathy)
Hypertension, antihypertensive medication
(tight
Addison’s disease
blood pressure control is important in diabetic individu-
Gastrointestinal loss
als). First-line agents are angiotensin converting enzyme
Vomiting
inhibitors
(ACEI) and angiotensin receptor blockers
Diarrhoea
(ARB)
Hypervolaemic patient (excess water)
Increased urinary albumin excretion (usually mea-
Congestive cardiac failure
sured using albumin/creatinine ratio): agents used are
Liver failure
ACEI and ARB
Nephrotic syndrome
Increased coagulation: antiplatelet agents, usually
Excess water intake: commonly seen in hospitalized
aspirin, are given to high-risk individuals, although
patients, who are given too much intravenous fluid not
recent evidence questions the efficacy of aspirin in dia-
containing salts (such as 5% dextrose)
betes subjects
Euvolaemic patient
Increased weight: diet and exercise are an important
Syndrome of inappropriate ADH secretion, character-
part of treatment and in difficult cases weight-reducing
ized by low plasma osmolarity and inappropriately high
agents, and even surgery, can be used
urine osmolarity with increased urinary sodium concen-
trations. Causes include:
8
Malignancy
a. Hyperparathyroidism, malignancy, familial
Respiratory disease (usually chest infections)
hypocalciuric hypercalcaemia, granulomatous
Central nervous system abnormalities
disease (sarcoidosis), vitamin D intoxication,
(encephalitis, meningitis, vascular event, head
thiazide diuretics, hyperthyroidism, Addison’s
injury)
disease.
Metabolic: hypothyroidism, acute intermittent
b. Patients with severe hypercalcaemia should be
porphyria
rehydrated first and can then be treated with
Drugs: a long list including antiepileptic
intravenous bisphosphonates.
treatment, chemotherapy agents, antidiabetic
Other treatment options for resistant hypercalcaemia
medications, psychiatric drugs
include calcitonin and high-dose steroids.
Idiopathic
Index of cases by diagnosis
Case 1 Diabetic ketoacidosis, 66
Case 14 Amiodarone-induced thyrotoxicosis, 120
Case 2 Thyrotoxicosis, 73
Case 15 Conn’s syndrome, 123
Case 3 Hypercalcaemia of malignancy, 79
Case 16 Type 1 and 2 diabetes mellitus (T1DM and
Case 4 Non-functioning pituitary tumour, 82
T2DM), 126
Case 5 Cushing’s syndrome, 86
Case 17 Paget’s disease, 132
Case 6 Hypothyroidism, 89
Case 18 Thyroid carcinoma, 135
Case 7 Hyperosmolar non-ketotic hyperglycaemia
Case 19 Pheochromocytoma, multiple endocrine
(HONK), 92
neoplasia type II (MEN II), 139
Case 8 Acromegaly, 98
Case 20 Hypoglycaemia, hypoadrenalism, 142
Case 9 Prolactinoma, 102
Case 21 Klinefelter’s syndrome, secondary
Case 10 Pituitary failure, 106
hypogonadism, 146
Case 11 Diabetes insipidus, 109
Case 22 Osteomalacia, 151
Case 12 Syndrome of inappropriate ADH secretion
Case 23 Osteoporosis, 154
(SIADH), 113
Case 24 Carcinoid syndrome, 158
Case 13 Polycystic ovary syndrome (PCOS), 117
Case 25 Hyperlipidaemia, 161
187
This page intentionally left blank
Index
Note: page numbers in italics refer to figures
adrenal medulla
30
in pregnancy
20, 76
and those in bold to tables.
adrenal tumours
6, 30, 35
side effects
16, 74
incidentalomas
35, 175, 186
arginine-vasopressin see antidiuretic hormone
abdominal pain
66-8
pheochromocytoma
139, 140
arterial blood gases (ABG)
93
abdominal X-ray (AXR)
68
virilizing
42
atrial fibrillation, paroxysmal
120
acanthosis nigricans
117
adrenal venous sampling
30, 33
autoimmune disorders
67, 90
acarbose
53, 129
adrenocorticotrophic hormone see ACTH
autonomic neuropathy, diabetic
49, 59, 167,
acetazolamide
180
agranulocytosis
16, 74
181
acromegaly
5-6, 98-101
alcohol intake, excess
6-7, 92, 154, 161,
clinical presentation
5, 82-3, 98-9,
179
β-blockers
75, 139
100
aldosterone
30
β-cells, pancreatic
46
complications
100
biosynthesis
41
destruction
46
investigations
5, 83, 99
control of secretion
32
dysfunction
47
MCQ
166, 179
deficiency
34, 42
bicarbonate
non-pituitary causes
100
excess
32-4
therapy
56, 57, 58, 70, 95
treatment
5-6, 99-100
serum
124
venous/plasma
56, 68
ACTH
1, 2, 30
aldosteronism see hyperaldosteronism
biguanides
53
abnormalities of secretion
6-8
alkaline phosphatase (AP)
132, 151
bile acid sequestrants
162
congenital adrenal hyperplasia
42
bony
132, 164, 177
biochemical abnormalities
170
deficiency
8, 10, 82, 106
α-blockers
33, 139
bisphosphonates
ectopic production
4, 6, 7, 31, 86-7
α-cells, pancreatic
46
hypercalcaemia
26, 80-1
excess
6-8, 82, 86-7
amenorrhoea
102
osteoporosis
28, 156
stimulation test
34, 143-4
causes
102, 103
Paget’s disease
29, 133
Addison’s disease
30, 34, 143-4
hypothalamic
44
side effects
156
differential diagnosis
142
primary
39, 102
body mass index (BMI)
61, 126
thyroid function tests
180
secondary
39, 102-3, 174, 185
bone isotope scan, Paget’s disease
28, 29, 132,
see also adrenal failure
amiodarone
19-20, 120, 181
133
ADH see antidiuretic hormone
amiodarone-induced hyperthyroidism (AIT)
bone mass, loss of
27
adrenal adenomas
32-3, 124-5
19-20, 120-2
bone metabolism
23-9
adrenal carcinoma
32
type 1
19, 20, 120-1
bone metastases
80
adrenal cortex
30
type 2
19, 20, 121
bone mineral density (BMD)
27
adrenal crisis (acute adrenal insufficiency)
amiodarone-induced hypothyroidism
120,
bromocriptine
104
144
121, 122
bruising, easy
86
adrenalectomy
8, 35, 87
anaemia, malignant disease
79, 80
adrenal failure (hypoadrenalism)
30, 34-5,
androgens
calcitonin
13, 23
142-5
adrenal
30
serum
140
acute (adrenal crisis)
144
secreting tumours
41, 117, 118
therapy
26, 28, 81, 156
autoimmune
144
see also testosterone
calcitriol
156
clinical presentation
34, 142-3
androstenedione
30, 41
calcium
investigations
34, 143
angiotensin I
32
24-h urinary
25
pituitary disease
106
angiotensin II
32
i.v. administration
24, 152-3
primary
34, 143-4
anion gap
68, 94, 180
metabolism
23-9
secondary
34
anti-androgen agents
41, 118
plasma
23, 24, 25, 79
treatment
34-5, 144
antidiuretic hormone (ADH)
1, 11-12
replacement therapy
24-5, 152
adrenal glands
30-5
deficiency
12, 110
cancer see malignant disease
anatomy
30, 31
excess secretion see syndrome of
cannabinoid receptor type 1 (CB1) blocker
hormone synthesis
41
inappropriate ADH secretion
54, 61-2
pathophysiology
30-5
antihypertensive therapy, pheochromocytoma
carbamazepine
114, 115
adrenal hyperplasia
33, 124
139
carbimazole
15-16, 74
congenital (CAH)
35, 41-2
antiplatelet treatment
127
in pregnancy
20, 76
adrenaline
30
antithyroid drugs
15-16, 74, 76
carcinoid syndrome
64, 65, 158-60
189
190
Index
carcinoid tumours
63-4, 158-60
cyclical
87
precipitants
57, 69, 70
cardiac arrhythmias
73, 181
differential diagnosis
123, 124
prognosis
70
cardiovascular disease, in diabetes
48-9,
investigations
4, 7, 31-2, 86-7
treatment
56-7, 69-70
59
pituitary-dependent see Cushing’s disease
diabetic nephropathy
49, 58, 167-8, 181-2
cardiovascular risk factors
161, 162, 186
treatment
7-8, 32, 87
diabetic neuropathy
49, 58-9
carpal tunnel syndrome
99, 179
cyproterone acetate
41, 118
diabetic retinopathy
49, 58
catecholamines
30
cystic fibrosis
179
Dianette
118
excess
33-4
diarrhoea
158
urinary
33, 139
dehydration
diet, weight losing
61
central pontine myelinolysis
115
diabetic ketoacidosis
67
dipeptidyl peptidase (DPP)-4
53-4
cerebral oedema
71
hyperosmolar non-ketotic hyperglycaemia
inhibitors
54, 55, 55
cerebrovascular disease
48
93, 94
diuretics
180
Charcot’s osteoarthropathy
49, 58
hyponatraemia and
113
potassium-sparing
125
chest infection
67, 93, 94
dehydroepiandrosterone (DHEA)
30, 41
thiazide
80, 177
chest X-ray (CXR)
δ-cells, pancreatic
46
dizziness
84, 106, 143
diabetes insipidus
110
demeclocycline
12, 115
dopamine
30
diabetic ketoacidosis
68
De Quervain’s thyroiditis
18, 75
dopamine agonists
6, 9, 100, 104
hypercalcaemia of malignancy
79, 80
desmopressin
110, 111
dual energy X-ray absorptiometry (DEXA)
hyperosmolar non-ketotic hyperglycaemia
dexamethasone suppression test
7, 83-4
154
94, 95
high dose
32, 87
dumping syndrome
142
cholesterol
60
low dose
4, 7, 31
raised see hypercholesterolaemia
overnight
7, 31
electrocardiogram (ECG)
total (TC)
60, 161
diabetes insipidus (DI)
109-12
Conn’s syndrome
123, 124
choriocarcinoma
15
causes
111
diabetic ketoacidosis
68, 70, 71
chromogranin A, plasma
64, 159
cranial
12, 110, 111
hyperosmolar non-ketotic hyperglycaemia
Chvostek’s sign
24, 151
differential diagnosis
67
94, 96
coarctation of aorta
123, 124
investigations
109-10
erectile dysfunction
147-8
coeliac disease
152, 177
nephrogenic
12, 111
erythema nodosum
110
computed tomography (CT)
177
treatment
110-11
exenatide
53-4
confusion, acute
92-7
diabetes mellitus
46-59, 126-31, 170-1
ezetimibe
59, 61, 162
congenital adrenal hyperplasia (CAH)
35,
classification
46-7, 127-8
41-2
clinical presentation
47-8
face
Conn’s syndrome
32-3, 123-5
diagnosis
49-50, 126, 130
acromegaly
99, 100
coronary artery disease risk factors
161, 162,
differentiation of type
47-8, 126-7, 130-1,
moon-like
86
186
174, 183
fasting tests
63
corticosteroids
30
gestational see gestational diabetes
female infertility
44, 45
see also cortisol; glucocorticoids;
investigations
49-50
female reproductive system
mineralocorticoids; steroids
lipid abnormalities
60
anatomy
36, 37
corticotrophin releasing hormone (CRH)
1, 2
lipid lowering therapy
59, 61
pathophysiology
38-42
stimulation test
8, 87
long-term management
127
physiology
36-8
cortisol
30
MCQs
166-8, 179-80, 181-2
ferritin, serum
148-9
24-h urinary
7, 31
newly diagnosed
86-8
fibrates
59, 61, 162
biosynthesis
41
secondary
47, 50, 86, 128
finasteride
41, 118
deficiency
8, 41
treatment
50-4, 172
fine needle aspiration (FNA), thyroid
21, 135,
excess
6-8
type 1 see type 1 diabetes mellitus
177
midnight
7, 31
type 2 see type 2 diabetes mellitus
fludrocortisone
35, 42, 144
negative feedback on pituitary
1, 2
diabetic complications
48-9
fluid replacement
replacement therapy see hydrocortisone
acute
48, 54-8
complications
71
stimulation tests
3, 4, 8, 143-4
chronic
48-9, 58-9
diabetic ketoacidosis
56, 57, 69
suppression tests
4, 7
investigations
50
hypercalcaemia
80
cough
79, 110
macrovascular (large vessel)
48-9, 59,
hyperosmolar non-ketotic hyperglycaemia
C peptide
46, 63, 142
127
(HONK)
58, 94, 95
craniopharyngioma
12
management
54-9
fluid restriction, SIADH
12, 115
CRH see corticotrophin releasing hormone
microvascular (small vessel)
49, 58-9,
flushing
64, 158
cultures
68
127
flutamide
41, 118
Cushing’s disease
6, 31, 86
prevention and screening
53, 71, 127
follicle stimulating hormone (FSH)
1, 36,
complications
87
diabetic hyperosmolar non-ketotic
38
diagnosis
4, 31-2, 83-4
hyperglycaemia see hyperosmolar
deficiency
82, 148
symptoms
82, 83
non-ketotic hyperglycaemia
excess
10, 82
Cushing’s syndrome
30-2, 86-8
diabetic ketoacidosis (DKA)
48, 54-7,
plasma
3, 39, 43
ACTH-dependent
6, 7-8, 31, 86-7
66-72
therapy
149
ACTH-independent
6, 31, 86
clinical presentation
56, 67-8
foot ulcers, diabetic
181
causes
31, 86
complications
70-1
fractures, osteoporotic
27-8, 154-6
clinical features
7, 30-1, 86, 87
investigations
50, 56, 68-9, 93
FSH see follicle stimulating hormone
complications
86, 87, 179
MCQ
165, 178
full blood count (FBC)
68, 79
Index
191
galactorrhoea
103
growth hormone releasing hormone (GHRH )
secondary
162, 163
gastric bypass surgery
62
1, 2
treatment
60-1, 162
gastrinomas
64, 65, 159
excess secretion
100
hyperosmolar non-ketotic hyperglycaemia
gestational diabetes
47, 128, 167,
gynaecomastia
43-4, 146
(HONK)
48, 57-8, 92-7
180
clinical presentation
57, 92-3
GH see growth hormone
haemochromatosis
148-50, 179, 181
complications
95-6
gigantism
5
haemoglobin, glycosylated (HbA1c)
52, 71,
investigations
57, 93-4
glibenclamide
53
128
MCQ
165, 178
gliclazide
53, 93, 129
hand size, changes in
99, 100
prognosis
95
glimepiride
53, 129
headache
98-9
treatment
58, 94-5
glitazones see thiazolidinediones
acute and recurrent
83, 139
hyperparathyroidism
25, 26
glucagon
46
sudden onset
84-5
primary
80, 139-40
glucagon-like peptide-1 (GLP-1) analogues
tension
99
tertiary
25, 27
53-4, 55, 55
heat intolerance
73, 139
hyperprolactinaemia
glucagonomas
64-5, 160
hemianopia, bitemporal
82, 83
amenorrhoea and
164, 176-7
glucagon stimulation test
3, 4, 6
heparin
causes
9, 104
glucocorticoids
diabetic ketoacidosis
57, 70
prolactinoma
8, 103-4
deficiency
34-5
hyperosmolar non-ketotic hyperglycaemia
hypertension
excess
30-2, 47
(HONK)
58, 95
acromegaly
99
replacement therapy
144
hepatic metastases
80, 115, 159
Cushing’s syndrome
86
see also cortisol; hydrocortisone
high-density lipoprotein cholesterol (HDLc)
essential
123
Glucophage see metformin
60, 161
hyperaldosteronism
32, 123
glucose
hilar lymphadenopathy
110
pheochromocytoma
33, 139
capillary
56, 57, 68, 70, 93
hirsutism
40, 117
secondary
123, 139, 174, 185
fasting plasma
49, 128
diagnosis of cause
117, 174, 185
hypertensive crisis
33
random plasma
49, 68, 79
treatment
41
hyperthyroidism (thyrotoxicosis)
13, 14-18
tight control in diabetes
52, 71
HONK see hyperosmolar non-ketotic
amiodarone-induced (AIT)
19-20, 120-2
glucose tolerance test, oral (OGT)
hyperglycaemia
apathetic
16
diabetes mellitus
49
hormone replacement therapy (HRT)
11, 28,
case
73-8
pituitary disease
4, 5, 83, 99
39, 156
causes
15, 76
α-glucosidase inhibitors
53, 55
human chorionic gonadotrophin (hCG)
149,
clinical presentation
14-15, 16, 73-4
glutamic acid decarboxylase (GAD) antibodies
180
diagnosis of cause
174, 185
50, 127
hydrocortisone (cortisol replacement)
hypercalcaemia
80
goitre
15
adrenal failure
34, 144
in pregnancy
20, 76
multinodular
20-1
pituitary failure
8, 11, 107
subclinical (SHyper)
19
thyroid
13, 20
5-hydroxyindolacetic acid (5-HIAA)
63, 64,
TSH-producing pituitary tumours
9
toxic multinodular (TMNG)
15, 17
159
see also Graves’ disease
gonadotrophin releasing hormone (GnRH)
1,
11β-hydroxylase deficiency
42
hypertriglyceridaemia
60, 161, 162
2, 36, 38
21α-hydroxylase deficiency
35, 41-2
hypoadrenalism see adrenal failure
gonadotrophins
1
complete
42
hypocalcaemia
23, 24-5
deficiency
10, 82, 148
partial
42
causes
24, 152
excess
10, 82, 146
hyperadrenalism
30-2
magnesium deficiency
153
investigations
3
hyperaldosteronism
30, 32-3
osteomalacia
151-3
Graves’ acropachy
15, 16
causes
32
postthyroidectomy
164-5, 177
Graves’ dermopathy
16
primary
32, 124-5
treatment
24-5, 152-3
Graves’ disease (GD)
14-17, 73-8
hyperandrogenism
42
hypoglycaemia
clinical presentation
14-15, 16, 73-4
hypercalcaemia
23, 25-6, 174
causes
168, 182
diagnosis
174, 185
causes
25, 80, 165, 177, 186
diabetes mellitus
52, 58
extrathyroidal manifestations
15, 16
clinical presentation
25, 79
factitious
142, 143
investigations
15, 74
differential diagnosis
67
hypoadrenalism
142-3
long-term management
76-7
familial hypocalciuric (FHH)
25, 80, 177
insulinoma
63
in pregnancy
20, 76
investigations
25, 80
symptoms
143
treatment
15-17, 74-5
of malignancy
26, 79-81
treatment
58
Graves’ ophthalmopathy (GO)
14, 16,
primary hyperparathyroidism
139-40
hypogonadism
73-4
treatment
25-6, 80-1, 174, 186
hypergonadotrophic
43, 45, 146
clinical features
74
hypercholesterolaemia
60, 161
hypogonadotrophic
43, 45, 148, 149, 167,
management
75
causes of secondary
162, 163
181
growth hormone (GH)
1, 2
familial
60
male
43, 146-50
abnormalities of secretion
4-6
hyperemesis gravidarum
15, 180
osteoporosis
155
deficiency (GHD)
6, 82
hyperglycaemia
93
primary
146-7
excess
4-6, 82
see also hyperosmolar non-ketotic
secondary
147, 148-9
plasma
5, 6
hyperglycaemia
hypokalaemia
replacement therapy
6, 11
hyperlipidaemia
60-1, 127, 161-3
diabetic ketoacidosis
70, 71
stimulation tests
3, 4
familial combined
60
ectopic ACTH production
86
suppression test
4, 5
MCQ
168, 182
hypokalaemic alkalosis
124
192
Index
hypomagnesaemia
24, 153
ketonuria
high anion gap
68, 167, 180
hyponatraemia
175
diabetic ketoacidosis
68, 69
normal anion gap
68, 180
causes
113, 114, 165, 177, 186
type 1 diabetes
50, 126
metabolic alkalosis
167, 180
diabetic ketoacidosis
69
Klinefelter’s syndrome
43, 165, 178
meta-iodobenzylguanidine (MIBG) scan
33,
management
115
investigations
146-7, 181
139
SIADH
11-12, 113
sexual dysfunction
146-7, 149-50
metformin
hypoparathyroidism
24
Kussmaul respiration
54
polycystic ovary syndrome
41, 118
hypophysitis, lymphocytic
12
side effects
129, 179
hypopituitarism see pituitary failure
lactic acidosis
53, 94, 129, 179
type 2 diabetes
53, 55, 55, 128
hypotension
67, 106
latent autoimmune diabetes of adults (LADA)
methimazole
15-16
postural
143, 181
46, 128
metoclopramide
164, 176
unexplained
144
left ventricular failure
95-6
microalbuminuria
49, 58, 167-8, 181
hypothalamic disorders
39, 43, 44
left ventricular hypertrophy (LVH)
123, 124
microprolactinomas
8, 104
hypothalamic-pituitary axis
1, 2
Leydig cells
36, 37
mineralocorticoids
30
hypothalamic-pituitary-ovarian axis
36,
LH see luteinizing hormone
deficiency
34-5
38
lifestyle changes
60, 128, 162
excess
32-4
hypothalamic-pituitary-testicular axis
38
lipid abnormalities
60-1
see also aldosterone; fludrocortisone
hypothalamus
1, 2
see also hyperlipidaemia
MODY see maturity onset diabetes of the young
hypothyroidism
13, 18-19, 89-91
lipid lowering therapy
60-1, 162
multinodular goitre
20-1
amiodarone-induced
19, 121, 122
diabetes mellitus
59, 127
multiple endocrine neoplasia (MEN)
autoimmune (AH)
18, 19, 89-90
MCQ
168, 182
type I (MEN I)
65, 140
causes
18, 91
lipoatrophy
52
type II (MEN II)
33, 65, 139-40
clinical presentation
18, 89
lipohypertrophy
52
muscle
hyperprolactinaemia
176
liver disease
aches and pains
151
investigations
19, 89-90
lipid abnormalities
60
weakness
73, 86, 151
lipid abnormalities
60
see also hepatic metastases
myeloma, multiple
25
myxoedema coma
20
liver function tests (LFTs)
myocardial infarction (MI)
pituitary failure
10, 19
haemochromatosis
147
acute, in diabetes
96
postpartum
106
malignant disease
79, 80
risk in diabetes
59
in pregnancy
20, 90
Paget’s disease
132
silent
68, 96, 179
secondary
106, 166, 178
Looser zone
26, 152
type 2 diabetes
174, 186
subclinical (SHypo)
19
low-density lipoprotein cholesterol (LDLc)
myopathy, proximal
151
treatment
19, 90
60, 161
myxoedema, pretibial
16, 74
lung cancer
79-81
myxoedema coma
20
incidentalomas, adrenal
35, 175, 186
luteinizing hormone (LH)
1, 36, 38
infertility
44-5, 147
deficiency
82, 148
nasogastric tube, diabetic ketoacidosis
57, 70
inhibin
36, 38
excess
82
natiglinide
53
insulin
46
plasma
3, 39, 43
nausea
142
deficiency
46
lymphocytic hypophysitis
12
neck
factitious hypoglycaemia
142, 143
examination
14, 135
injection regimes
50-1, 52, 71, 127
macroprolactinomas
84, 104
lumps/masses
14, 21, 135, 165, 177
injection site complications
52
magnesium deficiency
24, 153
pain and tenderness
18, 75
i.v. infusion
56, 57, 58, 70, 95
magnetic resonance imaging (MRI), pituitary
necrolytic migratory erythema
64
preparations
50, 52
gland
4, 5, 84
nephrotic syndrome
162
pumps
52
malabsorption
176
neuroendocrine syndromes
65
resistance
46, 47
male hypogonadism
43, 146-50
neuroendocrine system
63-5
sliding scale
56, 70
male infertility
44-5, 147
neuroendocrine tumours
63-5, 159-60
type 1 diabetes therapy
50-2, 127
male reproductive system
neurofibromatosis (NF)
33, 65, 140
type 2 diabetes therapy
54, 129
anatomy
36, 37
nicotinic acid
61, 162
insulin-like growth factor-1 (IGF-I)
5, 6
pathophysiology
39, 43-4
nocturia
67
insulinomas
63, 64, 159
physiology
38
non-thyroidal illness
176
clinical presentation
63, 65
malignant disease
noradrenaline
30
differential diagnosis
142, 143
hypercalcaemia
26, 79-81
insulin secretagogues
53
SIADH
115
obesity (and overweight)
61-2, 168, 182
insulin sensitizers
53
maturity onset diabetes of the young (MODY)
polycystic ovary syndrome
40, 117
insulin stress test
3, 4, 6, 148
127-8
secondary amenorrhoea and
174, 185
interferon therapy
159
diagnosis
48, 50, 126
truncal
86
iodine
13
genetics
47, 181
type 2 diabetes
126, 128
see also radioactive iodine
medical treatment
172
octreotide
irritability
73
meglitinides
53, 55
neuroendocrine tumours
63, 65, 159
ischaemic heart disease (IHD)
48
menopause, premature
39-40, 154
radionuclide scan
63, 159
islet cell antibodies
127
menstrual abnormalities
39, 102, 117
see also somatostatin analogues
menstrual cycle
36
oestradiol
3, 36, 38
Kallman’s syndrome
43, 149, 181
metabolic acidosis
67, 94
deficiency
10
ketoconazole
144
diabetic ketoacidosis
54, 69
oestrogen
36
Index
193
17-OH progesterone
42, 118
phosphate, urinary
26
hypothyroidism
20, 90
omega-3 fatty acids
61
pigmentation, in Addison’s disease
143
prolactinoma
104
optic chiasm lesions
82, 83
pins and needles
24, 99, 151
raised prolactin levels
164, 177
oral contraceptive pill (OCP)
41
pioglitazone
53, 179-80
type 2 diabetes therapy
54
polycystic ovary syndrome
118
pituitary adenomas see pituitary tumours
see also gestational diabetes
recent withdrawal bleed
66, 67
pituitary apoplexy
12, 85
premature ovarian failure
39-40
oral glucose tolerance test see glucose tolerance
pituitary failure (hypopituitarism)
2-3
pretibial myxoedema
16, 74
test, oral
causes
2, 11, 107
progesterone
36, 38
oral hypoglycaemic agents
53-4, 55, 128-9
clinical presentation
106
prolactin
1
orlistat
54, 55, 61
investigations
3, 4, 106, 107
deficiency
9, 106
osmolarity
non-functioning pituitary adenoma
10
raised serum see hyperprolactinaemia
plasma
94, 109-10, 111, 114
postpartum
106-8
serum
3, 4
urine
109-10, 111, 114
treatment
4, 107
prolactinomas
8-9, 102-5
osteogenesis imperfecta
29
pituitary function tests
3-4
clinical presentation
8, 82, 103
osteomalacia
23, 26-7, 151-3
pituitary gland
170
investigations
9, 83, 84, 103-4
differential diagnosis
132
anatomy
1, 2
treatment
9, 84, 104
investigations
26, 152
anterior
1, 2, 4-11
propylthiouracil
15-16, 74
osteoporosis
23, 27-8, 154-7
imaging
4, 5
in pregnancy
20, 76
clinical presentation
27, 154
investigations
3-4
proton pump inhibitors
64
early-onset
174, 185-6
oversecretion
1
pseudo-Cushing’s syndrome
6-7
investigations
27-8, 154-6
physiology
1, 2
pseudofractures
26, 152
treatment
28, 156
posterior
1, 2, 11-12
pseudohypoparathyroidism
24
ovarian failure, premature
39-40
surgery
84, 87, 99, 104
PTH see parathyroid hormone
ovarian tumours, virilizing
42
pituitary infarction
11, 12, 85, 107
puberty
45
ovaries
36, 37
pituitary tumours (adenomas)
1, 2-3, 10
delayed
45
polycystic
40
ACTH secreting
6, 7-8, 82
precocious
45
overweight see obesity
diagnosis
3-4
pulmonary embolus (PE)
67, 95
ovulation
36
gonadotrophin producing
10, 82
oxytocin
1, 12
growth hormone secreting
4-6, 82
radioactive iodine (RAI)
headache
99
hyperthyroidism
16-17, 74-5, 76
Paget’s disease
23, 28-9, 132-4
imaging
5, 84
thyroid carcinoma
135
clinical presentation
28, 132-3
investigations
3-4, 83-4
radiotherapy
investigations
28, 29, 132, 177
long-term follow up
85
head, complications
181
treatment
29, 133
MCQs
166, 178-79, 181
pituitary tumours
87, 99
palpitations
73
non functioning
9, 10-11, 82-5
ramipril
180
pamidronate
80-1
prolactin secreting see prolactinomas
5α-reductase inhibitors
41, 118
pancreas
46-59
treatment
4, 84, 87, 104
renal artery stenosis
123, 124
anatomy
46, 47
TSH producing (TSH-omas)
9, 15, 18, 82,
renal failure, chronic
pathophysiology
46
83
hypercalcaemia
25
physiology
46
visual field defects
82, 174, 185
hypertension
123, 124
pancreatic polypeptide
46
pleural effusion
79, 80
lipid abnormalities
60
pancreatitis, acute
162
pneumonia see chest infection
osteomalacia
26, 27
parathyroid adenoma
25
polycystic ovary syndrome (PCOS)
40-1,
polyuria
67
parathyroid glands
23, 24
117-19
raised alkaline phosphatase
177
parathyroid hormone (PTH)
23
clinical presentation
40, 117-18
renal tubular acidosis type IV
34
abnormalities of secretion
24, 25
diagnosis
174, 185
renin
30, 32
analogue therapy
156
investigations
40, 118
impaired secretion
34
plasma
80, 139
MCQ
166, 179
plasma activity (PRA)
124, 165, 177-8
resistance
24
raised prolactin levels
164, 176
repaglinide
53
see also hyperparathyroidism
treatment
41, 118
reproductive system
36-45, 169
parathyroid hormone (PTH)-related peptide
polydipsia
79, 86, 109, 126
anatomy
36, 37
(PTHrP)
80
psychogenic
111
pathophysiology
38-45
pegvisomant
6
polyuria
67, 79, 86, 109, 126
physiology
36-8
Pemberton’s sign
14
postpartum thyroiditis
18
respiratory alkalosis
54, 67
peripheral neuropathy, diabetic
49, 58-9
potassium, serum
ret proto-oncogene
140
peripheral vascular disease
49
diabetic ketoacidosis
69-70
rhabdomyolysis
180
petrosal sinus sampling
7, 87
see also hypokalaemia
rickets
26-7
pheochromocytoma
30, 33-4, 139-41
potassium therapy
rimonabant
54, 55, 61-2
clinical presentation
33, 139
diabetic ketoacidosis
56, 57, 70
risk factors, coronary artery disease
161, 162,
differential diagnosis
123, 124
hyperosmolar non-ketotic hyperglycaemia
186
familial
140
(HONK)
58, 95
rosiglitazone
53
investigations
33, 139
PP cells, pancreatic
46
management
33-4, 139-40
pregnancy
sarcoidosis
110
MCQ
166, 179
as cause of amenorrhoea
102
seizures
164-5, 177
rule of 10
33-4, 140
hyperthyroidism
20, 76, 180
semen analysis
44-5
194
Index
serotonin
63, 158
tachypnea
67
thyroid stimulating antibodies (TSAb)
14, 15,
sex hormone binding globulin (SHBG)
43,
testes
36, 37
74
118
trauma
181
thyroid stimulating hormone (TSH)
1, 2, 14
sex hormones
testicular failure
deficiency
10, 82
deficiency
10
primary
146-7
excess
9, 82, 83
negative feedback on pituitary
1, 2
secondary
147, 148-9
producing pituitary tumours
9, 15, 18, 82,
pituitary disease
3, 4
testicular feminization syndrome
39
83
see also androgens; oestradiol; testosterone
testosterone
3, 36, 38
serum
3, 74, 90
sexual dysfunction
98, 146-50
biosynthesis
41
suppression therapy
136
causes
149
deficiency
10, 43, 146
see also thyroid function tests
diabetes
59, 147-8
plasma
118
thyroid storm
16, 20, 77
Sheehan’s syndrome
9, 107
replacement therapy
28, 43, 107, 147
thyroid uptake scan
177
SIADH see syndrome of inappropriate ADH
tests, endocrine
171-2
Graves’ disease
15, 17
secretion
tetany
24
indications
174, 185
sibutramine
54, 61
TFTs see thyroid function tests
thyroiditis
75, 76
skin, dry
89
thiazide diuretics
80, 177
toxic nodule
17, 18, 136, 137
slimming tablets
54, 61-2, 129
thiazolidinediones
thyrotoxicosis see hyperthyroidism
smoking
79, 154, 161
polycystic ovary syndrome
41, 118
thyrotrophin releasing hormone (TRH)
1, 2
sodium, urinary
114
side effects
179-80
thyroxine see T4
soft tissue changes, acromegaly
99, 100
type 2 diabetes
53, 55, 55, 129
tiredness
89, 98, 106, 113, 164
somatostatin
46
thionamides see antithyroid drugs
toxic multinodular goitre (TMNG)
15, 17
somatostatin analogues
third nerve palsy
84
toxic thyroid nodule
15, 17, 18, 136-7
acromegaly
5, 100
thromboembolic disease
95-6
transglutaminase (tg) antibodies
152
neuroendocrine tumours
64, 65, 159
thyroglobulin (TG)
13, 22, 136
tremor, hand
13, 73, 142
see also octreotide
thyroid
13-22, 169
triglycerides
60, 161, 162
somatostatinoma
64, 65, 160
anatomy
13, 14
Trousseau’s sign
24, 151
spironolactone
41, 118, 125
examination
13-14
T score
154
statins
59, 60-1, 162
fine needle aspiration (FNA)
21, 135, 177
TSH see thyroid stimulating hormone
side effects
162, 180
pathophysiology
13
Turner’s syndrome
39-40, 166, 179
steroids
physiology
13
type 1 diabetes mellitus (T1DM)
46, 126-7
amiodarone-induced hyperthyroidism
thyroid cancer
21-2, 135-8
clinical presentation
47-8, 67, 126
121
investigations
21, 135
diagnosis
47-8, 126-7, 174, 185
hypercalcaemia of malignancy
81
medullary
136, 140
honeymoon period
127
osteoporosis and
154
papillary
135-6
hypothyroidism and
89-91
primary hypoadrenalism
144
prognosis
136
long-term management
52-3, 71, 127
replacement therapy
8, 11
treatment
22, 135-6
newly diagnosed
52, 127
see also cortisol; glucocorticoids;
types
136
treatment
50-3, 127
mineralocorticoids
thyroidectomy
type 2 diabetes mellitus (T2DM)
46, 47,
stimulation tests
3, 4
complications
137, 164-5, 177
128-9
striae, abdominal
86, Plate 3
Graves’ disease
17, 75
acute confusion
92-7
strontium
28, 156
thyroid cancer
135-6
cardiovascular disease prevention
59, 174,
struma ovarii
15
thyroid function tests (TFTs)
186
sulphonylureas (SU)
53, 128-9
amiodarone-induced hyperthyroidism
120
clinical presentation
47-8
deliberate ingestion
142, 143
hyperthyroidism
74, 75
diagnosis
47-8, 126-7, 128, 174, 185
mode of action
55
hypothyroidism
90
erectile dysfunction
147-8
side effects
55
MCQs
164, 167, 176, 180-1
MCQ
166-7, 179-80
suppression tests
3, 4
pituitary disease
3, 4, 106
treatment
53-4, 55, 128-9
sweating
thyroid hormones
13
tyrosine phosphatase (IA-2) antibodies
50
gustatory
181
excess ingestion
15
increased
98, 99, 139, 142
negative feedback on pituitary
1, 2
U&Es (urea and electrolytes)
synacthen test
34, 143-4
regulation of production
14
diabetic ketoacidosis
68, 69
syndrome of inappropriate ADH secretion
resistance
15, 176
hypercalcaemia of malignancy
79
(SIADH)
11-12, 113-16
see also T3; T4
hyperosmolar non-ketotic hyperglycaemia
causes
114
thyroiditis
15, 17-18, 91, 180
(HONK)
94
clinical presentation
113
De Quervain’s
18, 75
osteomalacia
151
investigations
113-15
postpartum
18
SIADH
113, 114
thyroid uptake scan
75, 76
ultrasound
T3
13, 14
thyroid nodules
13, 20-1, 135
liver
115
toxicosis
74
cold
135, 136-7
ovarian
40, 118
T4 (thyroxine)
13, 14
euthyroid subjects
20-1, 135
thyroid
121, 177
block and replace therapy
74
hot
17, 135, 136, 137
urinary tract infections (UTI)
66, 67,
poor compliance with therapy
164, 176
malignant
135
181-2
replacement therapy
10, 11, 19, 90, 136
toxic
15, 17, 18, 136-7
urine dipstick tests
serum free (FT4)
3, 9, 10
thyroid peroxidase (TPO)
13
diabetes
50, 93
see also thyroid function tests
antibodies
19, 74, 90
diabetic ketoacidosis
68, 69
Index
195
vertebral fractures
27-8, 154-5
vomiting
66, 84
white blood count (WBC), diabetic
VIPoma
64, 65, 160
von Hippel-Lindau disease
33, 65, 140
ketoacidosis
68, 69
virilization
117
wrist fracture
154
virilizing tumours
42
water deprivation test
110, 111
visual field defects
82, 83, 103,
water excess
113
Xenical see orlistat
185
weakness, muscle
73, 86, 151
X-rays
visual problems
82, 174, 185
weight gain
osteomalacia
26, 152
vitamin D
23
Cushing’s syndrome
86
osteoporosis
27-8, 154-5
deficiency
24, 26, 27, 152
hypothyroidism
89
Paget’s disease
28, 29, 132
dependent rickets
26
weight loss
intoxication
25
drugs for aiding
54, 61-2
Zollinger Ellison syndrome (gastrinoma)
64,
therapy
26-7, 152, 156
therapeutic
41, 53, 61, 118
65, 159
vitiligo
90
unintentional
47, 66, 79, 143
This page intentionally left blank